Integral Calculus For Science and Engineering Students 1
Integral Calculus For Science and Engineering Students 1
Integral Calculus For Science and Engineering Students 1
INTEGRAL CALCULUS
For Science and Engineering Students
Alan R. Cañares
0
Contents
1. The Indefinite Integral 1
7. Integration by Parts 94
References 325
0
1. The Indefinite Integral
𝐹 ′ (𝑥) = 𝑓(𝑥)
Anti-differentiation is also called integration and 𝐹(𝑥) is the integral of 𝑓(𝑥). Using
Leibniz’ notation, the integral of 𝑓(𝑥) is
𝐹(𝑥) = ∫ 𝑓(𝑥) 𝑑𝑥
The function 𝑓(𝑥) is called the integrand. The notation 𝑓(𝑥) 𝑑𝑥 is usually called the
element of integration. The differential 𝑑𝑥 defines the variable for which the integral of a
function is being evaluated. The notation that appears above is also called indefinite
integral to distinguish it from definite integral which will be discussed much later.
_____________________________________________________________________________________________________
Example 1.1.
Show that 𝐹(𝑥) = 3𝑥 2 − 4𝑥 is an anti-derivative of 𝑓(𝑥) = 6𝑥 − 4.
𝐹(𝑥) = 3𝑥 2 − 4𝑥
1
𝐹 ′ (𝑥) = 3 ∙ 2 ∙ 𝑥 2−1 − 4 ∙ 1 ∙ 𝑥1−1
𝐹 ′ (𝑥) = 6𝑥 − 4 = 𝑓(𝑥) 𝑞. 𝑒. 𝑑
_____________________________________________________________________________________________________
Even if a function 𝑓(𝑥) is defined on an interval [𝑎, 𝑏], its anti-derivative may not
exist on that interval or it may not exist at all. In many instances, where the anti-
derivative exists, it may not be expressed in terms of elementary functions.
If the integral of 𝑓(𝑥) is 𝐹(𝑥), the integral of 𝑓(𝑎𝑥 + 𝑏) where 𝑎 and 𝑏 are
constants, is
1
∫ 𝑓(𝑎𝑥 + 𝑏) 𝑑𝑥 = 𝐹(𝑎𝑥 + 𝑏) + 𝐶
𝑎 1.2
What is the integral of a power function 𝑓(𝑥) = 𝑥 𝑛 ? To solve this, we are looking
for a function 𝐹(𝑥) such that 𝐹 ′ (𝑥) = 𝑥 𝑛 . Recall in differential calculus that the
derivative of a power function 𝑥 𝑛 is
𝑑 𝑛
𝑥 = 𝑛𝑥 𝑛−1
𝑑𝑥
If we replace 𝑛 by 𝑛 + 1, we get
2
𝑑 𝑛+1
𝑥 = (𝑛 + 1)𝑥 (𝑛+1)−1
𝑑𝑥
= (𝑛 + 1)𝑥 𝑛
𝑑 𝑥 𝑛+1
= 𝑥𝑛
𝑑𝑥 𝑛 + 1
Hence,
𝑛
𝑥 𝑛+1
∫ 𝑥 𝑑𝑥 = + 𝐶, 𝑛 ≠ −1 1.3
𝑛+1
_____________________________________________________________________________________________________
Example 1.2.
Evaluate
∫(5𝑥 3 − 3𝑥 2 + 2𝑥 − 4) 𝑑𝑥
5 3 2 4
∫(5𝑥 3 − 3𝑥 2 + 2𝑥 − 4) 𝑑𝑥 = 3+1𝑥 3+1 − 2+1𝑥 2+1 + 1+1𝑥1+1 − 0+1𝑥 0+1 + 𝐶
5
= 𝑥 4 − 𝑥 3 + 𝑥 2 − 4𝑥 + 𝐶 ANS.
4
_____________________________________________________________________________________________________
Example 1.3.
Evaluate
1
∫ ( √𝑥 + ) 𝑑𝑥
√𝑥
2 3/2
= 𝑥 + 2√𝑥 + 𝐶 ANS.
3
_____________________________________________________________________________________________________
3
It should be noted that 𝑛 ≠ −1 because if so, then the anti-derivative of 𝑥 −1
becomes undefined using Eq. 1.3. Recall in differential calculus that the derivative of 𝑙𝑛 𝑥
is
𝑑 1
𝑙𝑛|𝑥| =
𝑑𝑥 𝑥
Hence,
∫ 𝑥 𝑛 𝑑𝑥 = 𝑙𝑛 |𝑥| + 𝐶, 𝑛 = −1
1
∫ 𝑑𝑥 = 𝑙𝑛 |𝑥| + 𝐶 1.4
𝑥
1 1
∫ 𝑑𝑥 = 𝑙𝑛 |𝑎𝑥 + 𝑏| + 𝐶
𝑎𝑥 + 𝑏 𝑎
_____________________________________________________________________________________________________
Example 1.4.
Evaluate
6𝑥 3 + 𝑥 2 − 10𝑥 − 3
∫ 𝑑𝑥
2𝑥 + 3
6𝑥 3 + 𝑥 2 − 10𝑥 − 3 6
∫ 𝑑𝑥 = ∫ (3𝑥 2 − 4𝑥 + 1 − ) 𝑑𝑥
2𝑥 + 3 2𝑥 + 3
= 𝑥 3 − 2𝑥 2 + 𝑥 − 3 𝑙𝑛 |2𝑥 + 3| + 𝐶 ANS.
_____________________________________________________________________________________________________
∫[𝑓(𝑥)]𝑛 𝑓 ′ (𝑥) 𝑑𝑥
4
𝑛 ′ (𝑥) 𝑛
𝑢𝑛+1
∫[𝑓(𝑥)] 𝑓 𝑑𝑥 = ∫ 𝑢 𝑑𝑢 = +𝐶
𝑛+1
Hence,
𝑛 ′ (𝑥)
[𝑓(𝑥)]𝑛+1
∫[𝑓(𝑥)] 𝑓 𝑑𝑥 = +𝐶 1.5
𝑛+1
𝑓′(𝑥)
∫ 𝑑𝑥
𝑓(𝑥)
𝑓′(𝑥) 𝑑𝑢
∫ 𝑑𝑥 = ∫ = 𝑙𝑛 |𝑢| + 𝐶
𝑓(𝑥) 𝑢
Hence,
𝑓′(𝑥)
∫ 𝑑𝑥 = 𝑙𝑛 |𝑓(𝑥)| + 𝐶 1.6
𝑓(𝑥)
_____________________________________________________________________________________________________
Example 1.5.
Evaluate
5𝑥 − 1
∫3 𝑑𝑥
√5𝑥 2 − 2𝑥 + 7
5𝑥 − 1 1
∫3 𝑑𝑥 = ∫ 𝑢−1/3 𝑑𝑢
2
√5𝑥 − 2𝑥 + 7 2
1 3 2/3
= ∙ 𝑢 +𝐶
2 2
3
(5𝑥 2 − 2𝑥 + 7)2/3 + 𝐶
= ANS.
4
_____________________________________________________________________________________________________
5
Example 1.6.
Evaluate
∫(3𝑥 − 9)√𝑥 + 5 𝑑𝑥
2 2
=3∙ (𝑥 + 5)5/2 − 24 ∙ (𝑥 + 5)3/2 + 𝐶
5 3
6
= (𝑥 + 5)5/2 − 16(𝑥 + 5)3/2 + 𝐶 ANS.
5
The above solution is a short cut. Using the substitution rule, we can let
𝑢 = 𝑥 + 5 such that 𝑥 = 𝑢 − 5 and 𝑑𝑥 = 𝑑𝑢. The integral becomes
= ∫(3𝑢3/2 − 24√𝑢) 𝑑𝑢
6 5/2
= 𝑢 − 24𝑢3/2 + 𝐶
5
6
= (𝑥 + 5)5/2 − 16(𝑥 + 5)3/2 + 𝐶 ANS.
5
_____________________________________________________________________________________________________
Example 1.7.
Evaluate
𝑑𝑦
∫
1 − 𝑦2
6
1 1 2
+ =
1 + 𝑦 1 − 𝑦 1 − 𝑦2
1 1 1
∫( + ) 𝑑𝑦
2 1+𝑦 1−𝑦
Integrating yields
1
[𝑙𝑛(1 + 𝑦) − 𝑙𝑛(1 − 𝑦)] + 𝐶
2
Simplifying gives
𝑑𝑦 1 1+𝑦
∫ = 𝑙𝑛 +𝐶 ANS.
1 − 𝑦2 2 1 − 𝑦
_____________________________________________________________________________________________________
Example 1.8.
Evaluate
4𝑥 − 7
∫ 𝑑𝑥
(2𝑥 + 1)(𝑥 − 4)
4𝑥 − 7
∫ 𝑑𝑥
2𝑥 2− 7𝑥 − 4
4𝑥 − 7
∫ 𝑑𝑥 = 𝑙𝑛|2𝑥 2 − 7𝑥 − 4| + 𝐶 ANS.
2𝑥 2− 7𝑥 − 4
_____________________________________________________________________________________________________
7
Example 1.9.
Evaluate
1 5
− ∫ 𝑢4 𝑑𝑢 = − 𝑢 +𝐶
5
1
= − (6 − 3𝑥 − 𝑥 2 )5 + 𝐶 ANS.
5
_____________________________________________________________________________________________________
Example 1.10.
Evaluate
2𝑥 3 − 1
∫ 𝑑𝑥
𝑥 + 𝑥4
Solution. This question was asked in the 2006 MIT Integration Bee. To solve it, we would
like to transform the integral into this form: ∫ 𝑑𝑢/𝑢. But the numerator does not seem to
be the derivative of the denominator. But if we divide the numerator and denominator
by 𝑥 2 /𝑥 2 , we get
2𝑥 3 − 1 𝑥 2 2𝑥 − 1/𝑥 2
∫ ÷ 2 𝑑𝑥 = ∫ 𝑑𝑥
𝑥 + 𝑥4 𝑥 1/𝑥 + 𝑥 2
2𝑥 − 1/𝑥 2 𝑑𝑢
∫ 2
𝑑𝑥 = ∫
1/𝑥 + 𝑥 𝑢
= 𝑙𝑛 |𝑢| + 𝐶
= 𝑙𝑛 |1/𝑥 + 𝑥 2 | + 𝐶
1 + 𝑥3
= 𝑙𝑛 | |+𝐶 ANS.
𝑥
8
Another approach uses the fact that if 𝑢 = 𝑥 + 𝑥 4 , 𝑑𝑢 = (1 + 4𝑥 3 ) 𝑑𝑥. So, we can
add (2𝑥 3 + 2) to the numerator and subtract it as well. The integral, thus, becomes
2𝑥 3 − 1 2𝑥 3 − 1 + (2𝑥 3 + 2) − (2𝑥 3 + 2)
∫ 𝑑𝑥 = ∫ 𝑑𝑥
𝑥 + 𝑥4 𝑥 + 𝑥4
(4𝑥 3 + 1) − (2𝑥 3 + 2)
=∫ 𝑑𝑥
𝑥 + 𝑥4
4𝑥 3 + 1 𝑥3 + 1
=∫ 𝑑𝑥 − 2 ∫ 𝑑𝑥
𝑥 + 𝑥4 𝑥(1 + 𝑥 3 )
𝑑𝑢 𝑑𝑥
=∫ − 2∫
𝑢 𝑥
= 𝑙𝑛 |𝑥 + 𝑥 4 | − 𝑙𝑛 |𝑥 2 | + 𝐶
1 + 𝑥3
= 𝑙𝑛 | |+𝐶 ANS.
𝑥
_____________________________________________________________________________________________________
Example 1.11
Evaluate
𝑤3
∫ 𝑑𝑤
√𝑤 2 + 9
𝑤2 1 𝑢−9
∫ ∙ 𝑤 𝑑𝑤 = ∫ 𝑑𝑢
√𝑤 2 + 9 2 √𝑢
1 9
= ∫ (√𝑢 − ) 𝑑𝑢
2 √𝑢
1 2
= ( 𝑢3/2 − 9 ∙ 2 𝑢1/2 ) + 𝐶
2 3
1
= (𝑤 2 + 9)3/2 − 9 √𝑤 2 + 9 + 𝐶 ANS.
3
We shall encounter this example in the later chapters where we will show how to
evaluate this integral using other methods or techniques.
9
Example 1.12.
Evaluate
𝑑𝑡
∫ 3
(1 + √𝑡)
Solution. To solve this, we need to substitute 𝑢 = 1 + √𝑡 for which 𝑑𝑢 = (1/2√𝑡) 𝑑𝑡. But
since 𝑢 − 1 = √𝑡, 𝑑𝑢 = 1/[2(𝑢 − 1)] 𝑑𝑡 and 2(𝑢 − 1) 𝑑𝑢 = 𝑑𝑡. Hence, the integral
reduces to
𝑑𝑡 𝑢−1
∫ 3 = 2∫ 𝑑𝑢
(1 + √𝑡) 𝑢3
1 1
= 2 ∫ ( 2 − 3 ) 𝑑𝑢
𝑢 𝑢
2 1
=− + 2+𝐶
𝑢 𝑢
2 1
=− + 2 +𝐶 ANS.
1 + √𝑡 (1 + √𝑡)
If you’re asking if there are other substitutions that we can use, the answer is yes.
But we must choose the better substitution or else, the solution may take longer. Consider
letting 𝑢 = √𝑡 instead. This means that 𝑢2 = 𝑡 and 2𝑢 𝑑𝑢 = 𝑑𝑡. The integral becomes
𝑑𝑡 2𝑢
∫ 3 =∫ 𝑑𝑢
(1 + √𝑡) (1 + 𝑢)3
Now, to solve the new integral, we need to use another substitution, say, 𝑣 = 1 + 𝑢 so
that 𝑢 = 𝑣 − 1 and 𝑑𝑣 = 𝑑𝑢. After that, we have
2𝑢 𝑣−1
∫ 3
𝑑𝑢 = 2 ∫ 3 𝑑𝑣
(1 + 𝑢) 𝑣
1 1
= 2 ∫ ( 2 − 3 ) 𝑑𝑣
𝑣 𝑣
1 1
= 2 (− + 2 ) + 𝐶
𝑣 2𝑣
2 1
=− + +𝐶
1 + 𝑢 (1 + 𝑢)2
10
2 1
=− + 2 +𝐶 ANS.
1 + √𝑡 (1 + √𝑡)
Although we got the same result, the solution is obviously longer, requiring two
substitutions instead of just one. The better substitution, therefore, is 𝑢 = 1 + √𝑡
because it clears out 1 in the denominator, making it easier to divide the numerator by
the denominator.
_____________________________________________________________________________________________________
Example 1.13.
Evaluate
∫ √1 − √𝑥 𝑑𝑥
1
Solution. We can let 𝑢 = 1 − √𝑥 so that 𝑑𝑢 = − 2 𝑑𝑥. But since 1 − 𝑢 = √𝑥, then
√𝑥
2(𝑢 − 1) 𝑑𝑢 = 𝑑𝑥. The integral reduces to
∫ √1 − √𝑥 𝑑𝑥 = 2 ∫(𝑢 − 1)√𝑢 𝑑𝑢
= 2 ∫(𝑢3/2 − 𝑢1/2 ) 𝑑𝑢
2 2
= 2 ∙ 𝑢5/2 − 2 ∙ 𝑢3/2 + 𝐶
5 3
4 5/2 4 3/2
= (1 − √𝑥) − (1 − √𝑥) + 𝐶 ANS.
5 3
We can try a different substitution though. By letting
𝑢 = √1 − √𝑥
𝑢 2 = 1 − √𝑥
1
2𝑢 𝑑𝑢 = − 𝑑𝑥
2 √𝑥
1
=− 𝑑𝑥
2(1 − 𝑢2 )
11
𝑑𝑥 = 4𝑢 (𝑢2 − 1) 𝑑𝑢
Substituting gives us
∫ √1 − √𝑥 𝑑𝑥 = ∫ 𝑢 ∙ 4𝑢 (𝑢2 − 1) 𝑑𝑢
= 4 ∫(𝑢4 − 𝑢2 ) 𝑑𝑢
1 1
= 4 ( 𝑢5 − 𝑢3 ) + 𝐶
5 3
4 4 3
5/2
= (1 − √𝑥) − (1 − √𝑥)2 + 𝐶 ANS.
5 3
____________________________________________________________________________________________________
Example 1.14.
Evaluate
1 − √𝑥
∫ 𝑑𝑥
1 + √𝑥
1 − √𝑥 √𝑥 √𝑥 − 𝑥
∫ 𝑑𝑥 ∙ =∫ 𝑑𝑥
1 + √𝑥 √𝑥 √𝑥 (1 + √𝑥)
The reason will become obvious subsequently. If we let 𝑢 = 1 + √𝑥, 𝑑𝑢 = (1/2√𝑥) 𝑑𝑥,
and 𝑥 = (𝑢 − 1)2 . Thus,
√𝑥 − 𝑥 (𝑢 − 1) − (𝑢 − 1)2
∫ 𝑑𝑥 = 2 ∫ 𝑑𝑢
√𝑥 (1 + √𝑥) 𝑢
−𝑢2 + 3𝑢 − 2
= 2∫ 𝑑𝑢
𝑢
2
= 2 ∫ (−𝑢 + 3 − ) 𝑑𝑢
𝑢
1
= 2 (− 𝑢2 + 3𝑢 − 2 𝑙𝑛 |𝑢|) + 𝐶
2
= −𝑢2 + 6𝑢 − 4 𝑙𝑛 |𝑢| + 𝐶
12
2
= −(1 + √𝑥) + 6(1 + √𝑥) − 4 𝑙𝑛 |1 + √𝑥| + 𝐶
1 − √𝑥
∫ 𝑑𝑥 = −𝑥 + 4√𝑥 − 4 𝑙𝑛 |1 + √𝑥| + 𝐶 ANS.
1 + √𝑥
Note: If it does not occur to you to multiply the integral by √𝑥/√𝑥, we can still use the
substitution above and the integral becomes
1 − √𝑥 1 − (𝑢 − 1)
∫ 𝑑𝑥 = ∫ 2(𝑢 − 1) 𝑑𝑢
1 + √𝑥 𝑢
(𝑢 − 1) − (𝑢 − 1)2
= 2∫ 𝑑𝑢
𝑢
−𝑢2 + 3𝑢 − 2
= 2∫ 𝑑𝑢
𝑢
= −𝑢2 + 6𝑢 − 4 𝑙𝑛 |𝑢| + 𝐶
2
= −(1 + √𝑥) + 6(1 + √𝑥) − 4 𝑙𝑛 |1 + √𝑥| + 𝐶
_____________________________________________________________________________________________________
Example 1.15.
Evaluate
1
∫ 𝑑𝑡
𝑡 2 √𝑡 2 − 1
Solution. Integrals of this form are usually evaluated using a technique called integration
by trigonometric substitution, which you will encounter later. In the meantime, we will
attempt to evaluate this integral using ordinary substitution although in a rather
unorthodox fashion.
1 1
∫ 𝑑𝑡 = ∫ 𝑑𝑡
𝑡 2 √𝑡 2 − 1 𝑡 3 √1 − 1/𝑡 2
If we let
13
1 2
𝑢 =1− , 𝑑𝑢 = 𝑑𝑡
𝑡2 𝑡3
Thus,
1 1 1
∫ 𝑑𝑡 = ∫ 𝑑𝑢
𝑡 3 √1 − 1/𝑡 2 2 √𝑢
= √𝑢 + 𝐶
= √1 − 1/𝑡 2 + 𝐶
Simplifying, we have
1 √𝑡 2 − 1
∫ 𝑑𝑡 = +𝐶 ANS.
𝑡 2 √𝑡 2 − 1 𝑡
_____________________________________________________________________________________________________
Example 1.16.
Evaluate
1
∫2 3 𝑑𝑥
√𝑥 + √𝑥
Solution. The choice of 𝑢 in this case is the function that will remove the square root and
6
cube root as well. The proper substitution is 𝑢 = √𝑥 or 𝑥 = 𝑢6 so that 𝑑𝑥 = 6𝑢5 𝑑𝑢.
Thus, the integral reduces to
1 𝑢5
∫2 𝑑𝑥 = 6 ∫ 𝑑𝑢
3
√𝑥 + √𝑥 𝑢3 + 𝑢2
𝑢3
= 6∫ 𝑑𝑢
𝑢+1
𝑡 3 − 3𝑡 2 + 3𝑡 − 1
= 6∫ 𝑑𝑡
𝑡
14
1
= 6 ∫ (𝑡 2 − 3𝑡 + 3 − ) 𝑑𝑡
𝑡
= 2𝑡 3 − 9𝑡 2 + 18𝑡 − 6 𝑙𝑛 |𝑡| + 𝐶
Hence,
1
∫2 3 𝑑𝑥 = 2𝑡 3 − 9𝑡 2 + 18𝑡 − 6 𝑙𝑛 |𝑡| + 𝐶
√ 𝑥 + √ 𝑥
1 3 6 6
∫2 3 𝑑𝑥 = 2√𝑥 − 3 √𝑥 + 6 √𝑥 − 6 𝑙𝑛 | √𝑥 + 1| + 𝐶 ANS.
√𝑥 + √𝑥
_____________________________________________________________________________________________________
Example 1.17.
Evaluate
3
√𝑦 + 1
∫2 𝑑𝑦
√𝑦 + 1
Solution. The appropriate substitution should clear out both square root and cube root.
If we let 𝑦 = 𝑢6 , then 𝑑𝑦 = 6𝑢5 𝑑𝑢 and the integral becomes
3
√𝑦 + 1 𝑢2 + 1 5
∫2 𝑑𝑦 = 6 ∫ 3 ∙ 𝑢 𝑑𝑢
√𝑦 + 1 𝑢 +1
𝑢7 + 𝑢5
= 6∫ 𝑑𝑢
𝑢3 + 1
𝑢7 + 𝑢5 4 2
−𝑢2 + 𝑢
= 𝑢 + 𝑢 − 𝑢 +
𝑢3 + 1 𝑢3 + 1
Hence, the integral further reduces to
3
√𝑦 + 1 4
−𝑢2 + 𝑢 2
∫2 𝑑𝑦 = 6 ∫ (𝑢 + 𝑢 − 𝑢 + 3 ) 𝑑𝑢
√𝑦 + 1 𝑢 +1
15
4 2
𝑢2 𝑢
= 6 ∫(𝑢 + 𝑢 − 𝑢) 𝑑𝑢 − 6 ∫ 3 𝑑𝑢 + 6 ∫ 3 𝑑𝑢
𝑢 +1 𝑢 +1
Solving the first integral on the right is straightforward. We just use the power formula
and we are done. To solve the second, we let 𝑡 = 𝑢3 + 1 so that 𝑑𝑡 = 3𝑢2 𝑑𝑢. Thus,
𝑢2 𝑑𝑡
−6 ∫ 3 𝑑𝑢 = −2 ∫
𝑢 +1 𝑡
= −2 𝑙𝑛 |𝑡| + 𝐶
= −2 𝑙𝑛 |𝑢3 + 1| + 𝐶
𝑢 𝑢−2
6∫ 𝑑𝑢 = 6 ∫ 𝑑𝑢
𝑢3 + 1 1 + 𝑢−3
𝑢−2 𝑑𝑡
6∫ −3
𝑑𝑢 = −2 ∫
1+𝑢 𝑡
= −2 𝑙𝑛 |𝑡| + 𝐶
= −2 𝑙𝑛 |1 + 𝑢−3 | + 𝐶
𝑢3 + 1
= −2 𝑙𝑛 | 3 | + 𝐶
𝑢
= −2 𝑙𝑛 |𝑢3 + 1| + 2 𝑙𝑛 |𝑢3 | + 𝐶
𝑢2 𝑢
6 ∫(𝑢4 + 𝑢2 − 𝑢) 𝑑𝑢 − 6 ∫ 3
𝑑𝑢 + 6 ∫ 3 𝑑𝑢
𝑢 +1 𝑢 +1
1 1 1
= 6 ( 𝑢5 + 𝑢3 − 𝑢2 ) − 2 𝑙𝑛 |𝑢3 + 1| − 2 𝑙𝑛 |𝑢3 + 1| + 2 𝑙𝑛 |𝑢3 | + 𝐶
5 3 2
6 5
= 𝑢 + 2𝑢3 − 3𝑢2 − 4 𝑙𝑛 |𝑢3 + 1| + 2 𝑙𝑛 |𝑢3 | + 𝐶
5
16
Example 1.18.
Evaluate
1
∫ 𝑑𝑥
𝑥5 + 𝑥
1 𝑥 −5
∫ 𝑑𝑥 = ∫ 𝑑𝑥
𝑥5 + 𝑥 1 + 𝑥 −4
𝑥 −5 1 𝑑𝑢
∫ −4
𝑑𝑥 = − ∫
1+𝑥 4 𝑢
1
=− 𝑙𝑛 |𝑢| + 𝐶
4
1
=− 𝑙𝑛 |1 + 𝑥 −4 | + 𝐶
4
1 𝑥4
= 𝑙𝑛 | 4 |+𝐶 ANS.
4 𝑥 +1
_____________________________________________________________________________________________________
17
Try these.
𝑥 2 − 5𝑥 + 5
1. ∫ 2 𝑑𝑥
𝑥 − 3𝑥 + 2
2. ∫(𝑥 + 1)√𝑥 − 3 𝑑𝑥
4. ∫ 3𝑥(2𝑥 − 5)7 𝑑𝑥
5. ∫ 𝑥 3 √𝑥 2 + 1 𝑑𝑥
1
∫ 𝑑𝑦
6. √𝑦(1 + √𝑦)
√𝑡
7. ∫ 𝑑𝑡
1 + √𝑡
1
8. ∫ 𝑑𝑟
1 + √𝑟
1
∫ 𝑑𝑦
9. 𝑦 + 3√𝑦
(𝑥 − 1)2
10. ∫ 𝑑𝑥
(𝑥 + 1)2
18
2. Integrals of Trigonometric Functions
𝑑
𝑠𝑖𝑛 𝑎𝑥 = 𝑎 𝑐𝑜𝑠 𝑎𝑥
𝑑𝑥
𝑑
𝑐𝑜𝑠 𝑎𝑥 = −𝑎 𝑠𝑖𝑛 𝑎𝑥
𝑑𝑥
𝑑
𝑡𝑎𝑛 𝑎𝑥 = 𝑎 𝑠𝑒𝑐 2 𝑎𝑥
𝑑𝑥
𝑑
𝑐𝑜𝑡 𝑎𝑥 = −𝑎 𝑐𝑠𝑐 2 𝑎𝑥
𝑑𝑥
𝑑
𝑠𝑒𝑐 𝑎𝑥 = 𝑎 𝑠𝑒𝑐 𝑎𝑥 ∙ 𝑡𝑎𝑛 𝑎𝑥
𝑑𝑥
𝑑
𝑐𝑠𝑐 𝑎𝑥 = −𝑎 𝑐𝑠𝑐 𝑎𝑥 ∙ 𝑐𝑜𝑡 𝑎𝑥
𝑑𝑥
From these, we derive the following relations:
1
∫ 𝑠𝑖𝑛 𝑎𝑥 𝑑𝑥 = − 𝑐𝑜𝑠 𝑎𝑥 + 𝐶 2.1
𝑎
1
∫ 𝑐𝑜𝑠 𝑎𝑥 𝑑𝑥 = 𝑠𝑖𝑛 𝑎𝑥 + 𝐶 2.2
𝑎
19
1
∫ 𝑠𝑒𝑐 2 𝑎𝑥 𝑑𝑥 = 𝑡𝑎𝑛 𝑎𝑥 + 𝐶 2.3
𝑎
1
∫ 𝑐𝑠𝑐 2 𝑎𝑥 𝑑𝑥 = − 𝑐𝑜𝑡 𝑎𝑥 + 𝐶 2.4
𝑎
1
∫ 𝑠𝑒𝑐 𝑎𝑥 ∙ 𝑡𝑎𝑛 𝑎𝑥 𝑑𝑥 = 𝑠𝑒𝑐 𝑎𝑥 + 𝐶 2.5
𝑎
1
∫ 𝑐𝑠𝑐 𝑎𝑥 ∙ 𝑐𝑜𝑡 𝑎𝑥 𝑑𝑥 = − 𝑐𝑠𝑐 𝑎𝑥 + 𝐶 2.6
𝑎
𝑠𝑖𝑛 𝑎𝑥
∫ 𝑡𝑎𝑛 𝑎𝑥 𝑑𝑥 = ∫ 𝑑𝑥
𝑐𝑜𝑠 𝑎𝑥
Note that the derivative of 𝑐𝑜𝑠 𝑎𝑥 is −𝑎 𝑠𝑖𝑛 𝑎𝑥 𝑑𝑥. Hence, using the substitution rule
where 𝑢 = 𝑐𝑜𝑠 𝑎𝑥,
1 𝑑𝑢 1
∫ 𝑡𝑎𝑛 𝑎𝑥 𝑑𝑥 = − ∫ = − 𝑙𝑛 |𝑢|
𝑎 𝑢 𝑎
1
=− 𝑙𝑛 |𝑐𝑜𝑠 𝑎𝑥| + 𝐶
𝑎
Following the same procedure, the integral of the cotangent of an angle is
𝑐𝑜𝑠 𝑎𝑥
∫ 𝑐𝑜𝑡 𝑎𝑥 𝑑𝑥 = ∫ 𝑑𝑥
𝑠𝑖𝑛 𝑎𝑥
1 𝑑𝑢
= ∫
𝑎 𝑢
1
= 𝑙𝑛 |𝑠𝑖𝑛 𝑎𝑥| + 𝐶
𝑎
In deriving the integral of the secant of an angle, we multiply the secant of an angle
by
20
1 𝑑𝑢 1
∫ 𝑠𝑒𝑐 𝑎𝑥 𝑑𝑥 = ∫ = 𝑙𝑛 |𝑢|
𝑎 𝑢 𝑎
1
= 𝑙𝑛 |𝑠𝑒𝑐 𝑎𝑥 + 𝑡𝑎𝑛 𝑎𝑥| + 𝐶
𝑎
To derive the integral of the cosecant of an angle, we multiply the cosecant of an
angle by
1 𝑑𝑢 1
∫ 𝑐𝑠𝑐 𝑎𝑥 𝑑𝑥 = ∫ = 𝑙𝑛 |𝑢|
𝑎 𝑢 𝑎
1
= 𝑙𝑛 |𝑐𝑠𝑐 𝑎𝑥 − 𝑐𝑜𝑡 𝑎𝑥| + 𝐶
𝑎
Summarizing,
1
∫ 𝑡𝑎𝑛 𝑎𝑥 𝑑𝑥 = − 𝑙𝑛 |𝑐𝑜𝑠 𝑎𝑥| + 𝐶 2.7
𝑎
1
∫ 𝑐𝑜𝑡 𝑎𝑥 𝑑𝑥 = 𝑙𝑛 |𝑠𝑖𝑛 𝑎𝑥| + 𝐶 2.8
𝑎
1
∫ 𝑠𝑒𝑐 𝑎𝑥 𝑑𝑥 = 𝑙𝑛 |𝑠𝑒𝑐 𝑎𝑥 + 𝑡𝑎𝑛 𝑎𝑥| + 𝐶 2.9
𝑎
1
∫ 𝑐𝑠𝑐 𝑎𝑥 𝑑𝑥 = 𝑙𝑛 |𝑐𝑠𝑐 𝑎𝑥 − 𝑐𝑜𝑡 𝑎𝑥| + 𝐶 2.10
𝑎
1 1 𝜋
∫ 𝑠𝑒𝑐 𝑎𝑥 𝑑𝑥 = 𝑙𝑛 |𝑡𝑎𝑛 (2 𝑎𝑥 + 4 )| + 𝐶
𝑎
1 1
∫ 𝑐𝑠𝑐 𝑎𝑥 𝑑𝑥 = 𝑙𝑛 |𝑡𝑎𝑛 2 𝑎𝑥| + 𝐶
𝑎
1
=− 𝑙𝑛 |𝑐𝑠𝑐 𝑎𝑥 + 𝑐𝑜𝑡 𝑎𝑥| + 𝐶
𝑎
_____________________________________________________________________________________________________
21
Example 2.1.
Evaluate
∫ 𝑠𝑖𝑛5 𝑥 𝑐𝑜𝑠 𝑥 𝑑𝑥
1 6
∫ 𝑠𝑖𝑛5 𝑥 𝑐𝑜𝑠 𝑥 𝑑𝑥 = ∫ 𝑢5 𝑑𝑢 = 𝑢 +𝐶
6
1
= 𝑠𝑖𝑛6 𝑥 + 𝐶 ANS.
6
In many instances, there is a need to transform the integrand into another form
using trigonometric identities to be able to evaluate the integral. The following examples
illustrate this.
_____________________________________________________________________________________________________
Example 2.2.
Evaluate
∫ 𝑐𝑜𝑠 5 𝑦 𝑑𝑦
Hence,
22
2 1
= 𝑢 − 𝑢3 + 𝑢5 + 𝐶
3 5
2 1
= 𝑠𝑖𝑛 𝑦 − 𝑠𝑖𝑛3 𝑦 + 𝑠𝑖𝑛5 𝑦 + 𝐶 ANS.
3 5
_____________________________________________________________________________________________________
Example 2.3.
Evaluate
𝑠𝑖𝑛 𝑥
∫ 𝑑𝑥
1 + 𝑠𝑖𝑛 𝑥
𝑠𝑖𝑛 𝑥
∴∫ 𝑑𝑥 = ∫(𝑠𝑒𝑐 𝑥 𝑡𝑎𝑛 𝑥 − 𝑠𝑒𝑐 2 𝑥 + 1) 𝑑𝑥
1 + 𝑠𝑖𝑛 𝑥
Example 2.4.
Evaluate
1 − 𝑐𝑜𝑠 𝑥
∫ 𝑑𝑥
1 + 𝑐𝑜𝑠 𝑥
Solution. To evaluate the integral, we will use the identity 𝑐𝑜𝑠 2𝑥 = 𝑐𝑜𝑠 2 𝑥 − 𝑠𝑖𝑛2 𝑥. The
integrand becomes
23
1 − 𝑐𝑜𝑠 𝑥 1 − (𝑐𝑜𝑠 2 12 𝑥 − 𝑠𝑖𝑛2 12 𝑥)
=
1 + 𝑐𝑜𝑠 𝑥 1 + (𝑐𝑜𝑠 2 12 𝑥 − 𝑠𝑖𝑛2 12 𝑥)
2𝑠𝑖𝑛2 12 𝑥 1 1
= = 𝑡𝑎𝑛2 2 𝑥 = 𝑠𝑒𝑐 2 2 𝑥 − 1
2𝑐𝑜𝑠 2 12 𝑥
Hence,
1 − 𝑐𝑜𝑠 𝑥 1
∫ 𝑑𝑥 = ∫ (𝑠𝑒𝑐 2 2 𝑥 − 1) 𝑑𝑥
1 + 𝑐𝑜𝑠 𝑥
1
= 2 𝑡𝑎𝑛 2 𝑥 − 𝑥 + 𝐶 ANS.
Note that we can obtain the same transformation using the half-angle identities for sine
and cosine, or
𝑥 1 − 𝑐𝑜𝑠 𝑥 𝑥 1 + 𝑐𝑜𝑠 𝑥
𝑠𝑖𝑛 =√ , 𝑐𝑜𝑠 =√
2 2 2 2
𝑥 1 − 𝑐𝑜𝑠 𝑥
𝑡𝑎𝑛2 =
2 1 + 𝑐𝑜𝑠 𝑥
Another way of evaluating the integral is by rationalizing the denominator by
multiplying the integrand by
24
= −2 𝑐𝑜𝑡 𝑥 + 2 𝑐𝑠𝑐 𝑥 − 𝑥 + 𝐶 ANS.
Comparing both answers, it appears that they are different, but technically, they are not.
It is left as an exercise to show that they are, in fact, equivalent.
_____________________________________________________________________________________________________
Example 2.5.
Evaluate
∫ √1 − 𝑠𝑖𝑛 𝑦 𝑑𝑦
Solution. The presence of the radical complicates the integrand so we will try to remove
it. Using the identity 𝑠𝑖𝑛 2𝑦 = 2 𝑠𝑖𝑛 𝑦 𝑐𝑜𝑠 𝑦, the term inside the radical reduces to
1 1
1 − 𝑠𝑖𝑛 𝑦 = 1 − 2 𝑠𝑖𝑛 2𝑦 𝑐𝑜𝑠 2𝑦
1 1 1 1
= (𝑐𝑜𝑠 2 2𝑦 + 𝑠𝑖𝑛2 2𝑦) − 2 𝑠𝑖𝑛 2𝑦 𝑐𝑜𝑠 2𝑦
1 1 2
= (𝑐𝑜𝑠 2𝑦 − 𝑠𝑖𝑛 2𝑦)
1 1
√1 − 𝑠𝑖𝑛 𝑦 = 𝑐𝑜𝑠 2𝑦 − 𝑠𝑖𝑛 2𝑦
1 1
∫ √1 − 𝑠𝑖𝑛 𝑦 𝑑𝑦 = ∫ (𝑐𝑜𝑠 2𝑦 − 𝑠𝑖𝑛 2𝑦) 𝑑𝑦
1 1
= 2𝑠𝑖𝑛 2𝑦 + 2𝑐𝑜𝑠 2𝑦 + 𝐶 ANS.
𝑐𝑜𝑠 𝑦
=∫ 𝑑𝑦
√1 + 𝑠𝑖𝑛 𝑦
25
It should be obvious by now why we did so. Note that if we let 𝑢 = 1 + 𝑠𝑖𝑛 𝑦, 𝑑𝑢 =
𝑐𝑜𝑠 𝑦 𝑑𝑦. The integral then becomes
𝑑𝑢
∫ √1 − 𝑠𝑖𝑛 𝑦 𝑑𝑦 = ∫
√𝑢
= 2√𝑢 + 𝐶
Example 2.6.
Evaluate
1
∫ 𝑑𝑥
√1 + 𝑠𝑖𝑛 𝑥
Solution. The integrand is similar to the integrand in the previous example. We note that
𝑠𝑖𝑛 𝑥 is positive and the integrand in the previous example is now in the denominator.
This complicates the problem, so our solution needs to reduce the integrand into
something that can be readily integrated. Using the solution to the previous example, it
follows that √1 + 𝑠𝑖𝑛 𝑥 = 𝑐𝑜𝑠 12𝑥 + 𝑠𝑖𝑛 12𝑥. Hence,
1 1
∫ 𝑑𝑥 = ∫ 𝑑𝑥
√1 + 𝑠𝑖𝑛 𝑥 𝑐𝑜𝑠 12𝑥 + 𝑠𝑖𝑛 12𝑥
We note that the numerator on the right side can be written as the sine of the sum of two
angles and this explains our action. Hence,
1 1 1 𝜋 1 𝜋
𝑐𝑜𝑠 2 𝑥 + 𝑠𝑖𝑛 2 𝑥 = √2 (𝑐𝑜𝑠 2 𝑥 𝑠𝑖𝑛 4 + 𝑠𝑖𝑛 2 𝑥 𝑐𝑜𝑠 4 )
26
1 𝜋
= √2 𝑠𝑖𝑛 (2 𝑥 + 4 )
Finally,
1 1
∫ 𝑑𝑥 = ∫ 𝑑𝑥
√1 + 𝑠𝑖𝑛 𝑥 𝑐𝑜𝑠 𝑥 + 𝑠𝑖𝑛 12 𝑥
1
2
1 1 1 𝜋
=∫ 𝑑𝑥 = ∫ 𝑐𝑠𝑐 (2 𝑥 + 4 ) 𝑑𝑥
√2 𝑠𝑖𝑛 (12 𝑥+ ) 𝜋
4 √2
2 1 1 𝜋
= 𝑙𝑛 |𝑡𝑎𝑛 2 (2𝑥 + 4 )| + 𝐶
√2
1 𝜋
= √2 𝑙𝑛 |𝑡𝑎𝑛 (4 𝑥 + 8 )| + 𝐶 ANS.
𝜋
Alternatively, we can express 𝑠𝑖𝑛 𝑥 = −𝑐𝑜𝑠 (𝑥 + 2 ) because the cosine of an angle
in the second quadrant is negative. Using the identity for half-angles, our integral reduces
to
1 1
∫ 𝑑𝑥 = ∫ 𝑑𝑥
√1 + 𝑠𝑖𝑛 𝑥 𝜋
√1 − 𝑐𝑜𝑠 (𝑥 + 2
)
1 1
= ∫ 𝑑𝑥
√2 𝑠𝑖𝑛 (12𝑥 + 𝜋4)
1 1 𝜋
= ∫ 𝑐𝑠𝑐 (2𝑥 + 4 ) 𝑑𝑥
√2
2
= 𝑙𝑛|𝑡𝑎𝑛 12(12 𝑥+𝜋4)| + 𝐶
√2
1 𝜋
= √2 𝑙𝑛 |𝑡𝑎𝑛 (4 𝑥 + 8 )| + 𝐶 ANS.
_____________________________________________________________________________________________________
Example 2.7.
Evaluate
∫ 𝑠𝑖𝑛 5𝑥 𝑐𝑜𝑠 𝑥 𝑑𝑥
27
Solution. To evaluate the integrand, we recall that
1
∴ 𝑠𝑖𝑛 𝐴 𝑐𝑜𝑠 𝐵 = [𝑠𝑖𝑛(𝐴 + 𝐵) + 𝑠𝑖𝑛(𝐴 − 𝐵)]
2
With 𝐴 = 5𝑥 and 𝐵 = 𝑥,
1
∴ ∫ 𝑠𝑖𝑛 5𝑥 𝑐𝑜𝑠 𝑥 𝑑𝑥 = ∫ [𝑠𝑖𝑛(5𝑥 + 𝑥) + 𝑠𝑖𝑛(5𝑥 − 𝑥)] 𝑑𝑥
2
1 1 1 1 1
= ∫(𝑠𝑖𝑛 6𝑥 + 𝑠𝑖𝑛 4𝑥) 𝑑𝑥 = ∙ (−𝑐𝑜𝑠 6𝑥) + ∙ (−𝑐𝑜𝑠 4𝑥)
2 2 6 2 4
1 1
=− 𝑐𝑜𝑠 6𝑥 − 𝑐𝑜𝑠 4𝑥 + 𝐶 ANS.
12 8
_____________________________________________________________________________________________________
Example 2.8.
Evaluate
𝑑𝑥
∫
2 𝑐𝑜𝑠 𝑥 + 3 𝑠𝑖𝑛 𝑥
Solution. To evaluate integrand of the form 1⁄(𝑎 𝑐𝑜𝑠 𝑥 + 𝑏 𝑠𝑖𝑛 𝑥), we use a different
substitution rule. If we let 𝑟 2 = 𝑎2 + 𝑏 2 , then 𝑎 = 𝑟 𝑠𝑖𝑛 𝛼 and 𝑏 = 𝑟 𝑐𝑜𝑠 𝛼, where 𝛼 =
𝑡𝑎𝑛−1 (𝑎⁄𝑏). With these substitutions to the above integrand, we identify 𝑎 = 2 and 𝑏 =
3. Hence,
𝑟 2 = 𝑎2 + 𝑏 2 = 22 +32 ; 𝑟 = √4 + 9 = √13
2 2
𝑟 𝑠𝑖𝑛 𝛼 = 2; 𝑟 𝑐𝑜𝑠 𝛼 = 3; 𝑡𝑎𝑛 𝛼 = 3; 𝛼 = 𝑡𝑎𝑛−1 3
𝑑𝑥 𝑑𝑥
∫ =∫
2 𝑐𝑜𝑠 𝑥 + 3 𝑠𝑖𝑛 𝑥 𝑟 𝑠𝑖𝑛 𝛼 𝑐𝑜𝑠 𝑥 + 𝑟 𝑐𝑜𝑠 𝛼 𝑠𝑖𝑛 𝑥
28
1 𝑑𝑥
= ∫
𝑟 𝑠𝑖𝑛 𝛼 𝑐𝑜𝑠 𝑥 + 𝑐𝑜𝑠 𝛼 𝑠𝑖𝑛 𝑥
1 𝑑𝑥
= ∫
𝑟 𝑠𝑖𝑛 (𝛼 + 𝑥)
which we obtain by using the identity: 𝑠𝑖𝑛 (𝐴 + 𝐵) = 𝑠𝑖𝑛 𝐴 𝑐𝑜𝑠 𝐵 + 𝑐𝑜𝑠 𝐴 𝑠𝑖𝑛 𝐵. The
integrand further reduces to
1 1 1
∫ 𝑐𝑠𝑐 (𝛼 + 𝑥) 𝑑𝑥 = 𝑙𝑛 |𝑡𝑎𝑛 2 (𝑥 + 𝛼)| + 𝐶
𝑟 𝑟
1 1 2
= 𝑙𝑛 |𝑡𝑎𝑛 2 (𝑥 + 𝑡𝑎𝑛−1 3)| + 𝐶 ANS.
√13
_____________________________________________________________________________________________________
Example 2.9.
Evaluate
3 𝑐𝑜𝑠 𝑥 − 𝑠𝑖𝑛 𝑥
∫ 𝑑𝑥
2 𝑐𝑜𝑠 𝑥 + 3 𝑠𝑖𝑛 𝑥
Solution. This integral needs a totally different technique. The integrand is a fraction
whose numerator 𝑁 = 3 𝑐𝑜𝑠 𝑥 − 𝑠𝑖𝑛 𝑥 and whose denominator 𝐷 = 2 𝑐𝑜𝑠 𝑥 + 3 𝑠𝑖𝑛 𝑥
We desire a fraction whose numerator 𝑁 = 𝐴𝐷 + 𝐵𝐷′ where 𝐴 and 𝐵 are unknown
coefficients to make the integrand readily integrable. Thus,
𝑁 𝐴𝐷 + 𝐵𝐷′
∫ 𝑑𝑡 = ∫ 𝑑𝑡
𝐷 𝐷
𝐷′
= 𝐴 ∫ 𝑑𝑡 + 𝐵 ∫ 𝑑𝑡
𝐷
= 𝐴𝑡 + 𝐵 𝑙𝑛 𝐷 + 𝐶
𝑁 = 𝐴𝐷 + 𝐵𝐷′
29
Thus, 2𝐴 + 3𝐵 = 3 and 3𝐴 − 2𝐵 = −1. This means that 𝐴 = 3/13, 𝐵 = 11/13 and
3 𝑐𝑜𝑠 𝑥 − 𝑠𝑖𝑛 𝑥 3 11
∫ 𝑑𝑥 = 𝑥+ 𝑙𝑛 |2 𝑐𝑜𝑠 𝑥 + 3 𝑠𝑖𝑛 𝑥| + 𝐶 ANS.
2 𝑐𝑜𝑠 𝑥 + 3 𝑠𝑖𝑛 𝑥 13 13
_____________________________________________________________________________________________________
Example 2.10.
Evaluate
𝑐𝑜𝑠 𝑡
∫ 𝑑𝑡
𝑐𝑜𝑠 𝑡 + 𝑠𝑖𝑛 𝑡
1 1 1
𝑐𝑜𝑠 2 𝑡 − 𝑐𝑜𝑠 𝑡 𝑠𝑖𝑛 𝑡 2 + 2 𝑐𝑜𝑠 2𝑡 2 𝑠𝑖𝑛 2𝑡
= −
𝑐𝑜𝑠 2 𝑡 − 𝑠𝑖𝑛2 𝑡 𝑐𝑜𝑠 2𝑡 𝑐𝑜𝑠 2𝑡
1 1 1
= 𝑠𝑒𝑐 2𝑡 + − 𝑡𝑎𝑛 2𝑡
2 2 2
Hence, the integral transforms into
𝑐𝑜𝑠 𝑡 1 1 1
∫ 𝑑𝑡 = ∫ ( + 𝑠𝑒𝑐 2𝑡 − 𝑡𝑎𝑛 2𝑡) 𝑑𝑡
𝑐𝑜𝑠 𝑡 + 𝑠𝑖𝑛 𝑡 2 2 2
1 1 1
= 𝑡 + 𝑙𝑛 |𝑠𝑒𝑐 2𝑡 + 𝑡𝑎𝑛 2𝑡| + 𝑙𝑛 |𝑐𝑜𝑠 2𝑡| + 𝐶
2 4 4
1 1
= 𝑡 + 𝑙𝑛 |1 + 𝑠𝑖𝑛 2𝑡| + 𝐶 ANS.
2 4
1 1
𝑐𝑜𝑠 𝑡 = (𝑐𝑜𝑠 𝑡 + 𝑠𝑖𝑛 𝑡) + (𝑐𝑜𝑠 𝑡 − 𝑠𝑖𝑛 𝑡)
2 2
The first term is easy to integrate. For the second term, we note that the derivative of the
numerator equals the denominator so this is a case of ∫ 𝑑𝑢/𝑢. Finally,
𝑐𝑜𝑠 𝑡 1 1
∫ 𝑑𝑡 = 𝑡 + 𝑙𝑛 |𝑐𝑜𝑠 𝑡 + 𝑠𝑖𝑛 𝑡| + 𝐶 ANS.
𝑐𝑜𝑠 𝑡 + 𝑠𝑖𝑛 𝑡 2 2
Example 2.11.
Evaluate
∫ 𝑐𝑜𝑠 4 𝑥 𝑑𝑥
Solution. To solve this problem, we recall the identity for the cosine of twice the angle in
our attempt to remove the exponent. The integral reduces to
1 1 1 1
= ∫ ( + 𝑐𝑜𝑠 2𝑥) ( + 𝑐𝑜𝑠 2𝑥) 𝑑𝑥
2 2 2 2
1
= ∫(1 + 2 𝑐𝑜𝑠 2𝑥 + 𝑐𝑜𝑠 2 2𝑥) 𝑑𝑥
4
1 1 1
= ∫ (1 + 2 𝑐𝑜𝑠 2𝑥 + ( + 𝑐𝑜𝑠 4𝑥)) 𝑑𝑥
4 2 2
1 3 1
= ∫ ( + 2 𝑐𝑜𝑠 2𝑥 + 𝑐𝑜𝑠 4𝑥) 𝑑𝑥
4 2 2
3 1 1
= 𝑥 + 𝑠𝑖𝑛 2𝑥 + 𝑠𝑖𝑛 4𝑥 + 𝐶 ANS.
8 4 32
_____________________________________________________________________________________________________
31
Example 2.12.
Evaluate
1 + 𝑠𝑖𝑛 𝑥
∫ 𝑑𝑥
1 + 𝑐𝑜𝑠 𝑥
1 + 𝑠𝑖𝑛 𝑥 1 𝑠𝑖𝑛 𝑥
∫ 𝑑𝑥 = ∫ 𝑑𝑥 + ∫ 𝑑𝑥
1 + 𝑐𝑜𝑠 𝑥 1 + 𝑐𝑜𝑠 𝑥 1 + 𝑐𝑜𝑠 𝑥
To solve the first integral on the right, we can multiply the integrand as follows:
1 1 − 𝑐𝑜𝑠 𝑥 1 − 𝑐𝑜𝑠 𝑥
∫ ∙ 𝑑𝑥 = ∫ 𝑑𝑥
1 + 𝑐𝑜𝑠 𝑥 1 − 𝑐𝑜𝑠 𝑥 1 − 𝑐𝑜𝑠 2 𝑥
1 − 𝑐𝑜𝑠 𝑥
=∫ 𝑑𝑥
𝑠𝑖𝑛2 𝑥
= −𝑐𝑜𝑡 𝑥 + 𝑐𝑠𝑐 𝑥 + 𝐶
To solve the second integral on the right, we note that if 𝑢 = 1 + 𝑐𝑜𝑠 𝑥, 𝑑𝑢 = −𝑠𝑖𝑛 𝑥 𝑑𝑥.
Thus,
𝑠𝑖𝑛 𝑥 𝑑𝑢
∫ 𝑑𝑥 = − ∫
1 + 𝑐𝑜𝑠 𝑥 𝑢
= −𝑙𝑛 |𝑢| + 𝐶
= −𝑙𝑛 |1 + 𝑐𝑜𝑠 𝑥| + 𝐶
Finally,
1 + 𝑠𝑖𝑛 𝑥
∫ 𝑑𝑥 = −𝑐𝑜𝑡 𝑥 + 𝑐𝑠𝑐 𝑥 − 𝑙𝑛 |1 + 𝑐𝑜𝑠 𝑥| + 𝐶 ANS.
1 + 𝑐𝑜𝑠 𝑥
32
(𝑐𝑜𝑠 2 12 𝑥 + 𝑠𝑖𝑛2 12 𝑥) + 2 𝑠𝑖𝑛 12 𝑥 𝑐𝑜𝑠 12 𝑥
=∫ 𝑑𝑥
(𝑐𝑜𝑠 2 12 𝑥 + 𝑠𝑖𝑛2 12 𝑥) + (𝑐𝑜𝑠 2 12 𝑥 − 𝑠𝑖𝑛2 12 𝑥)
2
(𝑐𝑜𝑠 12 𝑥 + 𝑠𝑖𝑛 12 𝑥)
=∫ 𝑑𝑥
2 𝑐𝑜𝑠 2 12 𝑥
2
1 𝑐𝑜𝑠 12 𝑥 + 𝑠𝑖𝑛 12 𝑥
= ∫( ) 𝑑𝑥
2 𝑐𝑜𝑠 12 𝑥
1 1 2
= ∫ (1 + 𝑡𝑎𝑛 2 𝑥) 𝑑𝑥
2
1 1 1
= ∫ (1 + 2 𝑡𝑎𝑛 2 𝑥 + 𝑡𝑎𝑛2 2 𝑥) 𝑑𝑥
2
1 1 1
= ∫ (𝑠𝑒𝑐 2 2 𝑥 + 2 𝑡𝑎𝑛 2 𝑥) 𝑑𝑥
2
1 1
= 𝑡𝑎𝑛 2 𝑥 + 2 𝑙𝑛 |𝑠𝑒𝑐 2 𝑥| + 𝐶 ANS.
Example 2.13.
Evaluate
∫ 𝑡𝑎𝑛3 𝑦 𝑑𝑦
= ∫(𝑠𝑒𝑐 2 𝑦 − 1) 𝑡𝑎𝑛 𝑦 𝑑𝑦
We can now proceed to break up the integrand into two and directly evaluate using
standard integrals. However, we choose a different approach by letting 𝑢 = 𝑠𝑒𝑐 𝑦. This
means that 𝑑𝑢 = 𝑠𝑒𝑐 𝑦 𝑡𝑎𝑛 𝑦 𝑑𝑦 or 𝑡𝑎𝑛 𝑦 𝑑𝑦 = 𝑑𝑢/ 𝑠𝑒𝑐 𝑥 = 𝑑𝑢/𝑢. Hence,
𝑑𝑢
∫(𝑠𝑒𝑐 2 𝑦 − 1) 𝑡𝑎𝑛 𝑦 𝑑𝑦 = ∫(𝑢2 − 1)
𝑢
33
1
= ∫ (𝑢 − ) 𝑑𝑢
𝑢
1 2
= 𝑢 − 𝑙𝑛 |𝑢| + 𝐶
2
1
= 𝑠𝑒𝑐 2 𝑦 − 𝑙𝑛 |𝑠𝑒𝑐 𝑦| + 𝐶 ANS.
2
_____________________________________________________________________________________________________
Example 2.14.
Evaluate
∫ 𝑐𝑜𝑡 4 𝑥 𝑑𝑥
1
∫ 𝑐𝑠𝑐 2 𝑥 𝑐𝑜𝑡 2 𝑥 𝑑𝑥 = − ∫ 𝑢2 𝑑𝑢 = − 𝑐𝑜𝑡 3 𝑥 + 𝐶
3
Finally,
1
∫ 𝑐𝑜𝑡 2 𝑥 𝑐𝑜𝑡 2 𝑥 𝑑𝑥 = − 𝑐𝑜𝑡 3 𝑥 + 𝑐𝑜𝑡 𝑥 + 𝑥 + 𝐶 ANS.
3
_____________________________________________________________________________________________________
34
Example 2.15.
Evaluate
∫ 𝑠𝑒𝑐 5 𝑥 ∙ 𝑡𝑎𝑛3 𝑥 𝑑𝑥
Note that if 𝑢 = 𝑠𝑒𝑐 𝑥, 𝑑𝑢 = 𝑠𝑒𝑐 𝑥 𝑡𝑎𝑛 𝑥 𝑑𝑥. Hence, the integral further reduces to
1 7 1 5
∫(𝑢6 − 𝑢4 ) 𝑑𝑢 = 𝑢 − 𝑢 +𝐶
7 5
Finally,
1 1
∫ 𝑠𝑒𝑐 5 𝑥 ∙ 𝑡𝑎𝑛3 𝑥 𝑑𝑥 = 𝑠𝑒𝑐 7 𝑥 − 𝑠𝑒𝑐 5 𝑥 + 𝐶 ANS.
7 5
_____________________________________________________________________________________________________
Example 2.16.
Evaluate
∫ 𝑠𝑒𝑐 4 𝑥 ∙ 𝑡𝑎𝑛2 𝑥 𝑑𝑥
35
= ∫(𝑡𝑎𝑛2 𝑥 + 𝑡𝑎𝑛4 𝑥) ∙ 𝑠𝑒𝑐 2 𝑥 𝑑𝑥
Note that if 𝑢 = 𝑡𝑎𝑛 𝑥, 𝑑𝑢 = 𝑠𝑒𝑐 2 𝑥 𝑑𝑥. Hence, the integral further reduces to
1 1
∫(𝑢2 + 𝑢4 ) 𝑑𝑢 = 𝑢3 + 𝑢5 + 𝐶
3 5
Finally,
1 1
∫ 𝑠𝑒𝑐 4 𝑥 ∙ 𝑡𝑎𝑛2 𝑥 𝑑𝑥 = 𝑡𝑎𝑛3 𝑥 + 𝑡𝑎𝑛5 𝑥 + 𝐶 ANS.
3 5
_____________________________________________________________________________________________________
Example 2.17.
Evaluate
∫ 𝑠𝑒𝑐 3 𝑥 𝑑𝑥
Solution. Usually, we solve this integral using different techniques which we shall learn
later. But we shall attempt an unorthodox approach here. We begin by rewriting the
integral as
1
∫ 𝑠𝑒𝑐 3 𝑥 𝑑𝑥 = ∫(𝑠𝑒𝑐 3 𝑥 + 𝑠𝑒𝑐 3 𝑥) 𝑑𝑥
2
1
∫ 𝑠𝑒𝑐 3 𝑥 𝑑𝑥 = ∫(𝑠𝑒𝑐 3 𝑥 + 𝑠𝑒𝑐 3 𝑥 − 𝑠𝑒𝑐 𝑥 + 𝑠𝑒𝑐 𝑥) 𝑑𝑥
2
1
∫ 𝑠𝑒𝑐 3 𝑥 𝑑𝑥 = ∫[𝑠𝑒𝑐 2 𝑥 𝑠𝑒𝑐 𝑥 + 𝑠𝑒𝑐 𝑥 (𝑠𝑒𝑐 2 𝑥 − 1) + 𝑠𝑒𝑐 𝑥] 𝑑𝑥
2
1
= ∫[𝑠𝑒𝑐 2 𝑥 𝑠𝑒𝑐 𝑥 + 𝑠𝑒𝑐 𝑥 𝑡𝑎𝑛2 𝑥 + 𝑠𝑒𝑐 𝑥] 𝑑𝑥
2
1
= ∫(𝑠𝑒𝑐 2 𝑥 𝑠𝑒𝑐 𝑥 + 𝑡𝑎𝑛 𝑥 𝑠𝑒𝑐 𝑥 𝑡𝑎𝑛 𝑥 + 𝑠𝑒𝑐 𝑥) 𝑑𝑥
2
Noting that 𝑑(𝑡𝑎𝑛 𝑥) = 𝑠𝑒𝑐 2 𝑥 𝑑𝑥 and 𝑑(𝑠𝑒𝑐 𝑥) = 𝑠𝑒𝑐 𝑥 𝑡𝑎𝑛 𝑥 𝑑𝑥, we have
36
1 1
∫ 𝑠𝑒𝑐 3 𝑥 𝑑𝑥 = ∫[𝑠𝑒𝑐 𝑥 𝑑(𝑡𝑎𝑛 𝑥) + 𝑡𝑎𝑛 𝑥 𝑑(𝑠𝑒𝑐 𝑥)] 𝑑𝑥 + ∫ 𝑠𝑒𝑐 𝑥 𝑑𝑥
2 2
1 1
= ∫ 𝑑(𝑠𝑒𝑐 𝑥 𝑡𝑎𝑛 𝑥) + ∫ 𝑠𝑒𝑐 𝑥 𝑑𝑥
2 2
1 1
∫ 𝑠𝑒𝑐 3 𝑥 𝑑𝑥 = 𝑠𝑒𝑐 𝑥 𝑡𝑎𝑛 𝑥 + 𝑙𝑛 |𝑠𝑒𝑐 𝑥 + 𝑡𝑎𝑛 𝑥| + 𝐶 ANS.
2 2
_____________________________________________________________________________________________________
Example 2.18.
Evaluate
∫ 𝑐𝑠𝑐 𝑡 𝑐𝑜𝑡 2 𝑡 𝑑𝑡
Solution. We can solve this integral using integration parts but we can also use the same
technique demonstrated above. We begin by rewriting the integral, thus,
1
∫ 𝑐𝑠𝑐 𝑡 𝑐𝑜𝑡 2 𝑡 𝑑𝑡 = ∫(𝑐𝑠𝑐 𝑡 𝑐𝑜𝑡 𝑡 𝑐𝑜𝑡 𝑡 + 𝑐𝑠𝑐 𝑡 𝑐𝑜𝑡 𝑡 𝑐𝑜𝑡 𝑡) 𝑑𝑡
2
and note that 𝑑(𝑐𝑠𝑐 𝑡) = − 𝑐𝑠𝑐 𝑡 𝑐𝑜𝑡 𝑡 𝑑𝑡. This means that 𝑐𝑠𝑐 𝑡 𝑐𝑜𝑡 𝑡 𝑐𝑜𝑡 𝑡 𝑑𝑡 =
−𝑐𝑜𝑡 𝑡 𝑑(𝑐𝑠𝑐 𝑡) and we want to add − 𝑐𝑠𝑐 𝑡 𝑑(𝑐𝑜𝑡 𝑡) = 𝑐𝑠𝑐 𝑡 𝑐𝑠𝑐 2 𝑡 𝑑𝑡 and subtract it as
well. Hence,
1
∫ 𝑐𝑠𝑐 𝑡 𝑐𝑜𝑡 2 𝑡 𝑑𝑡 = ∫(𝑐𝑠𝑐 𝑡 𝑐𝑜𝑡 𝑡 𝑐𝑜𝑡 𝑡 + 𝑐𝑠𝑐 𝑡 𝑐𝑜𝑡 𝑡 𝑐𝑜𝑡 𝑡 + 𝑐𝑠𝑐 𝑡 𝑐𝑠𝑐 2 𝑡 − 𝑐𝑠𝑐 𝑡 𝑐𝑠𝑐 2 𝑡) 𝑑𝑡
2
1
= ∫(𝑐𝑠𝑐 𝑡 𝑐𝑜𝑡 𝑡 𝑐𝑜𝑡 𝑡 + 𝑐𝑠𝑐 𝑡 𝑐𝑠𝑐 2 𝑡 + 𝑐𝑠𝑐 𝑡 𝑐𝑜𝑡 𝑡 𝑐𝑜𝑡 𝑡 − 𝑐𝑠𝑐 𝑡 𝑐𝑠𝑐 2 𝑡) 𝑑𝑡
2
1 1
= ∫(𝑐𝑠𝑐 𝑡 𝑐𝑜𝑡 𝑡 𝑐𝑜𝑡 𝑡 + 𝑐𝑠𝑐 𝑡 𝑐𝑠𝑐 2 𝑡) 𝑑𝑡 + ∫ 𝑐𝑠𝑐 𝑡 (𝑐𝑜𝑡 2 𝑡 − 𝑐𝑠𝑐 2 𝑡) 𝑑𝑡
2 2
1 1
= ∫ −𝑑(𝑐𝑠𝑐 𝑡 𝑐𝑜𝑡 𝑡) 𝑑𝑡 + ∫ 𝑐𝑠𝑐 𝑡 (−1) 𝑑𝑡
2 2
1 1
= − 𝑐𝑠𝑐 𝑡 𝑐𝑜𝑡 𝑡 + 𝑙𝑛 |𝑐𝑠𝑐 𝑡 + 𝑐𝑜𝑡 𝑡| + 𝐶 ANS.
2 2
_____________________________________________________________________________________________________
37
Try these.
1. ∫ 𝑐𝑜𝑠 4 𝑡 𝑠𝑖𝑛3 𝑡 𝑑𝑡
2. ∫ 𝑡𝑎𝑛4 𝑥 𝑑𝑥
𝑐𝑜𝑠 𝑤
4. ∫ 𝑑𝑤
1 − 𝑐𝑜𝑠 𝑤
1 + 𝑠𝑖𝑛 𝑡
5. ∫ 𝑑𝑡
1 − 𝑠𝑖𝑛 𝑡
6. ∫ √1 + 𝑐𝑜𝑠 𝑟 𝑑𝑟
7. ∫ 𝑠𝑖𝑛 3𝑥 𝑐𝑜𝑠 2𝑥 𝑑𝑥
1
8. ∫ 𝑑𝑟
𝑠𝑖𝑛 𝑟 − 2𝑐𝑜𝑠 𝑟
9. ∫ 𝑐𝑠𝑐 3 𝑦 𝑐𝑜𝑡 5 𝑦 𝑑𝑦
38
Integrals Yielding Inverse
3. Trigonometric Functions
If 𝑦 = 𝑠𝑖𝑛 𝑥 and we would like to determine the value of the angle 𝑥 whose sine
equals 𝑦, then 𝑥 = 𝑎𝑟𝑐𝑠𝑖𝑛 𝑦 or 𝑠𝑖𝑛−1 𝑦. It can be said that arcsine is the inverse function
of sine of an angle. So, if 𝑦 = 𝑓(𝑥) = 𝑠𝑖𝑛 𝑥, the inverse function is 𝑓 −1 (𝑥) = 𝑠𝑖𝑛−1 𝑥. Note
that the inverse of a function is not simply the reciprocal of the function or 1/𝑓(𝑥) and is
not obtained by simply interchanging the variables 𝑥 and 𝑦.
Just as there are six trigonometric functions, there are also six inverse
trigonometric functions. The following identities relate them to each other:
39
𝑐𝑜𝑠 −1 𝑥 = 𝜋 − 𝑠𝑖𝑛−1 √1 − 𝑥 2 𝑥≥0
𝑥
𝑠𝑖𝑛−1 𝑥 = 𝑡𝑎𝑛−1 √1−𝑥 2 𝑥 ∈ (0, 1)
𝑥
𝑡𝑎𝑛−1 𝑥 = 𝑠𝑖𝑛−1 √1−𝑥 2 𝑥>0
√1−𝑥2
𝑐𝑜𝑠 −1 𝑥 = 𝑡𝑎𝑛−1 𝑥 ∈ (0, 1)
𝑥
2
−1 √1−𝑥
𝑡𝑎𝑛−1 𝑥 = 𝑐𝑜𝑠 𝑥>0
𝑥
_____________________________________________________________________________________________________
Example 3.1.
Evaluate
∫ 𝑐𝑜𝑠 (2 𝑠𝑖𝑛−1 𝑥 ) 𝑑𝑥
Solution. If we let 𝑢 = 𝑠𝑖𝑛−1 𝑥, 𝑥 = 𝑠𝑖𝑛 𝑢 and 𝑑𝑥 = 𝑐𝑜𝑠 𝑢 𝑑𝑢. The integral becomes
Example 3.2.
Evaluate
∫ 𝑐𝑜𝑠 −1 (𝑠𝑖𝑛𝑥) 𝑑𝑥
Solution. To simplify the integrand, we use the identity 𝑠𝑖𝑛 𝑥 = 𝑐𝑜𝑠 (𝜋/2 − 𝑥). Thus,
𝜋
= ∫ ( − 𝑥) 𝑑𝑥
2
40
𝜋𝑥 1 2
= − 𝑥 +𝐶 ANS.
2 2
Suppose that 𝑥 = 𝑠𝑖𝑛 𝜃. This means that 𝜃 = 𝑠𝑖𝑛−1 𝑥 and 𝑑𝑥 = 𝑐𝑜𝑠 𝜃 𝑑𝜃. It
appears that
𝑑𝑥 𝑑𝑥
𝑑𝜃 = =
𝑐𝑜𝑠 𝜃 √1 − 𝑠𝑖𝑛2 𝜃
𝑑𝑥
∫ 𝑑𝜃 = ∫ = 𝑠𝑖𝑛−1 𝑥 + 𝐶
√1 − 𝑥2
Similarly, if we supposed that 𝑥 = 𝑐𝑜𝑠 𝜃, 𝜃 = 𝑐𝑜𝑠 −1 𝑥, and 𝑑𝑥 = −𝑠𝑖𝑛 𝜃 𝑑𝜃. Following the
same argument,
𝑑𝑥
∫ = −𝑐𝑜𝑠 −1 𝑥 + 𝐶
√1 − 𝑥2
Suppose this time that 𝑥 = 𝑡𝑎𝑛 𝜃, so that 𝜃 = 𝑡𝑎𝑛−1 𝑥 and 𝑑𝑥 = 𝑠𝑒𝑐 2 𝜃 𝑑𝜃. This
implies that
𝑑𝑥 𝑑𝑥
𝑑𝜃 = 2
=
𝑠𝑒𝑐 𝜃 1 + 𝑡𝑎𝑛2 𝜃
𝑑𝑥
∫ 𝑑𝜃 = ∫ = 𝑡𝑎𝑛−1 𝑥 + 𝐶
1 + 𝑥2
𝑑𝑥
∫ = −𝑐𝑜𝑡 −1 𝑥 + 𝐶
1 + 𝑥2
Finally, suppose that 𝑥 = 𝑠𝑒𝑐 𝜃, so that 𝜃 = 𝑠𝑒𝑐 −1 𝑥 and 𝑑𝑥 = 𝑠𝑒𝑐 𝜃 𝑡𝑎𝑛 𝜃 𝑑𝜃. As
a consequence,
𝑑𝑥 𝑑𝑥
𝑑𝜃 = =
𝑠𝑒𝑐 𝜃 𝑡𝑎𝑛 𝜃 𝑠𝑒𝑐 𝜃 √𝑠𝑒𝑐 2 𝜃 − 1
41
Substituting 𝑥 = 𝑠𝑒𝑐 𝜃 and integrating,
𝑑𝑥
∫ 𝑑𝜃 = ∫ = 𝑠𝑒𝑐 −1 𝑥 + 𝐶
𝑥√𝑥 2 − 1
Similarly, if 𝑥 = 𝑐𝑠𝑐 𝜃,
𝑑𝑥
∫ = −𝑐𝑠𝑐 −1 𝑥 + 𝐶
𝑥√𝑥 2 − 1
𝑑𝑥 𝑥
∫ = 𝑠𝑖𝑛−1 +𝐶 3.1
√𝑎2 − 𝑥 2 𝑎
𝑑𝑥 𝑥
∫ = −𝑐𝑜𝑠 −1 +𝐶 3.2
√𝑎2 − 𝑥 2 𝑎
𝑑𝑥 1 𝑥
∫ = 𝑡𝑎𝑛−1 + 𝐶 3.3
𝑎2 +𝑥 2 𝑎 𝑎
𝑑𝑥 1 −1
𝑥
∫ = − 𝑐𝑜𝑡 +𝐶 3.4
𝑎2 + 𝑥 2 𝑎 𝑎
𝑑𝑥 1 𝑥
∫ = 𝑠𝑒𝑐 −1 + 𝐶 3.5
𝑥√𝑥 2 − 𝑎2 𝑎 𝑎
𝑑𝑥 1 𝑥
∫ =− 𝑐𝑠𝑐 −1 + 𝐶 3.6
𝑥√𝑥 2 − 𝑎2 𝑎 𝑎
𝑑 𝑥 1
𝑠𝑖𝑛−1 =
𝑑𝑥 𝑎 √𝑎2 − 𝑥 2
𝑑 𝑥 1
𝑐𝑜𝑠 −1 = −
𝑑𝑥 𝑎 √𝑎2 − 𝑥 2
𝑑 𝑥 𝑎
𝑡𝑎𝑛−1 = 2
𝑑𝑥 𝑎 𝑎 + 𝑥2
𝑑 𝑥 𝑎
𝑐𝑜𝑡 −1 = − 2
𝑑𝑥 𝑎 𝑎 + 𝑥2
42
𝑑 𝑥 𝑎
𝑠𝑒𝑐 −1 =
𝑑𝑥 𝑎 𝑥 √𝑥 2 − 𝑎2
𝑑 𝑥 𝑎
𝑐𝑠𝑐 −1 = −
𝑑𝑥 𝑎 𝑥 √𝑥 − 𝑎2
2
_____________________________________________________________________________________________________
Example 3.3.
Evaluate
𝑑𝑥
∫
√3 − 2𝑥 2
𝑑𝑥 1 𝑑𝑢
∫ = ∫
√3 − 2𝑥 2 √2 √3 − 𝑢2
Finally,
1
𝑑𝑥 𝑠𝑖𝑛−1 √2𝑥 + 𝐶
√2 √3
∫ ={ ANS.
√3 − 2𝑥 2 1 −1 √2𝑥
− 𝑐𝑜𝑠 +𝐶
√2 √3
_____________________________________________________________________________________________________
Example 3.4.
Evaluate
𝑦
∫ 𝑑𝑦
𝑦4 + 9
𝑦 1 𝑑𝑢
∫ 𝑑𝑦 = ∫
𝑦4 + 9 2 𝑢2 + 9
43
Using either Eq. 3.3 or 3.4,
−1 1 𝑢
1 𝑑𝑢 1 3 𝑡𝑎𝑛 3 + 𝐶
∫ = ∙{
2 𝑢2 + 9 2 −1 𝑐𝑜𝑡 −1 𝑢 + 𝐶
3 3
Finally,
1 𝑦2
𝑦 6
𝑡𝑎𝑛−1 3 + 𝐶
∫ 4 𝑑𝑦 = { 1 𝑦2
ANS.
𝑦 +9 −6 𝑐𝑜𝑡 −1 3 + 𝐶
_____________________________________________________________________________________________________
Example 3.5.
Evaluate
𝑑𝑥
∫
√4𝑥 − 𝑥 2
Solution. To be able to use the standard integrals above, there is a need to reduce the
integrand to its recognizable form. The integrand may be written as
4𝑥 − 𝑥 2 = 4 − 4 + 4𝑥 − 𝑥 2 = 4 − (𝑥 − 2)2
𝑑𝑥 𝑑𝑥
∫ =∫
√4𝑥 − 𝑥 2 √4 − (𝑥 − 2)2
Example 3.6.
Evaluate
𝑡𝑎𝑛 𝑟
∫ 𝑑𝑟
√𝑠𝑒𝑐 2 𝑟 − 9
Solution. If we let 𝑢 = 𝑠𝑒𝑐 𝑟, 𝑑𝑢 = 𝑠𝑒𝑐 𝑟 𝑡𝑎𝑛 𝑟 𝑑𝑟. We can write the integral as follows:
44
𝑡𝑎𝑛 𝑟 𝑠𝑒𝑐 𝑟 𝑡𝑎𝑛 𝑟
∫ 𝑑𝑟 = ∫ 𝑑𝑟
√𝑠𝑒𝑐 2 𝑟 − 9 𝑠𝑒𝑐 𝑟 √𝑠𝑒𝑐 2 𝑟 − 9
𝑠𝑒𝑐 𝑟 𝑡𝑎𝑛 𝑟 𝑑𝑢
∫ 𝑑𝑟 = ∫
𝑠𝑒𝑐 𝑟 √𝑠𝑒𝑐 2 𝑟 − 9 𝑢 √𝑢2 − 9
1 𝑢
3
𝑠𝑒𝑐 −1 3 + 𝐶
={ 1 𝑢
−3 𝑐𝑠𝑐 −1 3 + 𝐶
1 𝑠𝑒𝑐 𝑟
3
𝑠𝑒𝑐 −1 ( 3 ) + 𝐶
={ 1 𝑠𝑒𝑐 𝑟 ANS.
−3 𝑐𝑠𝑐 −1 ( 3 ) + 𝐶
_____________________________________________________________________________________________________
Example 3.7.
Evaluate
𝑐𝑜𝑠 𝑦
∫ 𝑑𝑦
√𝑐𝑜𝑠 2 𝑦 + 3
Solution. At first look, the integrand does not appear recognizable to enable us to use the
standard integrals above. But if we write it as
𝑐𝑜𝑠 𝑦 𝑐𝑜𝑠 𝑦
∫ 𝑑𝑦 = ∫ 𝑑𝑦
√𝑐𝑜𝑠 2 𝑦 + 3 √(1 − 𝑠𝑖𝑛2 𝑦) + 3
𝑐𝑜𝑠 𝑦
=∫ 𝑑𝑦
√4 − 𝑠𝑖𝑛2 𝑦
it transforms into something more familiar. Letting 𝑢 = 𝑠𝑖𝑛 𝑦, 𝑑𝑢 = 𝑐𝑜𝑠 𝑦 𝑑𝑦. Hence, the
integral reduces to
𝑐𝑜𝑠 𝑦 𝑑𝑢
∫ 𝑑𝑦 = ∫
√4 − 𝑠𝑖𝑛2 𝑦 √4 − 𝑢2
𝑢
𝑠𝑖𝑛−1 2 + 𝐶
={ 𝑢
−𝑐𝑜𝑠 −1 2 + 𝐶
45
𝑠𝑖𝑛 𝑦
𝑠𝑖𝑛−1 ( 2
)+𝐶
= { 𝑠𝑖𝑛 𝑦
ANS.
−𝑐𝑜𝑠 −1 ( 2 ) + 𝐶
_____________________________________________________________________________________________________
Example 3.8.
Evaluate
√𝑐𝑜𝑠 −1 𝑥
∫ 𝑑𝑥
√1 − 𝑥 2
𝑑 1
𝑐𝑜𝑠 −1 𝑥 = −
𝑑𝑥 √1 − 𝑥 2
√𝑐𝑜𝑠 −1 𝑥 2 3/2
∫ 𝑑𝑥 = ∫ −√𝑢 𝑑𝑢 = − 𝑢 +𝐶
√1 − 𝑥 2 3
2
=− (𝑐𝑜𝑠 −1 𝑥)3/2 + 𝐶 ANS.
3
_____________________________________________________________________________________________________
Example 3.9.
Evaluate
∫ 𝑡𝑎𝑛−1 (𝑐𝑜𝑡 𝑥) 𝑑𝑥
−1 (𝑐𝑜𝑡
𝑡𝑎𝑛−1 𝑢
∫ 𝑡𝑎𝑛 𝑥) 𝑑𝑥 = − ∫ 𝑑𝑢
1 + 𝑢2
𝑡𝑎𝑛−1 𝑢
−∫ 𝑑𝑢 = − ∫ 𝑡 𝑑𝑡
1 + 𝑢2
1
= − 𝑡2 + 𝐶
2
46
1
= − (𝑡𝑎𝑛−1 𝑢)2 + 𝐶
2
1
= − (𝑡𝑎𝑛−1 𝑐𝑜𝑡 𝑥)2 + 𝐶 ANS.
2
_____________________________________________________________________________________________________
Example 3.10.
1
∫ 𝑑𝑧
√𝑧(1 + 𝑧)
1 1 1
∫ 𝑑𝑧 = ∫ ∙ 2 𝑑𝑧
√𝑧(1 + 𝑧) √𝑧 [1 + (√𝑧) ]
1
𝑑𝑢 = 𝑑𝑧
2√𝑧
1 1 𝑑𝑢 2 𝑡𝑎𝑛−1 𝑢 + 𝐶
∫ ∙ 𝑑𝑧 = 2 ∫ = {
√𝑧 [1 + (√𝑧) ]
2 1 + 𝑢2 −2 𝑐𝑜𝑡 −1 𝑢 + 𝐶
Hence,
1 2 𝑡𝑎𝑛−1 √𝑧 + 𝐶
∫ 𝑑𝑧 = { ANS.
√𝑧(1 + 𝑧) −2 𝑐𝑜𝑡 −1 √𝑧 + 𝐶
_____________________________________________________________________________________________________
Example 3.11.
Evaluate
1
∫ 𝑑𝑦
𝑦√𝑦 − 1
47
1 𝑑𝑢
∫ 𝑑𝑦 = 2 ∫
𝑦√𝑦 − 1 𝑢2+1
−1
= { 2 𝑡𝑎𝑛 −1𝑢 + 𝐶
−2 𝑐𝑜𝑡 𝑢 + 𝐶
2 𝑡𝑎𝑛−1 √𝑦 − 1 + 𝐶
={ ANS.
−2 𝑐𝑜𝑡 −1 √𝑦 − 1 + 𝐶
_____________________________________________________________________________________________________
Example 3.12.
Evaluate
𝑥3
∫√ 𝑑𝑥
9 − 𝑥5
Solution. At first glance, the integrand is not one of the recognizable forms which yields
an inverse trigonometric function. So, an approach to this problem requires that the
integrand be transformed into one with a square term inside the square root. The integral
can be rewritten as
𝑥3 𝑥 3/2
∫√ 𝑑𝑥 = ∫ 𝑑𝑥
9 − 𝑥5 √9 − (𝑥 5/2 )2
𝑥 3/2 2 𝑑𝑢
∫ 𝑑𝑥 = ∫
√9 − (𝑥 5/2 )2 5 √9 − 𝑢2
which is a recognizable form which yields inverse sine or cosine function. Thus,
2 𝑢
𝑥3 𝑠𝑖𝑛−1 + 𝐶
∫√ 𝑑𝑥 = { 5 3
9 − 𝑥5 2 𝑢
− 𝑐𝑜𝑠 −1 + 𝐶
5 3
2 𝑥 5/2
𝑠𝑖𝑛−1 +𝐶
= 5 3 ANS.
2 −1
𝑥 5/2
−
{ 5 𝑐𝑜𝑠 +𝐶
3
____________________________________________________________________________________________________
48
Try these.
1
1. ∫ 𝑑𝑥
3𝑥 2+5
1
2. ∫ 𝑑𝑥
√5 − 4𝑥 − 𝑥 2
𝑠𝑒𝑐 2 𝑥
3. ∫ 𝑑𝑥
√5 − 𝑠𝑒𝑐 2 𝑥
𝑐𝑜𝑡 𝑦
∫ 𝑑𝑥
4. √𝑐𝑜𝑡 2 𝑦 − 3
(𝑡𝑎𝑛−1 𝑦)2/3
5. ∫ 𝑑𝑦
𝑦2 + 1
1
6. ∫ 𝑑𝑟
√𝑟√1 − 𝑟
𝑥
7. ∫√ 𝑑𝑥
1 − 𝑥3
49
Integrals Involving Exponential and
4. Logarithmic Functions
𝑓(𝑥) = 𝑒 𝑥
𝑓 ′ (𝑥) = 𝑒 𝑥
Hence,
∫ 𝑒 𝑥 𝑑𝑥 = 𝑒 𝑥 + 𝐶
In general,
𝑒 𝑎𝑥
∫ 𝑒 𝑎𝑥 𝑑𝑥 = +𝐶
𝑎 4.1
𝑏 𝑎𝑥
∫ 𝑏 𝑎𝑥 𝑑𝑥 = +𝐶
𝑎 𝑙𝑛 𝑏 4.2
_____________________________________________________________________________________________________
50
Example 4.1.
Evaluate
2
∫ 𝑥 ∙ 2𝑥 𝑑𝑥
2 1
∫ 𝑥 ∙ 2𝑥 𝑑𝑥 = ∫ 2𝑢 𝑑𝑢
2
1 2𝑢
= +𝐶
2 𝑙𝑛 2
2
2𝑥
= +𝐶 ANS.
2 𝑙𝑛 2
_____________________________________________________________________________________________________
Example 4.2.
Evaluate
∫ 𝑒 𝑥 𝑐𝑜𝑠(𝑒 𝑥 ) 𝑑𝑥
Solution. We can use the substitution rule in this example by letting 𝑢 = 𝑒 𝑥 . Notably,
𝑑𝑢 = 𝑒 𝑥 𝑑𝑥, so
∫ 𝑒 𝑥 𝑐𝑜𝑠(𝑒 𝑥 ) 𝑑𝑥 = ∫ 𝑐𝑜𝑠 𝑢 𝑑𝑢
= 𝑠𝑖𝑛 𝑢 + 𝐶
= 𝑠𝑖𝑛 (𝑒 𝑥 ) + 𝐶 ANS.
_____________________________________________________________________________________________________
Example 4.3.
Evaluate
𝑒 √𝑥
∫ 𝑑𝑥
√𝑥
51
𝑒 √𝑥
∫ 𝑑𝑥 = 2 ∫ 𝑒 𝑢 𝑑𝑢
√𝑥
= 2 𝑒𝑢 + 𝐶
= 2 𝑒 √𝑥 + 𝐶 ANS.
_____________________________________________________________________________________________________
Example 4.4.
Evaluate
∫ 𝑠𝑒𝑐 2 𝑥 ∙ 3𝑡𝑎𝑛 𝑥 𝑑𝑥
Solution. This example needs no more detailed calculation. We note that if 𝑢 = 𝑡𝑎𝑛 𝑥,
𝑑𝑢 = 𝑠𝑒𝑐 2 𝑥 𝑑𝑥. Hence,
∫ 𝑠𝑒𝑐 2 𝑥 ∙ 3𝑡𝑎𝑛 𝑥 𝑑𝑥 = ∫ 3𝑢 𝑑𝑢
3𝑢 3𝑡𝑎𝑛 𝑥
= +𝐶 = +𝐶 ANS.
𝑙𝑛 3 𝑙𝑛 3
_____________________________________________________________________________________________________
Example 4.5.
Evaluate
𝑥 +𝑥)
∫ 𝑒 (𝑒 𝑑𝑥
Solution. At first glance, the integrand appears very complicated. But it may be written
as follows:
𝑥 +𝑥) 𝑥
∫ 𝑒 (𝑒 𝑑𝑥 = ∫ 𝑒 𝑒 ∙ 𝑒 𝑥 𝑑𝑥
If we let 𝑢 = 𝑒 𝑥 , we know that 𝑑𝑢 = 𝑒 𝑥 𝑑𝑥. So, the integral reduces to the integral of
𝑒 𝑢 , which is also 𝑒 𝑢 . Therefore,
𝑥 +𝑥)
∫ 𝑒 (𝑒 𝑑𝑥 = ∫ 𝑒 𝑢 𝑑𝑢
𝑥
= 𝑒𝑢 + 𝐶 = 𝑒𝑒 + 𝐶 ANS.
_____________________________________________________________________________________________________
52
Example 4.6.
Evaluate
𝑒 𝑦−1 + 𝑦 𝑒−1
∫ 𝑑𝑦
𝑒𝑦 + 𝑦𝑒
Solution. If we let 𝑢 = 𝑒 𝑦 + 𝑦 𝑒 ,
𝑒 𝑦−1 + 𝑦 𝑒−1 1 𝑑𝑢
∫ 𝑑𝑥 = ∫
𝑒𝑦 + 𝑦𝑒 𝑒 𝑢
1
= 𝑙𝑛 |𝑢| + 𝐶
𝑒
1
= 𝑙𝑛 (𝑒 𝑦 + 𝑦 𝑒 ) + 𝐶 ANS.
𝑒
_____________________________________________________________________________________________________
Example 4.7.
Evaluate
𝑒 2𝑥 − 1
∫ 𝑑𝑥
𝑒 2𝑥 + 1
𝑒 2𝑥 − 1 𝑒 𝑥 𝑒 𝑥 − 𝑒 −𝑥
÷ =
𝑒 2𝑥 + 1 𝑒 𝑥 𝑒 𝑥 + 𝑒 −𝑥
The reason becomes obvious. If we let 𝑢 = 𝑒 𝑥 + 𝑒 −𝑥 , then 𝑑𝑢 = (𝑒 𝑥 − 𝑒 −𝑥 ) 𝑑𝑥. The
integral reduces to
𝑒 2𝑥 − 1 𝑒 𝑥 − 𝑒 −𝑥 𝑑𝑢
∫ 𝑑𝑥 = ∫ 𝑑𝑥 = ∫
𝑒 2𝑥 + 1 𝑒 𝑥 + 𝑒 −𝑥 𝑢
53
Example 4.8.
Evaluate
𝑒𝑥 − 1
∫ 𝑥 𝑑𝑥
𝑒 +1
Solution. We note the similarity of the integrand to the previous example. Although it is
tempting to divide the integrand also by 𝑒 𝑥 /𝑒 𝑥 , that will not work in this example. If we
1 1
divide the integrand instead by 𝑒 2𝑥 /𝑒 2𝑥 , the reason becomes obvious. Thus,
1 1 1
𝑒 𝑥 − 1 𝑒 2𝑥 𝑒 2𝑥 − 𝑒 −2𝑥
÷ 1 = 1 1
𝑒𝑥 + 1
𝑒 2𝑥 𝑒 2𝑥 + 𝑒 −2𝑥
Notice that if we let 𝑢 be the denominator, 𝑑𝑢 is the numerator. Hence,
1 1
𝑒𝑥 − 1 𝑒 2𝑥 − 𝑒 −2𝑥 𝑑𝑢
∫ 𝑥 𝑑𝑥 = ∫ 1 1 𝑑𝑥 = ∫ 2
𝑒 +1 𝑢
𝑒 2𝑥 + 𝑒 −2𝑥
1 1
= 2 𝑙𝑛 |𝑢| + 𝐶 = 2 𝑙𝑛 |𝑒 2𝑥 + 𝑒 −2𝑥 | + 𝐶 ANS.
_____________________________________________________________________________________________________
Example 4.9.
Evaluate
2𝑒 𝑥 − 5
∫ 𝑥 𝑑𝑥
3𝑒 + 2
𝑁 𝐴𝐷 + 𝐵𝐷′
∫ 𝑑𝑥 = ∫ 𝑑𝑥
𝐷 𝐷
𝐷′
= 𝐴 ∫ 𝑑𝑥 + 𝐵 ∫ 𝑑𝑥
𝐷
= 𝐴𝑥 + 𝐵 𝑙𝑛 𝐷 + 𝐶
54
To determine the value of the coefficients 𝐴 and 𝐵, we proceed as follows:
𝑁 = 𝐴𝐷 + 𝐵𝐷′
2𝑒 𝑥 − 5 = 𝐴(3𝑒 𝑥 + 2) + 𝐵(3𝑒 𝑥 + 0)
2𝑒 𝑥 − 5 = 3𝑒 𝑥 (𝐴 + 𝐵) + 2𝐴
∴ 3(𝐴 + 𝐵) = 2 and 2𝐴 = −5
5 19
Solving for 𝐴 and 𝐵 yields 𝐴 = − 2 and 𝐵 = . Substituting these values gives
6
2𝑒 𝑥 − 5 5 19
∫ 𝑥
𝑑𝑥 = − 𝑥 + 𝑙𝑛 (3𝑒 𝑥 + 2) + 𝐶 ANS.
3𝑒 + 2 2 6
_____________________________________________________________________________________________________
Example 4.10.
Evaluate
1
∫ 𝑑𝑥
2𝑒 2𝑥 +3
𝑁 = 𝐴𝐷 + 𝐵𝐷′
1 = 𝐴(2𝑒 2𝑥 + 3) + 𝐵(4𝑒 2𝑥 + 0)
1 = 𝑒 2𝑥 (2𝐴 + 4𝐵) + 3𝐴
∴ 2𝐴 + 4𝐵 = 0 and 3𝐴 = 1
1 1
Solving for 𝐴 and 𝐵 yields 𝐴 = 3 and 𝐵 = − 6. Substituting these values gives
1 1 1
∫ 𝑑𝑥 = 𝑥 − 𝑙𝑛 (2𝑒 2𝑥 + 3) + 𝐶 ANS.
2𝑒 2𝑥 +3 3 6
There is a second way we can evaluate the integrand and this is more
straightforward using the substitution rule. Before that, let’s divide the integrand by
𝑒 2𝑥 /𝑒 2𝑥 , thus,
1 𝑒 2𝑥 𝑒 −2𝑥
÷ =
2𝑒 2𝑥 + 3 𝑒 2𝑥 2 + 3𝑒 −2𝑥
55
The reason becomes obvious later. If we let 𝑢 = 2 + 3𝑒 −2𝑥 , then 𝑑𝑢 = −6𝑒 −2𝑥 𝑑𝑥 and
1
− 6 𝑑𝑢 = 𝑒 −2𝑥 𝑑𝑥. The integrand further reduces to
1 1 𝑑𝑢
∫ 𝑑𝑥 = − ∫
2𝑒 2𝑥 + 3 6 𝑢
1
=− 𝑙𝑛 |𝑢| + 𝐶
6
1
=− 𝑙𝑛 (2 + 3𝑒 −2𝑥 ) + 𝐶
6
You might wonder why the resulting function seems different from the answer appearing
above. It is left as an exercise to show that these two functions are in fact equivalent.
_____________________________________________________________________________________________________
Example 4.11.
Evaluate
𝑐𝑜𝑡 (𝑙𝑛 𝑥)
∫ 𝑑𝑥
𝑥
𝑐𝑜𝑡 (𝑙𝑛 𝑥)
∫ 𝑑𝑥 = ∫ 𝑐𝑜𝑡 𝑢 𝑑𝑢 = 𝑙𝑛 |𝑠𝑖𝑛 𝑢| + 𝐶
𝑥
= 𝑙𝑛 |𝑠𝑖𝑛 𝑙𝑛 𝑥| + 𝐶 ANS.
_____________________________________________________________________________________________________
Example 4.12.
Evaluate
𝑑𝑥
∫
𝑥(𝑙𝑛 𝑥)(𝑙𝑛 𝑙𝑛 𝑥)
𝑑𝑥 𝑑𝑢
∫ =∫
𝑥(𝑙𝑛 𝑥)(𝑙𝑛 𝑙𝑛 𝑥) 𝑢
= 𝑙𝑛 |𝑢| + 𝐶.
56
Finally,
𝑑𝑥
∫ = 𝑙𝑛 𝑙𝑛 (𝑙𝑛 𝑥) + 𝐶 ANS.
𝑥(𝑙𝑛 𝑥)(𝑙𝑛 𝑙𝑛 𝑥)
_____________________________________________________________________________________________________
Example 4.13.
Evaluate
2 +1
∫ 𝑥𝑥 (𝑙𝑛 𝑥 2 + 1) 𝑑𝑥
2
Solution. We can let 𝑢 = 𝑥 𝑥 . This means that 𝑙𝑛 𝑢 = 𝑥 2 𝑙𝑛 𝑥 and
𝑑𝑢
𝑑(𝑙𝑛 𝑢) = = (2𝑥 𝑙𝑛 𝑥 + 𝑥) 𝑑𝑥
𝑢
𝑑𝑢 = 𝑢𝑥 (𝑙𝑛 𝑥 2 + 1) 𝑑𝑥
2 +1
= 𝑥𝑥 (𝑙𝑛 𝑥 2 + 1) 𝑑𝑥
Note that this resulting integral is exactly the same integral to be evaluated. Hence,
∫ 𝑑𝑢 = 𝑢 + 𝐶
2 +1 2
∫ 𝑥𝑥 (𝑙𝑛 𝑥 2 + 1) 𝑑𝑥 = 𝑥 𝑥 + 𝐶 ANS.
__________________________________________________________________________________________________
Example 4.14.
Evaluate
𝑒 𝑥 − 𝑒 −𝑥 2
∫( 𝑥 ) 𝑑𝑥
𝑒 + 𝑒 −𝑥
Solution. The terms 𝑒 𝑥 − 𝑒 −𝑥 and 𝑒 𝑥 + 𝑒 −𝑥 are special in that one is the derivative of the
other. To solve the problem, we need to reduce the integrand into the derivative of a
quotient or 𝑑(𝑢/𝑣) with 𝑢 = 𝑒 𝑥 − 𝑒 −𝑥 and 𝑣 = 𝑒 𝑥 + 𝑒 −𝑥 . To do this, we add and subtract
1, thus,
𝑒 𝑥 − 𝑒 −𝑥 2 𝑒 𝑥 − 𝑒 −𝑥 2
∫( 𝑥 ) 𝑑𝑥 = ∫ [1 + ( 𝑥 ) − 1] 𝑑𝑥
𝑒 + 𝑒 −𝑥 𝑒 + 𝑒 −𝑥
57
Further reduction yields
𝑒 𝑥 − 𝑒 −𝑥 2 (𝑒 𝑥 − 𝑒 −𝑥 )2 − (𝑒 𝑥 + 𝑒 −𝑥 )2
∫( 𝑥 ) 𝑑𝑥 = ∫ [1 + ] 𝑑𝑥
𝑒 + 𝑒 −𝑥 (𝑒 𝑥 + 𝑒 −𝑥 )2
(𝑒 𝑥 + 𝑒 −𝑥 )2 − (𝑒 𝑥 − 𝑒 −𝑥 )2
= ∫ [1 − ] 𝑑𝑥
(𝑒 𝑥 + 𝑒 −𝑥 )2
(𝑒 𝑥 + 𝑒 −𝑥 )(𝑒 𝑥 + 𝑒 −𝑥 ) − (𝑒 𝑥 − 𝑒 −𝑥 )(𝑒 𝑥 − 𝑒 −𝑥 )
= ∫ [1 − ] 𝑑𝑥
(𝑒 𝑥 + 𝑒 −𝑥 )2
(𝑒 𝑥 + 𝑒 −𝑥 )(𝑒 𝑥 + 𝑒 −𝑥 ) − (𝑒 𝑥 − 𝑒 −𝑥 )(𝑒 𝑥 − 𝑒 −𝑥 ) 𝑣 𝑑𝑢 − 𝑢 𝑑𝑣
∫[ 𝑥 −𝑥 2
] 𝑑𝑥 = ∫
(𝑒 + 𝑒 ) 𝑣2
𝑢 𝑢
= ∫𝑑( ) =
𝑣 𝑣
𝑒 𝑥 − 𝑒 −𝑥
=
𝑒 𝑥 + 𝑒 −𝑥
Hence,
2
𝑒 𝑥 − 𝑒 −𝑥 𝑒 𝑥 − 𝑒 −𝑥
∫( 𝑥 ) 𝑑𝑥 = 𝑥 − +𝐶 ANS.
𝑒 + 𝑒 −𝑥 𝑒 𝑥 + 𝑒 −𝑥
In the next chapter, you will recognize the second term as 𝑡𝑎𝑛ℎ 𝑥.
_____________________________________________________________________________________________________
Example 4.15.
Evaluate
𝑒𝑥
∫√ 𝑑𝑥
4 − 𝑒𝑥
Solution. We do not want to attempt to clear out the square root. Rather, we recognize
that since one over the square root of a binomial is usually the derivative of an inverse
trigonometric function, we wish to attempt to transform the integrand into one which is
recognizably yielding an inverse trigonometric function.
58
𝑒𝑥 𝑒 1/2 𝑥
∫√ 𝑑𝑥 = ∫ 𝑑𝑥
4 − 𝑒𝑥 √4 − (𝑒 1/2 𝑥 )2
𝑒 −1/2 𝑥 𝑑𝑢
∫ 𝑑𝑥 = 2 ∫
√4 − (𝑒 1/2 𝑥 )2 √4 − 𝑢2
𝑢
2 𝑠𝑖𝑛−1 +𝐶
={ 2
𝑢
−2 𝑐𝑜𝑠 −1 + 𝐶
2
√𝑒 𝑥
2 𝑠𝑖𝑛−1 +𝐶
2
={ ANS.
√𝑒 𝑥
−2 𝑐𝑜𝑠 −1 2 +𝐶
_____________________________________________________________________________________________________
Example 4.16.
Evaluate
𝑒 𝑥 − 2𝑥
∫ 𝑑𝑥
𝑒 𝑥 + 2𝑥
Solution. This integrand requires some magic trick to solve. What complicated the
integrand is the difference in the bases of the exponential functions. If we let 𝑢 = 𝑒 𝑥 + 2𝑥
we get 𝑑𝑢 = 𝑒 𝑥 + 2𝑥 𝑙𝑛 2 so we have to produce the term +2𝑥 𝑙𝑛 2 in the numerator by
all means possible. We can expand 𝑑𝑢 such that
𝑑𝑢 = 𝑒 𝑥 + 2𝑥 𝑙𝑛 2 + 2𝑥 − 2𝑥
= 𝑒 𝑥 + 2𝑥 + 2𝑥 (𝑙𝑛 2 − 1)
That’s the clue there. Multiplying the numerator by (𝑙𝑛 2 − 1)/(𝑙𝑛 2 − 1), we have
𝑙𝑛 2 − 1 𝑒 𝑥 𝑙𝑛 2 − 2𝑥 𝑙𝑛 2 − 𝑒 𝑥 + 2𝑥
𝑒 𝑥 − 2𝑥 ∙ =
𝑙𝑛 2 − 1 𝑙𝑛 2 − 1
But, remember, we need to produce the term +2𝑥 𝑙𝑛 2 in the numerator. We also need
the term −2𝑥 . Sadly, the expansion contains the opposite of both terms. We expand
further, thus,
59
𝑙𝑛 2 − 1 𝑒 𝑥 𝑙𝑛 2 − 2𝑥 𝑙𝑛 2 − 𝑒 𝑥 + 2𝑥 + (2𝑥 𝑙𝑛 2 − 2𝑥 𝑙𝑛 2) + (2𝑥 − 2𝑥 )
𝑒 𝑥 − 2𝑥 ∙ =
𝑙𝑛 2 − 1 𝑙𝑛 2 − 1
𝑒 𝑥 (𝑙𝑛 2 + 1) + 2𝑥 (𝑙𝑛 2 + 1) − 2𝑒 𝑥 − 2𝑥 𝑙𝑛 2
=
𝑙𝑛 2 − 1
(𝑒 𝑥 + 2𝑥 )(𝑙𝑛 2 + 1) − 2(𝑒 𝑥 + 2𝑥 𝑙𝑛 2)
=
𝑙𝑛 2 − 1
𝑙𝑛 2 + 1 2(𝑒 𝑥 + 2𝑥 𝑙𝑛 2)
= ∫[ − ] 𝑑𝑥
𝑙𝑛 2 − 1 (𝑙𝑛 2 − 1)(𝑒 𝑥 + 2𝑥 )
𝑙𝑛 2 + 1 2 𝑑𝑢
=∫ 𝑑𝑥 − ∫
𝑙𝑛 2 − 1 𝑙𝑛 2 − 1 𝑢
𝑙𝑛 2 + 1 2
=( )𝑥 −( ) 𝑙𝑛 |𝑒 𝑥 + 2𝑥 | + 𝐶 ANS.
𝑙𝑛 2 − 1 𝑙𝑛 2 − 1
_____________________________________________________________________________________________________
60
Try these.
𝑒𝑦
1. ∫3 𝑑𝑦
√1 + 𝑒 𝑦
𝑒 𝑥 𝑐𝑜𝑠 𝑒 𝑥
2. ∫ 𝑑𝑥
1 + 𝑠𝑖𝑛 𝑒 𝑥
3. ∫(1 − 𝑒 𝑦 )3 𝑑𝑦
1 1
4. ∫ 𝑠𝑒𝑐 𝑙𝑛 𝑑𝑥
𝑥 𝑥
1 − 2𝑡
5. ∫ 𝑑𝑡
1 + 2𝑡
𝑒𝑥
6. ∫ 𝑑𝑥
√1 − 𝑒 𝑥
𝑒 3𝑥 − 1
7. ∫ 𝑑𝑥
𝑒 3𝑥 + 1
8. ∫ 𝑥 𝑥 (𝑙𝑛 𝑥 + 1) 𝑑𝑥
3𝑒 𝑥 − 1
9. ∫ 𝑑𝑥
4𝑒 𝑥 + 5
1
10. ∫ 𝑑𝑥
2𝑒 3𝑥 −9
61
5. Integrals of Hyperbolic Functions
𝑒 𝑥 + 𝑒 −𝑥
𝑐𝑜𝑠ℎ 𝑥 =
2
𝑒 𝑥 − 𝑒 −𝑥
𝑠𝑖𝑛ℎ 𝑥 =
2
Using these definitions, the other hyperbolic functions are also defined as follows:
𝑠𝑖𝑛ℎ 𝑥 𝑒 𝑥 − 𝑒 −𝑥
𝑡𝑎𝑛ℎ 𝑥 = =
𝑐𝑜𝑠ℎ 𝑥 𝑒 𝑥 + 𝑒 −𝑥
𝑐𝑜𝑠ℎ 𝑥 𝑒 𝑥 + 𝑒 −𝑥
𝑐𝑜𝑡ℎ 𝑥 = =
𝑠𝑖𝑛ℎ 𝑥 𝑒 𝑥 − 𝑒 −𝑥
1 2
𝑠𝑒𝑐ℎ 𝑥 = = 𝑥
𝑐𝑜𝑠ℎ 𝑥 𝑒 + 𝑒 −𝑥
1 2
𝑐𝑠𝑐ℎ 𝑥 = = 𝑥
𝑠𝑖𝑛ℎ 𝑥 𝑒 − 𝑒 −𝑥
62
Like trigonometric functions, there are many useful relations that can be derived
for hyperbolic functions. The following hyperbolic identities will be useful in evaluating
the integrals of hyperbolic functions:
𝑐𝑜𝑠ℎ 𝑥 + 𝑠𝑖𝑛ℎ 𝑥 = 𝑒 𝑥
𝑐𝑜𝑠ℎ2 𝑥 − 𝑠𝑖𝑛ℎ2 𝑥 = 1
𝑠𝑒𝑐ℎ2 𝑥 + 𝑡𝑎𝑛ℎ2 𝑥 = 1
𝑐𝑜𝑡ℎ2 𝑥 − 𝑐𝑠𝑐ℎ2 𝑥 = 1
1 𝑐𝑜𝑠ℎ 𝑥 + 1
𝑐𝑜𝑠ℎ2 2𝑥 =
2
1 𝑐𝑜𝑠ℎ 𝑥 − 1
𝑠𝑖𝑛ℎ2 2𝑥 =
2
In differential calculus, the derivatives of the hyperbolic functions are given by the
following:
𝑑
𝑠𝑖𝑛ℎ 𝑎𝑥 = 𝑎 𝑐𝑜𝑠ℎ 𝑎𝑥
𝑑𝑥
𝑑
𝑐𝑜𝑠ℎ 𝑎𝑥 = 𝑎 𝑠𝑖𝑛ℎ 𝑎𝑥
𝑑𝑥
𝑑
𝑡𝑎𝑛ℎ 𝑎𝑥 = 𝑎 𝑠𝑒𝑐ℎ2 𝑎𝑥
𝑑𝑥
𝑑
𝑐𝑜𝑡ℎ 𝑎𝑥 = −𝑎 𝑐𝑠𝑐ℎ2 𝑎𝑥
𝑑𝑥
𝑑
𝑠𝑒𝑐ℎ 𝑎𝑥 = −𝑎 𝑠𝑒𝑐ℎ 𝑎𝑥 ∙ 𝑡𝑎𝑛ℎ 𝑎𝑥
𝑑𝑥
𝑑
𝑐𝑠𝑐ℎ 𝑎𝑥 = −𝑎 𝑐𝑠𝑐ℎ 𝑎𝑥 ∙ 𝑐𝑜𝑡ℎ 𝑎𝑥
𝑑𝑥
From these, we derive the following relations:
1
∫ 𝑠𝑖𝑛ℎ 𝑎𝑥 𝑑𝑥 = 𝑐𝑜𝑠ℎ 𝑎𝑥 + 𝐶 5.1
𝑎
63
1
∫ 𝑐𝑜𝑠ℎ 𝑎𝑥 𝑑𝑥 = 𝑠𝑖𝑛ℎ 𝑎𝑥 + 𝐶 5.2
𝑎
1
∫ 𝑠𝑒𝑐ℎ2 𝑎𝑥 𝑑𝑥 = 𝑡𝑎𝑛ℎ 𝑎𝑥 + 𝐶 5.3
𝑎
1
∫ 𝑐𝑠𝑐ℎ2 𝑎𝑥 𝑑𝑥 = − 𝑐𝑜𝑡ℎ 𝑎𝑥 + 𝐶 5.4
𝑎
1
∫ 𝑠𝑒𝑐ℎ 𝑎𝑥 ∙ 𝑡𝑎𝑛ℎ 𝑎𝑥 𝑑𝑥 = − 𝑠𝑒𝑐ℎ 𝑎𝑥 + 𝐶 5.5
𝑎
1
∫ 𝑐𝑠𝑐ℎ 𝑎𝑥 ∙ 𝑐𝑜𝑡ℎ 𝑎𝑥 𝑑𝑥 = − 𝑐𝑠𝑐ℎ 𝑎𝑥 + 𝐶 5.6
𝑎
𝑠𝑖𝑛ℎ 𝑎𝑥
∫ 𝑡𝑎𝑛ℎ 𝑎𝑥 𝑑𝑥 = ∫ 𝑑𝑥
𝑐𝑜𝑠ℎ 𝑎𝑥
Note that the derivative of 𝑐𝑜𝑠ℎ 𝑎𝑥 is 𝑎 𝑠𝑖𝑛ℎ 𝑎𝑥 𝑑𝑥. Hence, using the substitution rule
where 𝑢 = 𝑐𝑜𝑠ℎ 𝑎𝑥,
1 𝑑𝑢 1
∫ 𝑡𝑎𝑛ℎ 𝑎𝑥 𝑑𝑥 = ∫ = 𝑙𝑛 |𝑢|
𝑎 𝑢 𝑎
1
= 𝑙𝑛 |𝑐𝑜𝑠ℎ 𝑎𝑥| + 𝐶
𝑎
Similarly, the integral of the hyperbolic cotangent of an angle is
𝑐𝑜𝑠ℎ 𝑎𝑥
∫ 𝑐𝑜𝑡ℎ 𝑎𝑥 𝑑𝑥 = ∫ 𝑑𝑥
𝑠𝑖𝑛ℎ 𝑎𝑥
1 𝑑𝑢
= ∫
𝑎 𝑢
1
= 𝑙𝑛 |𝑠𝑖𝑛ℎ 𝑎𝑥| + 𝐶
𝑎
In deriving the integral of the hyperbolic secant of an angle, we write as follows:
1 𝑐𝑜𝑠ℎ 𝑥
∫ 𝑠𝑒𝑐ℎ 𝑥 𝑑𝑥 = ∫ 𝑑𝑥 = ∫ 𝑑𝑥
𝑐𝑜𝑠ℎ 𝑥 𝑐𝑜𝑠ℎ2 𝑥
𝑐𝑜𝑠ℎ 𝑥
=∫ 𝑑𝑥
1 + 𝑠𝑖𝑛ℎ2 𝑥
64
Note that when 𝑢 = 𝑠𝑖𝑛ℎ 𝑥, 𝑑𝑢 = 𝑐𝑜𝑠ℎ 𝑥 𝑑𝑥. The integral reduces to
𝑑𝑢
∫ = 𝑡𝑎𝑛−1 𝑢 + 𝐶
1 + 𝑢2
Generally,
1
∫ 𝑠𝑒𝑐ℎ 𝑎𝑥 𝑑𝑥 = 𝑡𝑎𝑛−1 (𝑠𝑖𝑛ℎ 𝑎𝑥) + 𝐶
𝑎
1
∫ 𝑡𝑎𝑛ℎ 𝑎𝑥 𝑑𝑥 = 𝑙𝑛 |𝑐𝑜𝑠ℎ 𝑎𝑥| + 𝐶 5.7
𝑎
1
∫ 𝑐𝑜𝑡ℎ 𝑎𝑥 𝑑𝑥 = 𝑙𝑛 |𝑠𝑖𝑛ℎ 𝑎𝑥| + 𝐶 5.8
𝑎
1
∫ 𝑠𝑒𝑐ℎ 𝑎𝑥 𝑑𝑥 = 𝑡𝑎𝑛−1 (𝑠𝑖𝑛ℎ 𝑎𝑥) + 𝐶 5.9
𝑎
1
∫ 𝑐𝑠𝑐ℎ 𝑎𝑥 𝑑𝑥 = 𝑙𝑛 |𝑐𝑠𝑐ℎ 𝑎𝑥 − 𝑐𝑜𝑡ℎ 𝑎𝑥| + 𝐶 5.10
𝑎
_____________________________________________________________________________________________________
65
Example 5.1.
Evaluate
𝑠𝑒𝑐ℎ √𝑥
∫ 𝑑𝑥
√𝑥
Example 5.2.
Evaluate
𝑠𝑖𝑛ℎ 3𝑦
∫ 𝑑𝑦
1 + 𝑠𝑖𝑛ℎ2 3𝑦
𝑠𝑖𝑛ℎ 3𝑦 𝑠𝑖𝑛ℎ 3𝑦
∫ 2
𝑑𝑦 = ∫ 𝑑𝑦
1 + (𝑐𝑜𝑠ℎ 3𝑦 − 1) 𝑐𝑜𝑠ℎ2 3𝑦
We note that the derivative of 𝑐𝑜𝑠ℎ 3𝑦 is 3 𝑠𝑖𝑛ℎ 3𝑦 𝑑𝑦. The integral reduces to
1 1 1 1
∫ 2 𝑑𝑢 = ∙ (− ) + 𝐶
3 𝑢 3 𝑢
1 1 1
=− ∙ + 𝐶 = − 𝑠𝑒𝑐ℎ 3𝑦 + 𝐶 ANS.
3 𝑐𝑜𝑠ℎ 3𝑦 3
_____________________________________________________________________________________________________
Example 5.3.
Evaluate
𝑐𝑜𝑠ℎ 𝑥 + 𝑠𝑖𝑛ℎ 𝑥
∫ 𝑑𝑥
𝑐𝑜𝑠ℎ 𝑥 − 𝑠𝑖𝑛ℎ 𝑥
66
Noting that the denominator is equal to 1 and by expanding the numerator, the integral
further reduces to
1 1
∫(𝑐𝑜𝑠ℎ 2𝑥 + 𝑠𝑖𝑛ℎ 2𝑥) 𝑑𝑥 = 𝑠𝑖𝑛ℎ 2𝑥 + 𝑐𝑜𝑠ℎ 2𝑥 + 𝐶 ANS.
2 2
_____________________________________________________________________________________________________
Example 5.4.
Evaluate
∫ 𝑐𝑜𝑠ℎ 𝑢 𝑑𝑢 = 𝑠𝑖𝑛ℎ 𝑢 + 𝐶
Example 5.5.
Evaluate
∫ 𝑡𝑎𝑛ℎ5 𝑥 𝑑𝑥
Solution. There are two ways we can approach the integral. First, we can transform it
into
𝑠𝑖𝑛ℎ5 𝑥
∫ 𝑡𝑎𝑛ℎ5 𝑥 𝑑𝑥 = ∫ 𝑑𝑥
𝑐𝑜𝑠ℎ5 𝑥
67
(𝑠𝑖𝑛ℎ2 𝑥)2
=∫ ∙ 𝑠𝑖𝑛ℎ 𝑥 𝑑𝑥
𝑐𝑜𝑠ℎ5 𝑥
(𝑐𝑜𝑠ℎ2 𝑥 − 1)2
=∫ ∙ 𝑠𝑖𝑛ℎ 𝑥 𝑑𝑥
𝑐𝑜𝑠ℎ5 𝑥
𝑐𝑜𝑠ℎ4 𝑥 − 2 𝑐𝑜𝑠ℎ2 𝑥 + 1
=∫ ∙ 𝑠𝑖𝑛ℎ 𝑥 𝑑𝑥
𝑐𝑜𝑠ℎ5 𝑥
1 2 1
= ∫( − + ) ∙ 𝑠𝑖𝑛ℎ 𝑥 𝑑𝑥
𝑐𝑜𝑠ℎ 𝑥 𝑐𝑜𝑠ℎ 𝑥 𝑐𝑜𝑠ℎ5 𝑥
3
1 2 1
∫ 𝑡𝑎𝑛ℎ5 𝑥 𝑑𝑥 = ∫ ( − 3 + 5 ) 𝑑𝑢
𝑢 𝑢 𝑢
1 1
= 𝑙𝑛 |𝑢| + − +𝐶
𝑢2 4𝑢4
1 1
= 𝑙𝑛 (𝑐𝑜𝑠ℎ 𝑥) + − +𝐶
𝑐𝑜𝑠ℎ2 𝑥 4 𝑐𝑜𝑠ℎ4 𝑥
1
= 𝑙𝑛 (𝑐𝑜𝑠ℎ 𝑥) + 𝑠𝑒𝑐ℎ2 𝑥 − 𝑠𝑒𝑐ℎ4 𝑥 + 𝐶 ANS.
4
For our second approach, we rewrite the integral as
= ∫ 𝑡𝑎𝑛ℎ3 𝑥 ∙ (1 − 𝑠𝑒𝑐ℎ2 𝑥) 𝑑𝑥
If we let 𝑢 = 𝑡𝑎𝑛ℎ 𝑥, 𝑑𝑢 = 𝑠𝑒𝑐ℎ2 𝑥 𝑑𝑥. Thus, the integral on the left becomes
68
1 1
∫ 𝑡𝑎𝑛ℎ 𝑥 𝑑𝑥 − ∫(𝑢 + 𝑢3 ) 𝑑𝑢 = 𝑙𝑛 (𝑐𝑜𝑠ℎ 𝑥) − 𝑢2 − 𝑢4 + 𝐶
2 4
Finally,
1 1
∫ 𝑡𝑎𝑛ℎ5 𝑥 𝑑𝑥 = 𝑙𝑛 (𝑐𝑜𝑠ℎ 𝑥) − 𝑡𝑎𝑛ℎ2 𝑥 − 𝑡𝑎𝑛ℎ4 𝑥 + 𝐶 ANS.
2 4
_____________________________________________________________________________________________________
Example 5.6.
Evaluate
∫ 𝑐𝑠𝑐ℎ3 𝑥 𝑐𝑜𝑡ℎ3 𝑥 𝑑𝑥
Solution. We factor out the term 𝑐𝑠𝑐ℎ 𝑥 𝑐𝑜𝑡ℎ 𝑥 and express the rest in terms of 𝑐𝑠𝑐ℎ 𝑥
using the proper hyperbolic identity. The integral reduces to
By now, it has become obvious why we factored out the term 𝑐𝑠𝑐ℎ 𝑥 𝑐𝑜𝑡ℎ 𝑥 and expressed
the rest in terms of 𝑐𝑠𝑐ℎ 𝑥. That is because the integrand can be reduced to an ordinary
power function where 𝑢 = 𝑐𝑠𝑐ℎ 𝑥 and 𝑑𝑢 = −𝑐𝑠𝑐ℎ 𝑥 𝑐𝑜𝑡ℎ 𝑥 𝑑𝑥. Substituting,
1 1
∫ 𝑐𝑠𝑐ℎ3 𝑥 𝑐𝑜𝑡ℎ3 𝑥 𝑑𝑥 = − ∫(𝑢2 + 𝑢4 ) 𝑑𝑢 = − 𝑢3 − 𝑢5 + 𝐶
3 5
1 1
= − 𝑐𝑠𝑐ℎ3 𝑥 − 𝑐𝑠𝑐ℎ5 𝑥 + 𝐶 ANS.
3 5
_____________________________________________________________________________________________________
69
Example 5.7.
Evaluate
𝑑𝑥
∫
√𝑐𝑜𝑠ℎ 𝑥 + 𝑠𝑖𝑛ℎ 𝑥
Solution. We know that the derivative of 𝑐𝑜𝑠ℎ 𝑥 + 𝑠𝑖𝑛ℎ 𝑥 is equal to itself. So, we need to
find 𝑐𝑜𝑠ℎ 𝑥 + 𝑠𝑖𝑛ℎ 𝑥 in the numerator. To do that, we need to multiply the integrand by
1 𝑐𝑜𝑠ℎ 𝑥 + 𝑠𝑖𝑛ℎ 𝑥
∙
√𝑐𝑜𝑠ℎ 𝑥 + 𝑠𝑖𝑛ℎ 𝑥 𝑐𝑜𝑠ℎ 𝑥 + 𝑠𝑖𝑛ℎ 𝑥
The result is
𝑑𝑥 𝑐𝑜𝑠ℎ 𝑥 + 𝑠𝑖𝑛ℎ 𝑥
∫ =∫ 𝑑𝑥 = ∫ 𝑢−3/2 𝑑𝑢
√𝑐𝑜𝑠ℎ 𝑥 + 𝑠𝑖𝑛ℎ 𝑥 (𝑐𝑜𝑠ℎ 𝑥 + 𝑠𝑖𝑛ℎ 𝑥)3/2
Finally,
𝑑𝑥 2
∫ =− +𝐶 ANS.
√𝑐𝑜𝑠ℎ 𝑥 + 𝑠𝑖𝑛ℎ 𝑥 √𝑐𝑜𝑠ℎ 𝑥 + 𝑠𝑖𝑛ℎ 𝑥
_____________________________________________________________________________________________________
Example 5.8.
Evaluate
Solution. Since the derivative of 𝑐𝑜𝑠ℎ 𝑥 − 𝑠𝑖𝑛ℎ 𝑥 is 𝑠𝑖𝑛ℎ 𝑥 − 𝑐𝑜𝑠ℎ 𝑥, we can rewrite the
integrand as
Hence,
1
∫(𝑐𝑜𝑠ℎ 𝑥 − 𝑠𝑖𝑛ℎ 𝑥)𝑛 𝑑𝑥 = − 𝑢𝑛 + 𝐶
𝑛
70
1
= − (𝑐𝑜𝑠ℎ 𝑥 − 𝑠𝑖𝑛ℎ 𝑥)𝑛 + 𝐶 ANS.
𝑛
_____________________________________________________________________________________________________
Example 5.9.
Evaluate
4 𝑐𝑜𝑠ℎ 𝑥 − 𝑠𝑖𝑛ℎ 𝑥
∫ 𝑑𝑥
2 𝑐𝑜𝑠ℎ 𝑥 + 3 𝑠𝑖𝑛ℎ 𝑥
Solution. Using the same technique in Example 3.9., we also need to reduce the integral
into this form:
𝑁 𝐴𝐷 + 𝐵𝐷′
∫ 𝑑𝑥 = ∫ 𝑑𝑥
𝐷 𝐷
𝐷′
∫ (𝐴 + 𝐵 ) 𝑑𝑥 = 𝐴𝑥 + 𝐵 𝑙𝑛 𝐷 + 𝐶
𝐷
Furthermore,
4 = 2𝐴 + 3𝐵
−1 = 3𝐴 + 2𝐵
4 𝑐𝑜𝑠ℎ 𝑥 − 𝑠𝑖𝑛ℎ 𝑥 11 14 𝐷′
∫ 𝑑𝑥 = ∫ (− + ) 𝑑𝑥
2 𝑐𝑜𝑠ℎ 𝑥 + 3 𝑠𝑖𝑛ℎ 𝑥 5 5 𝐷
11 14
=− 𝑥+ 𝑙𝑛 |2 𝑐𝑜𝑠ℎ 𝑥 + 3 𝑠𝑖𝑛ℎ 𝑥| + 𝐶 ANS.
5 5
_____________________________________________________________________________________________________
71
Example 5.10.
Evaluate
∫ 𝑐𝑠𝑐ℎ 𝑙𝑛 𝑥 𝑑𝑥
1 2
𝑐𝑠𝑐ℎ 𝑥 = = 𝑥
𝑠𝑖𝑛ℎ 𝑥 𝑒 − 𝑒 −𝑥
2 2 2𝑥
𝑐𝑠𝑐ℎ 𝑙𝑛 𝑥 = = = 2
𝑒 𝑙𝑛 𝑥 −𝑒 −𝑙𝑛 𝑥 1
𝑥−𝑥 𝑥 −1
Recognizing that the numerator is the derivative of the denominator, we finally have
2𝑥
∫ 𝑐𝑠𝑐ℎ 𝑙𝑛 𝑥 𝑑𝑥 = ∫ 𝑑𝑥
𝑥2 −1
= 𝑙𝑛 |𝑥 2 − 1| + 𝐶 ANS.
_____________________________________________________________________________________________________
Example 5.11.
Evaluate
∫ √𝑐𝑜𝑠ℎ 𝑥 + 1 𝑑𝑥
Solution. Since 𝑑(𝑐𝑜𝑠ℎ 𝑥) = 𝑠𝑖𝑛ℎ 𝑥 𝑑𝑥, we need to find 𝑠𝑖𝑛ℎ 𝑥 in the integrand and we can
do this by multiplying the integrand by 𝑠𝑖𝑛ℎ 𝑥/𝑠𝑖𝑛ℎ 𝑥, thus,
𝑠𝑖𝑛ℎ 𝑥
∫ √𝑐𝑜𝑠ℎ 𝑥 + 1 𝑑𝑥 = ∫ √𝑐𝑜𝑠ℎ 𝑥 + 1 𝑑𝑥
𝑠𝑖𝑛ℎ 𝑥
√𝑐𝑜𝑠ℎ 𝑥 + 1
= ∫ 𝑠𝑖𝑛ℎ 𝑥 𝑑𝑥
√𝑐𝑜𝑠ℎ2 𝑥 − 1
𝑠𝑖𝑛ℎ 𝑥
=∫ 𝑑𝑥
√𝑐𝑜𝑠ℎ 𝑥 − 1
If we let 𝑢 = 𝑐𝑜𝑠ℎ 𝑥 − 1, 𝑑𝑢 = 𝑠𝑖𝑛ℎ 𝑥 𝑑𝑥. The integral becomes
72
𝑠𝑖𝑛ℎ 𝑥 𝑑𝑢
∫ 𝑑𝑥 = ∫
√𝑐𝑜𝑠ℎ 𝑥 − 1 √𝑢
= 2 √𝑢 + 𝐶
= 2√𝑐𝑜𝑠ℎ 𝑥 − 1 + 𝐶 ANS.
1 𝑐𝑜𝑠ℎ 𝑥 + 1
𝑐𝑜𝑠ℎ2 2𝑥 =
2
we can rewrite the integral as
1
∫ √1 + 𝑐𝑜𝑠ℎ 𝑥 𝑑𝑥 = ∫ √2 𝑐𝑜𝑠ℎ 2𝑥 𝑑𝑥
Integrating,
1
∫ √1 + 𝑐𝑜𝑠ℎ 𝑥 𝑑𝑥 = 2√2 𝑠𝑖𝑛ℎ 2𝑥 + 𝐶 ANS.
Example 5.12.
Evaluate
1
∫ 𝑑𝑥
√1 + 𝑐𝑜𝑠ℎ 𝑥
Solution. Following the suggestion in the preceding example, the integral transforms into
1 𝑠𝑖𝑛ℎ 𝑥 1
∫ 𝑑𝑥 = ∫ 𝑑𝑥
√𝑐𝑜𝑠ℎ 𝑥 + 1 𝑠𝑖𝑛ℎ 𝑥 √𝑐𝑜𝑠ℎ 𝑥 + 1
𝑠𝑖𝑛ℎ 𝑥
=∫ 𝑑𝑥
√𝑐𝑜𝑠ℎ2 𝑥 − 1 √𝑐𝑜𝑠ℎ 𝑥 + 1
𝑠𝑖𝑛ℎ 𝑥
=∫ 𝑑𝑥
(𝑐𝑜𝑠ℎ 𝑥 + 1)√𝑐𝑜𝑠ℎ 𝑥 − 1
73
𝑠𝑖𝑛ℎ 𝑥 𝑑𝑢
∫ 𝑑𝑥 = ∫
(𝑐𝑜𝑠ℎ 𝑥 + 1)√𝑐𝑜𝑠ℎ 𝑥 − 1 (𝑢 + 2)√𝑢
𝑑𝑢 1
∫ = 2∫ 𝑑𝑡
(𝑢 + 2)√𝑢 𝑡2 +2
2 𝑡
= 𝑡𝑎𝑛−1 +𝐶
√2 √2
2 √𝑢
= 𝑡𝑎𝑛−1 +𝐶
√2 √2
2 √𝑐𝑜𝑠ℎ 𝑥 − 1
= 𝑡𝑎𝑛−1 ( )+𝐶
√2 √2
We recognize the term inside the parenthesis as the hyperbolic sine of half angle. Thus,
1 1
∫ 𝑑𝑥 = √2 𝑡𝑎𝑛−1 (𝑠𝑖𝑛ℎ 2𝑥) + 𝐶 ANS.
√𝑐𝑜𝑠ℎ 𝑥 + 1
Alternatively, using the identity for the half-angle, we can rewrite the integral as
1 1
∫ 𝑑𝑥 = ∫ 𝑑𝑥
√1 + 𝑐𝑜𝑠ℎ 𝑥 √2 𝑐𝑜𝑠ℎ 12𝑥
1 1
= ∫ 𝑠𝑒𝑐ℎ 2𝑥 𝑑𝑥
√2
1 1
∫ 𝑑𝑥 = √2 𝑡𝑎𝑛−1 (𝑠𝑖𝑛ℎ 2𝑥) + 𝐶 ANS.
√1 + 𝑐𝑜𝑠ℎ 𝑥
_____________________________________________________________________________________________________
74
Try these.
2. ∫ 𝑐𝑜𝑠ℎ5 𝑥 𝑑𝑥
3. ∫ 𝑠𝑖𝑛ℎ4 𝑥 𝑑𝑥
4. ∫ 𝑠𝑖𝑛ℎ 𝑥 𝑐𝑜𝑠ℎ(𝑐𝑜𝑠ℎ 𝑥) 𝑑𝑥
1
5. ∫ 𝑑𝑥
√𝑐𝑜𝑠ℎ 𝑥 − 1
6. ∫ 𝑡𝑎𝑛ℎ2 𝑥 𝑠𝑒𝑐ℎ4 𝑥 𝑑𝑥
7. ∫ 𝑐𝑜𝑠ℎ 𝑙𝑛 𝑥 𝑑𝑥
8. ∫ 𝑠𝑖𝑛ℎ 2𝑥 𝑐𝑜𝑠ℎ 3𝑥 𝑑𝑥
75
Integrals Yielding Inverse
6. Hyperbolic Functions
Looking at Eqs. 3.1 to 3.6, one can’t help notice that there seem to be missing
integrals. There appear no standard integrals included in the list for the familiar forms
√𝑥 2 − 𝑎2 , 𝑥 2 − 𝑎2 , and 𝑥√𝑎2 − 𝑥 2 . These forms will be considered here. These integrals
yield inverse hyperbolic functions.
𝑠𝑖𝑛ℎ−1 𝑥 = 𝑙𝑛 |𝑥 + √𝑥 2 + 1|
𝑐𝑜𝑠ℎ−1 𝑥 = 𝑙𝑛 |𝑥 + √𝑥 2 − 1|
1 1+𝑥
𝑡𝑎𝑛ℎ−1 𝑥 = 2 𝑙𝑛 | |
1−𝑥
1 𝑥+1
𝑐𝑜𝑡ℎ−1 𝑥 = 2 𝑙𝑛 | |
𝑥−1
1 + √1 − 𝑥 2
𝑠𝑒𝑐ℎ−1 𝑥 = 𝑙𝑛 | |
𝑥
−1
1 + √1 + 𝑥 2
𝑐𝑠𝑐ℎ 𝑥 = 𝑙𝑛 | |
𝑥
Suppose that 𝑥 = 𝑠𝑖𝑛ℎ 𝜃, so that 𝜃 = 𝑠𝑖𝑛ℎ−1 𝑥 and 𝑑𝑥 = 𝑐𝑜𝑠ℎ 𝜃 𝑑𝜃. This means
that
76
𝑑𝑥 𝑑𝑥
𝑑𝜃 = =
𝑐𝑜𝑠ℎ 𝜃 √1 + 𝑠𝑖𝑛ℎ2 𝜃
𝑑𝑥
∫ 𝑑𝜃 = ∫
√1 + 𝑥 2
Hence,
𝑑𝑥
∫ = 𝑠𝑖𝑛ℎ−1 𝑥 + 𝐶
√1 + 𝑥2
Suppose this time that 𝑥 = 𝑐𝑜𝑠ℎ 𝜃, so that 𝜃 = 𝑐𝑜𝑠ℎ−1 𝑥 and 𝑑𝑥 = 𝑠𝑖𝑛ℎ 𝜃 𝑑𝜃. This
means that
𝑑𝑥 𝑑𝑥
𝑑𝜃 = =
𝑠𝑖𝑛ℎ 𝜃 √𝑐𝑜𝑠ℎ2 𝜃 − 1
𝑑𝑥
∫ 𝑑𝜃 = ∫
√𝑥 2 − 1
Hence,
𝑑𝑥
∫ = 𝑐𝑜𝑠ℎ−1 𝑥 + 𝐶
√𝑥 2 − 1
𝑑𝑥 𝑑𝑥
𝑑𝜃 = =
𝑠𝑒𝑐ℎ 𝜃 1 − 𝑡𝑎𝑛ℎ2 𝜃
2
𝑑𝑥
∫ 𝑑𝜃 = ∫
1 − 𝑥2
Hence,
𝑑𝑥
∫ = 𝑡𝑎𝑛ℎ−1 𝑥 + 𝐶
1 − 𝑥2
𝑑𝑥
∫ 𝑑𝜃 = ∫
1 − 𝑥2
Hence,
𝑑𝑥
∫ = 𝑐𝑜𝑡ℎ−1 𝑥 + 𝐶
1 − 𝑥2
One will notice that this is identical to the integral yielding inverse hyperbolic tangent.
The distinction is in the value of 𝑥. If 𝑥 2 < 1, we use hyperbolic tangent. If 𝑥 2 > 1, we use
hyperbolic cotangent.
𝑑𝑥 𝑑𝑥
𝑑𝜃 = − =−
𝑠𝑒𝑐ℎ 𝜃 𝑡𝑎𝑛ℎ 𝜃 𝑠𝑒𝑐ℎ 𝜃 √1 − 𝑠𝑒𝑐ℎ2 𝜃
𝑑𝑥
∫ 𝑑𝜃 = ∫ −
𝑥 √1 − 𝑥 2
Hence,
𝑑𝑥
∫ = −𝑠𝑒𝑐ℎ−1 𝑥 + 𝐶
𝑥 √1 − 𝑥2
𝑑𝑥 𝑑𝑥
𝑑𝜃 = − =−
𝑐𝑠𝑐ℎ 𝜃 𝑐𝑜𝑡ℎ 𝜃 𝑐𝑠𝑐ℎ 𝜃 √1 + 𝑐𝑠𝑐ℎ2 𝜃
𝑑𝑥
∫ 𝑑𝜃 = ∫ −
𝑥 √1 + 𝑥 2
Hence,
78
𝑑𝑥
∫ = −𝑐𝑠𝑐ℎ−1 𝑥 + 𝐶
𝑥 √1 + 𝑥2
𝑑𝑥 𝑥
∫ = 𝑠𝑖𝑛ℎ−1 +𝐶 6.1
√𝑥 2 + 𝑎2 𝑎
𝑑𝑥 𝑥
∫ = 𝑐𝑜𝑠ℎ−1 +𝐶 6.2
√𝑥 2 − 𝑎2 𝑎
𝑑𝑥 1 𝑥
∫ = 𝑡𝑎𝑛ℎ−1 + 𝐶, 𝑥 2 < 𝑎2 6.3
𝑎2 −𝑥 2 𝑎 𝑎
𝑑𝑥 1 𝑥
∫ = 𝑐𝑜𝑡ℎ−1 + 𝐶, 𝑥 2 > 𝑎2 6.4
𝑎2 −𝑥 2 𝑎 𝑎
𝑑𝑥 1 𝑥
∫ =− 𝑠𝑒𝑐ℎ−1 + 𝐶 6.5
𝑥√𝑎2 − 𝑥 2 𝑎 𝑎
𝑑𝑥 1 𝑥
∫ =− 𝑐𝑠𝑐ℎ−1 + 𝐶 6.6
𝑥√𝑎2 + 𝑥 2 𝑎 𝑎
𝑑𝑥 𝑥
∫ = 𝑠𝑖𝑛−1 +𝐶 3.1
√𝑎2 − 𝑥 2 𝑎
𝑑𝑥 𝑥
∫ = −𝑐𝑜𝑠 −1 +𝐶 3.2
√𝑎2 − 𝑥 2 𝑎
𝑑𝑥 1 −1
𝑥
∫ = 𝑡𝑎𝑛 +𝐶 3.3
𝑎2 + 𝑥 2 𝑎 𝑎
𝑑𝑥 1 −1
𝑥
∫ = − 𝑐𝑜𝑡 +𝐶 3.4
𝑎2 + 𝑥 2 𝑎 𝑎
𝑑𝑥 1 𝑥
∫ = 𝑠𝑒𝑐 −1 + 𝐶 3.5
𝑥√𝑥 2 − 𝑎2 𝑎 𝑎
𝑑𝑥 1 𝑥
∫ =− 𝑐𝑠𝑐 −1 + 𝐶 3.6
𝑥√𝑥 2 − 𝑎2 𝑎 𝑎
79
Example 6.1.
Evaluate
1
∫ 𝑑𝑥
√𝑥 2 − 6𝑥
𝑑𝑥 𝑑𝑥 𝑑𝑥
∫ =∫ =∫
√𝑥 2 − 6𝑥 √𝑥 2 − 6𝑥 + 9 − 9 √(𝑥 − 3)2 − 9
Integrating,
𝑑𝑥 𝑥−3
∫ = 𝑐𝑜𝑠ℎ−1 ( )+𝐶 ANS.
√(𝑥 − 3)2 − 9 3
_____________________________________________________________________________________________________
Example 6.2.
Evaluate
𝑠𝑒𝑐ℎ2 𝑥
∫ 𝑑𝑥
5 − 𝑡𝑎𝑛ℎ2 𝑥
Solution. Notice that if we let 𝑢 = 𝑡𝑎𝑛ℎ 𝑥, 𝑑𝑢 = 𝑠𝑒𝑐ℎ2 𝑥 𝑑𝑥. The integral reduces to
𝑠𝑒𝑐ℎ2 𝑥 𝑑𝑢
∫ 2
𝑑𝑥 = ∫
5 − 𝑡𝑎𝑛ℎ 𝑥 5 − 𝑢2
1 𝑢
= 𝑡𝑎𝑛ℎ−1 +𝐶
√5 √5
1 𝑡𝑎𝑛ℎ 𝑥
= 𝑡𝑎𝑛ℎ−1 ( )+𝐶 ANS.
√5 √5
If you are wondering why the answer cannot be 𝑐𝑜𝑡ℎ−1 (𝑡𝑎𝑛ℎ 𝑥/√5), it would be helpful
to know that the range of 𝑡𝑎𝑛ℎ 𝑥 is (−1, 1) and 𝑡𝑎𝑛ℎ2 𝑥 < 5 for all values of 𝑥.
_____________________________________________________________________________________________________
80
Example 6.3.
Evaluate
𝑒 𝑥 + 𝑒 −𝑥
∫ 𝑑𝑥
√𝑒 2𝑥 + 𝑒 −2𝑥 − 1
𝑒 2𝑥 + 𝑒 −2𝑥 − 1 = (𝑒 𝑥 − 𝑒 −𝑥 )2 + 1
𝑒 𝑥 + 𝑒 −𝑥 𝑑𝑢
∫ 𝑑𝑥 = ∫
√𝑒 2𝑥 + 𝑒 −2𝑥 − 1 √𝑢2 + 1
= 𝑠𝑖𝑛ℎ−1 𝑢 + 𝐶
= 𝑠𝑖𝑛ℎ−1 (𝑒 𝑥 − 𝑒 −𝑥 ) + 𝐶
Example 6.4.
Evaluate
𝑐𝑜𝑡 𝑥
∫ 𝑑𝑥
√1 + 𝑐𝑠𝑐 2 𝑥
= 𝑐𝑠𝑐ℎ−1 𝑢 + 𝐶
81
Example 6.5.
Evaluate
𝑡𝑎𝑛ℎ 𝑥
∫ 𝑑𝑥
1 + 𝑡𝑎𝑛ℎ2 𝑥
𝑡𝑎𝑛ℎ 𝑥 1 𝑑𝑢
∫ 𝑑𝑥 = ∫
1 + 𝑡𝑎𝑛ℎ2 𝑥 2 1 − 𝑢2
1
2
𝑡𝑎𝑛ℎ−1 𝑢 + 𝐶, 𝑢<1
={ 1
2
𝑐𝑜𝑡ℎ−1 𝑢 + 𝐶, 𝑢>1
1
= 2 𝑡𝑎𝑛ℎ−1 (𝑡𝑎𝑛ℎ2 𝑥) + 𝐶 ANS.
Example 6.6.
Evaluate
1
∫ 𝑑𝑥
𝑥 √1 + 𝑥
Solution. To enable us to use as much as possible the standard integrals to facilitate our
calculation, we should be able to recognize the form of integrand to which the standard
integrals are applicable. The above integrand does not seem to fit any of the form of
standard integrals so it must be transformed into something more recognizable.
82
We realize that the integral may be written as
1 1 1
∫ 𝑑𝑥 = ∫ ∙ 𝑑𝑥
𝑥 √1 + 𝑥 √𝑥 √𝑥 √1 + √𝑥 2
1
𝑑𝑢 = 𝑑𝑥
2√𝑥
Hence,
1 1 𝑑𝑢
∫ ∙ 𝑑𝑥 = 2 ∫
√𝑥 √𝑥 √1 + √𝑥 2 𝑢 √1 + 𝑢2
= −2 𝑐𝑠𝑐ℎ−1 𝑢 + 𝐶
= −2 𝑐𝑠𝑐ℎ−1 √𝑥 + 𝐶 ANS.
___________________________________________________________________________
Example 6.7.
Evaluate
1
∫ 𝑑𝑥
√4 − 22𝑥
Solution. To enable us to use the standard integrals, we need to transform the integrand
into one of the recognizable forms. Notice that the integrand appears to have the form for
hyperbolic arcsecant of an angle. If we multiply the integrand by
1 2𝑥
∙
√4 − 22𝑥 2𝑥
2𝑥
∫ 𝑑𝑥
2𝑥 √4 − 22𝑥
2𝑥 1 𝑑𝑢
∫ 𝑑𝑥 = ∫
2𝑥 √4 − 22𝑥 𝑙𝑛 2 𝑢 √4 − 𝑢2
1 1 𝑢
= ∙ ∙ −𝑠𝑒𝑐ℎ−1 + 𝐶
𝑙𝑛 2 2 2
83
1 2𝑥
= ∙ −𝑠𝑒𝑐ℎ−1 +𝐶
2 𝑙𝑛 2 2
1
=− ∙ 𝑠𝑒𝑐ℎ−1 (2𝑥−1 ) + 𝐶 ANS.
𝑙𝑛 4
_____________________________________________________________________________________________________
Example 6.8.
Evaluate
1
∫ 𝑑𝑥
√𝑒 𝑥 + 1
Solution. Our approach here is similar to what we did in the previous example. We realize
that this appears to have the form for hyperbolic arccosecant of an angle if we rewrite
the integrand as
1 1
=
√𝑒 𝑥 + 1 √(𝑒 1/2𝑥 )2 + 1
It even becomes more obvious as one of the recognizable forms if we multiply the
integrand by
1 𝑒 1/2𝑥
∙
1/2𝑥
√(𝑒 1/2𝑥 )2 + 1 𝑒
1 1 1 1/2𝑥
∫ 𝑑𝑥 = 2 ∫ ∙ 𝑒 𝑑𝑥
√𝑒 𝑥 +1 𝑒 1/2𝑥 √(𝑒 1/2𝑥 )2 + 1 2
The factors 2 and ½ are introduced because if we let 𝑢 = 𝑒 1/2𝑥 , 𝑑𝑢 = 1/2𝑒 1/2𝑥 𝑑𝑥. Hence,
1 1 1/2𝑥 𝑑𝑢
2∫ 𝑒∙ 𝑑𝑥 = 2 ∫
𝑒 1/2𝑥 √(𝑒 1/2𝑥 )2 + 1 2 𝑢√𝑢2 + 1
= −2 𝑐𝑠𝑐ℎ−1 𝑢 + 𝐶
Finally,
1
∫ 𝑑𝑥 = −2 𝑐𝑠𝑐ℎ−1 (𝑒 1/2𝑥 ) + 𝐶 ANS.
√𝑒 𝑥
+1
_____________________________________________________________________________________________________
84
Example 6.9.
Evaluate
1
∫ 𝑑𝑥
√𝑥 5 + 4𝑥 2
Solution. Apparently, the integrand is not one of the recognizable forms which yields
inverse hyperbolic functions. But if we factor 𝑥 2 from the denominator, we can rewrite
our integral as
1 1 1
∫ 𝑑𝑥 = ∫ 𝑑𝑥 = ∫ 𝑑𝑥
√𝑥 5 + 4𝑥 2 𝑥 √𝑥 3 + 4 𝑥 √(𝑥 3/2 )2 + 4
√𝑥 √𝑥 2 𝑑𝑢
∫ 𝑑𝑥 = ∫ 𝑑𝑥 = ∫
𝑥√𝑥 ∙ √(𝑥 3/2 )2 + 4 𝑥 3/2 ∙ √(𝑥 3/2 )2 + 4 3 𝑢 √𝑢2 + 4
Finally,
1 2 1 𝑢
∫ 𝑑𝑥 = (− 𝑐𝑠𝑐ℎ−1 ) + 𝐶
𝑥 √𝑥 3 + 4 3 2 2
1 𝑥 3/2
= − 𝑐𝑠𝑐ℎ−1 +𝐶 ANS.
3 2
3 𝑥2
𝑑𝑢 = 𝑑𝑥
2 √𝑥 3 + 4
1 2 1 2 1
∫ 𝑑𝑥 =∫ 3 𝑑𝑢 = ∫ 2 𝑑𝑢
𝑥 √𝑥 3 + 4 3 𝑥 3 𝑢 −4
2 𝑢
− 𝑡𝑎𝑛ℎ−1 + 𝐶
={ 3 2
2 𝑢
− 𝑐𝑜𝑡ℎ−1 + 𝐶
3 2
85
2 √𝑥 3 + 4
− 𝑡𝑎𝑛ℎ−1 +𝐶
= 3 2 ANS.
2 √𝑥 3 + 4
− 𝑐𝑜𝑡ℎ−1 +𝐶
{ 3 2
Example 6.10.
Evaluate
1
∫ 𝑑𝑥
√1 + 𝑠𝑖𝑛 𝑥
1 𝑐𝑜𝑠 𝑥
∫ 𝑑𝑥 = ∫ 𝑑𝑥
√1 + 𝑠𝑖𝑛 𝑥 𝑐𝑜𝑠 𝑥 √1 + 𝑠𝑖𝑛 𝑥
The reason is because if we transform the denominator into a function of purely 𝑠𝑖𝑛 𝑥,
then 𝑑𝑥 = 𝑐𝑜𝑠 𝑥 𝑑𝑥. This means
𝑐𝑜𝑠 𝑥 𝑐𝑜𝑠 𝑥
∫ 𝑑𝑥 = ∫ 𝑑𝑥
𝑐𝑜𝑠 𝑥 √1 + 𝑠𝑖𝑛 𝑥 √1 − 𝑠𝑖𝑛2 𝑥 √1 + 𝑠𝑖𝑛 𝑥
𝑐𝑜𝑠 𝑥
=∫ 𝑑𝑥
(1 + 𝑠𝑖𝑛 𝑥) √1 − 𝑠𝑖𝑛 𝑥
𝑐𝑜𝑠 𝑥 𝑑𝑢
∫ 𝑑𝑥 = − ∫
(1 + 𝑠𝑖𝑛 𝑥) √1 − 𝑠𝑖𝑛 𝑥 (2 − 𝑢) √𝑢
𝑑𝑢
= −∫ 2 2
√𝑢 (√2 − √𝑢 )
86
To evaluate this last integral, we note that if we let 𝑡 = √𝑢, 𝑑𝑡 = (1/2√𝑢) 𝑑𝑢 and 2 𝑑𝑡 =
𝑑𝑢/√𝑢. This transforms into
𝑑𝑢 𝑑𝑡
−∫ 2 2
= −2 ∫ 2
√𝑢 (√2 − √𝑢 ) √2 − 𝑡 2
𝑑𝑡 2 𝑡
−2 ∫ 2 =− 𝑡𝑎𝑛ℎ−1 +𝐶
√2 − 𝑡 2 √2 √2
Ultimately,
𝑑𝑢 √𝑢
−∫ 2 2
= −√2 𝑡𝑎𝑛ℎ−1 +𝐶
√𝑢 (√2 − √𝑢 ) √2
1 √1 − 𝑠𝑖𝑛 𝑥
∫ 𝑑𝑥 = −√2 𝑡𝑎𝑛ℎ−1 +𝐶 ANS.
√1 + 𝑠𝑖𝑛 𝑥 √2
If you are wondering why we used inverse hyperbolic tangent instead of inverse
hyperbolic cotangent function, you will immediately see why our choice is correct since
2
(1 − 𝑠𝑖𝑛 𝑥)2 ≤ √2 for any value of 𝑥.
_____________________________________________________________________________________________________
Example 6.11.
Evaluate
𝑥2
∫ 𝑑𝑥
1 + 𝑥4
𝑥2 1 2𝑥 2
∫ 𝑑𝑥 = ∫ 𝑑𝑥
1 + 𝑥4 2 1 + 𝑥4
1 𝑥2 + 1 + 𝑥2 − 1
= ∫ 𝑑𝑥
2 1 + 𝑥4
1 𝑥2 + 1 1 𝑥2 − 1
= ∫ 𝑑𝑥 + ∫ 𝑑𝑥
2 1 + 𝑥4 2 1 + 𝑥4
Dividing by 𝑥 2 /𝑥 2 , we get
87
1 𝑥2 + 1 1 𝑥2 − 1 1 1 + 1/ 𝑥 2 1 1 − 1/𝑥 2
∫ 𝑑𝑥 + ∫ 𝑑𝑥 = ∫ 𝑑𝑥 + ∫ 𝑑𝑥
2 1 + 𝑥4 2 1 + 𝑥4 2 1/𝑥 2 + 𝑥 2 2 1/𝑥 2 + 𝑥 2
If we let
1
𝑢=𝑥−
𝑥
1
𝑣=𝑥+
𝑥
then
1
𝑑𝑢 = (1 + ) 𝑑𝑥
𝑥2
12
𝑑𝑣 = (1 − ) 𝑑𝑥
𝑥
and
1 2 1
𝑢2 = (𝑥 − ) = 𝑥 2 − 2 + 2
𝑥 𝑥
1 2 1
𝑣 = (𝑥 + ) = 𝑥 2 + 2 + 2
2
𝑥 𝑥
1 𝑥2 + 1 1 𝑥2 − 1 1 𝑑𝑢 1 𝑑𝑣
∫ 𝑑𝑥 + ∫ 𝑑𝑥 = ∫ + ∫
2 1 + 𝑥4 2 1 + 𝑥4 2 𝑢2 + 2 2 𝑣 2 − 2
1 𝑢 1 𝑣
= 𝑡𝑎𝑛−1 − 𝑡𝑎𝑛ℎ−1 +𝐶
2√2 √2 2√2 √2
1 𝑥 − 1/𝑥 1 𝑥 + 1/𝑥
= 𝑡𝑎𝑛−1 ( )− 𝑡𝑎𝑛ℎ−1 ( )+𝐶
2√2 √2 2√2 √2
1 𝑥2 − 1 𝑥2 + 1
= [𝑡𝑎𝑛−1 ( ) − 𝑡𝑎𝑛ℎ−1 ( )]
2√2 √2 𝑥 √2 𝑥
+𝐶 ANS.
1 −1
𝑥2 − 1 −1
𝑥2 + 1
[−𝑐𝑜𝑡 ( ) − 𝑐𝑜𝑡ℎ ( )] + 𝐶
2√2 √2 𝑥 √2 𝑥
88
Example 6.12.
Evaluate
1
∫ 𝑑𝑥
1 + 𝑠𝑖𝑛ℎ 𝑥
1 1
∫ 𝑑𝑥 = ∫ 1 𝑑𝑥
1 + 𝑠𝑖𝑛ℎ 𝑥 1 + 2(𝑒 𝑥 − 𝑒 −𝑥 )
2𝑒 𝑥
=∫ 𝑥 𝑑𝑥
2𝑒 + 𝑒 2𝑥 − 1
1 2𝑒 𝑥
∫ 𝑑𝑥 = ∫ 2𝑥 𝑑𝑥
1 + 𝑠𝑖𝑛ℎ 𝑥 (𝑒 + 2𝑥 + 1) − 2
2𝑒 𝑥
=∫ 𝑥 𝑑𝑥
(𝑒 + 1)2 − 2
1 𝑑𝑢 𝑑𝑢
∫ 𝑑𝑥 = 2 ∫ 2 = −2 ∫
1 + 𝑠𝑖𝑛ℎ 𝑥 𝑢 −2 2 − 𝑢2
2 𝑢
− 𝑡𝑎𝑛ℎ−1 +𝐶
= √2 √2
2 −1
𝑢
− 𝑐𝑜𝑡ℎ +𝐶
{ √2 √2
−1
𝑒𝑥 + 1
−√2 𝑡𝑎𝑛ℎ +𝐶
= √2 ANS.
−1
𝑒𝑥 + 1
−√2 𝑐𝑜𝑡ℎ +𝐶
{ √2
𝑒 𝑥 𝑠𝑖𝑛ℎ 𝑥 + 𝑐𝑜𝑠ℎ 𝑥
=
𝑒 𝑥 𝑠𝑖𝑛ℎ 𝑥 + 𝑐𝑜𝑠ℎ 𝑥
Thus, we have
89
1 𝑠𝑖𝑛ℎ 𝑥 + 𝑐𝑜𝑠ℎ 𝑥 𝑠𝑖𝑛ℎ 𝑥 + 𝑐𝑜𝑠ℎ 𝑥
∫ ∙ 𝑑𝑥 = ∫ 𝑑𝑥
1 + 𝑠𝑖𝑛ℎ 𝑥 𝑠𝑖𝑛ℎ 𝑥 + 𝑐𝑜𝑠ℎ 𝑥 (𝑠𝑖𝑛ℎ 𝑥 + 𝑐𝑜𝑠ℎ 𝑥) + 𝑠𝑖𝑛ℎ 𝑥 (𝑠𝑖𝑛ℎ 𝑥 + 𝑐𝑜𝑠ℎ 𝑥)
𝑠𝑖𝑛ℎ 𝑥 + 𝑐𝑜𝑠ℎ 𝑥
=∫ 𝑑𝑥
(𝑠𝑖𝑛ℎ 𝑥 + 𝑐𝑜𝑠ℎ 𝑥) + 𝑠𝑖𝑛ℎ2 𝑥 + 𝑠𝑖𝑛ℎ 𝑥 𝑐𝑜𝑠ℎ 𝑥
Multiplying by 2/2, and splitting 2 𝑠𝑖𝑛ℎ2 𝑥 into 𝑠𝑖𝑛ℎ2 𝑥 + 𝑐𝑜𝑠ℎ2 𝑥 − 1 plus a little
rearranging, we get
𝑠𝑖𝑛ℎ 𝑥 + 𝑐𝑜𝑠ℎ 𝑥
= 2∫ 𝑑𝑥
2(𝑠𝑖𝑛ℎ 𝑥 + 𝑐𝑜𝑠ℎ 𝑥) + 2 𝑠𝑖𝑛ℎ2 𝑥 + 2 𝑠𝑖𝑛ℎ 𝑥 𝑐𝑜𝑠ℎ 𝑥
𝑠𝑖𝑛ℎ 𝑥 + 𝑐𝑜𝑠ℎ 𝑥
= 2∫ 𝑑𝑥
2(𝑠𝑖𝑛ℎ 𝑥 + 𝑐𝑜𝑠ℎ 𝑥) + (𝑠𝑖𝑛ℎ2 𝑥 + 𝑐𝑜𝑠ℎ2 𝑥 − 1) + 2 𝑠𝑖𝑛ℎ 𝑥 𝑐𝑜𝑠ℎ 𝑥
𝑠𝑖𝑛ℎ 𝑥 + 𝑐𝑜𝑠ℎ 𝑥
= 2∫ 𝑑𝑥
2(𝑠𝑖𝑛ℎ 𝑥 + 𝑐𝑜𝑠ℎ 𝑥) + (𝑠𝑖𝑛ℎ2 𝑥 + 2 𝑠𝑖𝑛ℎ 𝑥 𝑐𝑜𝑠ℎ 𝑥 + 𝑐𝑜𝑠ℎ2 𝑥) − 1
𝑠𝑖𝑛ℎ 𝑥 + 𝑐𝑜𝑠ℎ 𝑥
= 2∫ 𝑑𝑥
2(𝑠𝑖𝑛ℎ 𝑥 + 𝑐𝑜𝑠ℎ 𝑥) + (𝑠𝑖𝑛ℎ 𝑥 + 𝑐𝑜𝑠ℎ 𝑥)2 − 1
𝑠𝑖𝑛ℎ 𝑥 + 𝑐𝑜𝑠ℎ 𝑥
= 2∫ 𝑑𝑥
(𝑠𝑖𝑛ℎ 𝑥 + 𝑐𝑜𝑠ℎ 𝑥)2 + 2(𝑠𝑖𝑛ℎ 𝑥 + 𝑐𝑜𝑠ℎ 𝑥) + 1 − 2
𝑠𝑖𝑛ℎ 𝑥 + 𝑐𝑜𝑠ℎ 𝑥
= 2∫ 𝑑𝑥
(𝑠𝑖𝑛ℎ 𝑥 + 𝑐𝑜𝑠ℎ 𝑥 + 1)2 − 2
1 𝑑𝑢 𝑑𝑢
∫ 𝑑𝑥 = 2 ∫ 2 = −2 ∫
1 + 𝑠𝑖𝑛ℎ 𝑥 𝑢 −2 2 − 𝑢2
2 𝑢
− 𝑡𝑎𝑛ℎ−1 +𝐶
= √2 √2
2 −1
𝑢
− 𝑐𝑜𝑡ℎ +𝐶
{ √2 √2
𝑠𝑖𝑛ℎ 𝑥 + 𝑐𝑜𝑠ℎ 𝑥 + 1
−√2 𝑡𝑎𝑛ℎ−1 +𝐶
= √2 ANS.
−1
𝑠𝑖𝑛ℎ 𝑥 + 𝑐𝑜𝑠ℎ 𝑥 + 1
−√2 𝑐𝑜𝑡ℎ +𝐶
{ √2
_____________________________________________________________________________________________________
90
Example 6.13.
Show that
1 𝑠𝑖𝑛ℎ 𝑥 𝑠𝑖𝑛ℎ 𝑥
∫ 𝑑𝑥 = ∫ 2
𝑑𝑥 = ∫ 𝑑𝑥
𝑠𝑖𝑛ℎ 𝑥 𝑠𝑖𝑛ℎ 𝑥 𝑐𝑜𝑠ℎ2 𝑥 − 1
𝑠𝑖𝑛ℎ 𝑥 𝑑𝑢 𝑑𝑢
∫ 2
𝑑𝑥 = ∫ 2 = −∫
𝑐𝑜𝑠ℎ 𝑥 − 1 𝑢 −1 1 − 𝑢2
Finally,
since 𝑐𝑜𝑠ℎ 𝑥 ≥ 1. It will just be left as an exercise to show that the answer is equivalent
to Eq. 5.10. It is also left as an exercise to show the following equivalent forms:
−𝑐𝑠𝑐ℎ−1 (𝑠𝑖𝑛ℎ 𝑥) + 𝐶
∫ 𝑐𝑠𝑐ℎ 𝑥 𝑑𝑥 = {−𝑠𝑒𝑐ℎ−1 (𝑡𝑎𝑛ℎ 𝑥) + 𝐶
−𝑠𝑖𝑛ℎ−1 (𝑐𝑠𝑐ℎ 𝑥) + 𝐶
Using parallel arguments, you can also try proving the following relations:
𝑠𝑖𝑛−1 (𝑡𝑎𝑛ℎ 𝑥) + 𝐶
𝑐𝑜𝑠 −1 (𝑠𝑒𝑐ℎ 𝑥) + 𝐶
∫ 𝑠𝑒𝑐ℎ 𝑥 𝑑𝑥 =
𝑡𝑎𝑛−1 (𝑠𝑖𝑛ℎ 𝑥) + 𝐶
{𝑠𝑒𝑐 −1 (𝑐𝑜𝑠ℎ 𝑥) + 𝐶
_____________________________________________________________________________________________________
Example 6.14.
Show that
91
𝑠𝑒𝑐 2 𝑥 𝑠𝑒𝑐 2 𝑥
∫ 𝑑𝑥 = ∫ 𝑑𝑥
𝑠𝑒𝑐 𝑥 √𝑡𝑎𝑛2 𝑥 + 1
𝑠𝑒𝑐 2 𝑥 𝑑𝑢
∫ 𝑑𝑥 = ∫ = 𝑠𝑖𝑛ℎ−1 𝑢 + 𝐶
√𝑡𝑎𝑛2 𝑥+1 √𝑢2 +1
Finally,
It will just be left as an exercise to show that the answer is equivalent to Eq. 2.9. It is also
left as an exercise to show the following equivalent forms:
𝑡𝑎𝑛ℎ−1 (𝑠𝑖𝑛 𝑥) + 𝐶
∫ 𝑠𝑒𝑐 𝑥 𝑑𝑥 = {𝑠𝑒𝑐ℎ−1 (𝑐𝑜𝑠 𝑥) + 𝐶
𝑐𝑜𝑠ℎ−1 (𝑠𝑒𝑐 𝑥) + 𝐶
Using parallel arguments, you can also try proving the following relations:
−𝑠𝑖𝑛ℎ−1 (𝑐𝑜𝑡 𝑥) + 𝐶
−𝑡𝑎𝑛ℎ−1 (𝑐𝑜𝑠 𝑥) + 𝐶
∫ 𝑐𝑠𝑐 𝑥 𝑑𝑥 =
−𝑠𝑒𝑐ℎ−1 (𝑠𝑖𝑛 𝑥) + 𝐶
{ −𝑐𝑜𝑠ℎ−1 (𝑐𝑠𝑐 𝑥) + 𝐶
_____________________________________________________________________________________________________
92
Try these.
1
1. ∫ 𝑑𝑥
4𝑥 2−9
1
2. ∫ 𝑑𝑥
√𝑥 2 − 4𝑥 + 3
𝑠𝑖𝑛 𝑥
3. ∫ 𝑑𝑥
√10 − 𝑠𝑖𝑛2 𝑥
𝑡𝑎𝑛ℎ 𝑦
∫ 𝑑𝑦
4. √𝑡𝑎𝑛ℎ2 𝑦 + 3
√𝑒 𝑦
5. ∫ 𝑑𝑦
√𝑒 𝑦 − 1
1
6. ∫ 𝑑𝑟
𝑟√1 − 𝑟
𝑥
7. ∫ 𝑑𝑥
√𝑥 4 + 9𝑥
93
7. Integration by Parts
There are many instances when the integral cannot be easily evaluated using the
substitution rule. In those instances, we resort to various techniques such as integration
by parts.
𝑑(𝑢𝑣) = 𝑢 𝑑𝑣 + 𝑣 𝑑𝑢
𝑢 𝑑𝑣 = 𝑑(𝑢𝑣) − 𝑣 𝑑𝑢
Integrating,
∫ 𝑢 𝑑𝑣 = ∫ 𝑑(𝑢𝑣) − ∫ 𝑣 𝑑𝑢
∫ 𝑢 𝑑𝑣 = 𝑢𝑣 − ∫ 𝑣 𝑑𝑢 7.1
94
Example 7.1.
Evaluate
∫ 𝑥 𝑠𝑖𝑛 𝑥 𝑑𝑥
𝑑𝑢 = 𝑑𝑥
𝑣 = ∫ 𝑠𝑖𝑛 𝑥 𝑑𝑥 = −𝑐𝑜𝑠 𝑥
Hence,
∫ 𝑥 𝑠𝑖𝑛 𝑥 𝑑𝑥 = 𝑢𝑣 − ∫ 𝑣 𝑑𝑢
= −𝑥 𝑐𝑜𝑠 𝑥 − ∫ −𝑐𝑜𝑠 𝑥 𝑑𝑥
The choice of 𝑢 and 𝑑𝑣 depends on the integrand. Remember that the goal in
substituting 𝑢 and 𝑑𝑣 is to simplify the integrand into something that can easily be
integrated. When the resulting integrand is more complicated, the choice must be
revised.
∫ 𝑥 𝑠𝑖𝑛 𝑥 𝑑𝑥 = 𝑢𝑣 − ∫ 𝑣 𝑑𝑢
1 1
= 𝑥 2 𝑠𝑖𝑛 𝑥 − ∫ 𝑥 2 𝑐𝑜𝑠 𝑥 𝑑𝑥
2 2
which is obviously more difficult to solve. Thus, care must be taken in deciding the proper
substitution. Usually, the choice for 𝑑𝑣 matters more because there is a need to integrate.
But in some instances, there is only one choice as the next example illustrates.
_____________________________________________________________________________________________________
Example 7.2.
Evaluate
∫ 𝑙𝑛 𝑦 𝑑𝑦
95
Solution. Using integration by parts, if we will let 𝑢 = 𝑙𝑛 𝑦 and 𝑑𝑣 = 𝑑𝑦, then
1
𝑑𝑢 = 𝑑𝑦
𝑦
𝑣 = ∫ 𝑑𝑦 = 𝑦
Hence,
∫ 𝑙𝑛 𝑦 𝑑𝑦 = 𝑢𝑣 − ∫ 𝑣 𝑑𝑢
1
= 𝑦 𝑙𝑛 𝑦 − ∫ 𝑦 ∙ 𝑑𝑦
𝑦
∫ 𝑙𝑛 𝑦 𝑑𝑦 = 𝑦 𝑙𝑛 𝑦 − 𝑦 + 𝐶 ANS.
In few instances, there are options in choosing the “parts”. The next example
illustrates this.
____________________________________________________________________________________________________
Example 7.3.
Evaluate
∫ 𝑦 𝑙𝑛 𝑦 𝑑𝑦
∫ 𝑦 𝑙𝑛 𝑦 𝑑𝑦 = 𝑦(𝑦 𝑙𝑛 𝑦 − 𝑦) − ∫(𝑦 𝑙𝑛 𝑦 − 𝑦) 𝑑𝑦
= 𝑦 2 𝑙𝑛 𝑦 − 𝑦 2 − ∫ 𝑦 𝑙𝑛 𝑦 𝑑𝑦 + ∫ 𝑦 𝑑𝑦
1
2 ∫ 𝑦 𝑙𝑛 𝑦 𝑑𝑦 = 𝑦 2 𝑙𝑛 𝑦 − 𝑦 2 + 𝑦 2
2
1 2 1
∫ 𝑦 𝑙𝑛 𝑦 𝑑𝑦 = 𝑦 𝑙𝑛 𝑦 − 𝑦 2 + 𝐶 ANS.
2 4
96
1 2 1 1
∫ 𝑦 𝑙𝑛 𝑦 𝑑𝑦 = 𝑦 𝑙𝑛 𝑦 − ∫ 𝑦 2 ∙ 𝑑𝑦
2 2 𝑦
1 1
= 𝑦 2 𝑙𝑛 𝑦 − ∫ 𝑦 𝑑𝑦
2 2
1 1
= 𝑦 2 𝑙𝑛 𝑦 − 𝑦 2 + 𝐶 ANS.
2 4
There are instances when the technique of integration by parts needs to be
applied successively to reduce the integrand into something that can readily be
integrated as in the next example.
_____________________________________________________________________________________________________
Example 7.4.
Evaluate
∫ 𝑥 2 𝑒 𝑥 𝑑𝑥
𝑑𝑢 = 2𝑥 𝑑𝑥
𝑣 = ∫ 𝑒 𝑥 𝑑𝑥 = 𝑒 𝑥
Hence,
∫ 𝑥 2 𝑒 𝑥 𝑑𝑥 = 𝑢𝑣 − ∫ 𝑣 𝑑𝑢
= 𝑥 2 𝑒 𝑥 − ∫ 2𝑥𝑒 𝑥 𝑑𝑥
Evaluating the integrand on the right requires integration by parts again so we let 𝑢 =
2𝑥 and 𝑑𝑣 = 𝑒 𝑥 𝑑𝑥. This means that 𝑑𝑢 = 2 𝑑𝑥 and 𝑣 = 𝑒 𝑥 . Hence,
∫ 𝑥 2 𝑒 𝑥 𝑑𝑥 = 𝑥 2 𝑒 𝑥 − ∫ 2𝑥𝑒 𝑥 𝑑𝑥
= 𝑥 2 𝑒 𝑥 − (2𝑥𝑒 𝑥 − ∫ 2𝑒 𝑥 𝑑𝑥)
= 𝑥 2 𝑒 𝑥 − (2𝑥𝑒 𝑥 − 2𝑒 𝑥 ) + 𝐶
97
∫ 𝑥 2 𝑒 𝑥 𝑑𝑥 = 𝑥 2 𝑒 𝑥 − 2𝑥𝑒 𝑥 + 2𝑒 𝑥 + 𝐶 ANS.
Example 7.5.
Evaluate
∫ 𝑒 𝑦 𝑠𝑖𝑛 𝑦 𝑑𝑦
𝑑𝑢 = 𝑒 𝑦 𝑑𝑦
𝑣 = ∫ 𝑠𝑖𝑛 𝑦 𝑑𝑦 = −𝑐𝑜𝑠 𝑦
Hence,
∫ 𝑒 𝑦 𝑠𝑖𝑛 𝑦 𝑑𝑦 = 𝑢𝑣 − ∫ 𝑣 𝑑𝑢
= −𝑒 𝑦 𝑐𝑜𝑠 𝑦 − ∫ −𝑒 𝑦 𝑐𝑜𝑠 𝑦 𝑑𝑦
= −𝑒 𝑦 𝑐𝑜𝑠 𝑦 + ∫ 𝑒 𝑦 𝑐𝑜𝑠 𝑦 𝑑𝑦
The integral on the right needs to be evaluated using integration by parts. If we let 𝑢 =
𝑒 𝑦 and 𝑑𝑣 = 𝑐𝑜𝑠 𝑦 𝑑𝑦, then 𝑑𝑢 = 𝑒 𝑦 𝑑𝑦 and 𝑣 = 𝑠𝑖𝑛 𝑦. Hence,
∫ 𝑒 𝑦 𝑠𝑖𝑛 𝑦 𝑑𝑦 = −𝑒 𝑦 𝑐𝑜𝑠 𝑦 + ∫ 𝑒 𝑦 𝑐𝑜𝑠 𝑦 𝑑𝑦
98
In some instances, substitution for 𝑢 and 𝑑𝑣 cannot be done immediately. There
is a need to reduce the integrand into a form where we can use the technique of
integration by parts. The following example illustrates this.
_____________________________________________________________________________________________________
Example 7.6.
Evaluate
∫ 𝑒 √𝑥 𝑑𝑥
Solution. Before we use the substitution for 𝑢 and 𝑑𝑣, we need to get past the radical first
which complicates our integrand. An appropriate substitution is to let 𝑡 = √𝑥 so that
1 1 1 1
𝑑𝑡 = ∙ 𝑑𝑥 = ∙ 𝑑𝑥
2 √𝑥 2 𝑡
∫ 𝑒 √𝑥 𝑑𝑥 = ∫ 2𝑡𝑒 𝑡 𝑑𝑡
= 2𝑡𝑒 𝑡 − ∫ 2𝑒 𝑡 𝑑𝑡
= 2𝑡𝑒 𝑡 − 2𝑒 𝑡 + 𝐶
∫ 𝑒 √𝑥 𝑑𝑥 = 2√𝑥𝑒 √𝑥 − 2𝑒 √𝑥 + 𝐶 ANS.
_____________________________________________________________________________________________________
Example 7.7.
Evaluate
∫ 𝑐𝑜𝑠 (𝑙𝑛 𝑥) 𝑑𝑥
99
Solution: We need to first make substitution for 𝑙𝑛 𝑥 to simplify the integrand. If we let
𝑡 = 𝑙𝑛 𝑥, then 𝑥 = 𝑒 𝑡 , 𝑑𝑡 = 1/𝑥 𝑑𝑥, and 𝑑𝑥 = 𝑒 𝑡 𝑑𝑡. Thus,
= 𝑒 𝑡 𝑠𝑖𝑛 𝑡 − ∫ 𝑒 𝑡 𝑠𝑖𝑛 𝑡 𝑑𝑡
Using successive integration by parts on the integrand to the right yields
∫ 𝑒 𝑡 𝑐𝑜𝑠 𝑡 𝑑𝑡 = 𝑒 𝑡 𝑠𝑖𝑛 𝑡 − (−𝑒 𝑡 𝑐𝑜𝑠 𝑡 − ∫ −𝑒 𝑡 𝑐𝑜𝑠 𝑡 𝑑𝑡)
Example 7.8.
Evaluate
∫ 𝑠𝑖𝑛 5𝑥 𝑐𝑜𝑠 𝑥 𝑑𝑥
Solution. We encountered this integral in Example 2.7. We shall now present a different
approach in evaluating this integral using integration by parts. If we let 𝑢 = 𝑠𝑖𝑛 5𝑥,
𝑑𝑢 = 5 𝑐𝑜𝑠 5𝑥 𝑑𝑥. On the other hand, if we let 𝑑𝑣 = 𝑐𝑜𝑠 𝑥 𝑑𝑥, 𝑣 = 𝑠𝑖𝑛 𝑥. Consequently,
100
∫ 𝑠𝑖𝑛 5𝑥 𝑐𝑜𝑠 𝑥 𝑑𝑥 = 𝑢𝑣 − ∫ 𝑣 𝑑𝑢
To solve the integral on the right, we will let 𝑢 = 𝑐𝑜𝑠 5𝑥, so 𝑑𝑢 = −5 𝑠𝑖𝑛 5𝑥 𝑑𝑥. If we let
𝑑𝑣 = 𝑠𝑖𝑛 𝑥 𝑑𝑥, 𝑣 = −𝑐𝑜𝑠 𝑥. Consequently,
∫ 𝑐𝑜𝑠 5𝑥 𝑠𝑖𝑛 𝑥 𝑑𝑥 = 𝑢𝑣 − ∫ 𝑣 𝑑𝑢
Substituting, we get
Finally,
1 5
∫ 𝑠𝑖𝑛 5𝑥 𝑐𝑜𝑠 𝑥 𝑑𝑥 = − 𝑠𝑖𝑛 5𝑥 𝑠𝑖𝑛 𝑥 − 𝑐𝑜𝑠 5𝑥 𝑐𝑜𝑠 𝑥 + 𝐶 ANS.
24 24
It is left as an exercise to show that this answer is equivalent to the answer in Example
2.7.
Integration by parts sometimes requires that we break one “part” into “sub-parts”
as we shall illustrate below.
_____________________________________________________________________________________________________
Example 7.9.
Evaluate
𝑤 3 𝑑𝑤
∫
√𝑤 2 + 9
101
Solution. We encountered this in Example 1.10. In this example, we will explore
integration of parts to evaluate the integral. For our purpose, we can rewrite the
integrand as follows:
1 2𝑤 𝑑𝑤
∫ 𝑤2 ∙
2 √𝑤 2 + 9
1 2𝑤 𝑑𝑤
𝑑𝑣 =
2 √𝑤 2 + 9
Then
1 2𝑤 𝑑𝑤
𝑣= ∫
2 √𝑤 2 + 9
1 𝑑𝑡 1
𝑣= ∫ = ∙ 2 √𝑡
2 √𝑡 2
𝑣 = √𝑤 2 + 9
𝑤 3 𝑑𝑤
∫ = 𝑢𝑣 − ∫ 𝑣 𝑑𝑢
√𝑤 2 + 9
= 𝑤 2 √𝑤 2 + 9 − ∫ 2𝑤 √𝑤 2 + 9 𝑑𝑤
2
= 𝑤 2 √𝑤 2 + 9 − (𝑤 2 + 9)3⁄2 + 𝐶 ANS.
3
_____________________________________________________________________________________________________
Example 7.10.
Evaluate
2𝑡 2 − 1
∫ 𝑑𝑡
√𝑡 2 − 1
102
Simplifying the integrand, we have
2𝑡 2 − 1 𝑡2 − 1 𝑡2
∫ 𝑑𝑡 = ∫ ( + ) 𝑑𝑡
√𝑡 2 − 1 √𝑡 2 − 1 √𝑡 2 − 1
𝑡2
= ∫ (√𝑡 2 − 1 + ) 𝑑𝑡
√𝑡 2 − 1
Hence,
𝑡2
∫ (√𝑡 2 − 1 + ) 𝑑𝑡 = ∫(𝑢 𝑑𝑣 + 𝑣 𝑑𝑢)
√𝑡 2 − 1
= ∫ 𝑑(𝑢𝑣) = 𝑢𝑣 + 𝐶
= 𝑡√𝑡 2 − 1 + 𝐶 ANS.
_____________________________________________________________________________________________________
Example 7.11.
Evaluate
∫ 𝑠𝑒𝑐 3 𝑟 𝑑𝑟
∫ 𝑠𝑒𝑐 3 𝑟 𝑑𝑟 = 𝑢𝑣 − ∫ 𝑣 𝑑𝑢
103
2 ∫ 𝑠𝑒𝑐 3 𝑟 𝑑𝑟 = 𝑠𝑒𝑐 𝑟 𝑡𝑎𝑛 𝑟 + ∫ 𝑠𝑒𝑐 𝑟 𝑑𝑟
1 1
∫ 𝑠𝑒𝑐 3 𝑟 𝑑𝑟 = 𝑠𝑒𝑐 𝑟 𝑡𝑎𝑛 𝑟 + 𝑙𝑛 |𝑠𝑒𝑐 𝑟 + 𝑡𝑎𝑛 𝑟| + 𝐶 ANS.
2 2
_____________________________________________________________________________________________________
Example 7.12.
Evaluate
∫ 𝑡𝑎𝑛ℎ5 𝑥 𝑑𝑥
Solution. Remember that this integral has already been solved in Example 5.5. using two
approaches. This new example presents the third approach using integration by parts.
5
𝑠𝑖𝑛ℎ4 𝑥 ∙ 𝑠𝑖𝑛ℎ 𝑥
∫ 𝑡𝑎𝑛ℎ 𝑥 𝑑𝑥 = ∫ 𝑑𝑥
𝑐𝑜𝑠ℎ5 𝑥
𝑠𝑖𝑛ℎ 𝑥 𝑑𝑡
𝑑𝑣 = 𝑑𝑥 =
𝑐𝑜𝑠ℎ5 𝑥 𝑡5
𝑠𝑖𝑛ℎ 𝑥 𝑑𝑡
𝑣=∫ 𝑑𝑥 = ∫
𝑐𝑜𝑠ℎ5 𝑥 𝑡5
1 1
=− = −
4𝑡 4 4 𝑐𝑜𝑠ℎ4 𝑥
Thus,
∫ 𝑡𝑎𝑛ℎ5 𝑥 𝑑𝑥 = 𝑢𝑣 − ∫ 𝑣 𝑑𝑢
1 𝑠𝑖𝑛ℎ4 𝑥 1
=− 4
− ∫− ∙ 4 𝑠𝑖𝑛ℎ3 𝑥 𝑐𝑜𝑠ℎ 𝑥 𝑑𝑥
4 𝑐𝑜𝑠ℎ 𝑥 4 𝑐𝑜𝑠ℎ4 𝑥
1 𝑠𝑖𝑛ℎ2 𝑥
= − 𝑡𝑎𝑛ℎ4 𝑥 + ∫ ∙ 𝑠𝑖𝑛ℎ 𝑥 𝑑𝑥
4 𝑐𝑜𝑠ℎ3 𝑥
104
To evaluate the integral on the left, we use integration by parts once again. Letting 𝑢 =
𝑠𝑖𝑛ℎ2 𝑥, 𝑑𝑢 = 2 𝑠𝑖𝑛ℎ 𝑥 𝑐𝑜𝑠ℎ 𝑥 𝑑𝑥. Letting
𝑠𝑖𝑛ℎ 𝑥 𝑑𝑡
𝑑𝑣 = 3
𝑑𝑥 = 3
𝑐𝑜𝑠ℎ 𝑥 𝑡
1 1
𝑣=− = −
2𝑡 2 2 𝑐𝑜𝑠ℎ2 𝑥
Thus,
𝑠𝑖𝑛ℎ2 𝑥
∫ ∙ 𝑠𝑖𝑛ℎ 𝑥 𝑑𝑥 = 𝑢𝑣 − ∫ 𝑣 𝑑𝑢
𝑐𝑜𝑠ℎ3 𝑥
1 𝑠𝑖𝑛ℎ2 𝑥 1
=− 2
− ∫− ∙ 2 𝑠𝑖𝑛ℎ 𝑥 𝑐𝑜𝑠ℎ 𝑥 𝑑𝑥
2 𝑐𝑜𝑠ℎ 𝑥 2 𝑐𝑜𝑠ℎ2 𝑥
1
= − 𝑡𝑎𝑛ℎ2 𝑥 + ∫ 𝑡𝑎𝑛ℎ 𝑥 𝑑𝑥
2
1
= − 𝑡𝑎𝑛ℎ2 𝑥 + 𝑙𝑛 (𝑐𝑜𝑠ℎ 𝑥) + 𝐶
2
Finally,
1 1
∫ 𝑡𝑎𝑛ℎ5 𝑥 𝑑𝑥 = − 𝑡𝑎𝑛ℎ4 𝑥 − 𝑡𝑎𝑛ℎ2 𝑥 + 𝑙𝑛 (𝑐𝑜𝑠ℎ 𝑥) + 𝐶 ANS.
4 2
_____________________________________________________________________________________________________
Example 7.13.
Evaluate
1 − 𝑙𝑛 𝑥 2
∫( ) 𝑑𝑥
𝑥
1 − 𝑙𝑛 𝑥 2 1 2 𝑙𝑛 𝑥 𝑙𝑛2 𝑥
∫( ) 𝑑𝑥 = ∫ 2 𝑑𝑥 − ∫ 2 𝑑𝑥 + ∫ 2 𝑑𝑥
𝑥 𝑥 𝑥 𝑥
Both second and third integral on the right can be solved rather tediously. To solve the
third, let us use integration by parts technique by letting
105
1
𝑢 = 𝑙𝑛2 𝑥 and 𝑑𝑣 = 𝑑𝑥
𝑥2
This means that
2 𝑙𝑛 𝑥 1
𝑑𝑢 = 𝑑𝑥 and 𝑣 = −
𝑥 𝑥
Thus,
𝑙𝑛2 𝑥
∫ 𝑑𝑥 = 𝑢𝑣 − ∫ 𝑣 𝑑𝑢
𝑥2
𝑙𝑛2 𝑥 2 𝑙𝑛 𝑥
=− + ∫ 2 𝑑𝑥
𝑥 𝑥
We stop here from further integrating. Instead, we substitute this result to the expanded
integral above, thus,
1 − 𝑙𝑛 𝑥 2 1 2 𝑙𝑛 𝑥 𝑙𝑛2 𝑥 2 𝑙𝑛 𝑥
∫( ) 𝑑𝑥 = ∫ 2 𝑑𝑥 − ∫ 2 𝑑𝑥 + (− + ∫ 2 𝑑𝑥)
𝑥 𝑥 𝑥 𝑥 𝑥
Note that we can cancel the second and the last integral. The result is
1 − 𝑙𝑛 𝑥 2 1 𝑙𝑛2 𝑥
∫( ) 𝑑𝑥 = ∫ 2 𝑑𝑥 −
𝑥 𝑥 𝑥
Finally,
1 − 𝑙𝑛 𝑥 2 1 𝑙𝑛2 𝑥
∫( ) 𝑑𝑥 = − − +𝐶
𝑥 𝑥 𝑥
1 + 𝑙𝑛2 𝑥
=− +𝐶 ANS.
𝑥
_____________________________________________________________________________________________________
Example 7.14.
Evaluate
Solution. This question appeared in the 2006 MIT Integration Bee. To solve it, we can let
𝑢 = 𝑠𝑖𝑛99 𝑥 and 𝑑𝑣 = 𝑠𝑖𝑛 100𝑥 𝑑𝑥 and use integration of parts repeatedly 98 times until
we get to 𝑢 = 𝑠𝑖𝑛 𝑥. We note, however, that
106
𝑠𝑖𝑛 101𝑥 = 𝑠𝑖𝑛 (𝑥 + 100𝑥)
Consequently,
∫ 𝑠𝑖𝑛 101𝑥 ∙ 𝑠𝑖𝑛99 𝑥 𝑑𝑥 = ∫(𝑠𝑖𝑛 𝑥 𝑐𝑜𝑠 100𝑥 + 𝑐𝑜𝑠 𝑥 𝑠𝑖𝑛 100𝑥) 𝑠𝑖𝑛99 𝑥 𝑑𝑥
To evaluate the first integral, we let 𝑢 = 𝑠𝑖𝑛100 𝑥 and 𝑑𝑣 = 𝑐𝑜𝑠 100𝑥 𝑑𝑥. This means that
𝑑𝑢 = 100 𝑠𝑖𝑛99 𝑥 𝑐𝑜𝑠 𝑥 𝑑𝑥 and 𝑣 = 𝑠𝑖𝑛 100𝑥/100. The integral, thus, becomes
Notice that the second term is the negative of the second integral appearing above. For
this reason,
Example 7.15.
Evaluate
∫ 𝑙𝑛 |𝑥 + √𝑥 2 + 1| 𝑑𝑥
107
Solution. If we let 𝑢 = 𝑙𝑛 |𝑥 + √𝑥 2 + 1| and 𝑑𝑣 = 𝑑𝑥, then 𝑣 = 𝑥 and
1 2𝑥
𝑑𝑢 = (1 + ) 𝑑𝑥
𝑥 + √𝑥 2 + 1 2√𝑥 2 + 1
1 𝑥 + √𝑥 2 + 1
=
𝑥 + √𝑥 2 + 1 √𝑥 2 + 1
1
=
√𝑥 2 + 1
∫ 𝑙𝑛 |𝑥 + √𝑥 2 + 1| 𝑑𝑥 = 𝑢𝑣 − ∫ 𝑣 𝑑𝑢
𝑥
= 𝑥 𝑙𝑛 |𝑥 + √𝑥 2 + 1| − ∫ 𝑑𝑥
√𝑥 2 + 1
= 𝑥 𝑙𝑛 |𝑥 + √𝑥 2 + 1| − √𝑥 2 + 1 + 𝐶 ANS.
Later in this chapter, you will recognize this as the integral of 𝑠𝑖𝑛ℎ−1 𝑥.
_____________________________________________________________________________________________________
Example 7.16.
Evaluate
𝑥 𝑐𝑜𝑠 𝑥 + 2 𝑠𝑖𝑛 𝑥
∫ 𝑑𝑥
√𝑠𝑖𝑛 𝑥
Solution. This is another interesting application similar to Example 7.12. Rewriting the
integral,
108
𝑑𝑡
𝑑𝑢 = 𝑑𝑥 and 𝑣 = ∫ = 2 √𝑠𝑖𝑛 𝑥
√𝑡
= 2𝑥 √𝑠𝑖𝑛 𝑥 − ∫ 2 √𝑠𝑖𝑛 𝑥 𝑑𝑥 + 𝐶
Once again, we can stop here. The integral on the right is difficult to evaluate because it
cannot be expressed in terms of elementary functions. Nevertheless, if we use this result
to evaluate the original integral, we get
𝑥 𝑐𝑜𝑠 𝑥 − 2 𝑠𝑖𝑛 𝑥
∫ 𝑑𝑥 = 2𝑥 √𝑠𝑖𝑛 𝑥 + ∫ 2 √𝑠𝑖𝑛 𝑥 𝑑𝑥 − ∫ 2 √𝑠𝑖𝑛 𝑥 𝑑𝑥 + 𝐶
√𝑠𝑖𝑛 𝑥
= 2𝑥 √𝑠𝑖𝑛 𝑥 + 𝐶 ANS.
Example 7.17.
Evaluate
∫ 𝑠𝑖𝑛−1 𝑦 𝑑𝑦
1
𝑑𝑢 = 𝑑𝑦
√1 − 𝑦 2
∫ 𝑠𝑖𝑛−1 𝑦 𝑑𝑦 = 𝑢𝑣 − ∫ 𝑣 𝑑𝑢
𝑦
= 𝑦 𝑠𝑖𝑛−1 𝑦 − ∫ 𝑑𝑦
√1 − 𝑦 2
109
1 −2𝑦
= 𝑦 𝑠𝑖𝑛−1 𝑦 + ∫ 𝑑𝑦
2 √1 − 𝑦 2
We can now make use of ordinary substitution to evaluate the integral on the right
(noting that the numerator is the derivative of the term inside the square root), thus,
1
∫ 𝑡𝑎𝑛−1 𝑥 𝑑𝑥 = 𝑥 𝑡𝑎𝑛−1 𝑥 − 2 𝑙𝑛 (1 + 𝑥 2 ) + 𝐶 7.4
1
∫ 𝑐𝑜𝑡 −1 𝑥 𝑑𝑥 = 𝑥 𝑐𝑜𝑡 −1 𝑥 + 2 𝑙𝑛 (1 + 𝑥 2 ) + 𝐶 7.5
_____________________________________________________________________________________________________
Example 7.18.
Evaluate
∫ 𝑐𝑜𝑡ℎ−1 𝑦 𝑑𝑦
1 1
𝑑𝑢 = 2
𝑑𝑦 = − 2 𝑑𝑦
1−𝑦 𝑦 −1
∫ 𝑐𝑜𝑡ℎ−1 𝑦 𝑑𝑦 = 𝑢𝑣 − ∫ 𝑣 𝑑𝑢
110
𝑦
= 𝑦 𝑐𝑜𝑡ℎ−1 𝑦 − ∫ − 𝑑𝑦
𝑦2 −1
1 2𝑦
= 𝑦 𝑐𝑜𝑡ℎ−1 𝑦 + ∫ 2 𝑑𝑦
2 𝑦 −1
Using ordinary substitution to evaluate the integral on the right (noting that the
numerator is the derivative of the term inside the square root), we have
1
∫ 𝑐𝑜𝑡ℎ−1 𝑦 𝑑𝑦 = 𝑦 𝑐𝑜𝑡ℎ−1 𝑦 + 2 𝑙𝑛 (𝑦 2 − 1) + 𝐶 ANS.
Using the same technique, we can derive a general formula for the integrals of
inverse hyperbolic functions:
1
∫ 𝑡𝑎𝑛ℎ−1 𝑥 𝑑𝑥 = 𝑥 𝑡𝑎𝑛ℎ−1 𝑥 + 2 𝑙𝑛 (1 − 𝑥 2 ) + 𝐶 7.10
1
∫ 𝑐𝑜𝑡ℎ−1 𝑥 𝑑𝑥 = 𝑥 𝑐𝑜𝑡ℎ−1 𝑥 + 2 𝑙𝑛 (𝑥 2 − 1) + 𝐶 7.11
_____________________________________________________________________________________________________
111
∫ 𝑠𝑖𝑛−1 𝑥 𝑑𝑥 = 𝑥 𝑠𝑖𝑛−1 𝑥 + 𝑐𝑜𝑠 (𝑠𝑖𝑛−1 𝑥) + 𝐶
1
x
𝑠𝑖𝑛−1 𝑥
√1 − 𝑥 2
∫ 𝑠𝑖𝑛−1 𝑥 𝑑𝑥 = 𝑥 𝑠𝑖𝑛−1 𝑥 + √1 − 𝑥 2 + 𝐶
_____________________________________________________________________________________________________
Example 7.19.
Show that
2
∫ 𝑒 𝑥 (2𝑥 2 + 1) 𝑑𝑥
∫ 𝑓(𝑥) 𝑑𝑥 + ∫ 𝑥 𝑓′(𝑥) 𝑑𝑥
To evaluate the second integral, we let 𝑢 = 𝑥 and 𝑑𝑣 = 𝑓 ′ (𝑥) 𝑑𝑥. This means that 𝑑𝑢 =
𝑑𝑥 and 𝑣 = 𝑓(𝑥). Thus,
∫ 𝑥 𝑓′(𝑥) 𝑑𝑥 = 𝑢𝑣 − ∫ 𝑣 𝑑𝑢
= 𝑥 𝑓(𝑥) − ∫ 𝑓(𝑥) 𝑑𝑥 + 𝐶
Transposing the integral to the left gives
112
2 2 2
∫ 𝑒 𝑥 (2𝑥 2 + 1) 𝑑𝑥 = ∫ 𝑒 𝑥 𝑑𝑥 + ∫ 𝑥(2𝑥𝑒 𝑥 ) 𝑑𝑥
2 2
We note that if 𝑓(𝑥) = 𝑒 𝑥 , 𝑓′(𝑥) = 2𝑥𝑒 𝑥 . Hence,
2 2
∫ 𝑒 𝑥 (2𝑥 2 + 1) 𝑑𝑥 = 𝑥𝑒 𝑥 + 𝐶 ANS.
_____________________________________________________________________________________________________
Example 7.20.
Show that
1 − 𝑠𝑖𝑛 𝑥
∫ 𝑒𝑥 ( ) 𝑑𝑥
1 − 𝑐𝑜𝑠 𝑥
∫ 𝑒 𝑥 𝑓(𝑥) 𝑑𝑥 + ∫ 𝑒 𝑥 𝑓 ′ (𝑥) 𝑑𝑥
To evaluate the second integral, we let we let 𝑢 = 𝑒 𝑥 and 𝑑𝑣 = 𝑓 ′ (𝑥) 𝑑𝑥. This means that
𝑑𝑢 = 𝑒 𝑥 𝑑𝑥 and 𝑣 = 𝑓(𝑥). Thus,
∫ 𝑒 𝑥 𝑓′(𝑥) 𝑑𝑥 = 𝑢𝑣 − ∫ 𝑣 𝑑𝑢
= 𝑒 𝑥 𝑓(𝑥) − ∫ 𝑒 𝑥 𝑓(𝑥) 𝑑𝑥 + 𝐶
Transposing the integral to the left gives
∫ 𝑒 𝑥 𝑓(𝑥) 𝑑𝑥 + ∫ 𝑒 𝑥 𝑓 ′(𝑥) 𝑑𝑥 = 𝑒 𝑥 𝑓(𝑥) + 𝐶 𝑞. 𝑒. 𝑑.
𝑥
1 − 𝑠𝑖𝑛 𝑥 𝑥
1 − 2 𝑠𝑖𝑛 12𝑥 𝑐𝑜𝑠 12𝑥
∫𝑒 ( ) 𝑑𝑥 = ∫ 𝑒 ( ) 𝑑𝑥
1 − 𝑐𝑜𝑠 𝑥 2 𝑠𝑖𝑛2 12𝑥
1 1 1
= ∫ 𝑒 𝑥 𝑐𝑠𝑐 2 2𝑥 𝑑𝑥 − ∫ 𝑒 𝑥 𝑐𝑜𝑡 2𝑥 𝑑𝑥
2
113
1 1
We note that if 𝑓(𝑥) = −𝑐𝑜𝑡 12𝑥, 𝑓 ′ (𝑥) = 2 𝑐𝑠𝑐 2 𝑥. Hence,
2
1 − 𝑠𝑖𝑛 𝑥 1
∫ 𝑒𝑥 ( ) 𝑑𝑥 = −𝑒 𝑥 𝑐𝑜𝑡 2𝑥 + 𝐶 ANS.
1 − 𝑐𝑜𝑠 𝑥
There are many other relations similar to the two relations given in the previous
examples that can also be derived using integration by parts. These relations allow us to
evaluate integrals which are difficult, if not impossible, to express in terms of elementary
functions. It is left as an exercise to prove the following:
_____________________________________________________________________________________________________
Example 7.21.
Evaluate
3 1
∫ [ (𝑐𝑜𝑠ℎ 𝑥)3/2 − (𝑐𝑜𝑠ℎ 𝑥)−1/2 ] 𝑑𝑥
2 2
Solution. We need to transform the integral into this form to determine the function 𝑓(𝑥):
1 1
∫ [(𝑐𝑜𝑠ℎ 𝑥)(𝑐𝑜𝑠ℎ 𝑥)1/2 + (𝑐𝑜𝑠ℎ 𝑥)3/2 − (𝑐𝑜𝑠ℎ 𝑥)−1/2 ] 𝑑𝑥
2 2
1
∫ [(𝑐𝑜𝑠ℎ 𝑥)(𝑐𝑜𝑠ℎ 𝑥)1/2 + (𝑐𝑜𝑠ℎ 𝑥)−1/2 (𝑐𝑜𝑠ℎ2 𝑥 − 1)] 𝑑𝑥
2
1
∫ [(𝑐𝑜𝑠ℎ 𝑥)(𝑐𝑜𝑠ℎ 𝑥)1/2 + (𝑐𝑜𝑠ℎ 𝑥)−1/2 (𝑠𝑖𝑛ℎ2 𝑥)] 𝑑𝑥
2
114
1 𝑠𝑖𝑛ℎ 𝑥
∫ [(𝑐𝑜𝑠ℎ 𝑥)(𝑐𝑜𝑠ℎ 𝑥)1/2 + (𝑠𝑖𝑛ℎ 𝑥) ( )] 𝑑𝑥
2 (𝑐𝑜𝑠ℎ 𝑥)1/2
1 𝑠𝑖𝑛ℎ 𝑥
𝑓 ′ (𝑥) =
2 (𝑐𝑜𝑠ℎ 𝑥)1/2
Hence,
1 𝑠𝑖𝑛ℎ 𝑥
∫ [(𝑐𝑜𝑠ℎ 𝑥)(𝑐𝑜𝑠ℎ 𝑥)1/2 + (𝑠𝑖𝑛ℎ 𝑥) ( )] 𝑑𝑥 = ∫[𝑐𝑜𝑠ℎ 𝑥 𝑓(𝑥) + 𝑠𝑖𝑛ℎ 𝑥 𝑓′(𝑥)] 𝑑𝑥
2 (𝑐𝑜𝑠ℎ 𝑥)1/2
3 1
∫ [ (𝑐𝑜𝑠ℎ 𝑥)3/2 − (𝑐𝑜𝑠ℎ 𝑥)−1/2 ] 𝑑𝑥 = (𝑠𝑖𝑛ℎ 𝑥)(𝑐𝑜𝑠ℎ 𝑥)1/2 + 𝐶 ANS.
2 2
_____________________________________________________________________________________________________
Example 7.22.
Prove the reduction formula
1 𝑛−1
∫ 𝑠𝑖𝑛𝑛 𝑥 𝑑𝑥 = − 𝑐𝑜𝑠 𝑥 𝑠𝑖𝑛𝑛−1 𝑥 + ∫ 𝑠𝑖𝑛𝑛−2 𝑥 𝑑𝑥
𝑛 𝑛
𝑑𝑢 = (𝑛 − 1) 𝑠𝑖𝑛𝑛−1 𝑥 𝑐𝑜𝑠 𝑥 𝑑𝑥
𝑣 = ∫ 𝑠𝑖𝑛 𝑥 𝑑𝑥 = −𝑐𝑜𝑠 𝑥
∫ 𝑠𝑖𝑛𝑛 𝑥 𝑑𝑥 = 𝑢𝑣 − ∫ 𝑣 𝑑𝑢
115
Note that the integral for evaluation re-appears on the right. Transposing this to the left,
1 (𝑛 − 1)
∫ 𝑐𝑜𝑠 𝑛 𝑥 𝑑𝑥 = 𝑠𝑖𝑛 𝑥 𝑐𝑜𝑠 𝑛−1 𝑥 + ∫ 𝑐𝑜𝑠 𝑛−2 𝑥 𝑑𝑥
𝑛 𝑛
∫ 𝑥 𝑛 𝑒 𝑥 𝑑𝑥 = 𝑥 𝑛 𝑒 𝑥 − 𝑛 ∫ 𝑥 𝑛−1 𝑒 𝑥 𝑑𝑥
_____________________________________________________________________________________________________
Example 7.23.
Evaluate
∫ 𝑐𝑜𝑠 4 𝑟 𝑑𝑟
1 4−1
∫ 𝑐𝑜𝑠 4 𝑟 𝑑𝑟 = 𝑠𝑖𝑛 𝑟 𝑐𝑜𝑠 4−1 𝑟 + ∫ 𝑐𝑜𝑠 4−2 𝑟 𝑑𝑟
4 4
1 3
= 𝑠𝑖𝑛 𝑟 𝑐𝑜𝑠 3 𝑟 + ∫ 𝑐𝑜𝑠 2 𝑟 𝑑𝑟
4 4
1 3 1 1
= 𝑠𝑖𝑛 𝑟 𝑐𝑜𝑠 3 𝑟 + ∫ ( + 𝑐𝑜𝑠 2𝑟) 𝑑𝑟
4 4 2 2
1 3 3
= 𝑠𝑖𝑛 𝑟 𝑐𝑜𝑠 3 𝑟 + ∫ 𝑑𝑟 + ∫ 𝑐𝑜𝑠 2𝑟 𝑑𝑟
4 8 8
1 3 3
= 𝑠𝑖𝑛 𝑟 𝑐𝑜𝑠 3 𝑟 + 𝑟 + 𝑠𝑖𝑛 2𝑟 + 𝐶
4 8 16
3 1 3
= 𝑟 + 𝑠𝑖𝑛 𝑟 𝑐𝑜𝑠 3 𝑟 + 𝑠𝑖𝑛 2𝑟 + 𝐶 ANS.
8 4 16
_____________________________________________________________________________________________________
116
Try these.
1. ∫ 𝑦 2 𝑙𝑛 𝑦 𝑑𝑦
2. ∫ 𝑙𝑛 √𝑡 𝑑𝑡
3. ∫ 𝑐𝑜𝑠 𝑥 𝑙𝑛 𝑐𝑜𝑠 𝑥 𝑑𝑥
4. ∫ 𝑐𝑠𝑐 3 𝑤 𝑑𝑤
5. ∫ 𝑠𝑒𝑐ℎ3 𝑡 𝑑𝑡
6. ∫ 𝑐𝑜𝑡 4 𝑥 𝑑𝑥
7. ∫(𝑐𝑜𝑠 −1 𝑥)2 𝑑𝑥
8. ∫(𝑙𝑛 𝑟)2 𝑑𝑟
2𝑦 − 1
∫ 𝑒𝑦 ∙ 𝑑𝑦
9. √𝑦 − 1
10. ∫ 𝑥 𝑡𝑎𝑛ℎ2 𝑥 𝑑𝑥
117
Integration by Trigonometric
8. Substitution
118
Example 8.1.
Evaluate
∫ √𝑥 2 + 9 𝑑𝑥
∫ √𝑥 2 + 9 𝑑𝑥 = ∫ 3 𝑠𝑒𝑐 𝜃 ∙ 3 𝑠𝑒𝑐 2 𝜃 𝑑𝜃
= 9 ∫ 𝑠𝑒𝑐 3 𝜃 𝑑𝜃
In Example 7.8., we found out that
1 1
∫ 𝑠𝑒𝑐 3 𝑟 𝑑𝑟 = 𝑠𝑒𝑐 𝑟 ∙ 𝑡𝑎𝑛 𝑟 + 𝑙𝑛|𝑠𝑒𝑐 𝑟 + 𝑡𝑎𝑛 𝑟| + 𝐶
2 2
1 1
∫ 𝑠𝑒𝑐 3 𝜃 𝑑𝜃 = 𝑠𝑒𝑐 𝜃 ∙ 𝑡𝑎𝑛 𝜃 + 𝑙𝑛|𝑠𝑒𝑐 𝜃 + 𝑡𝑎𝑛 𝜃| + 𝐶
2 2
1 1 2 1 1 1 1
= ∙ √𝑥 + 9 ∙ 𝑥 + 𝑙𝑛 | √𝑥 2 + 9 + 𝑥| + 𝐶
2 3 3 2 3 3
Therefore,
1 1 1 9 1 1
∫ √𝑥 2 + 9 = 9 ∙ ∙ √𝑥 2 + 9 ∙ 𝑥 + 𝑙𝑛 | √𝑥 2 + 9 + 𝑥| + 𝐶
2 3 3 2 3 3
1 9 9
= 𝑥√𝑥 2 + 9 + 𝑙𝑛 |√𝑥 2 + 9 + 𝑥| − 𝑙𝑛 3 + 𝐶
2 2 2
1 9
= 𝑥√𝑥 2 + 9 + 𝑙𝑛 |√𝑥 2 + 9 + 𝑥| + 𝐶 ANS.
2 2
Note that −92 𝑙𝑛 3 being a constant, is already absorbed by the constant 𝐶.
_____________________________________________________________________________________________________
119
Example 8.2.
Evaluate
∫ √𝑥 2 − 25 𝑑𝑥
Solution. Using the proper substitution, 𝑥 = 5 𝑠𝑒𝑐 𝜃, 𝑑𝑥 = 5 𝑠𝑒𝑐 𝜃 𝑡𝑎𝑛 𝜃 𝑑𝜃, and
√𝑥 2 − 25 = √25 𝑠𝑒𝑐 2 𝜃 − 25 = 5 𝑡𝑎𝑛 𝜃. Hence,
= 25 ∫ 𝑡𝑎𝑛2 𝜃 𝑠𝑒𝑐 𝜃 𝑑𝜃
= 25 ∫(𝑠𝑒𝑐 2 𝜃 − 1) 𝑠𝑒𝑐 𝜃 𝑑𝜃
= 25 ∫(𝑠𝑒𝑐 3 𝜃 − 𝑠𝑒𝑐 𝜃) 𝑑𝜃
Because we already know the solution to the first term, we will just take care of the
second term. The integral of 𝑠𝑒𝑐 𝜃 is 𝑙𝑛 |𝑠𝑒𝑐 𝜃 + 𝑡𝑎𝑛 𝜃|. Hence,
1 1
∫(𝑠𝑒𝑐 3 𝜃 − 𝑠𝑒𝑐 𝜃) 𝑑𝜃 = 𝑠𝑒𝑐 𝜃 ∙ 𝑡𝑎𝑛 𝜃 + 𝑙𝑛|𝑠𝑒𝑐 𝜃 + 𝑡𝑎𝑛 𝜃| − 𝑙𝑛 |𝑠𝑒𝑐 𝜃 + 𝑡𝑎𝑛 𝜃| + 𝐶
2 2
1 1
= 𝑠𝑒𝑐 𝜃 ∙ 𝑡𝑎𝑛 𝜃 − 𝑙𝑛|𝑠𝑒𝑐 𝜃 + 𝑡𝑎𝑛 𝜃| + 𝐶
2 2
Hence,
1 1 1 25 1 1
∫ √𝑥 2 − 25 𝑑𝑥 = 25 ∙ ∙ √𝑥 2 − 25 ∙ 𝑥 − 𝑙𝑛 | √𝑥 2 − 25 + 𝑥| + 𝐶
2 5 5 2 5 5
1 25 25
= 𝑥√𝑥 2 − 25 − 𝑙𝑛 |√𝑥 2 − 25 + 𝑥| + 𝑙𝑛 5 + 𝐶
2 2 2
1 25
= 𝑥√𝑥 2 − 25 − 𝑙𝑛 |√𝑥 2 − 25 + 𝑥| + 𝐶 ANS.
2 2
Note also that 252
𝑙𝑛 5 being a constant, is already absorbed by the constant 𝐶.
_____________________________________________________________________________________________________
120
Example 8.3.
Evaluate
∫ √4 − 𝑥 2 𝑑𝑥
∫ √4 − 𝑥 2 𝑑𝑥 = ∫ 2 𝑐𝑜𝑠 𝜃 ∙ 2 𝑐𝑜𝑠 𝜃 𝑑𝜃
1 + 𝑐𝑜𝑠 2𝜃
= 4 ∫ 𝑐𝑜𝑠 2 𝜃 𝑑𝜃 = 4 ∫ 𝑑𝜃
2
𝑥 𝑥 √4 − 𝑥 2
= 2 𝑠𝑖𝑛−1 + 2 ∙ ∙ +𝐶
2 2 2
𝑥 1
= 2 𝑠𝑖𝑛−1 + 𝑥√4 − 𝑥 2 + 𝐶 ANS.
2 2
Summarizing, we have additional standard integrals as follows:
1 𝑎2
∫ √𝑥 2 + 𝑎2 𝑑𝑥 = 𝑥 √𝑥 2 + 𝑎2 + 𝑙𝑛 |√𝑥 2 + 𝑎2 + 𝑥| + 𝐶 8.1
2 2
1 𝑎2
∫ √𝑥 2 − 𝑎2 𝑑𝑥 = 𝑥 √𝑥 − 𝑎 − 𝑙𝑛 |√𝑥 2 − 𝑎2 + 𝑥| + 𝐶
2 2 8.2
2 2
1 𝑎2 𝑥
∫ √𝑎2 − 𝑥 2 𝑑𝑥 = 𝑥 √𝑎2 − 𝑥 2 + 𝑠𝑖𝑛−1 + 𝐶 8.3
2 2 𝑎
Equivalently,
1 𝑎2 𝑥
∫ √𝑥 2 + 𝑎2 𝑑𝑥 = 𝑥 √𝑥 2 + 𝑎2 + 𝑠𝑖𝑛ℎ−1 + 𝐶
2 2 𝑎
1 𝑎2 𝑥
∫ √𝑥 2 − 𝑎2 𝑑𝑥 = 𝑥 √𝑥 2 − 𝑎2 − 𝑐𝑜𝑠ℎ−1 + 𝐶
2 2 𝑎
_____________________________________________________________________________________________________
121
Example 8.4.
Evaluate
𝑑𝑥
∫
𝑥 √𝑥 2 − 16
Solution. Immediately, one will recognize that this is a standard integral which yields
inverse secant. Evaluating the integral yields
𝑑𝑥 1 𝑥
∫ = 𝑠𝑒𝑐 −1 + 𝐶
𝑥 √𝑥 2 − 16 4 4
However, using trigonometric substitution, we can let 𝑥 = 4 𝑠𝑒𝑐 𝜃 and this gives us
𝑑𝑥 = 4 𝑠𝑒𝑐 𝜃 𝑡𝑎𝑛 𝜃 𝑑𝜃 and √𝑥 2 − 16 = 4 𝑡𝑎𝑛 𝜃. The integral becomes
𝑑𝑥 4 𝑠𝑒𝑐 𝜃 𝑡𝑎𝑛 𝜃 𝑑𝜃
∫ =∫
𝑥 √𝑥 2 − 16 4 𝑠𝑒𝑐 𝜃 ∙ 4 𝑡𝑎𝑛 𝜃
1 1
= ∫ 𝑑𝜃 = 𝜃 + 𝐶
4 4
1 𝑥
= 𝑠𝑒𝑐 −1 + 𝐶 ANS.
4 4
_____________________________________________________________________________________________________
Example 8.5.
Evaluate
∫ 𝑠𝑒𝑐 2 𝑥√𝑡𝑎𝑛2 𝑥 + 4 𝑑𝑥
Solution. Trigonometric substitution does not work with irrational algebraic functions
only. It applies also to irrational trigonometric functions as well. We let
𝑡𝑎𝑛 𝑥 = 2 𝑡𝑎𝑛 𝜃
𝑠𝑒𝑐 2 𝑥 𝑑𝑥 = 2 𝑠𝑒𝑐 2 𝜃 𝑑𝜃
√𝑡𝑎𝑛2 𝑥 + 4 = 2 𝑠𝑒𝑐 𝜃
Thus,
122
Following the answer in Example 7.8.,
1 1
∫ 𝑠𝑒𝑐 3 𝜃 𝑑𝑟 = 𝑠𝑒𝑐 𝜃 ∙ 𝑡𝑎𝑛 𝜃 + 𝑙𝑛|𝑡𝑎𝑛 𝜃 + 𝑠𝑒𝑐 𝜃| + 𝐶
2 2
1 1 1 1 1 1
= ∙ √𝑡𝑎𝑛2 𝑥 + 4 ∙ 𝑡𝑎𝑛 𝑥 + 𝑙𝑛 | ∙ 𝑡𝑎𝑛 𝑥 + √𝑡𝑎𝑛2 𝑥 + 4| + 𝐶
2 2 2 2 2 2
1 1
= 𝑡𝑎𝑛 𝑥 √𝑡𝑎𝑛2 𝑥 + 4 + 𝑙𝑛 |𝑡𝑎𝑛 𝑥 + √𝑡𝑎𝑛2 𝑥 + 4| + 𝐶
8 2
Finally,
1 1
∫ 𝑠𝑒𝑐 2 𝑥√𝑡𝑎𝑛2 𝑥 + 4 𝑑𝑥 = 4 ( 𝑡𝑎𝑛 𝑥√𝑡𝑎𝑛2 𝑥 + 4 + 𝑙𝑛 |𝑡𝑎𝑛 𝑥 + √𝑡𝑎𝑛2 𝑥 + 4| + 𝐶)
8 2
1
= 𝑡𝑎𝑛 𝑥√𝑡𝑎𝑛2 𝑥 + 4 + 2 𝑙𝑛 |𝑡𝑎𝑛 𝑥 + √𝑡𝑎𝑛2 𝑥 + 4| + 𝐶 ANS.
2
_____________________________________________________________________________________________________
Example 8.6.
Evaluate
6𝑥 − 3
∫ 𝑑𝑥
√4𝑥 2 − 4𝑥 − 3
6𝑥 − 3 2𝑥 − 1
=3∙
√4𝑥 2 − 4𝑥 − 3 √(2𝑥 − 1)2 − 4
By letting
2𝑥 − 1 = 2 𝑠𝑒𝑐 𝜃
2 𝑑𝑥 = 2 𝑠𝑒𝑐 𝜃 𝑡𝑎𝑛 𝜃 𝑑𝜃
Thus,
6𝑥 − 3 2 𝑠𝑒𝑐 𝜃
∫ 𝑑𝑥 = 3 ∫ ∙ 𝑠𝑒𝑐 𝜃 𝑡𝑎𝑛 𝜃 𝑑𝜃
√4𝑥 2 − 4𝑥 − 3 2 𝑡𝑎𝑛 𝜃
= 3 ∫ 𝑠𝑒𝑐 2 𝜃 𝑑𝜃 = 3 𝑡𝑎𝑛 𝜃 + 𝐶
123
Finally,
6𝑥 − 3 3
∫ 𝑑𝑥 = √4𝑥 2 − 4𝑥 − 3 + 𝐶 ANS.
√4𝑥 2 − 4𝑥 − 3 2
There are instances when it is not more practical to use trigonometric substitution
especially if it makes the process of integration more tedious. In this example, notice that
1
if we let 𝑢 = 4𝑥 2 − 4𝑥 − 3, 𝑑𝑢 = (8𝑥 − 4) 𝑑𝑥 and 4 𝑑𝑢 = (2𝑥 − 1) 𝑑𝑥. Using these
substitutions, the integral reduces to
6𝑥 − 3 3 𝑑𝑢 3
∫ 𝑑𝑥 = ∫ = √𝑢 + 𝐶
√4𝑥 2 − 4𝑥 − 3 4 √𝑢 2
3
= √4𝑥 2 − 4𝑥 − 3 + 𝐶
2
which is equal to the answer above after the substitution.
_____________________________________________________________________________________________________
Example 8.7.
Evaluate
𝑥2
∫ 𝑑𝑥
(9 − 𝑥 2 )3/2
𝑥2 (3 𝑠𝑖𝑛 𝜃)2
∫ 𝑑𝑥 = ∫ ∙ 3 𝑐𝑜𝑠 𝜃 𝑑𝜃
(9 − 𝑥 2 )3/2 (9 − 9 𝑠𝑖𝑛2 𝜃)3/2
9 𝑠𝑖𝑛2 𝜃
=∫ ∙ 3 cos 𝜃 𝑑𝜃
(9 𝑐𝑜𝑠 2 𝜃)3/2
= ∫ 𝑡𝑎𝑛2 𝜃 𝑑𝜃 = ∫(𝑠𝑒𝑐 2 𝜃 − 1) 𝑑𝜃
= 𝑡𝑎𝑛 𝜃 − 𝜃 + 𝐶
𝑥 𝑥
= − 𝑠𝑖𝑛−1 +𝐶 ANS.
√9 − 𝑥 2 3
_____________________________________________________________________________________________________
124
Example 8.8.
Evaluate
√4𝑥 − 𝑥 2
∫ 𝑑𝑥
𝑥
√4𝑥 − 𝑥 2 √4 − (4 − 4𝑥 + 𝑥 2 ) √4 − (𝑥 − 2)2
= =
𝑥 𝑥 𝑥
The integral, thus, reduces to
√4𝑥 − 𝑥 2 √4 − (𝑥 − 2)2
∫ 𝑑𝑥 = ∫ 𝑑𝑥
𝑥 𝑥
√4 − (𝑥 − 2)2 2 𝑐𝑜𝑠 𝜃
∫ 𝑑𝑥 = ∫ ∙ 2 𝑐𝑜𝑠 𝜃 𝑑𝜃
𝑥 2 (1 + 𝑠𝑖𝑛 𝜃)
𝑐𝑜𝑠 2 𝜃
= 2∫ 𝑑𝜃
1 + 𝑠𝑖𝑛 𝜃
1 − 𝑠𝑖𝑛2 𝜃
= 2∫ 𝑑𝜃 = 2 ∫(1 − 𝑠𝑖𝑛 𝜃) 𝑑𝜃
1 + 𝑠𝑖𝑛 𝜃
= 2𝜃 + 2 𝑐𝑜𝑠 𝜃 + 𝐶
Finally,
√4𝑥 − 𝑥 2 𝑥−2
∫ 𝑑𝑥 = 2 𝑠𝑖𝑛−1 ( ) + √4𝑥 − 𝑥 2 + 𝐶 ANS.
𝑥 2
125
Example 8.9.
Evaluate
𝑥2 − 1
∫ 𝑑𝑥
𝑥2 + 1
Solution. We can let 𝑥 = 𝑡𝑎𝑛 𝜃 so that 𝑑𝑥 = 𝑠𝑒𝑐 2 𝜃 𝑑𝜃. This means also that
𝑥 2 − 1 = 𝑡𝑎𝑛2 𝜃 − 1 = 𝑠𝑒𝑐 2 𝜃 − 2
𝑥 2 + 1 = 𝑡𝑎𝑛2 𝜃 + 1 = 𝑠𝑒𝑐 2 𝜃
Hence,
𝑥2 − 1 𝑠𝑒𝑐 2 𝜃 − 2
∫ 𝑑𝑥 = ∫ ∙ 𝑠𝑒𝑐 2 𝜃 𝑑𝜃
𝑥2 + 1 𝑠𝑒𝑐 2 𝜃
= ∫(𝑠𝑒𝑐 2 𝜃 − 2) 𝑑𝜃
= 𝑡𝑎𝑛 𝜃 − 2𝜃 + 𝐶
Finally,
𝑥2 − 1
∫ 𝑑𝑥 = 𝑥 − 2 𝑡𝑎𝑛−1 𝑥 + 𝐶 ANS.
𝑥2 + 1
If the above calculation proves to be too tedious, there is another way we can
evaluate the integral. Note that when you divide the numerator by the denominator, the
result is
𝑥2 − 1 2
∫ 2 𝑑𝑥 = ∫ (1 − 2 ) 𝑑𝑥
𝑥 +1 𝑥 +1
which can readily be integrated using standard integrals. We recognize this as Eq. 3.3.
Integration yields
2
∫ (1 − ) 𝑑𝑥 = 𝑥 − 2 𝑡𝑎𝑛−1 𝑥 + 𝐶
𝑥2 +1
126
Example 8.10.
Evaluate
𝑥2 + 1
∫ 𝑑𝑥
𝑥2 − 1
𝑥 2 + 1 = 𝑠𝑒𝑐 2 𝜃 + 1 = 𝑡𝑎𝑛2 𝜃 + 2
𝑥 2 − 1 = 𝑠𝑒𝑐 2 𝜃 − 1 = 𝑡𝑎𝑛2 𝜃
𝑥2 + 1 𝑡𝑎𝑛2 𝜃 + 2
∫ 𝑑𝑥 = ∫ ∙ 𝑠𝑒𝑐 𝜃 𝑡𝑎𝑛 𝜃 𝑑𝜃
𝑥2 − 1 𝑡𝑎𝑛2 𝜃
If 𝑥 = 𝑠𝑒𝑐 𝜃, it can be shown that 𝑐𝑜𝑠 𝜃 = 1/𝑥 and 𝑠𝑖𝑛 𝜃 = √𝑥 2 − 1/𝑥. Therefore,
𝑥
𝑐𝑠𝑐 𝜃 =
√𝑥 2 − 1
1
𝑐𝑜𝑡 𝜃 =
√𝑥 2 − 1
Finally,
𝑥2 + 1 𝑥−1
∫ 2 𝑑𝑥 = 𝑥 + 2 𝑙𝑛 | |+𝐶
𝑥 −1 √𝑥 2 − 1
√𝑥 − 1
= 𝑥 + 2 𝑙𝑛 | |+𝐶
√𝑥 + 1
𝑥−1
= 𝑥 + 𝑙𝑛 | |+𝐶 ANS.
𝑥+1
127
Fortunately, there is another approach to the problem. Notice that the integrand
can be broken down into partial fractions as follows:
𝑥2 + 1 1 1
2
= 1+ −
𝑥 −1 𝑥−1 𝑥+1
Hence,
𝑥2 + 1 1 1
∫ 2
𝑑𝑥 = ∫ (1 + − ) 𝑑𝑥
𝑥 −1 𝑥−1 𝑥+1
= 𝑥 + 𝑙𝑛 |𝑥 − 1| − 𝑙𝑛 |𝑥 + 1| + 𝐶
which is exactly the same as the answer above. This example and the previous one only
shows that trigonometric substitution is not the only choice of technique in evaluating an
integral.
_____________________________________________________________________________________________________
Example 8.11.
Evaluate
1
∫ 𝑑𝑦
𝑦√4 − 𝑦 2
1 2 𝑐𝑜𝑠 𝜃 𝑑𝜃
∫ 𝑑𝑦 = ∫
𝑦√4 − 𝑦 2 2 𝑠𝑖𝑛 𝜃 ∙ 2 𝑐𝑜𝑠 𝜃
1
= ∫ 𝑐𝑠𝑐 𝜃 𝑑𝜃
2
1
= 𝑙𝑛 |𝑐𝑠𝑐 𝜃 − 𝑐𝑜𝑡 𝜃| + 𝐶
2
Finally,
1 1 2 − √4 − 𝑦 2
∫ 𝑑𝑦 = 𝑙𝑛 | |+𝐶 ANS.
𝑦√4 − 𝑦 2 2 𝑦
Note that we can also use the standard integral for hyperbolic arcsecant of an
angle. Using Eq. 6.5,
128
1 1 𝑦
∫ 𝑑𝑦 = − 𝑠𝑒𝑐ℎ−1 + 𝐶
𝑦√4 − 𝑦 2 2 2
Example 8.12.
Evaluate
1
∫ 𝑑𝑡
𝑡 2 √𝑡 2 − 1
1 1
∫ 𝑑𝑡 = ∫ ∙ 𝑠𝑒𝑐 𝜃 𝑡𝑎𝑛 𝜃 𝑑𝜃
𝑡 2 √𝑡 2 − 1 𝑠𝑒𝑐 2 𝜃 𝑡𝑎𝑛 𝜃
= ∫ 𝑐𝑜𝑠 𝜃 𝑑𝜃 = 𝑠𝑖𝑛 𝜃 + 𝐶
1
∫ 𝑑𝑡 = √1 − 𝑐𝑜𝑠 2 𝜃 + 𝐶
𝑡 2 √𝑡 2 −1
= √1 − 1/𝑡 2 + 𝐶
√𝑡 2 − 1
= +𝐶 ANS.
𝑡
_____________________________________________________________________________________________________
Example 8.13.
Evaluate
∫ 𝑒 2𝑥 √1 + 𝑒 2𝑥 𝑑𝑥
Solution. We can let 𝑒 𝑥 = 𝑡𝑎𝑛 𝜃, so that 𝑒 𝑥 𝑑𝑥 = 𝑠𝑒𝑐 2 𝜃 𝑑𝜃. The integral becomes
129
= ∫ 𝑠𝑒𝑐 𝜃 𝑡𝑎𝑛 𝜃 ∙ 𝑠𝑒𝑐 2 𝜃 𝑑𝜃
1
∫ 𝑒 2𝑥 √1 + 𝑒 2𝑥 𝑑𝑥 = (1 + 𝑒 2𝑥 )3/2 + 𝐶 ANS.
3
1
∫ 𝑒 2𝑥 √1 + 𝑒 2𝑥 𝑑𝑥 = ∫ √𝑢 𝑑𝑢
2
1 2 3/2
= ∙ 𝑢 +𝐶
2 3
1
= (1 + 𝑒 2𝑥 )3/2 + 𝐶
3
_____________________________________________________________________________________________________
Example 8.14.
Evaluate
√𝑥
∫ 𝑑𝑥
√1 − 𝑥
√𝑥 √1 − 𝑢
∫ 𝑑𝑥 = ∫ − 𝑑𝑢
√1 − 𝑥 √𝑢
1 𝑑𝑢
= 𝑐𝑜𝑠 𝜃 𝑑𝜃
2 √𝑢
130
√1 − 𝑢
∫− 𝑑𝑢 = ∫ −𝑐𝑜𝑠 𝜃 ∙ 2 𝑐𝑜𝑠 𝜃 𝑑𝜃
√𝑢
= −2 ∫ 𝑐𝑜𝑠 2 𝜃 𝑑𝜃
1 1
= −2 ∫ ( + 𝑐𝑜𝑠 2𝜃) 𝑑𝜃
2 2
1
= −𝜃 − 𝑠𝑖𝑛 2𝜃 + 𝐶
2
1
= −𝜃 − ∙ 2 𝑠𝑖𝑛 𝜃 𝑐𝑜𝑠 𝜃 + 𝐶
2
1
= −𝑠𝑖𝑛−1 √𝑢 − ∙ 2√𝑢 ∙ √1 − 𝑢 + 𝐶
2
= −𝑠𝑖𝑛−1 √1 − 𝑥 − √1 − 𝑥 ∙ √𝑥 + 𝐶 ANS.
This just shows that trigonometric substitutions equally apply to irrational functions.
_____________________________________________________________________________________________________
Example 8.15.
Evaluate
𝑤 3 𝑑𝑤
∫
√𝑤 2 + 9
Solution. This is Example 7.7. where we used integration by parts. The integral may also
be evaluated using trigonometric substitution. If we let 𝑤 = 3 𝑡𝑎𝑛 𝜃, 𝑑𝑤 = 3 𝑠𝑒𝑐 2 𝜃 𝑑𝜃.
Hence, the integral becomes
𝑤 3 𝑑𝑤 (3 𝑡𝑎𝑛 𝜃)3
∫ =∫ 3 𝑠𝑒𝑐 2 𝜃 𝑑𝜃
√𝑤 2 + 9 3 𝑠𝑒𝑐 𝜃
= 27 ∫ 𝑡𝑎𝑛3 𝜃 𝑠𝑒𝑐 𝜃 𝑑𝜃
= 9 𝑠𝑒𝑐 3 𝜃 − 27 𝑠𝑒𝑐 𝜃 + 𝐶
Substituting 𝑤,
131
3
𝑤 3 𝑑𝑤 √𝑤 2 + 9 √𝑤 2 + 9
∫ =9∙( ) − 27 ∙ ( )+𝐶
√𝑤 2 + 9 3 3
1
= (𝑤 2 + 9)3/2 − 9√𝑤 2 + 9 + 𝐶 ANS.
3
It is left as an exercise to show that this answer is equivalent to the answer in Example
7.7.
_____________________________________________________________________________________________________
Example 8.16.
Evaluate
(𝑡 − 1)3/2
∫ 𝑑𝑡
𝑡2
(𝑡 − 1)3/2 𝑢3
∫ 𝑑𝑡 = ∫ ∙ 2𝑢 𝑑𝑢
𝑡2 (𝑢2 + 1)2
𝑢4
= 2∫ 𝑑𝑢
(𝑢2 + 1)2
𝑢4 𝑡𝑎𝑛4 𝜃
2∫ 𝑑𝑢 = 2 ∫ ∙ 𝑠𝑒𝑐 2 𝜃 𝑑𝜃
(𝑢2 + 1)2 (𝑠𝑒𝑐 2 𝜃)2
𝑡𝑎𝑛4 𝜃
= 2∫ 𝑑𝜃
𝑠𝑒𝑐 2 𝜃
(𝑠𝑒𝑐 2 𝜃 − 1)2
= 2∫ 𝑑𝜃
𝑠𝑒𝑐 2 𝜃
𝑠𝑒𝑐 4 𝜃 − 2 𝑠𝑒𝑐 2 𝜃 + 1
= 2∫ 𝑑𝜃
𝑠𝑒𝑐 2 𝜃
= 2 ∫(𝑠𝑒𝑐 2 𝜃 − 2 + 𝑐𝑜𝑠 2 𝜃) 𝑑𝜃
132
1 1
= 2 ∫ (𝑠𝑒𝑐 2 𝜃 − 2 + + 𝑐𝑜𝑠 2𝜃) 𝑑𝜃
2 2
Going back to the original substitution, 𝑢 = 𝑡𝑎𝑛 𝜃, √𝑢2 + 1 = 𝑠𝑒𝑐 𝜃, 1/√𝑢2 + 1 = 𝑐𝑜𝑠 𝜃,
and 𝑢/√𝑢2 + 1 = 𝑠𝑖𝑛 𝜃.
𝑢4 𝑢
2∫ 2 2
𝑑𝑢 = 2𝑢 − 3 𝑡𝑎𝑛−1 𝑢 + 2 +𝐶
(𝑢 + 1) 𝑢 +1
Finally, if 𝑢2 = 𝑡 − 1,
(𝑡 − 1)3/2 −1 √𝑡 − 1
∫ 𝑑𝑡 = 2√𝑡 − 1 − 3 𝑡𝑎𝑛 √𝑡 − 1 + +𝐶 ANS.
𝑡2 𝑡
𝑢4 𝑢4
2∫ 2 𝑑𝑢 = 2 ∫ 4 𝑑𝑢
(𝑢 + 1)2 𝑢 + 2𝑢2 + 1
2𝑢2 + 1
= 2 ∫ [1 − 2 ] 𝑑𝑢
(𝑢 + 1)2
2𝑢2 + 1
= 2 ∫ 𝑑𝑢 − 2 ∫ 2 𝑑𝑢
(𝑢 + 1)2
Using the same trigonometric substitution, the second integral on the right becomes
2𝑢2 + 1 2 𝑡𝑎𝑛2 𝜃 + 1
2∫ 𝑑𝑢 = 2 ∫ ∙ 𝑠𝑒𝑐 2 𝜃 𝑑𝜃
(𝑢2 + 1)2 (𝑠𝑒𝑐 2 𝜃)2
2 (𝑡𝑎𝑛2 𝜃 + 1) − 1
= 2∫ 𝑑𝜃
𝑠𝑒𝑐 2 𝜃
= 2 ∫(2 − 𝑐𝑜𝑠 2 𝜃) 𝑑𝜃
= 2 ∫(2 − 𝑐𝑜𝑠 2 𝜃) 𝑑𝜃
133
1 1
= 2 ∫ (2 − − 𝑐𝑜𝑠 2𝜃) 𝑑𝜃
2 2
1
= 3𝜃 − 𝑠𝑖𝑛 2𝜃 + 𝐶
2
It is left as an exercise to continue this to show that it leads to the same answer.
(𝑡𝑎𝑛2 𝜃)3/2
= 2∫ ∙ 𝑡𝑎𝑛 𝜃 𝑑𝜃
𝑠𝑒𝑐 2 𝜃
𝑡𝑎𝑛4 𝜃
= 2∫ 𝑑𝜃
𝑠𝑒𝑐 2 𝜃
Note that the result is the same as the transformed integrand shown above. Hence, if 𝑡 =
𝑠𝑒𝑐 2 𝜃, 𝑡𝑎𝑛 𝜃 = √𝑡 − 1, 𝑐𝑜𝑠 𝜃 = 1/𝑡, 𝑎𝑛𝑑 𝑠𝑖𝑛 𝜃 = √𝑡 − 1/𝑡. Finally,
(𝑡 − 1)3/2 −1 √𝑡 − 1
∫ 𝑑𝑡 = 2√𝑡 − 1 − 3 𝑡𝑎𝑛 √𝑡 − 1 + +𝐶
𝑡2 𝑡
_____________________________________________________________________________________________________
134
The following examples will illustrate the technique of integration by hyperbolic
substitution.
_____________________________________________________________________________________________________
Example 8.17.
Evaluate
∫ √𝑥 2 + 9 𝑑𝑥
= 9 ∫ 𝑐𝑜𝑠ℎ2 𝜃 𝑑𝜃
1 1
= 9 ∫ ( 𝑐𝑜𝑠ℎ 2𝜃 + ) 𝑑𝜃
2 2
1 1
= 9 ( 𝑠𝑖𝑛ℎ 2𝜃 + 𝜃 +) + 𝐶
4 2
1 1
= 9 ( ∙ 2 𝑠𝑖𝑛ℎ 𝜃 𝑐𝑜𝑠ℎ 𝜃 + 𝜃) + 𝐶
4 2
9 9
= 𝑠𝑖𝑛ℎ 𝜃 𝑐𝑜𝑠ℎ 𝜃 + 𝜃 + 𝐶
2 2
Going back to the original substitution, we get
9 9 9 𝑥 √𝑥 2 + 9
𝑠𝑖𝑛ℎ 𝜃 𝑐𝑜𝑠ℎ 𝜃 + 𝜃 + 𝐶 = ∙ ∙ +𝐶
2 2 2 3 3
Finally,
1 9 𝑥
∫ √𝑥 2 + 9 𝑑𝑥 = 𝑥√𝑥 2 + 9 + 𝑠𝑖𝑛ℎ−1 + 𝐶 ANS.
2 2 3
_____________________________________________________________________________________________________
135
Example 8.18.
Evaluate
∫ √𝑥 2 − 25 𝑑𝑥
= 25 ∫ 𝑠𝑖𝑛ℎ2 𝜃 𝑑𝜃
1 1
= 25 ∫ ( 𝑐𝑜𝑠ℎ 2𝜃 − ) 𝑑𝜃
2 2
1 1
= 25 ( 𝑠𝑖𝑛ℎ 2𝜃 − 𝜃) + 𝐶
4 2
1 1
= 25 ( ∙ 2 𝑠𝑖𝑛ℎ 𝜃 𝑐𝑜𝑠ℎ 𝜃 − 𝜃) + 𝐶
4 2
25 25
= 𝑠𝑖𝑛ℎ 𝜃 𝑐𝑜𝑠ℎ 𝜃 − 𝜃 + 𝐶
2 2
Going back to the original substitution, we get
25 25 25 𝑥 √𝑥 2 − 25 25 𝑥
𝑠𝑖𝑛ℎ 𝜃 𝑐𝑜𝑠ℎ 𝜃 − 𝜃 + 𝐶 = ∙ ∙ − 𝑐𝑜𝑠ℎ−1 + 𝐶
2 2 2 5 5 2 5
Finally,
1 25 𝑥
∫ √𝑥 2 − 25 𝑑𝑥 = 𝑥 √𝑥 2 − 25 − 𝑐𝑜𝑠ℎ−1 + 𝐶 ANS.
2 2 5
_____________________________________________________________________________________________________
Example 8.19.
Evaluate
∫ √4 − 𝑥 2 𝑑𝑥
136
Solution. If we let 𝑥 = 2 𝑡𝑎𝑛ℎ 𝜃, 𝑑𝑥 = 2 𝑠𝑒𝑐ℎ2 𝜃 𝑑𝜃, and √4 − 𝑥 2 = 2 𝑠𝑒𝑐ℎ 𝜃. The
integral reduces to
= 4 ∫ 𝑠𝑒𝑐ℎ3 𝜃 𝑑𝜃
It is left as an exercise to show, using integration by parts similar to Example 7.8., that
1 1
∫ 𝑠𝑒𝑐ℎ3 𝜃 𝑑𝜃 = 𝑠𝑒𝑐ℎ 𝑥 𝑡𝑎𝑛ℎ 𝑥 + 𝑠𝑖𝑛−1 (𝑡𝑎𝑛ℎ 𝑥) + 𝐶
2 2
Thus,
1 1
4 ∫ 𝑠𝑒𝑐ℎ3 𝜃 𝑑𝜃 = 4 ( 𝑠𝑒𝑐ℎ 𝑥 𝑡𝑎𝑛ℎ 𝑥 + 𝑠𝑖𝑛−1 (𝑡𝑎𝑛ℎ 𝑥)) + 𝐶
2 2
−1 (𝑡𝑎𝑛ℎ
√4 − 𝑥 2 𝑥 𝑥
2 𝑠𝑒𝑐ℎ 𝑥 𝑡𝑎𝑛ℎ 𝑥 + 2 𝑠𝑖𝑛 𝑥) + 𝐶 = 2 ∙ ∙ + 2 𝑠𝑖𝑛−1 + 𝐶
2 2 2
Finally,
1 𝑥
∫ √4 − 𝑥 2 𝑑𝑥 = 𝑥 √4 − 𝑥 2 + 2 𝑠𝑖𝑛−1 + 𝐶 ANS.
2 2
_____________________________________________________________________________________________________
Example 8.20.
Evaluate
∫ 𝑠𝑒𝑐 3 𝑥 𝑑𝑥
Solution. We encountered this integral in Example 7.10. This is one occasion where we
can also use hyperbolic substitution instead of trigonometric substitution. Rewriting the
integral,
𝑐𝑜𝑠 𝑥
∫ 𝑠𝑒𝑐 3 𝑥 𝑑𝑥 = ∫ 𝑑𝑥
𝑐𝑜𝑠 4 𝑥
137
𝑐𝑜𝑠 𝑥
=∫ 𝑑𝑥
(1 − 𝑠𝑖𝑛2 𝑥)2
𝑑𝑢
=∫
(1 − 𝑢2 )2
noting that 𝑢 = 𝑠𝑖𝑛 𝑥 and 𝑑𝑢 = 𝑐𝑜𝑠 𝑥 𝑑𝑥. To solve this transformed integral, we let 𝑢 =
𝑡𝑎𝑛ℎ 𝜃 so that 𝑑𝑢 = 𝑠𝑒𝑐ℎ2 𝜃 𝑑𝜃 and 1 − 𝑢2 = 𝑠𝑒𝑐ℎ2 𝜃. Hence,
𝑑𝑢 𝑠𝑒𝑐ℎ2 𝜃
∫ =∫ 𝑑𝜃
(1 − 𝑢2 )2 𝑠𝑒𝑐ℎ4 𝜃
= ∫ 𝑐𝑜𝑠ℎ2 𝜃 𝑑𝜃
1 1
= ∫ ( 𝑐𝑜𝑠ℎ 2𝜃 + ) 𝑑𝑥
2 2
1 1
= 𝑠𝑖𝑛ℎ 2𝜃 + 𝜃 + 𝐶
4 2
1 1
= 𝑠𝑖𝑛ℎ 𝜃 𝑐𝑜𝑠ℎ 𝜃 + 𝜃 + 𝐶
2 2
If 𝑢 = 𝑡𝑎𝑛ℎ 𝜃 and 1 − 𝑢2 = 𝑠𝑒𝑐ℎ2 𝜃,
1 1
𝑐𝑜𝑠ℎ 𝜃 = =
𝑠𝑒𝑐ℎ 𝜃 √1 − 𝑢2
𝑠𝑖𝑛ℎ 𝜃
𝑡𝑎𝑛ℎ 𝜃 = =𝑢
𝑐𝑜𝑠ℎ 𝜃
𝑢
𝑠𝑖𝑛ℎ 𝜃 =
√1 − 𝑢2
Thus,
𝑑𝑢 1 1
∫ 2 2
= 𝑠𝑖𝑛ℎ 𝜃 𝑐𝑜𝑠ℎ 𝜃 + 𝜃 + 𝐶
(1 − 𝑢 ) 2 2
1 𝑢 1 1
= ∙ ∙ + 𝑡𝑎𝑛ℎ−1 𝑢 + 𝐶
2 √1 − 𝑢2 √1 − 𝑢2 2
1 𝑢 1
= + 𝑡𝑎𝑛ℎ−1 𝑢 + 𝐶
2 1 − 𝑢2 2
Finally,
138
1 𝑠𝑖𝑛 𝑥 1
∫ 𝑠𝑒𝑐 3 𝑥 𝑑𝑥 = 2
+ 𝑡𝑎𝑛ℎ−1 (𝑠𝑖𝑛 𝑥) + 𝐶
2 1 − 𝑠𝑖𝑛 𝑥 2
1 𝑠𝑖𝑛 𝑥 1
= 2
+ 𝑡𝑎𝑛ℎ−1 (𝑠𝑖𝑛 𝑥) + 𝐶
2 𝑐𝑜𝑠 𝑥 2
1 1
= 𝑠𝑒𝑐 𝑥 𝑡𝑎𝑛 𝑥 + 𝑡𝑎𝑛ℎ−1 (𝑠𝑖𝑛 𝑥) + 𝐶 ANS.
2 2
It is left as an exercise to show that this is equivalent to the answer in Example 7.10.
_____________________________________________________________________________________________________
Example 8.21.
Show that
𝑑𝑥 1 𝑥
∫ = − 𝑠𝑒𝑐ℎ−1 + 𝐶
𝑥√𝑎2 − 𝑥 2 𝑎 𝑎
𝑑𝑥 𝑎 𝑠𝑒𝑐ℎ2 𝜃 𝑑𝜃
∫ =∫
𝑥√𝑎2 − 𝑥 2 𝑎 𝑡𝑎𝑛ℎ 𝜃 ∙ 𝑎 𝑠𝑒𝑐ℎ 𝜃
1
= ∫ 𝑐𝑠𝑐ℎ 𝜃 𝑑𝜃
𝑎
1
= − 𝑠𝑒𝑐ℎ−1 (𝑡𝑎𝑛ℎ 𝜃) + 𝐶
𝑎
Going back to the original substitution, we have finally
𝑑𝑥 1 𝑥
∫ = − 𝑠𝑒𝑐ℎ−1 + 𝐶 𝑞. 𝑒. 𝑑.
𝑥√𝑎2 − 𝑥 2 𝑎 𝑎
_____________________________________________________________________________________________________
139
Try these.
1
∫ 𝑑𝑦
1. 𝑦√9 − 𝑦 2
2. ∫ 𝑥 2 √𝑥 2 − 4 𝑑𝑥
𝑠𝑖𝑛 𝑥
3. ∫ 𝑑𝑥
√1 + 𝑐𝑜𝑠 2 𝑥
√1 + (𝑙𝑛 𝑥)2
4. ∫ 𝑑𝑥
𝑥
√𝑒 𝑥
5. ∫ 𝑑𝑥
√1 + 𝑒 𝑥
6. ∫ √8𝑥 − 𝑥 2 𝑑𝑥
1
7. ∫ 𝑑𝑥
√𝑥 2 − 6𝑥 + 5
8. ∫(𝑥 2 − 1)3/2 𝑑𝑥
𝑡−1
9. ∫√ 𝑑𝑡
𝑡+1
1
10. ∫ 𝑑𝑡
(1 + 𝑡 2 )2
140
Integration by Weierstrass and
9. Euler Substitutions
Weierstrass Substitution
So far, we have learned about algebraic, trigonometric, and hyperbolic
substitutions. There is another substitution procedure for integrating trigonometric
functions 𝑓(cos 𝑥, sin 𝑥) using the tangent of half angles. This procedure is usually
attributed to Karl Weierstrass, a German mathematician (although this is being
disputed). If we let 𝑡 = 𝑡𝑎𝑛 𝑥2, 𝑥 = 2 𝑡𝑎𝑛−1 𝑡 for which
2
𝑑𝑥 = 𝑑𝑡
1 + 𝑡2
Using basic trigonometric identities, we derive 𝑠𝑒𝑐 12 𝑥 and eventually, 𝑐𝑜𝑠 12 𝑥 and 𝑠𝑖𝑛 12 𝑥
as follows:
1 1
𝑠𝑒𝑐 2 𝑥 = √1 + 𝑡𝑎𝑛2 2 𝑥 = √1 + 𝑡 2
1 1 1
𝑐𝑜𝑠 2 𝑥 = 1 =
𝑠𝑒𝑐 2 𝑥 √1 + 𝑡 2
1 1 𝑡
𝑠𝑖𝑛 2 𝑥 = √1 − 𝑐𝑜𝑠 2 2 𝑥 =
√1 + 𝑡 2
141
1 12𝑡
𝑠𝑖𝑛 𝑥 = 2 𝑐𝑜𝑠 2 𝑥 𝑠𝑖𝑛 2 𝑥 =
1 + 𝑡2
_____________________________________________________________________________________________________
Example 9.1.
Evaluate
𝑑𝑥
∫
2 𝑐𝑜𝑠 𝑥 + 3 𝑠𝑖𝑛 𝑥
𝑑𝑥 1 2 𝑑𝑡
∫ =∫ ∙
2 𝑐𝑜𝑠 𝑥 + 3 𝑠𝑖𝑛 𝑥 1− 𝑡2 2𝑡 1 + 𝑡2
2( 2 ) + 3( )
1+𝑡 1 + 𝑡2
2 𝑑𝑡
=∫
2 − 2𝑡 2 + 6𝑡
𝑑𝑡
=∫
1 + 3𝑡 − 𝑡 2
𝑑𝑡
=∫
9 9
1 + 4 − (𝑡 2 − 3𝑡 + 4)
𝑑𝑡
=∫
13 3 2
− (𝑡 − )
4 2
2 𝑡−3
= 𝑡𝑎𝑛ℎ−1 ( √13 2) + 𝐶
√13 2
2 2𝑡 − 3
= 𝑡𝑎𝑛ℎ−1 ( )+𝐶
√13 √13
2 2 𝑡𝑎𝑛 𝑥2 − 3
= 𝑡𝑎𝑛ℎ−1 ( )+𝐶 ANS.
√13 √13
It is left as an exercise to show that this is equivalent to the answer in Example 2.8. (Note:
The answer may also be expressed as 𝑎𝑟𝑐𝑜𝑡ℎ if |2 𝑡𝑎𝑛 𝑥2 − 3| > √13.)
_____________________________________________________________________________________________________
142
Example 9.2.
Obtain new relations for the integral of 𝑡𝑎𝑛 𝑥, 𝑐𝑜𝑡 𝑥, 𝑠𝑒𝑐 𝑥 and 𝑐𝑠𝑐 𝑥.
Solutions.
a)
𝑠𝑖𝑛 𝑥
∫ 𝑡𝑎𝑛 𝑥 𝑑𝑥 = ∫ 𝑑𝑥
𝑐𝑜𝑠 𝑥
2𝑡/1 + 𝑡 2 2 𝑑𝑡
=∫
1 − 𝑡 /1 + 𝑡 1 + 𝑡 2
2 2
2𝑡 𝑑𝑡
= 2∫ ∙
1 − 𝑡2 1 + 𝑡2
2𝑡
= 2∫ 𝑑𝑡
1 − 𝑡4
2𝑡 1
2∫ 𝑑𝑡 = 2 ∫ 𝑑𝑢
1 − 𝑡4 1 − 𝑢2
= 2 𝑡𝑎𝑛ℎ−1 𝑢 + 𝐶
= 2 𝑡𝑎𝑛ℎ−1 𝑡 2 + 𝐶
1
∫ 𝑡𝑎𝑛 𝑥 𝑑𝑥 = 2 𝑡𝑎𝑛ℎ−1 (𝑡𝑎𝑛2 2𝑥) + 𝐶 ANS.
b)
𝑐𝑜𝑠 𝑥
∫ 𝑐𝑜𝑡 𝑥 𝑑𝑥 = ∫ 𝑑𝑥
𝑠𝑖𝑛 𝑥
1 − 𝑡 2 /1 + 𝑡 2 2 𝑑𝑡
=∫ ∙
2𝑡/1 + 𝑡 2 1 + 𝑡2
1 − 𝑡2 𝑑𝑡
=∫ ∙
𝑡 1 + 𝑡2
1 − 𝑡2
=∫ 𝑑𝑡
𝑡 + 𝑡3
Dividing by 𝑡 2 /𝑡 2 gives
143
1 − 𝑡2 𝑡2 1/𝑡 2 − 1
∫ ÷ 𝑑𝑡 = ∫ 𝑑𝑡
𝑡 + 𝑡3 𝑡2 1/𝑡 + 𝑡
1/𝑡 2 − 1 𝑑𝑢
∫ 𝑑𝑡 = − ∫
1/𝑡 + 𝑡 𝑢
= −𝑙𝑛 |𝑢| + 𝐶
1
= −𝑙𝑛 | + 𝑡| + 𝐶
𝑡
1 1
= −𝑙𝑛 |𝑐𝑜𝑡 2𝑥 + 𝑡𝑎𝑛 2𝑥| + 𝐶
1
= −𝑙𝑛 | 1 |+𝐶
𝑠𝑖𝑛 2
𝑥 𝑐𝑜𝑠 12𝑥
1 1
= 𝑙𝑛 |𝑠𝑖𝑛 2𝑥 𝑐𝑜𝑠 2𝑥| + 𝐶
𝑠𝑖𝑛 𝑥
= 𝑙𝑛 | |+𝐶
2
c)
1
∫ 𝑠𝑒𝑐 𝑥 𝑑𝑥 = ∫ 𝑑𝑥
𝑐𝑜𝑠 𝑥
1 + 𝑡 2 2 𝑑𝑡
=∫ ∙
1 − 𝑡2 1 + 𝑡2
𝑑𝑡
= 2∫
1 − 𝑡2
144
= 2 𝑡𝑎𝑛ℎ−1 𝑡 + 𝐶
1
∫ 𝑠𝑒𝑐 𝑥 𝑑𝑥 = 2 𝑡𝑎𝑛ℎ−1 (𝑡𝑎𝑛 2𝑥) + 𝐶 ANS.
d)
1
∫ 𝑐𝑠𝑐 𝑥 𝑑𝑥 = ∫ 𝑑𝑥
𝑠𝑖𝑛 𝑥
1 + 𝑡 2 2 𝑑𝑡
=∫ ∙
2𝑡 1 + 𝑡2
𝑑𝑡
=∫
𝑡
= 𝑙𝑛 |𝑡| + 𝐶
1
∫ 𝑐𝑠𝑐 𝑥 𝑑𝑥 = 𝑙𝑛 |𝑡𝑎𝑛 2𝑥| + 𝐶 ANS.
_____________________________________________________________________________________________________
Example 9.3.
Evaluate
1 + 𝑠𝑖𝑛 𝑥
∫ 𝑑𝑥
1 + 𝑐𝑜𝑠 𝑥
2𝑡
1 + 𝑠𝑖𝑛 𝑥 1+( )
∫ 𝑑𝑥 = ∫ 1 + 𝑡 2 ∙ 2 𝑑𝑡
1 + 𝑐𝑜𝑠 𝑥 1 − 𝑡2 1 + 𝑡2
1+( )
1 + 𝑡2
1 + 𝑡 2 + 2𝑡 2 𝑑𝑡
=∫ ∙
2 1 + 𝑡2
2𝑡
= ∫ (1 + ) 𝑑𝑡
1 + 𝑡2
= 𝑡 + 𝑙𝑛 (1 + 𝑡 2 ) + 𝐶
1 1
= 𝑡𝑎𝑛 2𝑥 + 𝑙𝑛 (1 + 𝑡𝑎𝑛2 2𝑥) + 𝐶
145
1 1
= 𝑡𝑎𝑛 2𝑥 + 2 𝑙𝑛 (𝑠𝑒𝑐 2𝑥) + 𝐶 ANS.
_____________________________________________________________________________________________________
Example 9.4.
Evaluate
∫ 𝑐𝑠𝑐 𝑡 𝑐𝑜𝑡 2 𝑡 𝑑𝑡
Solution. We evaluated this integral in Example 2.18. The Weierstrass procedure also
works as well for other trigonometric functions. Using the Weierstrass substitution, we
transform the integral into
2
2
1 + 𝑡2 1 − 𝑡2 2 𝑑𝑡
∫ 𝑐𝑠𝑐 𝑡 𝑐𝑜𝑡 𝑡 𝑑𝑡 = ∫ ∙( ) ∙
2𝑡 2𝑡 1 + 𝑡2
𝑡 4 − 2𝑡 2 + 1
=∫ 𝑑𝑡
4𝑡 3
1 2 1
= ∫ (𝑡 − + 3 ) 𝑑𝑡
4 𝑡 𝑡
1 1 1
= ( 𝑡 2 − 2 𝑙𝑛 𝑡 − 2 ) + 𝐶
4 2 2𝑡
1 1 1 1 1 1
= 𝑡𝑎𝑛2 2𝑥 − 𝑙𝑛 𝑡𝑎𝑛 2𝑥 − 𝑐𝑜𝑡 2 2𝑥 + 𝐶 ANS.
8 2 8
It is left as an exercise to show that this answer is equivalent to the answer in Example
2.18.
_____________________________________________________________________________________________________
We can derive similar substitutions for hyperbolic functions 𝑓(cosh 𝑥, sinh 𝑥). If
we let 𝑡 = 𝑡𝑎𝑛ℎ 𝑥2, 𝑥 = 2 𝑡𝑎𝑛ℎ−1 𝑡 for which
2
𝑑𝑥 = 𝑑𝑡
1 − 𝑡2
146
Using basic hyperbolic identities, we derive 𝑠𝑒𝑐ℎ 12 𝑥 and eventually, 𝑐𝑜𝑠ℎ 12 𝑥 and 𝑠𝑖𝑛ℎ 12 𝑥
as follows:
1 1
𝑠𝑒𝑐ℎ 2 𝑥 = √1 − 𝑡𝑎𝑛ℎ2 2 𝑥 = √1 − 𝑡 2
1 1 1
𝑐𝑜𝑠ℎ 2 𝑥 = 1 =
𝑠𝑒𝑐ℎ 2 𝑥 √1 − 𝑡 2
1 1 𝑡
𝑠𝑖𝑛ℎ 2 𝑥 = √𝑐𝑜𝑠ℎ2 2 𝑥 − 1 =
√1 − 𝑡 2
1 1 2𝑡
𝑠𝑖𝑛ℎ 𝑥 = 2 𝑐𝑜𝑠ℎ 2 𝑥 𝑠𝑖𝑛ℎ 2 𝑥 =
1 − 𝑡2
Note the difference in the sign of 𝑡 2 in the equations: +𝑡 2 for trigonometric and −𝑡 2 for
hyperbolic functions.
_____________________________________________________________________________________________________
Example 9.5.
Obtain new relations for the integral of 𝑡𝑎𝑛ℎ 𝑥, 𝑐𝑜𝑡ℎ 𝑥, 𝑠𝑒𝑐ℎ 𝑥 and 𝑐𝑠𝑐ℎ 𝑥.
Solutions.
a)
𝑠𝑖𝑛ℎ 𝑥
∫ 𝑡𝑎𝑛ℎ 𝑥 𝑑𝑥 = ∫ 𝑑𝑥
𝑐𝑜𝑠ℎ 𝑥
2𝑡/1 − 𝑡 2 2 𝑑𝑡
=∫ ∙
1 + 𝑡 2 /1 − 𝑡 2 1 − 𝑡 2
2𝑡 𝑑𝑡
= 2∫ ∙
1 + 𝑡 1 − 𝑡2
2
2𝑡
= 2∫ 𝑑𝑡
1 − 𝑡4
2𝑡 1
2∫ 4
𝑑𝑡 = 2 ∫ 𝑑𝑢
1−𝑡 1 − 𝑢2
147
= 2 𝑡𝑎𝑛ℎ−1 𝑢 + 𝐶
= 2 𝑡𝑎𝑛ℎ−1 𝑡 2 + 𝐶
1
∫ 𝑡𝑎𝑛ℎ 𝑥 𝑑𝑥 = 2 𝑡𝑎𝑛ℎ−1 (𝑡𝑎𝑛ℎ2 2𝑥) + 𝐶 ANS.
b)
𝑐𝑜𝑠ℎ 𝑥
∫ 𝑐𝑜𝑡ℎ 𝑥 𝑑𝑥 = ∫ 𝑑𝑥
𝑠𝑖𝑛ℎ 𝑥
1 + 𝑡 2 /1 − 𝑡 2 2 𝑑𝑡
=∫ ∙
2𝑡/1 − 𝑡 2 1 − 𝑡2
1 + 𝑡2 𝑑𝑡
=∫ ∙
𝑡 1 − 𝑡2
1 + 𝑡2
=∫ 𝑑𝑡
𝑡 − 𝑡3
Dividing by 𝑡 2 /𝑡 2 gives
1 + 𝑡2 𝑡2 1/𝑡 2 + 1
∫ ÷ 𝑑𝑡 = ∫ 𝑑𝑡
𝑡 − 𝑡3 𝑡2 1/𝑡 − 𝑡
1/𝑡 2 + 1 𝑑𝑢
∫ 𝑑𝑡 = − ∫
1/𝑡 − 𝑡 𝑢
= −𝑙𝑛 |𝑢| + 𝐶
1
= −𝑙𝑛 | − 𝑡| + 𝐶
𝑡
1 1
= −𝑙𝑛 |𝑐𝑜𝑡ℎ 2𝑥 − 𝑡𝑎𝑛ℎ 2𝑥| + 𝐶
148
1
= −𝑙𝑛 | |+𝐶
𝑠𝑖𝑛ℎ 12𝑥 𝑐𝑜𝑠ℎ 12𝑥
1 1
= 𝑙𝑛 |𝑠𝑖𝑛ℎ 2𝑥 𝑐𝑜𝑠ℎ 2𝑥| + 𝐶
𝑠𝑖𝑛ℎ 𝑥
= 𝑙𝑛 | |+𝐶
2
c)
1
∫ 𝑠𝑒𝑐ℎ 𝑥 𝑑𝑥 = ∫ 𝑑𝑥
𝑐𝑜𝑠ℎ 𝑥
1 − 𝑡 2 2 𝑑𝑡
=∫ ∙
1 + 𝑡2 1 − 𝑡2
𝑑𝑡
= 2∫
1 + 𝑡2
= 2 𝑡𝑎𝑛−1 𝑡 + 𝐶
1
∫ 𝑠𝑒𝑐ℎ 𝑥 𝑑𝑥 = 2 𝑡𝑎𝑛−1 (𝑡𝑎𝑛ℎ 2𝑥) + 𝐶 ANS.
d)
1
∫ 𝑐𝑠𝑐ℎ 𝑥 𝑑𝑥 = ∫ 𝑑𝑥
𝑠𝑖𝑛ℎ 𝑥
1 − 𝑡 2 2 𝑑𝑡
=∫ ∙
2𝑡 1 − 𝑡2
𝑑𝑡
=∫
𝑡
= 𝑙𝑛 |𝑡| + 𝐶
1
∫ 𝑐𝑠𝑐ℎ 𝑥 𝑑𝑥 = 𝑙𝑛 |𝑡𝑎𝑛ℎ 2𝑥| + 𝐶 ANS.
_____________________________________________________________________________________________________
149
Example 9.6.
Evaluate
1
∫ 𝑑𝑥
1 + 𝑠𝑖𝑛ℎ 𝑥
Solution. We encountered this integral in Example 6.12. and we offered two solutions.
We shall present a third solution here using Weierstrass substitution. The integral may
be rewritten as
1 1 2 𝑑𝑡
∫ 𝑑𝑥 = ∫ ∙
1 + 𝑠𝑖𝑛ℎ 𝑥 2𝑡 2
1+( 2 ) 1−𝑡
1−𝑡
2
=∫ 𝑑𝑡
1 − 𝑡 2 + 2𝑡
𝑑𝑡
= 2∫
2 − (𝑡 2 − 2𝑡 + 1)
𝑑𝑡
= 2∫
2 − (𝑡 − 1)2
2 𝑡−1
𝑡𝑎𝑛ℎ−1 ()+𝐶
= √2 √2
2 𝑡−1
𝑐𝑜𝑡ℎ−1 ( )+𝐶
{ √2 √2
−1
𝑡𝑎𝑛ℎ 𝑥2 − 1
√2 𝑡𝑎𝑛ℎ ( )+𝐶
√2
= ANS.
𝑡𝑎𝑛ℎ 𝑥2 − 1
√2 𝑐𝑜𝑡ℎ−1 ( )+𝐶
{ √2
_____________________________________________________________________________________________________
Example 9.7.
Evaluate
𝑑𝑥
∫
√1 + 𝑐𝑜𝑠ℎ 𝑥
Solution. We encountered this integral in Example 5.12. Using Weierstrass substitution,
we have
150
𝑑𝑥 1 2 𝑑𝑡
∫ =∫ ∙
√1 + 𝑐𝑜𝑠ℎ 𝑥 2 1 − 𝑡2
√1 + 1 + 𝑡 2
1−𝑡
1 2 𝑑𝑡
=∫ ∙
2 2 1 − 𝑡2
√1 − 𝑡 + 12 + 𝑡
1−𝑡
2 𝑑𝑡
= ∫
√2 √1 − 𝑡 2
−1
= { √2 𝑠𝑖𝑛 −1𝑡 + 𝐶
−√2 𝑐𝑜𝑠 𝑡 + 𝐶
𝑥
√2 𝑠𝑖𝑛−1 (𝑡𝑎𝑛ℎ 2) + 𝐶
={ 𝑥
ANS.
−√2 𝑐𝑜𝑠 −1 (𝑡𝑎𝑛ℎ 2) + 𝐶
It will be left as an exercise to show that these answers are equivalent to the answer in
Example 5.12.
_____________________________________________________________________________________________________
Example 9.8.
Evaluate
𝑑𝑥
∫
2 𝑐𝑜𝑠ℎ 𝑥 − 𝑠𝑖𝑛ℎ 𝑥 − 3
𝑑𝑥 1 2 𝑑𝑡
∫ =∫ ∙
2 𝑐𝑜𝑠ℎ 𝑥 − 𝑠𝑖𝑛ℎ 𝑥 − 3 1+ 𝑡22𝑡 1−𝑡 2 1 − 𝑡2
2( 2 )−( 2 ) − 3( 2 )
1−𝑡 1−𝑡 1−𝑡
𝑑𝑡
=∫
2 + 𝑡 2 − 2𝑡 − 3 + 3𝑡 2
𝑑𝑡
=∫
4𝑡 2 − 2𝑡 − 1
1 𝑑𝑡
= ∫ 2 1
4 𝑡 − 2𝑡 − 1 + 14 − 14
151
1 𝑑𝑡
= ∫
4 (𝑡 − 1)2 − 5
4 4
1 2 −1
𝑡 − 12
=− ∙ 𝑡𝑎𝑛ℎ ( )+𝐶
4 √5 √5/2
1 −1
2 𝑡𝑎𝑛ℎ 𝑥2 − 1
=− 𝑡𝑎𝑛ℎ ( )+𝐶 ANS.
2√5 √5
(Note: The answer may also be
1 2 𝑡𝑎𝑛ℎ 𝑥2 − 1
− 𝑐𝑜𝑡ℎ−1 ( )+𝐶
2√5 √5
Euler Substitution
Another less common method of evaluating integrals is Euler substitution. Euler
substitution is commonly used to evaluate the integrals of rational functions of the form
∫ ℝ (𝑥, √𝑎𝑥 2 + 𝑏𝑥 + 𝑐) 𝑑𝑥
There are three cases to consider when using Euler substitution. In each case, we
express the variable 𝑥 in terms of another variable 𝑡.
√𝑎𝑥 2 + 𝑏𝑥 + 𝑐 = ±𝑥√𝑎 + 𝑡
2
𝑎𝑥 2 + 𝑏𝑥 + 𝑐 = (±𝑥√𝑎 + 𝑡)
= 𝑎𝑥 2 ± 2𝑥𝑡 √𝑎 + 𝑡 2
(±2𝑡√𝑎 − 𝑏)𝑥 = 𝑐 − 𝑡 2
152
𝑐 − 𝑡2
𝑥=
±2𝑡√𝑎 − 𝑏
In using this substitution, either + or – can be used. The coefficient 𝑐 may or may not be
equal to zero. The idea behind this substitution is to remove the 𝑎𝑥 2 term.
√𝑎𝑥 2 + 𝑏𝑥 + 𝑐 = 𝑥𝑡 ± √𝑐
2
𝑎𝑥 2 + 𝑏𝑥 + 𝑐 = (𝑥𝑡 ± √𝑐)
= 𝑥 2 𝑡 2 ± 2𝑥𝑡√𝑐 + 𝑐
(𝑎 − 𝑡 2 )𝑥 2 = (±2𝑡√𝑐 − 𝑏)𝑥
±2𝑡√𝑐 − 𝑏
𝑥=
𝑎 − 𝑡2
In using this substitution, either + or – can be used. The coefficient 𝑎 may or may not be
equal to zero. The idea behind this substitution is to remove the coefficient 𝑐.
𝑎(𝑥 − 𝛽) = (𝑥 − 𝛼)𝑡 2
(𝑎 − 𝑡 2 )𝑥 = 𝑎𝛽 − 𝛼𝑡 2
𝑎𝛽 − 𝛼𝑡 2
𝑥=
𝑎 − 𝑡2
Of course, you may also use the root 𝛽. The idea behind this substitution is to remove the
term 𝑥 − 𝛼 or 𝑥 − 𝛽.
Euler substitution is not the technique of choice in most situations. While the idea
is to remove the irrational form √𝑎𝑥 2 + 𝑏𝑥 + 𝑐, sometimes the resulting integrand which
is a rational function of 𝑡 is also difficult to integrate and there is a need to decompose it
into partial fractions which we shall take up in the next chapter.
_____________________________________________________________________________________________________
153
Example 9.9.
Evaluate
𝑑𝑥
∫
√𝑥 2 − 3𝑥 + 2
Solution. We notice that 𝑎 > 0, 𝑐 > 0 and 𝑥 2 − 3𝑥 + 2 = (𝑥 − 1)(𝑥 − 2) with real roots.
So, we can use all the cases of Euler substitution here.
Case I. Let
√𝑥 2 − 3𝑥 + 2 = 𝑥 + 𝑡
𝑥 2 − 3𝑥 + 2 = 𝑥 2 + 2𝑥𝑡 + 𝑡 2
2 − 𝑡 2 = (2𝑡 + 3)𝑥
2 − 𝑡2
𝑥=
2𝑡 + 3
Getting the derivative of 𝑥,
𝑡 2 + 3𝑡 + 2
= −2 𝑑𝑡
(2𝑡 + 3)2
𝑑𝑥 1 𝑡 2 + 3𝑡 + 2
∫ =∫ ∙ (−2 ) 𝑑𝑡
√𝑥 2 − 3𝑥 + 2 2 − 𝑡2 (2𝑡 + 3)2
(2𝑡 + 3) + 𝑡
2𝑡 + 3 𝑡 2 + 3𝑡 + 2
= −2 ∫ ∙ 𝑑𝑡
(2 − 𝑡 2 ) + (2𝑡 + 3)𝑡 (2𝑡 + 3)2
𝑑𝑡
= −2 ∫
2𝑡 + 3
= −𝑙𝑛 (2𝑡 + 3) + 𝐶
154
Case II. Let
√𝑥 2 − 3𝑥 + 2 = 𝑥𝑡 + √2
𝑥 2 − 3𝑥 + 2 = 𝑥 2 𝑡 2 + 2√2 𝑥𝑡 + 2
𝑥 − 3 = 𝑥𝑡 2 + 2√2 𝑡
(1 − 𝑡 2 )𝑥 = 2√2 𝑡 + 3
2√2 𝑡 + 3
𝑥=
1 − 𝑡2
Getting the derivative of 𝑥,
√2 𝑡 2 + 3𝑡 + √2
=2 𝑑𝑡
(1 − 𝑡 2 )2
𝑑𝑥 1 √2 𝑡 2 + 3𝑡 + √2
∫ =∫ ∙ (2 ) 𝑑𝑡
√𝑥 2 − 3𝑥 + 2 2√2 𝑡 + 3 (1 − 𝑡 2 )2
( ) 𝑡 + √2
1 − 𝑡2
1 − 𝑡2 √2 𝑡 2 + 3𝑡 + √2
= 2∫ ∙( ) 𝑑𝑡
(2√2 𝑡 + 3)𝑡 + √2(1 − 𝑡 2 ) (1 − 𝑡 2 )2
1 √2 𝑡 2 + 3𝑡 + √2
= 2∫ ∙( ) 𝑑𝑡
(2√2 𝑡 2 + 3𝑡) + (√2 − √2𝑡 2 ) 1 − 𝑡2
𝑑𝑡
= 2∫
1 − 𝑡2
= 2 𝑡𝑎𝑛ℎ−1 𝑡 + 𝐶
√𝑥 2 − 3𝑥 + 2 − √2
= 2 𝑡𝑎𝑛ℎ−1 ( )+𝐶 ANS.
𝑥
√𝑥 2 − 3𝑥 + 2 = √(𝑥 − 1)(𝑥 − 2)
155
= (𝑥 − 1)𝑡
(𝑥 − 1)(𝑥 − 2) = (𝑥 − 1)2 𝑡 2
𝑥 − 2 = 𝑥𝑡 2 − 𝑡 2
(1 − 𝑡 2 )𝑥 = 2 − 𝑡 2
2 − 𝑡2
𝑥=
1 − 𝑡2
Getting the derivative of 𝑥,
(1 − 𝑡 2 )(−2𝑡) − (2 − 𝑡 2 )(−2𝑡)
𝑑𝑥 = 𝑑𝑡
(1 − 𝑡 2 )2
2𝑡
= 𝑑𝑡
(1 − 𝑡 2 )2
𝑑𝑥 1 2𝑡
∫ =∫ ∙ 𝑑𝑡
√𝑥 2 − 3𝑥 + 2 2− 𝑡2 (1 − 𝑡 2 )2
( − 1) 𝑡
1 − 𝑡2
1 − 𝑡2 2𝑡
=∫ ∙ 𝑑𝑡
𝑡 (1 − 𝑡 2 )2
𝑑𝑡
= 2∫
1 − 𝑡2
= 2 𝑡𝑎𝑛ℎ−1 𝑡 + 𝐶
√𝑥 2 − 3𝑥 + 2
= 2 𝑡𝑎𝑛ℎ−1 ( )+𝐶
𝑥−1
√𝑥 − 2
= 2 𝑡𝑎𝑛ℎ−1 ( )+𝐶 ANS.
√𝑥 − 1
𝑑𝑥 𝑑𝑥
∫ =∫
√𝑥 2 − 3𝑥 + 2
√𝑥 2 − 3𝑥 + 49 + 2 − 49
156
𝑑𝑥
=∫
2
√(𝑥 − 32) − 14
𝑥 − 3/2
= 𝑐𝑜𝑠ℎ−1 ( )+𝐶
1/2
= 𝑐𝑜𝑠ℎ−1(2𝑥 − 3) + 𝐶 ANS.
It is left as an exercise to show that these four answers are equivalent to each other.
_____________________________________________________________________________________________________
Example 9.10.
Evaluate
√4𝑥 − 𝑥 2
∫ 𝑑𝑥
𝑥
√−𝑥(𝑥 − 4) = 𝑥𝑡
−𝑥(𝑥 − 4) = 𝑥 2 𝑡 2
−(𝑥 − 4) = 𝑥𝑡 2
𝑥(𝑡 2 + 1) = 4
4
𝑥=
𝑡2 + 1
8𝑡
𝑑𝑥 = − 𝑑𝑡
(𝑡 2 + 1)2
Substituting yields
4𝑡
√4𝑥 − 𝑥 2 2 8𝑡
∫ 𝑑𝑥 = ∫ 𝑡 + 1 ∙ (− 2 𝑑𝑡)
𝑥 4 (𝑡 + 1)2
𝑡2 + 1
157
2𝑡 2
= −4 ∫ 2 𝑑𝑡
(𝑡 + 1)2
We can solve this integral using integration by parts. We let 𝑢 = 𝑡 so that 𝑑𝑢 = 𝑑𝑡. If we
let
2𝑡
𝑑𝑣 = 𝑑𝑡
(𝑡 2 + 1)2
1
𝑣=−
𝑡2 +1
Hence,
2𝑡 2
∫ 2 𝑑𝑡 = 𝑢𝑣 − ∫ 𝑣 𝑑𝑢
(𝑡 + 1)2
𝑡 1
=− − ∫− 2 𝑑𝑡
𝑡2 +1 𝑡 +1
𝑡
=− + 𝑡𝑎𝑛−1 𝑡 + 𝐶
𝑡2 +1
Substituting the expression for 𝑡 from the Euler substitution we used, i.e.,
√4𝑥 − 𝑥 2
𝑡=
𝑥
we finally have,
√4𝑥 − 𝑥 2 𝑡
∫ 𝑑𝑥 = −4 (− 2 + 𝑡𝑎𝑛−1 𝑡) + 𝐶
𝑥 𝑡 +1
√4𝑥 − 𝑥 2
𝑥 √4𝑥 − 𝑥 2
=4 2 − 4 𝑡𝑎𝑛−1 ( )+𝐶
√4𝑥 − 𝑥 2 𝑥
( ) +1
𝑥
√4𝑥 − 𝑥 2
𝑥 −1
√4𝑥 − 𝑥 2
= − 4 𝑡𝑎𝑛 ( )+𝐶
4𝑥 − 𝑥 2 𝑥
+ 1
𝑥2
√4𝑥 − 𝑥 2
= √4𝑥 − 𝑥 2 − 4 𝑡𝑎𝑛−1 ( )+𝐶 ANS.
𝑥
158
It will be left as an exercise to show that this is equivalent to the answer in Example 8.8.
_____________________________________________________________________________________________________
Example 9.11.
Evaluate
1
∫ 𝑑𝑥
𝑥 2 √𝑥 2 − 1
Solution. We encountered this integral in Example 1.15 and 8.12. We can attempt to use
Euler substitution using either Case I or III. Using Case I, the proper substitution is
√𝑥 2 − 1 = 𝑥 + 𝑡
𝑥 2 − 1 = 𝑥 2 + 2𝑥𝑡 + 𝑡 2
𝑡2 + 1
𝑥=−
2𝑡
Getting the derivative of 𝑥,
2𝑡(2𝑡) − (𝑡 2 + 1)(2)
𝑑𝑥 = − 𝑑𝑡
4𝑡 2
1 − 𝑡2
= 𝑑𝑡
2𝑡 2
The integral reduces to
1 1 1 − 𝑡2
∫ 𝑑𝑥 = ∫ 2 ∙ 𝑑𝑡
𝑥 2 √𝑥 2 − 1 𝑡2 + 1 𝑡2 + 1 2𝑡 2
(− ) [(−
2𝑡 2𝑡 ) + 𝑡]
8𝑡 3 1 − 𝑡2
=∫ 2 ∙ 𝑑𝑡
(𝑡 + 1)2 (𝑡 2 − 1) 2𝑡 2
2𝑡
= −2 ∫ 𝑑𝑡
(𝑡 2 + 1)2
2
= +𝐶
𝑡2 +1
Substituting 𝑡, we have
159
1 2
∫ 𝑑𝑥 = 2 +𝐶
𝑥 2 √𝑥 2 − 1 (√𝑥 2 − 1 − 𝑥) + 1
Although this will now pass for an answer, we can further simplify this. The result is
1 √𝑥 2 − 1 − 𝑥
∫ 𝑑𝑥 = +𝐶 ANS.
𝑥 2 √𝑥 2 − 1 𝑥
Note that this is exactly the same answer we have in Example 1.15. and 8.12. when −𝑥/𝑥
is absorbed by the constant 𝐶 being equal to –1.
_____________________________________________________________________________________________________
Example 9.12.
Evaluate
𝑤3
∫ 𝑑𝑤
√𝑤 2 + 9
Solution. This is a familiar integral to you because we have evaluated this integral using
three different techniques. We’ll present a fourth one here using Euler substitution. Let’s
try a Case II substitution
√𝑤 2 + 9 = 𝑤𝑡 + 3
𝑤 2 + 9 = 𝑤 2 𝑡 2 + 6𝑤𝑡 + 9
𝑤(1 − 𝑡 2 ) = 6𝑡
6𝑡
𝑤=
1 − 𝑡2
Getting the derivative of 𝑤,
(1 − 𝑡 2 )(6) − 6𝑡(−2𝑡)
𝑑𝑤 = 𝑑𝑡
(1 − 𝑡 2 )2
6 − 6𝑡 2 + 12𝑡 2
= 𝑑𝑡
(1 − 𝑡 2 )2
1 + 𝑡2
𝑑𝑤 = 6 𝑑𝑡
(1 − 𝑡 2 )2
Substituting yields
160
6𝑡 3
𝑤 3 ] [ 1 + 𝑡2
∫ 𝑑𝑤 = ∫ 1 − 𝑡2 ∙ [6 ] 𝑑𝑡
√𝑤 2 + 9 6𝑡 (1 − 𝑡 2 )2
[( ) 𝑡 + 3]
1 − 𝑡2
3
𝑡
( ) 1 + 𝑡2
= 1296 ∫ 1 − 𝑡2 ∙ ( ) 𝑑𝑡
6𝑡 2 + 3 − 3𝑡 2 (1 − 𝑡 2 )2
( )
1 − 𝑡2
𝑡3
= 432 ∫ 𝑑𝑡
(1 − 𝑡 2 )4
1
To solve this, let 𝑢 = 1 − 𝑡 2 so that 𝑡 2 = 1 − 𝑢 and 𝑑𝑢 = −2𝑡 𝑑𝑡 or − 2 𝑑𝑢 = 𝑡𝑑𝑡. Thus,
𝑡3 1−𝑢
432 ∫ 2 4
𝑑𝑡 = −216 ∫ 4 𝑑𝑢
(1 − 𝑡 ) 𝑢
1 1
= −216 ∫ ( 4 − 3 ) 𝑑𝑢
𝑢 𝑢
72 108
= − 2 +𝐶
𝑢3 𝑢
72 108
= 2 3
− +𝐶
(1 − 𝑡 ) (1 − 𝑡 2 )2
𝑤3 72 108
∫ 𝑑𝑤 = − +𝐶
√𝑤 2 + 9 2 3 2 2
√𝑤 2 +9−3 √𝑤 2 +9−3
[1 − ( 𝑤 ) ] [1 − ( 𝑤 ) ]
72𝑤 6 108𝑤 4
= − +𝐶
2 3 2 2
[𝑤 2 − (√𝑤 2 + 9 − 3) ] [𝑤 2 − (√𝑤 2 + 9 − 3) ]
𝑤6 3𝑤 4
= 3− 2 +𝐶
3(√𝑤 2 + 9 − 3) (√𝑤 2 + 9 − 3)
1 3 2
= (√𝑤 2 + 9 + 3) − 3 (√𝑤 2 + 9 + 3) + 𝐶 ANS.
3
This will pass for an answer. But further simplifying gives
161
𝑤3 1 2
∫ 𝑑𝑤 = (𝑤 + 9)3/2 − 9√𝑤 2 + 9 − 18 + 𝐶
√𝑤 2 + 9 3
1
= (𝑤 2 + 9)3/2 − 9√𝑤 2 + 9 + 𝐶
3
which is exactly equal to the answer in Examples 1.11 and 8.14.
_____________________________________________________________________________________________________
162
Try these:
1
∫ 𝑑𝑦
1. 𝑦√9 − 𝑦 2
2. ∫ 𝑥 2 √𝑥 2 − 4 𝑑𝑥
𝑤
3. ∫ 𝑑𝑤
√1 + 𝑤 2
√1 + 𝑥 2
4. ∫ 𝑑𝑥
𝑥
√𝑥
5. ∫ 𝑑𝑥
√1 + 𝑥
6. ∫ √8𝑥 − 𝑥 2 𝑑𝑥
1
7. ∫ 𝑑𝑥
√𝑥 2 − 6𝑥 + 5
8. ∫(𝑥 2 − 1)3/2 𝑑𝑥
𝑡2 − 1
9. ∫√ 𝑑𝑡
𝑡2 + 1
1
10. ∫ 𝑑𝑡
(1 + 𝑡 2 )3/2
163
10. Integration by Partial Fractions
𝑃(𝑥)
∫ 𝑑𝑥
𝑄(𝑥)
𝑃(𝑥) 𝐴1 𝐴2 𝐴3 𝐴𝑛
= + + + . . .+
𝑄(𝑥) 𝑎1 𝑥 + 𝑏1 𝑎2 𝑥 + 𝑏2 𝑎3 𝑥 + 𝑏3 𝑎𝑛 𝑥 + 𝑏𝑛
_____________________________________________________________________________________________________
Example 10.1.
Evaluate
−6𝑥 − 7
∫ 𝑑𝑥
6𝑥 2 − 19𝑥 + 10
Solution. Identifying the factors of the denominator, the integrand can be written as
−6𝑥 − 7 𝐴 𝐵
= +
6𝑥 2 − 19𝑥 + 10 3𝑥 − 2 2𝑥 − 5
164
𝐴(2𝑥 − 5) + 𝐵(3𝑥 − 2)
=
(3𝑥 − 2)(2𝑥 − 5)
Thus,
𝑥: 2𝐴 + 3𝐵 = −6
𝑐: − 5𝐴 − 2𝐵 = −7
To solve for 𝐴 and 𝐵, we can multiply the first equation by 2 and the second by 3 and then
add to eliminate 𝐵 and solve for 𝐴. Then, we substitute the value of 𝐴 to either equation
to solve for 𝐵. The results are 𝐴 = 3 and 𝐵 = −4. Hence,
−6𝑥 − 7 3 4
∫ 𝑑𝑥 = ∫ ( − ) 𝑑𝑥
6𝑥 2 − 19𝑥 + 10 3𝑥 − 2 2𝑥 − 5
1 1
= 3 ∙ 𝑙𝑛 |3𝑥 − 2| − 4 ∙ 𝑙𝑛 |2𝑥 − 5| + 𝐶
3 2
= 𝑙𝑛 |3𝑥 − 2| − 2 𝑙𝑛 |2𝑥 − 5| + 𝐶 ANS.
_____________________________________________________________________________________________________
Example 10.2.
Evaluate
3𝑥 2 + 6
∫ 3 𝑑𝑥
2𝑥 − 3𝑥 2 − 3𝑥 + 2
Solution. Identifying the factors of the denominator, the integrand can be written as
3𝑥 2 + 6 𝐴 𝐵 𝐶
= + +
2𝑥 3 − 3𝑥 2 − 3𝑥 + 2 𝑥 + 1 𝑥 − 2 2𝑥 − 1
𝐴 𝐵 𝐶 𝐴(𝑥 − 2)(2𝑥 − 1) + 𝐵(𝑥 + 1)(2𝑥 − 1) + 𝐶(𝑥 + 1)(𝑥 − 2)
+ + =
𝑥 + 1 𝑥 − 2 2𝑥 − 1 (𝑥 + 1)(𝑥 − 2)(2𝑥 − 1)
Thus,
𝑥2: 2𝐴 + 2𝐵 + 𝐶 = 3
𝑥: − 5𝐴 + 𝐵 − 𝐶 = 0
𝑐: 2𝐴 − 𝐵 − 2𝐶 = 6
165
We have three equations containing three unknowns. To solve for 𝐴, 𝐵, and 𝐶, we
can obtain an expression for 𝐶 from the second equation and substitute this expression
for 𝐶 in the first and third equations. Thus, 𝐶 = −5𝐴 + 𝐵 and the first and third equations
become
−3𝐴 + 3𝐵 = 3
12𝐴 − 3𝐵 = 6
If we add these equations, we can eliminate 𝐵 and solve for 𝐴. The results are 𝐴 = 1, 𝐵 =
2 and 𝐶 = −3. Hence,
3𝑥 2 + 6 1 2 3
∫ 3 2
𝑑𝑥 = ∫ ( + − ) 𝑑𝑥
2𝑥 − 3𝑥 − 3𝑥 + 2 𝑥 + 1 𝑥 − 2 2𝑥 − 1
3
= 𝑙𝑛 |𝑥 + 1| + 2 𝑙𝑛 |𝑥 − 2| − 𝑙𝑛 |2𝑥 − 1| + 𝐶 ANS.
2
There are other ways of calculating the value of the coefficients. Using matrix
notation, we can write the system of equations above as
2 2 1 3
−5 1 −1 0
2 −1 −2 6
Our goal is to reduce to zero all the values on the matrix in the left, except the main
diagonal which should be reduced to 1, using row operations.
2 2 1 3 2 2 1 3
5R1 + 2R2 −5 1 −1 0 = 0 12 3 15
R1 − R3 2 −1 −2 6 0 3 3 −3
−3R1 + R2 2 2 1 3 −6 6 0 6
R2 – R3 0 12 3 15 = 0 9 0 18
R2 − 4R3 0 3 3 −3 0 0 −9 27
166
−3R1 + 2R2 −6 6 0 6 18 0 0 18
0 9 0 18 = 0 9 0 18
0 0 −9 27 0 0 −9 27
R1 ÷ 18 18 0 0 18 1 0 0 1
R1 ÷ 9 0 9 0 18 = 0 1 0 2
R1 ÷ − 9 0 0 −9 27 0 0 1 −3
In determining the value of the coefficients, we can dispense with the more
tedious process of solving simultaneous equations. We note that if 𝑄(𝑥) can be factored
into 𝑛 linear fractions, then for every non-repeated fraction 𝑥 − 𝑟𝑗 , there corresponds a
partial fraction 𝐴𝑗 /(𝑥 − 𝑟𝑗 ) such that
(𝑥 − 𝑟𝑗 )𝑃(𝑥)
𝐴𝑗 = lim
𝑥→𝑟𝑗 𝑄(𝑥)
Thus,
(𝑥 + 1)(3𝑥 2 + 6)
𝐴 = lim
𝑥→−1 (𝑥 + 1)(𝑥 − 2)(2𝑥 − 1)
3𝑥 2 + 6
= lim
𝑥→−1 (𝑥 − 2)(2𝑥 − 1)
3(−1)2 + 6
=
(−1 − 2)(2(−1) − 1)
3+6
𝐴= =1
(−3)(−3)
Similarly,
(𝑥 − 2)(3𝑥 2 + 6)
𝐵 = lim
𝑥→2 (𝑥 + 1)(𝑥 − 2)(2𝑥 − 1)
3𝑥 2 + 6
= lim
𝑥→2 (𝑥 + 1)(2𝑥 − 1)
167
3(2)2 + 6
=
(2 + 1)(2(2) − 1)
12 + 6
𝐵= =2
(3)(3)
(2𝑥 − 1)(3𝑥 2 + 6)
𝐶 = lim
𝑥→1/2 (𝑥 + 1)(𝑥 − 2)(2𝑥 − 1)
3𝑥 2 + 6
= lim
𝑥→1/2 (𝑥 + 1)(𝑥 − 2)
2
3(12) + 6
=
(12 + 1)(12 − 2)
3
4
+6
𝐶= = −3
(2)(−32)
3
_____________________________________________________________________________________________________
Example 10.3.
Evaluate
𝑒 2𝑥 − 3𝑒 𝑥
∫ 𝑑𝑥
𝑒 2𝑥 − 1
𝑒𝑥 − 3 𝐴 𝐵
𝑒 𝑥 ( 2𝑥 ) = 𝑒𝑥 ( 𝑥 + 𝑥 )
𝑒 −1 𝑒 +1 𝑒 −1
𝐴(𝑒 𝑥 − 1) + 𝐵(𝑒 𝑥 + 1)
= 𝑒 𝑥 [( )]
(𝑒 𝑥 + 1)(𝑒 𝑥 − 1)
(𝐴 + 𝐵)𝑒 𝑥 + (−𝐴 + 𝐵)
= 𝑒𝑥 [ ]
(𝑒 𝑥 + 1)(𝑒 𝑥 − 1)
Thus,
𝐴+𝐵 =1
−𝐴 + 𝐵 = −3
Adding both eliminates 𝐴 and solves for 𝐵. The results are 𝐴 = 2 and 𝐵 = −1. Hence,
168
𝑒𝑥 − 3 2 1
∫ 𝑒 𝑥 ( 2𝑥 ) 𝑑𝑥 = ∫ 𝑒 𝑥 ( 𝑥 − 𝑥 ) 𝑑𝑥
𝑒 −1 𝑒 +1 𝑒 −1
= 2 𝑙𝑛 (𝑒 𝑥 + 1) − 𝑙𝑛 (𝑒 𝑥 − 1) + 𝐶 ANS.
_____________________________________________________________________________________________________
Example 10.4.
Evaluate
2𝑥 3 + 13𝑥 2 − 5𝑥 − 34
∫ 𝑑𝑥
𝑥 4 − 5𝑥 2 + 4
2𝑥 3 + 13𝑥 2 − 5𝑥 − 34 𝐴 𝐵 𝐶 𝐷
= + + +
𝑥 4 − 5𝑥 2 + 4 𝑥+1 𝑥−1 𝑥+2 𝑥−2
Thus, using the same technique employed in Example 9.2.,
2𝑥 3 + 13𝑥 2 − 5𝑥 − 34
= lim
𝑥→−1 (𝑥 − 1)(𝑥 + 2)(𝑥 − 2)
18
=− = −3
6
Similar calculations reveal that
169
(𝑥 − 2)(2𝑥 3 + 13𝑥 2 − 5𝑥 − 34)
𝐷 = lim =2
𝑥→2 (𝑥 + 1)(𝑥 − 1)(𝑥 + 2)(𝑥 − 2)
2𝑥 3 + 13𝑥 2 − 5𝑥 − 34 3 4 1 2
∫ 4 2
𝑑𝑥 = ∫ (− + − + ) 𝑑𝑥
𝑥 − 5𝑥 + 4 𝑥+1 𝑥−1 𝑥+2 𝑥−2
= −3 𝑙𝑛 |𝑥 + 1| + 4 𝑙𝑛 |𝑥 − 1| − 𝑙𝑛 |𝑥 + 2|
+ 2 𝑙𝑛 |𝑥 − 2| + 𝐶
(𝑥 − 1)4 (𝑥 − 2)2
= 𝑙𝑛 | |+𝐶 ANS.
(𝑥 + 2)(𝑥 + 1)3
_____________________________________________________________________________________________________
Case II. All the factors of 𝑄(𝑥) are linear with at least one factor repeated.
𝐴1 𝐴2 𝐴3 𝐴𝑚
+ + + . . . +
(𝑎𝑥 + 𝑏)𝑚 (𝑎𝑥 + 𝑏)𝑚−1 (𝑎𝑥 + 𝑏)𝑚−2 𝑎𝑥 + 𝑏
_____________________________________________________________________________________________________
Example 10.5.
Evaluate
4𝑥 2 − 14𝑥 + 14
∫ 𝑑𝑥
4𝑥 2 − 12𝑥 + 9
Solution. Because the degree of the numerator equals the degree of the denominator, we
divide first the numerator by the denominator, thus,
4𝑥 2 − 14𝑥 + 14 −2𝑥 + 5
2
= 1+ 2
4𝑥 − 12𝑥 + 9 4𝑥 − 12𝑥 + 9
−2𝑥 + 5 𝐴 𝐵
= +
4𝑥 2 − 12𝑥 + 9 (2𝑥 − 3)2 2𝑥 − 3
𝐴 + 𝐵(2𝑥 − 3)
=
(2𝑥 − 3)2
170
2𝐵𝑥 + (𝐴 − 3𝐵)
=
(2𝑥 − 3)2
Thus,
2𝐵 = −2
𝐴 − 3𝐵 = 5
4𝑥 2 − 14𝑥 + 14 2 1
∫ 2
𝑑𝑥 = ∫ (1 + 2
− ) 𝑑𝑥
4𝑥 − 12𝑥 + 9 (2𝑥 − 3) 2𝑥 − 3
1 1
=𝑥− − 𝑙𝑛|2𝑥 − 3| + 𝐶 ANS.
2𝑥 − 3 2
_____________________________________________________________________________________________________
Example 10.6.
Evaluate
8𝑥 2 − 27𝑥 + 27
∫ 𝑑𝑥
𝑥 4 − 6𝑥 3 + 9𝑥 2
Solution. The factors of the denominator are 𝑥 2 and (𝑥 − 3)2 . Since both factors repeat,
the integrand may be written as
8𝑥 2 − 27𝑥 + 27 𝐴 𝐵 𝐶 𝐷
= + + +
𝑥 4 − 6𝑥 3 + 9𝑥 2 𝑥 2 𝑥 (𝑥 − 3)2 𝑥 − 3
Thus,
𝑥3: 𝐵+ 𝐷=0
𝑥2: 𝐴 − 6𝐵 + 𝐶 − 3𝐷 = 8
𝑥: − 6𝐴 + 9𝐵 = −27
𝑐: 9𝐴 = 27
171
Solving for the values, 𝐴 = 3, 𝐵 = −1, 𝐶 = 2 and 𝐷 = 1. Hence,
8𝑥 2 − 27𝑥 + 27 3 1 2 1
∫ 4 3 2
𝑑𝑥 = ∫ ( 2 − + 2
+ ) 𝑑𝑥
𝑥 − 6𝑥 + 9𝑥 𝑥 𝑥 (𝑥 − 3) 𝑥−3
3 2
= − − 𝑙𝑛 |𝑥| − + 𝑙𝑛 |𝑥 − 3| + 𝐶 ANS.
𝑥 𝑥−3
___________________________________________________________________________
Example 10.7.
Evaluate
∫ 𝑠𝑒𝑐 3 𝑥 𝑑𝑥
𝑐𝑜𝑠 𝑥 𝑑𝑢
∫ 𝑠𝑒𝑐 3 𝑥 𝑑𝑥 = ∫ 2 2
𝑑𝑥 = ∫
(1 − 𝑠𝑖𝑛 𝑥) (1 − 𝑢2 )2
1 1
2 2
=
(1 − 𝑢 ) (1 + 𝑢) (1 − 𝑢)2
2
𝐴 𝐵 𝐶 𝐷
= + + +
(1 + 𝑢)2 1 + 𝑢 (1 − 𝑢)2 1 − 𝑢
1 = 𝐴(1 − 𝑢)2 + 𝐵(1 + 𝑢)(1 − 𝑢)2 + 𝐶(1 + 𝑢)2 + 𝐷(1 − 𝑢)(1 + 𝑢)2
If you expand this and gather similar terms, we get the following:
𝑢3 ∶ 𝐵−𝐷 =0
𝑢2 ∶ 𝐴−𝐵+𝐶−𝐷 =0
𝑢 ∶ −2𝐴 − 𝐵 + 2𝐶 + 𝐷 = 0
𝑐 ∶ 𝐴+𝐵+𝐶+𝐷 =1
𝑑𝑢 𝐴 𝐵 𝐶 𝐷
∫ 2 2
= ∫[ 2
+ + 2
+ ] 𝑑𝑢
(1 − 𝑢 ) (1 + 𝑢) 1 + 𝑢 (1 − 𝑢) 1−𝑢
172
1 1 1 1 1
= ∫[ 2
+ + 2
+ ] 𝑑𝑢
4 (1 + 𝑢) 1 + 𝑢 (1 − 𝑢) 1−𝑢
1 1 1
= (− + 𝑙𝑛 |1 + 𝑢| + − 𝑙𝑛 |1 − 𝑢|) + 𝐶
4 1+𝑢 1−𝑢
1 1 1 1 1+𝑢
= ( − ) + 𝑙𝑛 | |+𝐶
4 1−𝑢 1+𝑢 4 1−𝑢
1 2𝑢 1 1+𝑢
= ( 2
) + 𝑙𝑛 | |+𝐶
4 1−𝑢 4 1−𝑢
1 2𝑢 1 1+𝑢
∫ 𝑠𝑒𝑐 3 𝑥 𝑑𝑥 = ( ) + 𝑙𝑛 | |+𝐶
4 1 − 𝑢2 4 1−𝑢
1 𝑠𝑖𝑛 𝑥 1 1 + 𝑠𝑖𝑛 𝑥
= ( 2
) + 𝑙𝑛 | |+𝐶
2 1 − 𝑠𝑖𝑛 𝑥 4 1 − 𝑠𝑖𝑛 𝑥
1 𝑠𝑖𝑛 𝑥 1 1 + 𝑠𝑖𝑛 𝑥 1 + 𝑠𝑖𝑛 𝑥
= ( 2 ) + 𝑙𝑛 | ∙ |+𝐶
2 𝑐𝑜𝑠 𝑥 4 1 − 𝑠𝑖𝑛 𝑥 1 + 𝑠𝑖𝑛 𝑥
1 1 𝑠𝑖𝑛 𝑥 1 1 + 𝑠𝑖𝑛 𝑥 2
= ( ∙ ) + 𝑙𝑛 |( ) |+𝐶
2 𝑐𝑜𝑠 𝑥 𝑐𝑜𝑠 𝑥 4 𝑐𝑜𝑠 𝑥
1 1
= 𝑠𝑒𝑐 𝑥 𝑡𝑎𝑛 𝑥 + 𝑙𝑛 |𝑠𝑒𝑐 𝑥 + 𝑡𝑎𝑛 𝑥| + 𝐶 ANS.
2 2
_____________________________________________________________________________________________________
We can deal with Case II rational functions using what is called the Ostrogradski
method. If 𝑃(𝑥) is a polynomial whose degree is less than 𝑄(𝑥) so that 𝑃(𝑥)/𝑄(𝑥) is a
proper fraction, then the integral of 𝑃(𝑥)/𝑄(𝑥) is
where 𝑄1 (𝑥) is the greatest divisor of 𝑄(𝑥) and 𝑄′(𝑥) with 𝑄2 (𝑥) = 𝑄(𝑥)/𝑄1 (𝑥). 𝑃1 (𝑥) and
𝑃2 (𝑥) are polynomials with unknown coefficients which may be obtained by
differentiating the above relation.
_____________________________________________________________________________________________________
173
Example 10.8.
Evaluate
1
∫ 𝑑𝑥
(𝑥 + 1)2 (𝑥 + 2)2
Solution. In this case, 𝑃(𝑥) = 1 and 𝑄(𝑥) = (𝑥 + 1)2 (𝑥 + 2)2 . Differentiating 𝑄(𝑥) gives
Hence, the greatest divisor of 𝑄(𝑥) and 𝑄′(𝑥) is 𝑄1 (𝑥) = (𝑥 + 1)(𝑥 + 2). This means that
𝑄2 (𝑥) = 𝑄(𝑥)/𝑄1 (𝑥) = (𝑥 + 1)(𝑥 + 2). Thus,
1 𝐴𝑥 + 𝐵 𝐶𝑥 + 𝐷
∫ 𝑑𝑥 = +∫ 𝑑𝑥
(𝑥 + 1)2 (𝑥 + 2)2 (𝑥 + 1)(𝑥 + 2) (𝑥 + 1)(𝑥 + 2)
Getting the derivative of the above equation removes the integration sign, thus,
𝑑 1 𝑑 𝐴𝑥 + 𝐵 𝑑 𝐶𝑥 + 𝐷
∫ 2 2
𝑑𝑥 = + ∫ 𝑑𝑥
𝑑𝑥 (𝑥 + 1) (𝑥 + 2) 𝑑𝑥 (𝑥 + 1)(𝑥 + 2) 𝑑𝑥 (𝑥 + 1)(𝑥 + 2)
1 2𝑥 + 3 2
∫ 𝑑𝑥 = − −∫ 𝑑𝑥
(𝑥 + 1)2 (𝑥 + 2)2 (𝑥 + 1)(𝑥 + 2) (𝑥 + 1)(𝑥 + 2)
To solve the integral on the right, we decompose the integrand into partial fractions, thus,
2 𝐸 𝐹
∫ 𝑑𝑥 = ∫ ( + ) 𝑑𝑥
(𝑥 + 1)(𝑥 + 2) 𝑥+1 𝑥+2
𝐸(𝑥 + 2) + 𝐹(𝑥 + 1)
=∫ 𝑑𝑥
(𝑥 + 1)(𝑥 + 2)
(𝐸 + 𝐹)𝑥 + (2𝐸 + 𝐹)
=∫ 𝑑𝑥
(𝑥 + 1)(𝑥 + 2)
174
This means that 𝐸 + 𝐹 = 0 and 2𝐸 + 𝐹 = 2. This results in 𝐸 = 2 and 𝐹 = −2. Thus,
2 2 2
∫ 𝑑𝑥 = ∫ ( − ) 𝑑𝑥
(𝑥 + 1)(𝑥 + 2) 𝑥+1 𝑥+2
= 2 𝑙𝑛 |𝑥 + 1| − 2 𝑙𝑛 |𝑥 + 2| + 𝐶
Finally,
1 2𝑥 + 3
∫ 𝑑𝑥 = − − (2 𝑙𝑛 |𝑥 + 1| − 2 𝑙𝑛 |𝑥 + 2|) + 𝐶
(𝑥 + 1)2 (𝑥 + 2)2 (𝑥 + 1)(𝑥 + 2)
2𝑥 + 3 𝑥+2
=− + 2 𝑙𝑛 | |+𝐶 ANS.
(𝑥 + 1)(𝑥 + 2) 𝑥+1
_____________________________________________________________________________________________________
Case III. Some of the factors of 𝑄(𝑥) are quadratic with no repetition.
𝑃(𝑥) 𝐴1 𝑥 + 𝐵1 𝐴2 𝑥 + 𝐵2 𝐴𝑛 𝑥 + 𝐵𝑛
= 2
+ 2
+ . . .+
𝑄(𝑥) 𝑎1 𝑥 + 𝑏1 𝑥 + 𝑐1 𝑎2 𝑥 + 𝑏2 𝑥 + 𝑐2 𝑎𝑛 𝑥 2 + 𝑏𝑛 𝑥 + 𝑐𝑛
_____________________________________________________________________________________________________
Example 10.9.
Evaluate
4𝑥 2 − 3𝑥 − 7
∫ 3 𝑑𝑥
𝑥 − 5𝑥 2 + 𝑥 − 5
Solution. The identified factors for the denominator are 𝑥 2 + 1 and 𝑥 − 5. The integrand
decomposes into
4𝑥 2 − 3𝑥 − 7 𝐴𝑥 + 𝐵 𝐶
= +
𝑥 3 − 5𝑥 2 + 𝑥 − 5 𝑥 2 + 1 𝑥 − 5
(𝐴𝑥 + 𝐵)(𝑥 − 5) + 𝐶(𝑥 2 + 1)
=
(𝑥 2 + 1)(𝑥 − 5)
Thus,
𝑥2 ∶ 𝐴+ 𝐶= 4
175
𝑥 ∶ −5𝐴 + 𝐵 = −3
𝑐 ∶ −5𝐵 + 𝐶 = −7
4𝑥 2 − 3𝑥 − 7 𝑥+2 3
∫ 3 2
𝑑𝑥 = ∫ ( 2 + ) 𝑑𝑥
𝑥 − 5𝑥 + 𝑥 − 5 𝑥 +1 𝑥−5
1 2𝑥 2 3
= ∫( ∙ 2 + 2 + ) 𝑑𝑥
2 𝑥 +1 𝑥 +1 𝑥−5
1
= 𝑙𝑛|𝑥 2 + 1| + 2 𝑡𝑎𝑛−1 𝑥 + 3 𝑙𝑛|𝑥 − 5| + 𝐶 ANS.
2
_____________________________________________________________________________________________________
Example 10.10.
Evaluate
3𝑥 2 + 10𝑥 + 4
∫ 𝑑𝑥
𝑥 3 + 5𝑥 2 + 4𝑥 − 4
3𝑥 2 + 10𝑥 + 4 𝐴𝑥 + 𝐵 𝐶
3 2
= 2 +
𝑥 + 5𝑥 + 4𝑥 − 4 𝑥 + 3𝑥 − 2 𝑥 + 2
(𝐴𝑥 + 𝐵)(𝑥 + 2) + 𝐶(𝑥 2 + 3𝑥 − 2)
=
(𝑥 2 + 3𝑥 − 2)(𝑥 + 2)
Thus,
𝑥2: 𝐴+ 𝐶=3
𝑥: 2𝐴 + 𝐵 + 3𝐶 = 10
𝑐: 2𝐵 − 2𝐶 = 4
176
3𝑥 2 + 10𝑥 + 4 2𝑥 + 3 1
∫ 3 𝑑𝑥 = ∫ ( + ) 𝑑𝑥
𝑥 + 5𝑥 2 + 4𝑥 − 4 𝑥 2 + 3𝑥 − 2 𝑥 + 2
= 𝑙𝑛 |𝑥 2 + 3𝑥 − 2| + 𝑙𝑛 |𝑥 + 2| + 𝐶 ANS.
𝑑𝑢
∫ = 𝑙𝑛 |𝑢| = 𝑙𝑛 |𝑥 3 + 5𝑥 2 + 4𝑥 − 4| + 𝐶
𝑢
Example 10.11.
Evaluate
𝑑𝑦
∫
𝑦4 +1
1 1
= 4
𝑦4 + 1 𝑦 + 2𝑦 + 1 − 2𝑦 2
2
1
= 2
(𝑦 2 + 1)2 − (√2 𝑦)
1
=
(𝑦 2 + √2 𝑦 + 1)(𝑦 2 − √2 𝑦 + 1)
1 𝐴𝑦 + 𝐵 𝐶𝑦 + 𝐷
= +
(𝑦 2 + √2 𝑦 + 1)(𝑦 2 − √2 𝑦 + 1) 𝑦 2 + √2 𝑦 + 1 𝑦 2 − √2 𝑦 + 1
1 √2
4
𝑦 + 12 −√2
4
𝑦 + 12
= +
(𝑦 2 + √2 𝑦 + 1)(𝑦 2 − √2 𝑦 + 1) 𝑦 2 + √2 𝑦 + 1 𝑦 2 − √2 𝑦 + 1
177
√2
4
𝑦 + 12 −√2
4
𝑦 + 12 √2
8
(2𝑦 + √2) + 14 −√2
8
(2𝑦 − √2) + 14
+ = +
𝑦 2 + √2 𝑦 + 1 𝑦 2 − √2 𝑦 + 1 𝑦 2 + √2 𝑦 + 1 𝑦 2 − √2 𝑦 + 1
This is deliberate because the term inside the parenthesis is equal to the derivative of its
denominator. The integral transforms into
𝑑𝑦 √2 2𝑦 + √2 1 𝑑𝑦
∫ = ∫ 𝑑𝑦 + ∫ 2
𝑦4+1 8 𝑦 + √2 𝑦 + 1
2 4 1
(𝑦 + ) + 12
√2
√2 2𝑦 − √2 1 𝑑𝑦
− ∫ 𝑑𝑦 + ∫ 2
8 𝑦 − √2 𝑦 + 1
2 4 1
(𝑦 − ) + 12
√2
√2 √2
= 𝑙𝑛|𝑦 2 + √2 𝑦 + 1| − 𝑙𝑛|𝑦 2 − √2 𝑦 + 1|
8 8
1 1
1 𝑦+ 1 𝑦−
+ ∙ √2 𝑡𝑎𝑛−1 ( 1 √2) + ∙ √2 𝑡𝑎𝑛−1 ( 1 √2)
4 √2
4 √2
√2 𝑦 2 + √2 𝑦 + 1 √2 √2
𝑙𝑛 | |+ 𝑡𝑎𝑛−1 (√2𝑦 + 1) + 𝑡𝑎𝑛−1 (√2𝑦 − 1) + 𝐶 ANS.
8 2
𝑦 − √2 𝑦 + 1 4 4
_____________________________________________________________________________________________________
Example 10.12.
Evaluate
1
∫ 𝑑𝑥
1 + 𝑡𝑎𝑛 𝑥
Solution. The reader will find out that this is equivalent to the integral in Example 2.10.
But, we shall present a different approach here using partial fractions. If we let 𝑢 =
𝑡𝑎𝑛 𝑥, 𝑑𝑢 = 𝑠𝑒𝑐 2 𝑥 𝑑𝑥. This suggests multiplying the integrand by 𝑠𝑒𝑐 2 𝑥/ 𝑠𝑒𝑐 2 𝑥 or
1 𝑠𝑒𝑐 2 𝑥 1 𝑠𝑒𝑐 2 𝑥
∫ ∙ 𝑑𝑥 = ∫ ∙ 𝑑𝑥
1 + 𝑡𝑎𝑛 𝑥 𝑠𝑒𝑐 2 𝑥 1 + 𝑡𝑎𝑛 𝑥 1 + 𝑡𝑎𝑛2 𝑥
1 𝑠𝑒𝑐 2 𝑥 1
∫ ∙ 2
𝑑𝑥 = ∫ 𝑑𝑢
1 + 𝑡𝑎𝑛 𝑥 1 + 𝑡𝑎𝑛 𝑥 (1 + 𝑢)(1 + 𝑢2 )
178
Now, decomposing the fraction, we get
1 𝐴 𝐵𝑢 + 𝐶
= +
(1 + 𝑢)(1 + 𝑢2 ) 1 + 𝑢 1 + 𝑢2
1 = (𝐴 + 𝐵)𝑢2 + (𝐵 + 𝐶)𝑢 + (𝐴 + 𝐶)
1 1 1 1 𝑢−1
∫ 2
𝑑𝑢 = ∫ ( − ) 𝑑𝑢
(1 + 𝑢)(1 + 𝑢 ) 2 1 + 𝑢 2 1 + 𝑢2
1 1 1 2𝑢 1 1
= ∫ 𝑑𝑢 − ∫ 2
𝑑𝑢 + ∫ 𝑑𝑢
2 1+𝑢 4 1+𝑢 2 1 + 𝑢2
1 1 1
= 𝑙𝑛 |1 + 𝑢| − 𝑙𝑛 |1 + 𝑢2 | + 𝑡𝑎𝑛−1 𝑥 + 𝐶
2 4 2
Hence, our original integral becomes
1 1 1 1
∫ 𝑑𝑥 = 𝑙𝑛 |1 + 𝑡𝑎𝑛 𝑥| − 𝑙𝑛 |1 + 𝑡𝑎𝑛 2 𝑥| + 𝑡𝑎𝑛−1 (𝑡𝑎𝑛 𝑥) + 𝐶
1 + 𝑡𝑎𝑛 𝑥 2 4 2
1 1 1
= 𝑙𝑛 |1 + 𝑡𝑎𝑛 𝑥| − 𝑙𝑛 |1 + 𝑡𝑎𝑛 2 𝑥| + 𝑡𝑎𝑛−1 (𝑡𝑎𝑛 𝑥) + 𝐶
2 4 2
1 1 1
= 𝑙𝑛 |1 + 𝑡𝑎𝑛 𝑥| − 𝑙𝑛 |𝑠𝑒𝑐 2 𝑥| + 𝑥 + 𝐶
2 4 2
1 1 + 𝑡𝑎𝑛 𝑥 1
= 𝑙𝑛 | |+ 𝑥+𝐶
2 𝑠𝑒𝑐 𝑥 2
1 1
= 𝑙𝑛 |𝑐𝑜𝑠 𝑥 + 𝑠𝑖𝑛 𝑥| + 𝑥 + 𝐶 ANS.
2 2
which is the same answer in Example 2.10.
_____________________________________________________________________________________________________
Example 10.13.
Evaluate
∫ √𝑡𝑎𝑛ℎ 𝑥 𝑑𝑥
179
Solution. If we let 𝑢 = 𝑡𝑎𝑛ℎ 𝑥, 𝑑𝑢 = 𝑠𝑒𝑐ℎ2 𝑥 𝑑𝑥. So, we begin by multiplying the
integrand by 𝑠𝑒𝑐ℎ2 𝑥/ 𝑠𝑒𝑐ℎ2 𝑥 and get
𝑠𝑒𝑐ℎ2 𝑥
∫ √𝑡𝑎𝑛ℎ 𝑥 𝑑𝑥 = ∫ √𝑡𝑎𝑛ℎ 𝑥 𝑑𝑥
𝑠𝑒𝑐ℎ2 𝑥
𝑠𝑒𝑐ℎ2 𝑥
=∫ √𝑡𝑎𝑛ℎ 𝑥 𝑑𝑥
1 − 𝑡𝑎𝑛ℎ2 𝑥
√𝑢
=∫ 𝑑𝑢
1 − 𝑢2
√𝑢 𝑡2 𝑡2
∫ 𝑑𝑢 = 2 ∫ 𝑑𝑡 = −2 ∫ 𝑑𝑡
1 − 𝑢2 1 − 𝑡4 𝑡4 − 1
𝑡2 𝐴𝑡 + 𝐵 𝐶 𝐷
4
= 2 + +
𝑡 −1 𝑡 +1 𝑡+1 𝑡−1
Hence,
𝐴+𝐶+𝐷 = 0
𝐵−𝐶+𝐷 = 1
−𝐴 + 𝐶 + 𝐷 = 0
−𝐵 − 𝐶 + 𝐷 = 0
From the first and third equations, it seems that 𝐴 = 0 for them to be true. This means
that 𝐶 = −𝐷. If this is so, adding the second and third equations yields 𝐷 = 1/4, 𝐶 =
−1/4, and 𝐵 = 1/2. Thus,
180
𝑡2 1 1 1 1 1 1
−2 ∫ 4
𝑑𝑡 = −2 ∫ ( 2 − + ) 𝑑𝑡
𝑡 −1 2𝑡 +1 4𝑡 + 1 4𝑡 − 1
1 1
= − 𝑡𝑎𝑛−1 𝑡 + 𝑙𝑛 |𝑡 + 1| − 𝑙𝑛 |𝑡 − 1| + 𝐶
2 2
= − 𝑡𝑎𝑛−1 𝑡 + 𝑐𝑜𝑡ℎ−1 𝑡 + 𝐶
= − 𝑡𝑎𝑛−1 √𝑢 + 𝑐𝑜𝑡ℎ−1 √𝑢 + 𝐶
__________________________________________________________________________
Example 10.14.
Evaluate
∫ 𝑡𝑎𝑛ℎ−1 𝑥 2 𝑑𝑥
𝑢 = 𝑡𝑎𝑛ℎ−1 𝑥 2 and 𝑑𝑣 = 𝑑𝑥
2𝑥
𝑑𝑢 = 𝑑𝑥 and 𝑣 = 𝑥
1 − 𝑥4
Hence, the integral becomes
∫ 𝑡𝑎𝑛ℎ−1 𝑥 2 𝑑𝑥 = 𝑢𝑣 − ∫ 𝑣 𝑑𝑢
𝑥2
= 𝑥 𝑡𝑎𝑛ℎ−1 𝑥 2 − 2 ∫ 𝑑𝑥
1 − 𝑥4
−1 2
𝑥2
= 𝑥 𝑡𝑎𝑛ℎ 𝑥 + 2∫ 4 𝑑𝑥
𝑥 −1
Notice that we encountered a similar integral as this one in Example 9.12. Following the
solution there,
𝑥2 1 1 1 1 1
2∫ 4
𝑑𝑥 = − ∫ 𝑑𝑥 + ∫ 𝑑𝑥 + ∫ 2 𝑑𝑥
𝑥 −1 2 𝑥+1 2 𝑥−1 𝑥 +1
181
1 1
= − 𝑙𝑛 |𝑥 + 1| + 𝑙𝑛 |𝑥 − 1| + 𝑡𝑎𝑛−1 𝑥 + 𝐶
2 2
1 𝑥+1
= − 𝑙𝑛 | | + 𝑡𝑎𝑛−1 𝑥 + 𝐶
2 𝑥−1
= − 𝑐𝑜𝑡ℎ−1 𝑥 + 𝑡𝑎𝑛−1 𝑥 + 𝐶
Finally,
∫ 𝑡𝑎𝑛ℎ−1 𝑥 2 𝑑𝑥 = 𝑥 𝑡𝑎𝑛ℎ−1 𝑥 2 − 𝑐𝑜𝑡ℎ−1 𝑥 + 𝑡𝑎𝑛−1 𝑥 + 𝐶 ANS.
__________________________________________________________________________
Case IV. One of the factors of 𝑄(𝑥) is quadratic that is repeated m times.
𝐴1 𝑥 + 𝐵1 𝐴2 𝑥 + 𝐵2 𝐴𝑚 𝑥 + 𝐵𝑚
+ + . . . +
(𝑎𝑥 2 + 𝑏𝑥 + 𝑐)𝑚 (𝑎𝑥 2 + 𝑏𝑥 + 𝑐 )𝑚−1 𝑎𝑥 2 + 𝑏𝑥 + 𝑐
_____________________________________________________________________________________________________
Example 10.15.
Evaluate
2𝑥 3 + 𝑥 2 − 2
∫ 4 𝑑𝑥
𝑥 − 4𝑥 2 + 4
Solution. The denominator is actually equal to (𝑥 2 − 2)2 . Hence, the integrand may be
written as
2𝑥 3 + 𝑥 2 − 2 𝐴𝑥 + 𝐵 𝐶𝑥 + 𝐷
4 2
= 2 2
+ 2
𝑥 − 4𝑥 + 4 (𝑥 − 2) 𝑥 −2
𝐶𝑥 3 + 𝐷𝑥 2 + (𝐴 − 2𝐶)𝑥 + (𝐵 − 2𝐷)
=
(𝑥 2 − 2)2
Thus,
𝐶=2
𝐷=1
182
𝐴 − 2𝐶 = 0; 𝐴 = 4
𝐵 − 2𝐷 = −2; 𝐵 = 0
2𝑥 3 + 𝑥 2 + 1 4𝑥 2𝑥 + 1
∫ 4 2
𝑑𝑥 = ∫ ( 2 2
+ 2 ) 𝑑𝑥
𝑥 − 4𝑥 + 4 (𝑥 − 2) 𝑥 −2
2𝑥 2𝑥 1
= ∫ (2 ∙ + + ) 𝑑𝑥
(𝑥 2 − 2)2 𝑥 2 − 2 𝑥 2 − 2
2 1 𝑥
=− + 𝑙𝑛 |𝑥 2 − 2| − 𝑡𝑎𝑛ℎ−1 +𝐶 ANS.
𝑥2 −2 √2 √2
_____________________________________________________________________________________________________
Example 10.16.
Evaluate
𝑥3
∫ 𝑑𝑥
(𝑥 2 + 1)3
Solution. The integrand may be broken down into component partial fractions, i.e.,
𝑥3 𝐴𝑥 + 𝐵 𝐶𝑥 + 𝐷 𝐸𝑥 + 𝐹
= + +
(𝑥 2 + 1)3 (𝑥 2 + 1)3 (𝑥 2 + 1)2 𝑥 2 + 1
𝑥3 −𝑥 𝑥
∫ 2 3
𝑑𝑥 = ∫ ( 2 3
+ 2 ) 𝑑𝑥
(𝑥 + 1) (𝑥 + 1) (𝑥 + 1)2
−𝑥 𝑥 1 1 1
∫( + ) 𝑑𝑥 = ∫ (− + ) 𝑑𝑢
(𝑥 2 + 1)3 (𝑥 2 + 1)2 2 𝑢3 𝑢2
1 1 1 1
= ∙ − ∙ +𝐶
4 𝑢2 2 𝑢
1 1 1 1
= ∙ 2 2
− ∙ 2 +𝐶 ANS.
4 (𝑥 + 1) 2 (𝑥 + 1)
183
This example may also be solved using trigonometric substitution and ordinary
substitution. Using ordinary substitution, we need to simplify the denominator by
reducing it to a single term only instead of a binomial. If we let
𝑢 = 𝑥 2 + 1, 𝑥 = √𝑢 − 1
1 𝑑𝑢
𝑑𝑥 =
2 √𝑢 − 1
1 𝑢−1 1 1 1
= ∫ 3 𝑑𝑢 = ∫ ( 2 − 3 ) 𝑑𝑢
2 𝑢 2 𝑢 𝑢
1 1 1 1
= ∙ 2 2
− ∙ 2 +𝐶
4 (𝑥 + 1) 2 (𝑥 + 1)
_____________________________________________________________________________________________________
Example 10.17.
Evaluate
2𝑥 + 1
∫ 𝑑𝑥
(𝑥 2 + 1)2
Solution. We can use Ostrogradski’s method. In this case, 𝑃(𝑥) = 2𝑥 + 1 and 𝑄(𝑥) =
(𝑥 2 + 1)2 . Differentiating 𝑄(𝑥) gives
Hence, the greatest divisor of 𝑄(𝑥) and 𝑄′(𝑥) is 𝑄1 (𝑥) = (𝑥2 + 1). This means that
𝑄2 (𝑥) = 𝑄(𝑥)/𝑄1 (𝑥) = (𝑥2 + 1). Thus,
2𝑥 + 1 𝐴𝑥 + 𝐵 𝐶𝑥 + 𝐷
∫ 2 2
𝑑𝑥 = 2 +∫ 2 𝑑𝑥
(𝑥 + 1) 𝑥 +1 𝑥 +1
Getting the derivative of the above equation removes the integration sign, thus,
𝑑 2𝑥 + 1 𝑑 𝐴𝑥 + 𝐵 𝑑 𝐶𝑥 + 𝐷
∫ 2 2
𝑑𝑥 = + ∫ 𝑑𝑥
𝑑𝑥 (𝑥 + 1) 𝑑𝑥 𝑥 2 + 1 𝑑𝑥 𝑥 2 + 1
184
2𝑥 + 1 (𝑥2 + 1)𝐴 − (𝐴𝑥 + 𝐵)(2𝑥) 𝐶𝑥 + 𝐷
= + 2
(𝑥2 + 1)2 (𝑥 2 + 1)2 𝑥 +1
2𝑥 + 1 = 𝐴𝑥 2 + 𝐴 − 2𝐴𝑥 2 − 2𝐵𝑥 + 𝐶𝑥 3 + 𝐷𝑥 2 + 𝐶𝑥 + 𝐷
1 1
A little calculation reveals that 𝐴 = 2 , 𝐵 = −1, 𝐶 = 0 and 𝐷 = 2. Thus,
2𝑥 + 1 1 𝑥−2 1 1
∫ 2 2
𝑑𝑥 = ( 2 )+ ∫ 2 𝑑𝑥
(𝑥 + 1) 2 𝑥 +1 2 𝑥 +1
1 𝑥−2 1
= ( 2 ) + 𝑡𝑎𝑛−1 𝑥 + 𝐶 ANS.
2 𝑥 +1 2
_____________________________________________________________________________________________________
185
Try these.
4𝑥 2 + 3𝑥 + 2
1. ∫ 2 𝑑𝑥
𝑥 (𝑥 + 1)2
5𝑥 2 − 39𝑥 + 18
2. ∫ 4 𝑑𝑥
𝑥 − 2𝑥 3 − 9𝑥 2 + 18
𝑒 2𝑥 − 7𝑒 𝑥
3. ∫ 2𝑥 𝑑𝑥
2𝑒 − 3𝑒 𝑥 − 2
3𝑥 3 − 2𝑥 2 + 3𝑥 − 1
4. ∫ 𝑑𝑥
𝑥 4 − 𝑥 3 + 2𝑥 2 + 1
−5𝑥 2 + 29𝑥 − 40
5. ∫ 𝑑𝑥
𝑥 3 − 8𝑥 2 + 21𝑥 − 18
2𝑥 3 − 𝑥 2 + 9𝑥 − 3
6. ∫ 𝑑𝑥
𝑥 4 + 8𝑥 2 + 16
𝑑𝑡
7. ∫
𝑡4 − 𝑡2
186
11. The Definite Integral
𝑏−𝑎
∆𝑥 =
𝑛
60
50
x=a x=b
40
30
∆x
20
f(x)
10 R
0
0 2 4 6 8 10 12
187
The height of the first strip is 𝑓(𝑥1 ) = 𝑓(𝑎 + ∆𝑥). For the second strip, the height is
𝑓(𝑥2 ) = 𝑓(𝑎 + 2∆𝑥), for the third 𝑓(𝑥3 ) = 𝑓(𝑎 + 3∆𝑥), etc. For the rth strip, the height is
𝑓(𝑥𝑟 ) = 𝑓(𝑎 + 𝑟∆𝑥). If the area of each strip is 𝑓(𝑥𝑖 )∆𝑥, the area of the entire region R is
approximately equal to the sum of the area of all strips. Hence,
𝑛
𝐴𝑟𝑒𝑎 ≈ ∑ 𝑓(𝑥𝑖 ) ∆𝑥
𝑖=1
The method of approximating the area of the region under the curve by partitioning the
region into smaller shapes that together form a region similar to the region being
calculated is known as the Riemann sum.
It can be shown that the area of the entire region is better approximated by these
strips if we have a bigger 𝑛, i.e., if more strips are used. Consequently, we allow ∆𝑥 to
become smaller, i.e., the width of each strip becomes narrower until it reaches a limiting
value. The area, therefore, of the region R is
𝑛
where 𝑎 and 𝑏 are called the lower and upper limits, respectively, of the integration of
the function 𝑓(𝑥). An integral which is evaluated on an interval [𝑎, 𝑏] is called a definite
integral.
where 𝐹 ′ (𝑥) = 𝑓(𝑥). This is the first part of the fundamental theorem of calculus. If 𝐹(𝑥) is
any anti-derivative of 𝑓(𝑥), then
𝑏
∫ 𝑓(𝑥)𝑑𝑥 = 𝐹(𝑏) − 𝐹(𝑎) 11.1
𝑎
188
This is known as the second part of the fundamental theorem of calculus.
_____________________________________________________________________________________________________
Example 11.1.
Evaluate
1
∫ 𝑥 𝑑𝑥
0
as the area of the region bounded by the curve 𝑦 = 𝑥, the x-axis and the lines 𝑥 =
0 and 𝑥 = 1,
Solution. If we divide the region into 𝑛 strips, each strip will have a width of
𝑏−𝑎 1−0 1
∆𝑥 = = =
𝑛 𝑛 𝑛
Hence, the area under the curve is
𝑛
𝑛 𝑛
1 1 1 1
= lim ∑ 𝑓 (𝑟𝑖 ) = lim ∑ 𝑟𝑖
𝑛→∞ 𝑛 𝑛 𝑛→0 𝑛 𝑛
𝑖=1 𝑖=1
𝑛
1 1
= lim 2 ∑ 𝑟𝑖 = lim 2 (1 + 2 + 3 + . . . + 𝑛)
𝑛→∞ 𝑛 𝑛→0 𝑛
𝑖=1
But
𝑛(𝑛 + 1)
1 + 2 + 3 + ...+ 𝑛 =
2
Consequently,
1 𝑛(𝑛 + 1) 1 1
𝐴 = lim ∙ = lim ( + )
𝑛→∞ 𝑛2 2 𝑛→∞ 2 2𝑛
If 𝑛 → ∞, 𝐴 = 1/2.
189
1
1 21
∫ 𝑥 𝑑𝑥 = 𝑥 |
0 2 0
1 1 1
= (12 ) − (02 ) = ANS.
2 2 2
In getting the definite integral, it is customary not to write the constant C
anymore. The reason is that the constant will cancel anyway once the limits of integration
are substituted to the anti-derivative.
_____________________________________________________________________________________________________
Example 11.2.
Evaluate
5
1
∫ 𝑑𝑥
0 𝑥−2
Example 11.3.
Evaluate
𝜋/2
∫ 𝑐𝑜𝑠 2 𝑥 𝑑𝑥
𝜋/4
Solution. Using the trigonometric identity for the square of cosine, we get
𝜋/2 𝜋/2
2
1 1
∫ 𝑐𝑜𝑠 𝑥 𝑑𝑥 = ∫ ( + 𝑐𝑜𝑠 2𝑥) 𝑑𝑥
𝜋/4 𝜋/4 2 2
𝜋/2
1 1
= 𝑥 + 𝑠𝑖𝑛 2𝑥|
2 4 𝜋/4
190
1 𝜋 1 2𝜋 1 𝜋 1 2𝜋
= ( ∙ + 𝑠𝑖𝑛 ) − ( ∙ + 𝑠𝑖𝑛 )
2 2 4 2 2 4 4 4
𝜋 𝜋 1
= ( + 0) − ( + )
4 8 4
𝜋 1
= − ANS.
8 4
_____________________________________________________________________________________________________
Example 11.4.
Evaluate
5
𝑒 2𝑥
∫ 2𝑥 𝑑𝑥
3 𝑒 +1
5 5
𝑒 2𝑥 𝑙𝑛 (𝑒 2𝑥 + 1)
∫ 2𝑥 𝑑𝑥 = |
3 𝑒 +1 2 3
1 1
= 2 𝑙𝑛 (𝑒 10 + 1) − 2 𝑙𝑛 (𝑒 6 + 1)
𝑒 10 + 1
1
= 𝑙𝑛 ( 6 ) ANS.
𝑒 +1
2
_____________________________________________________________________________________________________
Example 11.5.
Evaluate
4
∫ 2𝑥(𝑥 2 + 4)2 𝑑𝑥
2
1 2 1
= (4 + 4)3 − (22 + 4)3
3 3
8000 512
= − = 2496 ANS.
3 3
191
There is another way we can evaluate the definite integral immediately after
making the substitution. If 𝑢 = 𝑥 2 + 4, then 𝑢 = 22 + 4 = 8 for 𝑥 = 2 and 𝑢 = 42 + 4 =
20 for 𝑥 = 4. Consequently,
4 20
∫ 2𝑥(𝑥 2 + 4)2 𝑑𝑥 = ∫ 𝑢2 𝑑𝑢
2 8
1 3 20 1
= 𝑢 | = (203 − 83 )
3 8 3
1
(8000 − 512) = 2496
=
3
_____________________________________________________________________________________________________
Example 11.6.
Evaluate
√2
∫ 𝑥 (4 − 𝑥 2 )3/2 𝑑𝑥
0
Solution. We can use trigonometric substitution by letting 𝑥 = 2 𝑠𝑖𝑛 𝜃. This means that
𝑑𝑥 = 2 𝑐𝑜𝑠 𝜃 𝑑𝜃 and √4 − 𝑥 2 = 2 𝑐𝑜𝑠 𝜃. If so, then 𝜃 = 𝑠𝑖𝑛−1 𝑥/2 which is equal to
√2 𝜋
𝑠𝑖𝑛−1 or for 𝑥 = √2 and 0 for 𝑥 = 0. The integral then reduces to
2 4
√2 𝜋/4
∫ 𝑥 (4 − 𝑥 2 )3/2 𝑑𝑥 = ∫ (2 𝑠𝑖𝑛 𝜃)(2 𝑐𝑜𝑠 𝜃)3 (2 𝑐𝑜𝑠 𝜃 𝑑𝜃)
0 0
𝜋/4
= 32 ∫ 𝑠𝑖𝑛 𝜃 ∙ 𝑐𝑜𝑠 4 𝜃 𝑑𝜃
0
𝜋 √2
Note that if we let 𝑢 = 𝑐𝑜𝑠 𝜃, 𝑑𝑢 = −𝑠𝑖𝑛 𝜃 𝑑𝜃. This means that 𝑢 = 𝑐𝑜𝑠 4 𝑜𝑟 𝑎𝑛𝑑 𝑢 =
2
𝑐𝑜𝑠 0 𝑜𝑟 1. Thus, the reduced integral further reduces to
𝜋/4 √2/2
32 ∫ 𝑠𝑖𝑛 𝜃 ∙ 𝑐𝑜𝑠 4 𝜃 𝑑𝜃 = −32 ∫ 𝑢4 𝑑𝑢
0 1
√2/2 5
32 32 √2
= − 𝑢5 | = − [( ) − 15 ]
5 1 5 2
32 − 4√2
= ANS.
5
192
Example 11.7.
Evaluate
𝜋/3
∫ 𝑥 𝑠𝑒𝑐 2 𝑥 𝑑𝑥
𝜋/6
𝜋/3
= 𝑥 𝑡𝑎𝑛 𝑥 − ∫ 𝑡𝑎𝑛 𝑥 𝑑𝑥
𝜋/6
𝜋/3 𝜋/3
= 𝑥 𝑡𝑎𝑛 𝑥|𝜋/6 + 𝑙𝑛 |𝑐𝑜𝑠 𝑥||𝜋/6
𝜋 𝜋 𝜋 𝜋 𝜋 𝜋
= ( 𝑡𝑎𝑛 − 𝑡𝑎𝑛 ) + (𝑙𝑛 |𝑐𝑜𝑠 | − 𝑙𝑛 |𝑐𝑜𝑠 |)
3 3 6 6 3 6
√3 𝜋 √3 𝜋 1 √3
=( − ) + (𝑙𝑛 − 𝑙𝑛 )
3 18 2 2
5√3 𝜋 1
= − 𝑙𝑛 3 ANS.
18 2
_____________________________________________________________________________________________________
Example 11.8.
Evaluate
𝜋/2
1
∫ 𝑑𝑥
0 1 + 𝑠𝑖𝑛 𝑥
193
𝜋/2
𝑠𝑖𝑛 𝑥
=∫ (𝑠𝑒𝑐 2 𝑥 − ) 𝑑𝑥
0 𝑐𝑜𝑠 2 𝑥
𝜋/2
1
= (𝑡𝑎𝑛 𝑥 − )|
𝑐𝑜𝑠 𝑥 0
Here’s the complication. If we directly substitute 𝜋/2 to the antiderivative, we will get an
𝜋
undefined term because 𝑐𝑜𝑠 2 = 0. However, we take note that
1 𝑠𝑖𝑛 𝑥 1
𝑡𝑎𝑛 𝑥 − = −
𝑐𝑜𝑠 𝑥 𝑐𝑜𝑠 𝑥 𝑐𝑜𝑠 𝑥
𝑠𝑖𝑛 𝜋2 1
= 𝜋−
𝑐𝑜𝑠 2 𝑐𝑜𝑠 𝜋2
1 1
= 𝜋− =0
𝑐𝑜𝑠 2 𝑐𝑜𝑠 𝜋2
1
= 0 − (𝑡𝑎𝑛 0 − )
𝑐𝑜𝑠 0
= 0 − (0 − 1) = 1 ANS.
_____________________________________________________________________________________________________
Example 11.9.
Evaluate
𝜋/2
𝑑𝑥
∫ 4
0 (√𝑠𝑖𝑛 𝑥 + √𝑐𝑜𝑠 𝑥)
Solution. This integral appeared in the 2013 Stanford Math Tournament. To solve the
problem, we divide the integrand by 𝑐𝑜𝑠 2 𝑥/ 𝑐𝑜𝑠 2 𝑥, thus
𝜋/2 𝜋/2
𝑑𝑥 𝑐𝑜𝑠 2 𝑥 𝑠𝑒𝑐 2 𝑥 𝑑𝑥
∫ 4 ÷ 𝑐𝑜𝑠 2 𝑥 = ∫ 4
0 (√𝑠𝑖𝑛 𝑥 + √𝑐𝑜𝑠 𝑥) 0 (√𝑡𝑎𝑛 𝑥 + 1)
By letting 𝑢2 = 𝑡𝑎𝑛 𝑥, 2𝑢 𝑑𝑢 = 𝑠𝑒𝑐 2 𝑥 𝑑𝑥. The limits of integration also change from
𝑥 = 0 and 𝑥 = 𝜋/2 to 𝑢 = 𝑡𝑎𝑛 0 = 0 and 𝑢 = 𝑡𝑎𝑛 𝜋/2 = ∞. The integral becomes
194
𝜋/2 ∞
𝑠𝑒𝑐 2 𝑥 𝑑𝑥 2𝑢 𝑑𝑢
∫ 4 =∫
0 (√𝑡𝑎𝑛 𝑥 + 1) 0 (𝑢 + 1)4
1 1 ∞
= 2 (− 2 + 4 )|
2𝑡 3𝑡 1
1 1
= 2 [0 − (− + )]
2 3
1
= ANS.
3
_____________________________________________________________________________________________________
Example 11.10
Evaluate
1
∫ (𝑙𝑛 𝑥)20 𝑑𝑥
0
20 (𝑙𝑛 𝑥)19
𝑑𝑢 = 𝑑𝑥 and 𝑣 = 𝑥
𝑥
The integral then becomes
1 1
∫ (𝑙𝑛 𝑥)20 𝑑𝑥 = 𝑥(𝑙𝑛 𝑥)20 |10 − ∫ 20 (𝑙𝑛 𝑥)19 𝑑𝑥
0 0
1
= 0 − 20 ∫ (𝑙𝑛 𝑥)19 𝑑𝑥
0
195
19 (𝑙𝑛 𝑥)18
𝑑𝑢 = 𝑑𝑥 and 𝑣 = 𝑥
𝑥
The integral then becomes
1 1
−20 ∫ (𝑙𝑛 𝑥)19 𝑑𝑥 = −20 𝑥(𝑙𝑛 𝑥)19 |10 − (−20) ∫ 19 (𝑙𝑛 𝑥)18 𝑑𝑥
0 0
1
= 0 − (20)(−19) ∫0 (𝑙𝑛 𝑥)18 𝑑𝑥
If we continue this, we reduce the exponent of 𝑙𝑛 𝑥 each time we integrate by parts until
1 1
20
∫ (ln 𝑥) 𝑑𝑥 = −(20)(−19)(−18) ∫ (𝑙𝑛 𝑥)17 𝑑𝑥
0 0
1
= (−20)(−19)(−18) … (−3)(−2) ∫ 𝑙𝑛 𝑥 𝑑𝑥
0
= (−20)(−19)(−18) … (−3)(−2)(−1)
= 20! ANS.
_____________________________________________________________________________________________________
Example 11.11
Evaluate
𝜋/2
𝑑𝑥
∫
0 1 + 𝑡𝑎𝑛20 𝑥
Solution. Like the preceding integral, the solution to this problem involves trickery. We
can write 𝑡𝑎𝑛 𝑥 as a co-function of 𝑐𝑜𝑡 𝑥, thus,
𝜋
𝑡𝑎𝑛 𝑥 = 𝑐𝑜𝑡 (2 − 𝑥)
𝜋/2 𝜋/2
𝑑𝑥 𝑑𝑥
∫ = ∫
0 1 + 𝑡𝑎𝑛20 𝑥 0 1 + 𝑐𝑜𝑡 20 (𝜋2 − 𝑥)
𝜋 𝜋
If we let 𝑢 = 𝜋2 − 𝑥, 𝑑𝑢 = −𝑑𝑥. If 𝑥 = 2 , 𝑢 = 0. If 𝑥 = 0, 𝑢 = 2 . Thus, the integral becomes
𝜋/2 0
𝑑𝑥 −𝑑𝑢
∫ 20
= ∫ 20 𝑢
0 1 + 𝑡𝑎𝑛 𝑥 𝜋/2 1 + 𝑐𝑜𝑡
196
which we can further rewrite as
𝜋/2 𝜋/2
𝑑𝑥 𝑑𝑢
∫ 20
= ∫
0 1 + 𝑡𝑎𝑛 𝑥 0 1 + 1/𝑡𝑎𝑛20 𝑢
𝜋/2
𝑡𝑎𝑛20 𝑢
=∫ 𝑑𝑢
0 𝑡𝑎𝑛20 𝑢 + 1
𝜋/2
1
=∫ (1 − ) 𝑑𝑢
0 𝑡𝑎𝑛20 𝑢 +1
𝜋/2 𝜋/2
𝑑𝑢
=∫ 𝑑𝑢 − ∫
0 0 𝑡𝑎𝑛20 𝑢 +1
Notice that the integral at the right has the same form as the integral we wish to evaluate.
Even if that integral is expressed in terms of another variable, it cannot be denied that
they have the same value. If the integral we want to evaluate is I, then
𝜋/2
𝐼=∫ 𝑑𝑢 − 𝐼
0
𝜋
2𝐼 =
2
𝜋
𝐼= ANS.
4
_____________________________________________________________________________________________________
𝑏 𝑏 𝑏
Prop. 1 ∫ [𝑓(𝑥) + 𝑔(𝑥)] 𝑑𝑥 = ∫ 𝑓(𝑥) 𝑑𝑥 + ∫ 𝑔(𝑥) 𝑑𝑥 11.2
𝑎 𝑎 𝑎
Proof: If 𝐹(𝑥) and 𝐺(𝑥) are the anti-derivatives of 𝑓(𝑥) and 𝑔(𝑥) respectively, then
𝑏
∫ [𝑓(𝑥) + 𝑔(𝑥)] 𝑑𝑥 = [𝐹(𝑏) + 𝐺(𝑏)] − [𝐹(𝑎) + 𝐺(𝑎)]
𝑎
197
= [𝐹(𝑏) − 𝐹(𝑎)] + [𝐺(𝑏) − 𝐺(𝑎)]
𝑏 𝑏
= ∫ 𝑓(𝑥) 𝑑𝑥 + ∫ 𝑔(𝑥) 𝑑𝑥 𝑞. 𝑒. 𝑑.
𝑎 𝑎
𝑎
Prop. 2 ∫ 𝑓(𝑥) 𝑑𝑥 = 0 11.3
𝑎
𝑏 𝑎
Prop. 3 ∫ 𝑓(𝑥) 𝑑𝑥 = ∫ −𝑓(𝑥) 𝑑𝑥 11.4
𝑎 𝑏
= −[𝐹(𝑎) − 𝐹(𝑏)]
𝑎
= ∫ −𝑓(𝑥) 𝑑𝑥 𝑞. 𝑒. 𝑑.
𝑏
𝑏 𝑐 𝑏
Prop. 4 ∫ 𝑓(𝑥) 𝑑𝑥 = ∫ 𝑓(𝑥) 𝑑𝑥 + ∫ 𝑓(𝑥) 𝑑𝑥, 𝑎<𝑐<𝑏 11.5
𝑎 𝑎 𝑐
198
𝑎 𝑎
Prop. 5 ∫ 𝑓(𝑥) 𝑑𝑥 = ∫ [𝑓(𝑥) + 𝑓(−𝑥)] 𝑑𝑥 11.6
−𝑎 0
−𝑎
∫ 𝑓(𝑥) 𝑑𝑥 = 𝐹(−𝑎) − 𝐹(0)
0
𝑎
= ∫ 𝑓(−𝑥) 𝑑𝑥
0
Hence,
𝑎 𝑎 𝑎
∫ 𝑓(𝑥) 𝑑𝑥 = ∫ 𝑓(−𝑥) 𝑑𝑥 + ∫ 𝑓(𝑥) 𝑑𝑥
−𝑎 0 0
𝑎
= ∫ [𝑓(𝑥) + 𝑓(−𝑥)] 𝑑𝑥 𝑞. 𝑒. 𝑑.
0
𝑏 𝑏
Prop. 6 ∫ 𝑓(𝑥) 𝑑𝑥 = ∫ 𝑓(𝑎 + 𝑏 − 𝑥) 𝑑𝑥 11.7
𝑎 𝑎
𝑎 𝑏
= ∫ −𝑓(𝑢) 𝑑𝑢 = ∫ 𝑓(𝑢) 𝑑𝑢
𝑏 𝑎
199
𝑏
= ∫ 𝑓(𝑥) 𝑑𝑥 𝑞. 𝑒. 𝑑.
𝑎
_____________________________________________________________________________________________________
Example 11.12.
Prove that
𝑎 𝑎
∫ 𝑓(𝑥) 𝑑𝑥 = ∫ 𝑓(𝑎 − 𝑥) 𝑑𝑥
0 0
𝑎
= ∫ 𝑓(𝑎 − 𝑥) 𝑑𝑥 𝑞. 𝑒. 𝑑.
0
_____________________________________________________________________________________________________
Example 11.13.
Prove that
2𝑎 𝑎 𝑎
∫ 𝑓(𝑥) 𝑑𝑥 = ∫ 𝑓(𝑥) 𝑑𝑥 + ∫ 𝑓(2𝑎 − 𝑥) 𝑑𝑥
0 0 0
𝑎
= ∫ 𝑓(2𝑎 − 𝑢) 𝑑𝑢
0
𝑎
= ∫ 𝑓(2𝑎 − 𝑥) 𝑑𝑥
0
200
2𝑎 𝑎 𝑎
∫ 𝑓(𝑥) 𝑑𝑥 = ∫ 𝑓(𝑥) 𝑑𝑥 + ∫ 𝑓(2𝑎 − 𝑥) 𝑑𝑥 𝑞. 𝑒. 𝑑.
0 0 0
_____________________________________________________________________________________________________
Example 11.14.
Evaluate
𝜋/3
𝑐𝑜𝑠 𝑡
𝑦=∫ 𝑑𝑡
𝜋/6 𝑠𝑖𝑛 𝑡 + 𝑐𝑜𝑠 𝑡
𝜋/3
𝑠𝑖𝑛 𝑡
∴𝑦=∫ 𝑑𝑡
𝜋/6 𝑐𝑜𝑠 𝑡 + 𝑠𝑖𝑛 𝑡
Addition gives
𝜋/3 𝜋/3
𝑐𝑜𝑠 𝑡 𝑠𝑖𝑛 𝑡
𝑦+𝑦 =∫ 𝑑𝑡 + ∫ 𝑑𝑡
𝜋/6 𝑠𝑖𝑛 𝑡 + 𝑐𝑜𝑠 𝑡 𝜋/6 𝑐𝑜𝑠 𝑡 + 𝑠𝑖𝑛 𝑡
𝜋/3
𝑐𝑜𝑠 𝑡 + 𝑠𝑖𝑛 𝑡
2𝑦 = ∫ 𝑑𝑡
𝜋/6 𝑠𝑖𝑛 𝑡 + 𝑐𝑜𝑠 𝑡
1 𝜋/3 1 𝜋/3
𝑦 = ∫ 𝑑𝑡 = 𝑡|
2 𝜋/6 2 𝜋/6
1 𝜋 𝜋 𝜋
𝑦=( − )= ANS.
2 3 6 12
_____________________________________________________________________________________________________
Example 11.15.
Evaluate
201
𝜋
2𝑥 𝑠𝑖𝑛 𝑥
∫ 𝑑𝑥
0 3 + 𝑐𝑜𝑠 2𝑥
Solution. This integral appeared in the 2018 Stanford Math Tournament. Using the
relation in Example 11.12, we have
𝑎 𝑎
∫ 𝑓(𝑥) 𝑑𝑥 = ∫ 𝑓(𝑎 − 𝑥) 𝑑𝑥
0 0
𝜋 𝜋
2𝑥 𝑠𝑖𝑛 𝑥 2(𝜋 − 𝑥) 𝑠𝑖𝑛 (𝜋 − 𝑥)
∫ 𝑑𝑥 = ∫ 𝑑𝑥
0 3 + 𝑐𝑜𝑠 2𝑥 0 3 + 𝑐𝑜𝑠 2(𝜋 − 𝑥)
But since
we have
𝜋 𝜋
2𝑥 𝑠𝑖𝑛 𝑥 2(𝜋 − 𝑥) 𝑠𝑖𝑛 𝑥
∫ 𝑑𝑥 = ∫ 𝑑𝑥
0 3 + 𝑐𝑜𝑠 2𝑥 0 3 + 𝑐𝑜𝑠 2𝑥
𝜋 𝜋
2𝜋 𝑠𝑖𝑛 𝑥 2𝑥 𝑠𝑖𝑛 𝑥
=∫ 𝑑𝑥 − ∫ 𝑑𝑥
0 3 + 𝑐𝑜𝑠 2𝑥 0 3 + 𝑐𝑜𝑠 2𝑥
𝜋 𝜋
2𝑥 𝑠𝑖𝑛 𝑥 2𝜋 𝑠𝑖𝑛 𝑥
2∫ 𝑑𝑥 = ∫ 𝑑𝑥
0 3 + 𝑐𝑜𝑠 2𝑥 0 3 + 𝑐𝑜𝑠 2𝑥
𝜋
𝑠𝑖𝑛 𝑥
= 2𝜋 ∫ 𝑑𝑥
0 3 + 𝑐𝑜𝑠 2𝑥
𝜋
𝑠𝑖𝑛 𝑥
= 2𝜋 ∫ 𝑑𝑥
0 3 + (2𝑐𝑜𝑠 2 𝑥 − 1)
𝜋
𝑠𝑖𝑛 𝑥
= 2𝜋 ∫ 𝑑𝑥
0 2 + 2 𝑐𝑜𝑠 2 𝑥
𝜋
𝑠𝑖𝑛 𝑥
= 𝜋∫ 𝑑𝑥
0 1 + 𝑐𝑜𝑠 2 𝑥
202
= (−𝜋 𝑡𝑎𝑛−1 𝑢)|1−1
𝜋 𝜋
= −𝜋 (− − )
4 4
𝜋2
=
2
Finally,
𝜋
2𝑥 𝑠𝑖𝑛 𝑥 𝜋2
2∫ 𝑑𝑥 =
0 3 + 𝑐𝑜𝑠 2𝑥 2
𝜋
2𝑥 𝑠𝑖𝑛 𝑥 𝜋2
∫ 𝑑𝑥 = ANS.
0 3 + 𝑐𝑜𝑠 2𝑥 4
_____________________________________________________________________________________________________
Example 11.16.
Evaluate
6
√7 − 𝑥
𝑦=∫ 𝑑𝑥
3 √7 − 𝑥 + √𝑥 − 2
6
√𝑥 − 2
𝑦=∫ 𝑑𝑥
3 √𝑥 − 2 + √7 − 𝑥
1 6 1 6
𝑦 = ∫ 𝑑𝑥 = 𝑥|
2 3 2 3
203
1 3
= (6 − 3) = ANS.
2 2
Do not think that this technique is well applicable in all scenarios. It only applies
because the function is symmetric between the interval. Notice that 6 + 3 = 7 + 2 and
this is the reason why we were able to reduce the integrand to its equivalent.
_____________________________________________________________________________________________________
Example 11.17.
Evaluate
𝜋
𝑦 = ∫ 𝑥 𝑠𝑖𝑛2 𝑥 𝑑𝑥
0
𝜋
= ∫ (𝜋 − 𝑥) 𝑠𝑖𝑛2 𝑥 𝑑𝑥
0
𝜋 𝜋
= ∫ 𝜋 𝑠𝑖𝑛2 𝑥 𝑑𝑥 − ∫ 𝑥 𝑠𝑖𝑛2 𝑥 𝑑𝑥
0 0
𝜋
𝑦 = ∫ 𝜋 𝑠𝑖𝑛2 𝑥 𝑑𝑥 − 𝑦
0
𝜋
2𝑦 = ∫ 𝜋 𝑠𝑖𝑛2 𝑥 𝑑𝑥
0
𝜋 𝜋 1 1
𝑦= ∫ ( − 𝑐𝑜𝑠 2𝑥) 𝑑𝑥
2 0 2 2
𝜋
𝜋 1 1 1
= ( 𝑥 − ∙ 𝑠𝑖𝑛 2𝑥)|
2 2 2 2 0
𝜋
𝜋 1
= 𝑥 − 𝑠𝑖𝑛 2𝑥|
4 4 0
𝜋 1
= (𝜋 − 0) − (𝑠𝑖𝑛 2𝜋 − 𝑠𝑖𝑛 0)
4 4
204
𝜋2
= ANS.
4
_____________________________________________________________________________________________________
Example 11.18.
Evaluate
4
𝑦 = ∫ 𝑡 2 √4 − 𝑡 𝑑𝑡
0
4
= ∫ (4 − 𝑡)2 √𝑡 𝑑𝑡
0
4
= ∫ (𝑡 5/2 − 8 𝑡 3/2 + 16 𝑡1/2 ) 𝑑𝑡
0
4
2 16 32
= 𝑡 7/2 − 𝑡 3/2 + 𝑡1/2 |
7 5 3 0
2 16 3/2 32 1/2
= 47/2 − 4 + 4 −0
7 5 3
2048
= ANS.
105
_____________________________________________________________________________________________________
Examples are polynomials whose exponents are odd numbers. On the other hand, a
function is even if
Polynomials whose exponents are even numbers are even functions. The ratio of
two odd functions or two even functions is even. A constant is also an even function
205
although the constant itself may be an odd number. The ratio of an odd function to an
even function, or an even function to an odd function, is odd. Similarly, the product of odd
functions and the product of even functions are even. The product of an odd function and
an even function is actually odd. In many instances, a function is neither odd nor even.
The sum of an odd and an even function is neither odd nor even.
𝑎
∫ 𝑓(𝑥) 𝑑𝑥 = 0 𝑖𝑓 𝑓(𝑥) 𝑖𝑠 𝑜𝑑𝑑
−𝑎
Prop. 7 11.8
𝑎 𝑎
∫ 𝑓(𝑥) 𝑑𝑥 = 2 ∫ 𝑓(𝑥)𝑑𝑥 𝑖𝑓 𝑓(𝑥) 𝑖𝑠 𝑒𝑣𝑒𝑛
−𝑎 0
_____________________________________________________________________________________________________
Example 11.19.
Evaluate
𝜋/3
∫ 𝑠𝑖𝑛3 𝑡 𝑑𝑡
−𝜋/3
Solution, We know that 𝑠𝑖𝑛 (−𝑡) = −𝑠𝑖𝑛 𝑡. Now, (𝑠𝑖𝑛 (−𝑡))3 = (−𝑠𝑖𝑛 𝑡)3 = −𝑠𝑖𝑛3 𝑡.
Hence, 𝑠𝑖𝑛3 𝑡 is an odd function. Consequently,
𝜋/3
∫ 𝑠𝑖𝑛3 𝑡 𝑑𝑡 = 0 ANS.
−𝜋/3
_____________________________________________________________________________________________________
Example 11.20.
Evaluate
5
𝑦3 + 𝑦
∫ 4 2
𝑑𝑦
−5 𝑦 − 3𝑦 − 7
Solution. Notice that the numerator is odd while the denominator is even (despite the
constant which is an odd number because a constant, itself, is an even function). Since
the ratio of an odd function to an even function is itself odd, then
5
𝑦3 + 𝑦
∫ 4 2
𝑑𝑦 = 0 ANS.
−5 𝑦 − 3𝑦 − 7
_____________________________________________________________________________________________________
206
Example 11.21.
Evaluate
𝜋/2
∫ 𝑦 𝑠𝑖𝑛2 (𝑦 3 ) 𝑐𝑜𝑠 (𝑦 3 ) 𝑑𝑦
−𝜋/2
Solution. The monomial 𝑦 is itself an odd function because of its exponent. Although the
sine of an angle is odd, the square of sine is even. The cosine of an angle is actually even
despite the fact that 𝑦 3 is odd. Since the integrand is a product of odd and even functions,
the combination is in fact odd. Consequently,
𝜋/2
∫ 𝑦 𝑠𝑖𝑛2 (𝑦 3 ) 𝑐𝑜𝑠 (𝑦 3 ) 𝑑𝑦 = 0 ANS.
−𝜋/2
_____________________________________________________________________________________________________
f (b)
f (x)
f (a)
x
a b
207
Geometrically, the definite integral of a function 𝑓(𝑥) between 𝑎 and 𝑏 is equal to
the area under the curve bounded by the lines 𝑥 = 𝑎 and 𝑥 = 𝑏. But if we view the
cartesian coordinate axes from another angle, we interchange the x- and y-axes and the
curve is now defined by another function 𝑓−1 (𝑦) which is the inverse of the original
function. The definite integral of this new function is
𝑓(𝑏)
∫ 𝑓−1 (𝑦) 𝑑𝑦
𝑓(𝑎)
which, geometrically, is equal to the area ‘under the curve’ of 𝑓−1 (𝑦) bounded by the lines
𝑦 = 𝑓(𝑎) and 𝑦 = 𝑓(𝑏) although in the diagram, it appears to be the area to its ‘side’.
From the diagram, these areas plus that of the rectangle 𝑎 𝑓(𝑎) are part of the bigger
rectangle whose area is 𝑏 𝑓(𝑏). Thus,
𝑏 𝑓(𝑏)
∫ 𝑓(𝑥) 𝑑𝑥 + ∫ 𝑓−1 (𝑦) 𝑑𝑦 + 𝑎 𝑓(𝑎) = 𝑏 𝑓(𝑏)
𝑎 𝑓(𝑎)
Hence,
𝑏 𝑓(𝑏)
Prop. 8 ∫ 𝑓(𝑥) 𝑑𝑥 = [𝑏 𝑓(𝑏) − 𝑎 𝑓(𝑎)] − ∫ 𝑓−1 (𝑦) 𝑑𝑦 11.9
𝑎 𝑓(𝑎)
____________________________________________________________________________________________________
Example 11.22.
Evaluate
7
2
∫ 𝑠𝑖𝑛−1 ( ) 𝑑𝑥
3 √𝑥 + 1
Solution. To solve this ordinarily, we can use integration by parts and proceed to evaluate
the definite integral between the limits. But the solution may be tedious. However, there
is a much simpler route using the integral of the inverse of a function. Let 𝑦 be
2
𝑦 = 𝑠𝑖𝑛−1 ( )
√𝑥 + 1
2
𝑠𝑖𝑛 𝑦 =
√𝑥 + 1
𝑥 + 1 = 4 𝑐𝑠𝑐 2 𝑦
208
𝑥 = 4 𝑐𝑠𝑐 2 𝑦 − 1
7 𝜋/4
2 𝜋 𝜋
∫ 𝑠𝑖𝑛−1 ( ) 𝑑𝑥 = (7 ∙ − 3 ∙ ) − ∫ (4 𝑐𝑠𝑐 2 𝑦 − 1) 𝑑𝑦
3 √𝑥 + 1 4 2 𝜋/2
𝜋 𝜋/4
= − (−𝑐𝑜𝑡 𝑦 − 𝑦)|𝜋/2
4
𝜋 𝜋 𝜋 𝜋 𝜋
= + (4𝑐𝑜𝑡 4 + ) − (4𝑐𝑜𝑡 2 + )
4 4 2
=4 ANS.
_____________________________________________________________________________________________________
Example 11.23.
Evaluate
2
√𝑥 2 + 1
∫ 𝑑𝑥
1 𝑥
Solution. To evaluate this integral, we can use trigonometric substitution. Let 𝑥 = 𝑡𝑎𝑛 𝜃
such that 𝑑𝑥 = 𝑠𝑒𝑐 2 𝜃 𝑑𝜃. If 𝑥 = 1, 𝜃 = 𝜋/4. If 𝑥 = 2, 𝜃 = 𝑎𝑟𝑐𝑡𝑎𝑛 2. Thus,
2 tan −1 2
√𝑥 2 + 1 𝑠𝑒𝑐 𝜃
∫ 𝑑𝑥 = ∫ 𝑠𝑒𝑐 2 𝜃 𝑑𝜃
1 𝑥 𝜋/4 𝑡𝑎𝑛 𝜃
tan−1 2
𝑠𝑒𝑐 𝜃
=∫ (1 + 𝑡𝑎𝑛2 𝜃) 𝑑𝜃
𝜋/4 𝑡𝑎𝑛 𝜃
tan−1 2
=∫ 𝑠𝑒𝑐 𝜃 (𝑐𝑜𝑡 𝜃 + 𝑡𝑎𝑛 𝜃) 𝑑𝜃
𝜋/4
tan−1 2
=∫ (𝑐𝑠𝑐 𝜃 + 𝑠𝑒𝑐 𝜃 𝑡𝑎𝑛 𝜃) 𝑑𝜃
𝜋/4
−1 2
= −𝑙𝑛 |𝑐𝑠𝑐 𝜃 + 𝑐𝑜𝑡 𝜃| + 𝑠𝑒𝑐 𝜃|tan
𝜋/4
209
1 𝑥 1
𝑡𝑎𝑛−1 𝑥 = 𝑐𝑜𝑡 −1 = 𝑠𝑖𝑛−1 = 𝑐𝑜𝑠 −1
𝑥 √1 + 𝑥 2 √1 + 𝑥 2
we have
√1 + 4 √5
𝑐𝑠𝑐 (𝑡𝑎𝑛−1 2) = =
2 2
𝑠𝑒𝑐 (𝑡𝑎𝑛−1 2) = √1 + 4 = √5
1
𝑐𝑜𝑡(𝑡𝑎𝑛−1 2) =
2
Finally, we have
−1 2 √5 + 1
−𝑙𝑛 |𝑐𝑠𝑐 𝜃 + 𝑐𝑜𝑡 𝜃| + 𝑠𝑒𝑐 𝜃|tan
𝜋/4 = [−𝑙𝑛 ( ) + √5] − [−𝑙𝑛 (√2 + 1) + √2]
2
2
√𝑥 2 + 1 √5 + 1
∫ 𝑑𝑥 = − 𝑙𝑛 [ ] + (√5 − √2) ANS.
1 𝑥 2(√2 + 1)
Notice that we went through great lengths finding the final answer. However, using the
integral of inverse functions, we have
√𝑥 2 + 1
𝑦=
𝑥
𝑦2𝑥2 = 𝑥2 + 1
𝑥 2 (𝑦 2 − 1) = 1
1
𝑥=
√𝑦 2 − 1
√5
2
√𝑥 2 + 1 √5 2 1
∫ 𝑑𝑥 = (2 ∙ − 1 ∙ √2) − ∫ 𝑑𝑦
1 𝑥 2 2
√2 √𝑦 − 1
210
= (√5 − √2) − 𝑐𝑜𝑠ℎ−1 𝑦|√5/2
√2
√5
= (√5 − √2) − (𝑐𝑜𝑠ℎ−1 − 𝑐𝑜𝑠ℎ−1 √2)
2
Note that
𝑐𝑜𝑠ℎ−1 𝑥 = 𝑙𝑛 |𝑥 + √𝑥 2 − 1|
√5 √5 1
𝑐𝑜𝑠ℎ−1 − 𝑐𝑜𝑠ℎ−1 √2 = 𝑙𝑛 ( + ) − 𝑙𝑛 (√2 + 1)
2 2 2
√5 + 1
= 𝑙𝑛 [ ]
2(√2 + 1)
2
√𝑥 2 + 1 √5 + 1
∫ 𝑑𝑥 = (√5 − √2) − 𝑙𝑛 [ ] ANS.
1 𝑥 2(√2 + 1)
_____________________________________________________________________________________________________
Example 11.24.
Evaluate
1 𝑥5 −1
∫ 𝑑𝑥
0 𝑙𝑛 𝑥
Solution. The solution to this integral involves special functions known as exponential
integrals which are beyond the scope of this text. But definite integrals such as this
example can be evaluated using the Feynman technique, which involves differentiating
‘under the integral’. The technique involves carefully introducing a parameter into the
integrand and differentiating the integrand with respect to that parameter.
Let’s start by introducing a parameter, say, 𝑡 into the integrand this wise:
1
𝑥𝑡 − 1
𝐼(𝑡) = ∫ 𝑑𝑥
0 𝑙𝑛 𝑥
This means that, in the end, we will evaluate this integral with 𝑡 = 5 or 𝐼(5) to solve this
problem. Differentiating 𝐼(𝑡) with respect to 𝑡 means treating the variable 𝑥 constant.
Hence,
211
𝜕 1 𝑥𝑡 − 1
𝐼′(𝑡) = ∫ 𝑑𝑥
𝜕𝑡 0 𝑙𝑛 𝑥
1
𝜕 𝑥𝑡 − 1
=∫ ( ) 𝑑𝑥
0 𝜕𝑡 𝑙𝑛 𝑥
1
𝑥 𝑡 𝑙𝑛 𝑥
=∫ 𝑑𝑥
0 𝑙𝑛 𝑥
1
= ∫ 𝑥 𝑡 𝑑𝑥
0
1
𝑥 𝑡+1
= |
𝑡+1 0
1
𝐼′ (𝑡) =
𝑡+1
This means that
𝐼(𝑡) = ∫ 𝐼 ′ (𝑡) 𝑑𝑡
= 𝑙𝑛 |𝑡 + 1| + 𝐶
To get the value of the constant 𝐶, we note that if 𝑡 = 0, 𝑥 𝑡 = 1 and this cancels with 1 in
the numerator. Hence, 𝐼(0) = 0 and 𝐶 = −𝑙𝑛 1 = 0. Further,
𝐼(𝑡) = 𝑙𝑛 |𝑡 + 1|
Ultimately,
1 𝑥3 −1
∫ 𝑑𝑥 = 𝐼(5)
0 𝑙𝑛 𝑥
= 𝑙𝑛 |5 + 1|
= 𝑙𝑛 6 ANS.
_____________________________________________________________________________________________________
212
Example 11.25.
Show that
∞
∫ 𝑥𝑛 𝑒−𝑥 𝑑𝑥 = 𝑛!
0
Proof. We can use here the technique of integration by parts repeatedly to come up with
the answer or employ Feynman’s trick to show the above relation. Before that, we note
that
∞
∫ 𝑒−𝑥 𝑑𝑥 = 1
0
∞ 1
∫ 𝑒−𝑡𝑥 𝑑𝑥 =
0 𝑡
for 𝑡 > 0. If we differentiate both sides of the equation with respect to 𝑡, the result is
𝜕 −𝑡𝑥∞ 𝜕1
∫ 𝑒 𝑑𝑥 =
0 𝜕𝑡 𝜕𝑡 𝑡
∞ 1
∫ −𝑥𝑒−𝑡𝑥 𝑑𝑥 = −
0 𝑡2
∞ 1
∫ 𝑥𝑒−𝑡𝑥 𝑑𝑥 =
0 𝑡2
213
If 𝑡 = 1,
∞
∫ 𝑥𝑛 𝑒−𝑥 𝑑𝑥 = 𝑛! 𝑞. 𝑒. 𝑑.
0
_____________________________________________________________________________________________________
Improper Integrals
So far, we have been discussing integration of functions that are continuous and
integrable on an interval, i.e., the domains are bounded. But there are also definite
integrals of functions whose domains are unbounded. When either or both limits of
integration are infinite or when the function itself becomes infinite at or between the
limits of integration, such integrals are known as improper integrals. Improper integrals
come in the following forms:
∞ 𝑏 ∞
∫ 𝑓(𝑥) 𝑑𝑥 ∫ 𝑓(𝑥) 𝑑𝑥 ∫ 𝑓(𝑥) 𝑑𝑥
𝑎 −∞ −∞
Some definite integrals with finite upper and lower limits may also be considered
improper if the function is discontinuous at a certain value on the interval. An example
of such improper integral is Example 10.2. Although its anti-derivative exists and,
therefore, can be evaluated between the limits, the value, thus, obtained is known as the
Cauchy Principal Value.
Evaluation of the improper integrals that appear above is not as easy as getting
the anti-derivative of the integrand and substituting the infinite value of one or both
limits of integration. Instead, we evaluate these improper integrals as the limiting value
of the integral as the limit of integration approaches infinity. Thus,
∞ 𝑏
∫ 𝑓(𝑥) 𝑑𝑥 = lim ∫ 𝑓(𝑥) 𝑑𝑥
𝑎 𝑏→∞ 𝑎
𝑏 𝑏
∫ 𝑓(𝑥) 𝑑𝑥 = lim ∫ 𝑓(𝑥) 𝑑𝑥
−∞ 𝑎→−∞ 𝑎
∞ 𝜆
∫ 𝑓(𝑥) 𝑑𝑥 = lim ∫ 𝑓(𝑥) 𝑑𝑥
−∞ 𝜆→∞ −𝜆
If the limit on the right side of the equation exists, then the improper integral is said to
be convergent. Otherwise, the improper integral is said to be divergent.
The same is true with improper integrals which are discontinuous at a finite number
𝑎 or 𝑏. The integral
214
𝑏 𝑏
∫ 𝑓(𝑥) 𝑑𝑥 = lim ∫ 𝑓(𝑥) 𝑑𝑥
𝑎 𝜆→𝑎 𝑎
because it may occur that the first or second term on the right side of the equation exists.
It is necessary, however, that both terms must exist for the improper integral on the left
side of the equation to be considered convergent.
_____________________________________________________________________________________________________
Example 11.26.
Determine if the following integrals are either convergent or divergent:
∞
a) ∫ 𝑥 𝑑𝑥
−∞
∞
1
b) ∫ 𝑑𝑥
1 𝑥𝑝
1
1
c) ∫ 𝑑𝑥
0 𝑥𝑝
0
1
d) ∫ 𝑑𝑥
−∞ 𝑥2 +1
∞
e) ∫ 𝑥𝑒 −𝑥 𝑑𝑥
1
5
1
f) ∫ 𝑑𝑥
3 √2𝑥 − 6
215
Solution.
you will notice that each term on the right diverges. But if we evaluate the improper
integral, thus,
∞ 𝜆
∫ 𝑥 𝑑𝑥 = lim ∫ 𝑥 𝑑𝑥
−∞ 𝜆→∞ −𝜆
1 2 𝜆
= lim ( 𝑥 )|
𝜆→∞ 2 −𝜆
1 1
= lim ( 𝜆2 − 𝜆2 ) = 0
𝜆→∞ 2 2
b) To evaluate
∞
1
∫ 𝑑𝑥
1 𝑥𝑝
𝑏
1 1−𝑝
= lim 𝑥 |
𝑏→∞ 1 − 𝑝
1
1
= lim (𝑏1−𝑝 − 11−𝑝 )
𝑏→∞ 1 − 𝑝
1 1
= lim ( 𝑝−1 − 1)
𝑏→∞ 1 − 𝑝 𝑏
1
= (0 − 1)
1−𝑝
1
=
𝑝−1
216
On the other hand, if 𝑝 ≤ 1,
∞ 𝑏
1 1
∫ 𝑝
𝑑𝑥 = lim ∫ 𝑑𝑥
1 𝑥 𝑏→∞ 1 𝑥 𝑝
𝑏
1 1−𝑝
= lim 𝑥 |
𝑏→∞ 1 − 𝑝
1
1
= lim (𝑏1−𝑝 − 11−𝑝 )
𝑏→∞ 1 − 𝑝
Note that 𝑥1−𝑝 equals ∞ if 𝑥 = 0. Hence, the integral diverges. The integral is an improper
integral because the function itself becomes infinite at 0. On the other hand, if 𝑝 < 1,
1 1
1 1 1−𝑝
∫ 𝑝 𝑑𝑥 = 𝑥 |
0 𝑥 1−𝑝 0
1
1
= 𝑥 −(𝑝−1) |
1−𝑝 0
1
= (1 − 0)
1−𝑝
1
=
1−𝑝
1 −1
𝑥0
= lim 𝑡𝑎𝑛 |
𝑎→−∞ 2 2𝑎
1 𝑎
= lim ( 𝑡𝑎𝑛−1 0 − 𝑡𝑎𝑛−1 )
2 𝑎→−∞ 2
217
1 𝜋
= 0 − (− )
2 2
𝜋
=
4
Hence, the integral is convergent because it exists.
∫ 𝑥𝑒 −𝑥 𝑑𝑥 = − 𝑥𝑒 −𝑥 − 𝑒 −𝑥 + 𝐶
Thus,
∞
1 1 𝑏
∫ 𝑥𝑒 −𝑥 𝑑𝑥 = lim (−𝑥 − )|
1 𝑏→∞ 𝑒𝑥 𝑒𝑥 1
1 1 1 1
= lim [(−𝑏 𝑏
− 𝑏 ) − (− − )]
𝑏→∞ 𝑒 𝑒 𝑒 𝑒
2 2
= 0+ =
𝑒 𝑒
Hence, the integral is convergent.
=1
Hence, the integral is convergent.
_____________________________________________________________________________________________________
Example 11.27
Evaluate the improper integral
∞
𝑑𝑥
∫
0 (𝑥100 + 1)(1 + 𝑥 2 )
Solution. We can try using repeated integration by parts or decompose the integrand into
more than a hundred partial fractions but that would be too tedious. But with a little
218
trickery, can solve the problem using an inventive substitution. If we let 𝑢 = 1/𝑥, then
𝑥 = 1/𝑢 and 𝑑𝑥 = −𝑑𝑢/𝑢2 . If 𝑥 = 0, 𝑢 = ∞. If 𝑥 = ∞, 𝑢 = 0. Thus,
∞ 0
1 1 −𝑑𝑢
∫ 100 2
𝑑𝑥 = ∫ 100 2
∙ 2
0 (𝑥 + 1)(1 + 𝑥 ) ∞ (1/𝑢 + 1)(1 + 1/𝑢 ) 𝑢
0
1
= −∫ 𝑑𝑢
∞ (1/𝑢100 + 1)(𝑢2 + 1)
∞
𝑢100
=∫ 𝑑𝑢
0 (𝑢100 + 1)(1 + 𝑢2 )
But
𝑢100 1
100
= 1 − 100
𝑢 +1 𝑢 +1
Thus,
∞ ∞ ∞
𝑢100 1 1
∫ 100 2
𝑑𝑢 = ∫ 2
𝑑𝑢 − ∫ 100
𝑑𝑢
0 (𝑢 + 1)(1 + 𝑢 ) 0 1+𝑢 0 (𝑢 + 1)(1 + 𝑢2 )
Notice that the integral to the right has the same form as the integral we are trying to
evaluate except that we are using a different variable. Be that as it may, even arguing that
𝑢 ≠ 𝑥 because of the original substitution, they should have the same value whatever
variable was used. So, if we let the original integral to be equal to I, we have
∞
1
𝐼=∫ 𝑑𝑢 − 𝐼
0 1 + 𝑢2
∞
1
2𝐼 = ∫ 𝑑𝑢
0 1 + 𝑢2
𝜋
= 𝑡𝑎𝑛−1 𝑢|∞
0 =
2
𝜋
𝐼= ANS.
4
As a matter of fact, you will find out if you give it a try that the answer is 𝜋/4 if we replace
the exponent 100 by any other number. Do you think that this trick applies if the integral
is not improper?
219
Try these.
b. ∫ |𝑐𝑜𝑠 𝑥| 𝑑𝑥
0
𝑙𝑛 2
c. ∫ 𝑠𝑖𝑛ℎ 𝑥 𝑑𝑥
0
5
1
d. ∫ 𝑑𝑥
3 √𝑥 2 − 9
4
1
e. ∫ 𝑑𝑥
1 1 + √𝑥
2. Evaluate
6
∫ 𝑓(𝑥) 𝑑𝑥
2
where
𝑥2 + 1 𝑥>5
𝑓(𝑥) = {
2𝑥 𝑥≤5
a. ∫ (𝑥 2 − 4𝑥 + 3) 𝑑𝑥 > 0
1
−2
b. ∫ (−𝑥 3 + 2𝑥 2 + 𝑥 − 3) 𝑑𝑥 > 0
−4
4. If 𝑦𝑚𝑖𝑛 = 𝑓(𝑎) and 𝑦𝑚𝑎𝑥 = 𝑓(𝑏) represent the minimum and maximum values of
𝑓(𝑥) on the interval [𝑎, 𝑏], then
220
𝑏
𝑦𝑚𝑖𝑛 (𝑏 − 𝑎) ≤ ∫ 𝑓(𝑥) 𝑑𝑥 ≤ 𝑦𝑚𝑖𝑛 (𝑏 − 𝑎)
𝑎
Using this property, estimate the value of the following integrals:
3/2
2
a. ∫ 𝑒 −𝑥 𝑑𝑥
1/2
2𝜋/3
𝑐𝑜𝑠 𝑡
b. ∫ 𝑑𝑡
𝜋/6 𝑡
6
c. ∫ 𝑙𝑜𝑔 (𝑥 − 1) 𝑑𝑥
3
a. ∫ 𝑥 2 𝑠𝑖𝑛2 𝑥 𝑑𝑥
−𝜋
𝜋
b. ∫ 𝑥 𝑠𝑖𝑛 𝑥 𝑑𝑥
−𝜋
3
7𝑥 3 + 3𝑥
c. ∫ 5 3
𝑑𝑥
−3 𝑥 − 2𝑥
221
4
2𝑥 3 + 4𝑥
d. ∫ 2
𝑑𝑥
−4 𝑥 − 5
10. Evaluate
1
∫ 𝑥𝑒 𝑥 𝑑𝑥
−∞
222
Applications of Definite Integrals:
12. Area Under the Curve
for all 𝑦 ≥ 0 on the interval [𝑎, 𝑏]. If on the interval [𝑎, 𝑏], 𝑦 ≤ 0, then the area of the
region with the same boundaries is
𝑏
𝐴 = ∫ −𝑓(𝑥) 𝑑𝑥 12.2
𝑎
Suppose the region is bounded by two curves defined by 𝑦 = 𝑓(𝑥) and 𝑦 = 𝑔(𝑥)
such that 𝑓(𝑥) ≥ 𝑔(𝑥) for all 𝑥 in the region defined on an interval [𝑎, 𝑏]. We draw a
rectangular strip whose length is 𝑓(𝑥) − 𝑔(𝑥) and whose width is ∆𝑥. The area of the
region bounded by the curves on the interval [𝑎, 𝑏] is then
223
70
60 x=b
x=a
50
40
f(x)
30
20
g(x)
10
0
0 2 4 6 8 10 12 14
which is equivalent to the area of the region bounded by 𝑓(𝑥) on the interval less the area
of the region bounded by 𝑔(𝑥) on the same interval, or
𝑏
𝐴 = ∫ [𝑓(𝑥) − 𝑔(𝑥)] 𝑑𝑥 12.4
𝑎
_____________________________________________________________________________________________________
Example 12.1.
Find the area of the plane region bounded by 𝑦 = 𝑥 2 − 2𝑥 + 4, the x- and y-axis, and the
line 𝑥 = 5.
Solution. We envision a parabola that opens upward because of the positive coefficient
of 𝑥 2 . The vertex of the parabola is at (1, 3) and is obtained by setting 𝑦 ′ = 2𝑥 − 2 = 0,
so that 𝑥 = 1 and 𝑦 = 3. The graph of the function is described below. The region R is
bounded at the top by the parabola, the line 𝑥 = 0 corresponding to the y-axis, the line
𝑥 = 5, and the x-axis. Since 𝑦 ≥ 0 for all 0 ≤ 𝑥 ≤ 5, then the area of the defined region is
5 5
1 3
∫ (𝑥 2 2
− 2𝑥 + 4) 𝑑𝑥 = 𝑥 − 𝑥 + 4𝑥|
0 3 0
224
1 3
= (5 − 0) − (52 − 0) + 4(5 − 0)
3
110
= 𝑠𝑞. 𝑢𝑛𝑖𝑡𝑠 ANS.
3
_____________________________________________________________________________________________________
Example 12.2.
Find the area of the region bounded by the lines 𝑦 = 𝑥 + 3 and 𝑦 = 6 − 2𝑥 and the x-axis.
225
To enable us to calculate the area of the region R, we need to divide the region bounded
by the three lines into two sub-regions: first, the region under the line 𝑦 = 𝑥 + 3 and
second, the region under the line 𝑦 = 6 − 2𝑥. Note that the two regions meet at the point
of intersection of the two lines whose x-coordinate is given by
𝑥 + 3 = 6 − 2𝑥
𝑥=1
1 3
1 2 1
= 𝑥 + 3𝑥| + 6𝑥 − 2 ∙ 𝑥 2 |
2 −3 2 1
Although it is usually convenient to use vertical strips to find the area bounded by
a curve, sometimes it is more convenient to use horizontal strips instead. If in using
vertical strips, we integrate with respect to 𝑥, in using horizontal strips, we integrate with
respect to 𝑦 and the function must be expressed as 𝑥 = 𝑓(𝑦) instead of 𝑦 = 𝑓(𝑥). So, the
area bounded by the curve 𝑥 = 𝑓(𝑦) and the lines 𝑦 = 𝑐 and 𝑦 = 𝑑 is
𝑑
𝐴 = ∫ 𝑓(𝑦) 𝑑𝑦
𝑐
For regions bounded by 𝑥 = 𝑓(𝑦) to the right and 𝑥 = 𝑔(𝑦) to the left and the lines 𝑦 = 𝑐
and 𝑦 = 𝑑, the area is
𝑑
𝐴 = ∫ [𝑓(𝑦) − 𝑔(𝑦)] 𝑑𝑦
𝑐
So, in this example, the area of the region bounded by the two lines 𝑦 = 𝑥 + 3 (which is
1
the same as 𝑥 = 𝑦 − 3) and 𝑦 = 6 − 2𝑥 (which is the same as 𝑥 = 3 − 2 𝑦) is obtained by
evaluating the integral from the x-axis (𝑦 = 0) to the point of intersection of the lines at
𝑥 = 1 (or 𝑦 = 4), thus,
4
1
𝐴 = ∫ [(3 − 𝑦) − (𝑦 − 3)] 𝑑𝑦
0 2
226
4
3
= ∫ (6 − 𝑦) 𝑑𝑦
0 2
3 24
= 6𝑦 − 𝑦 |
4 0
Example 12.3.
Find the area of a circle defined by 𝑥 2 + 𝑦 2 = 52 .
Solution. We envision a circle whose center is (0, 0) and whose radius is 5 as shown
below.
If we only consider that part of the circle which lies in the first quadrant, then the equation
of the circle is
𝑦 = √52 − 𝑥 2
bounded by the x- and y-axis and defined on the interval [0, 5]. The area of one quadrant
of the circle is, thus,
5
1
4
𝐴 = ∫ √52 − 𝑥 2 𝑑𝑥
0
𝑟2 𝑥 1
∫ √𝑟 2 − 𝑥 2 𝑑𝑥 = 𝑠𝑖𝑛−1 + 𝑥√𝑟 2 − 𝑥 2
2 𝑟 2
227
Hence,
5
1 52 𝑥 1
𝐴= 𝑠𝑖𝑛−1 + 𝑥√52 − 𝑥 2 |
4 2 5 2 0
52 25𝜋
= ( 𝑠𝑖𝑛−1 1 + 0) − (0 − 0) = 𝑠𝑞. 𝑢𝑛𝑖𝑡𝑠
2 4
The area of the full circle, therefore, is four times that or
Example 12.4.
Find the area of an ellipse defined by
𝑥2 𝑦2
+ =1
16 9
Solution. We envision an ellipse whose center is (0, 0) and whose semi-major axis is 4
and its semi-minor axis is 3 as shown below.
Like our approach with the circle, we will only consider that part of the ellipse which lies
in the first quadrant defined on the interval [0, 4]. The area of one quadrant of the ellipse
is
4
1 3
𝐴 = ∫ √16 − 𝑥 2 𝑑𝑥
4 0 4
228
Using standard integral 8.3,
4
1 3 42 𝑥 1
𝐴 = ∙ ( 𝑠𝑖𝑛−1 + 𝑥√42 − 𝑥 2 )|
4 4 2 4 2 0
Note that the area of an ellipse is 𝐴 = 𝜋𝑎𝑏 where 𝑎 and 𝑏 are its semi-major and semi-
minor axes.
_____________________________________________________________________________________________________
Example 12.5.
Find the area of the intersection between two circles as shown.
𝑥2 + 𝑦2 = 𝑟2
(𝑥 − 𝑟)2 + 𝑦 2 = 𝑟 2
𝑟 2 − 𝑥 2 = 𝑟 2 − (𝑥 − 𝑟)2
229
1 √3
Hence, 𝑥 = 2 𝑟 and 𝑦 = ± 𝑟.
2
To determine the area of the intersection of the circles, we divide the region into
two along the line where the circles overlap. The section at the right is further divided
into two by the x-axis. It is this one-fourth of the total area of the intersection that we will
consider. The length of the strip is the value of 𝑦 equal to √𝑟 2 − 𝑥 2 . The area of this
section is
𝑟
𝐴 = ∫𝑟/2 √𝑟 2 − 𝑥 2 𝑑𝑥
𝑟
𝑟2 𝑥 1
= 𝑠𝑖𝑛−1 + 𝑥 √𝑟 2 − 𝑥 2 |
2 𝑟 2 𝑟/2
𝑟2 1 1 1 √3
= (𝑠𝑖𝑛−1 1 − 𝑠𝑖𝑛−1 2) + (0 − 𝑟 ∙ 𝑟)
2 2 2 2
𝑟2 𝜋 𝜋 1 √3
= ( − ) + (− ) 𝑟 2
2 2 6 2 4
𝜋 √3
= ( − ) 𝑟2
6 8
(𝑥 − 𝑟)2 = 𝑟 2 − 𝑦 2
𝑥 = 𝑟 − √𝑟 2 − 𝑦 2
Note that we took the negative root of the radical because it is the left portion of the circle
that is involved. The area of the overlap is
√3
𝑟
2
𝐴=∫ [(√𝑟 2 − 𝑦 2 ) − (𝑟 − √𝑟 2 − 𝑦 2 )] 𝑑𝑦
√3
− 𝑟
2
√3
𝑟
2
=∫ (2√𝑟 2 − 𝑦 2 − 𝑟) 𝑑𝑦
√3
− 𝑟
2
230
√3
𝑟
𝑟2 𝑦 1 2
= 2 ( 𝑠𝑖𝑛−1 + 𝑦√𝑟 2 − 𝑦 2 ) − 𝑟𝑦|
2 𝑟 2 √3
− 𝑟
2
√3 −√3 √3 2 √3 √ 2 −√3
2
= 𝑟 2 (𝑠𝑖𝑛−1 − 𝑠𝑖𝑛−1 ) + ( 𝑟 ∙ √𝑟 2 − (√3𝑟) + 𝑟 ∙ 𝑟 − ( 𝑟) )
2 2 2 2 2 2
√3 √3
−𝑟( 𝑟+ 𝑟)
2 2
𝜋 𝜋 1
= 𝑟 2 ( + ) + √3 𝑟 ∙ 𝑟 − √3 𝑟 2
3 3 2
2𝜋 √3
= ( − ) 𝑟2 ANS.
3 2
_____________________________________________________________________________________________________
Example 12.6.
Find the area of the region bounded by 4𝑦 = 𝑥 2 and 𝑦 2 = 4𝑥.
We have two parabolas, one opening upward and the other, to the right. The two
parabolas intersect at 𝑥 = 0, 4. The area of the region bounded by these curves is
4
1
𝐴 = ∫ (2√𝑥 − 𝑥 2 ) 𝑑𝑥
0 4
4 3/2 1 34
= 𝑥 − 𝑥 |
3 12 0
231
4 1
= (8 − 0) − (64 − 0)
3 12
16
= 𝑠𝑞. 𝑢𝑛𝑖𝑡𝑠 ANS.
3
It is left as an exercise to solve this example using horizontal strips instead.
_____________________________________________________________________________________________________
Example 12.7.
Find the area of the region bounded by 𝑦 2 = 2𝑥 and the line 𝑦 = 𝑥 − 4.
Solution. The region bounded by the line and the curve is shown below.
To approximate the area of the region, we can use horizontal strips. That means
we will integrate with respect to the variable 𝑦. The length of the strip is equal to 𝑥 = 𝑦 +
4 minus 𝑥 = 12𝑦 2. The parabola intersects the straight line at
1
𝑦 + 4 = 2𝑦 2
𝑦 2 − 2𝑦 − 8 = 0
(𝑦 + 2)(𝑦 − 4) = 0
232
4
1
𝐴 = ∫ [(𝑦 + 4) − 2𝑦 2 ] 𝑑𝑦
−2
4
1 1
= 𝑦 2 + 4𝑦 − 𝑦 3 |
2 6 −2
1 1
= (16 − 4) + 4(4 + 2) − (64 + 8)
2 6
= 18 𝑠𝑞. 𝑢𝑛𝑖𝑡𝑠 ANS.
_____________________________________________________________________________________________________
Example 12.8.
Find the area of the region bounded by the curve 𝑦 = 𝑥 3 − 2𝑥 2 − 5𝑥 + 6 and the x-axis.
233
𝑥 3 − 2𝑥 2 − 5𝑥 + 6 = (𝑥 − 1)(𝑥 + 2)(𝑥 − 3) = 0
so the roots are 𝑥 = 1, 𝑥 = −2 and 𝑥 = 3. Thus, the first sub-region has 𝑦 ≥ 0 on the
interval [−2, 1] while the second sub-region has 𝑦 ≤ 0 on the interval [1, 3]. The area of
the entire region, therefore, is given by
1 3
𝐴=∫ (𝑥 3 − 2𝑥 − 5𝑥 + 6) 𝑑𝑥 + ∫ −(𝑥 3 − 2𝑥 2 − 5𝑥 + 6) 𝑑𝑥
2
−2 1
1 2 5
= [ (1 − 16) − (1 + 8) − (1 − 4) + 6(1 + 2)]
4 3 2
1 2 5
+ [− (81 − 1) + (27 − 1) + (9 − 1) − 6(3 − 1)]
4 3 2
63 16 253
= + = 𝑠𝑞. 𝑢𝑛𝑖𝑡𝑠 ANS.
4 3 12
_____________________________________________________________________________________________________
Example 12.9.
Find the area of the region bounded by 𝑦 = 𝑥 2 + 1 and 𝑦 = 2𝑥 2 − 3.
234
We envision two parabolas opening upward. The upper parabola opens wider and has a
vertex at (0, 1). The lower parabola opens narrower and has a vertex at (0, −3).
𝑥 2 + 1 = 2𝑥 2 − 3
Hence, 𝑥 = ±2. The interval is [−2, 2]. The area of the region bounded by the curves,
therefore, is
2
𝐴 = ∫ [(𝑥 2 + 1) − (2𝑥 2 − 3)] 𝑑𝑥
−2
2
= ∫ (−𝑥 2 + 4) 𝑑𝑥
−2
2
1
= − 𝑥 3 + 4𝑥|
3 −2
1
= − (8 + 8) + 4(2 + 2)
3
32
= 𝑠𝑞. 𝑢𝑛𝑖𝑡𝑠 ANS.
3
_____________________________________________________________________________________________________
Example 12.10.
Find the area of that part of the circle 𝑥 2 + 𝑦 2 = 25 which is under the parabola 𝑦 =
𝑥 2 /5 − 5.
Solution. The part of the circle which is under the parabola is shown in the graph below.
Note that the parabola intersects the circle at 𝑥 = 0, ±5. The upper curve is the parabola
while the lower curve is the bottom, of the circle whose equation is −√25 − 𝑥 2 . The area
of that part of the circle intersected by the parabola is given by
5
1
𝐴 = ∫ [( 𝑥 2 − 5) − (−√25 − 𝑥 2 )] 𝑑𝑥
−5 5
5
1 3 52 𝑥 1
= 𝑥 − 5𝑥 + 𝑠𝑖𝑛−1 + 𝑥√52 − 𝑥 2 |
15 2 5 2 −5
235
1 52 𝜋 𝜋 1
= (125 + 125) − 5(5 + 5) + ( + ) + (0 − 0)
15 2 2 2 2
25𝜋 100
= − 𝑠𝑞. 𝑢𝑛𝑖𝑡𝑠 ANS.
2 3
Another approach to the problem is to get the area of the region bounded by the
parabola and the x-axis and subtract this area from the area of the lower half of the circle
1
𝜋𝑟 2 . Thus,
2
5
25𝜋 1
𝐴= − ∫ − ( 𝑥 2 − 5) 𝑑𝑥
2 −5 5
5
25𝜋 1 3
= − (− 𝑥 + 5𝑥)|
2 15 −5
25𝜋 100
= − 𝑠𝑞. 𝑢𝑛𝑖𝑡𝑠
2 3
_____________________________________________________________________________________________________
Example 12.11.
Find the area of the region bounded by 𝑦 = 𝑥 2 and the lines 𝑦 = 4 and 𝑦 = 9.
Solution. The region referred to is shown in the graph below. Notably, the parabola
intersects the line 𝑦 = 9 at 𝑥 = ±3. We draw a vertical line crossing these points for
reference. The parabola also intersects the line 𝑦 = 4 at 𝑥 = ±2. The region R is part of
the big rectangle whose dimensions are 9 − 0 = 9 and 3 − (−3) = 6. The area of that
reference rectangle is 9 × 6 = 54.
236
The area of the reference triangle is equal to the area of the region R plus the area
of the left part of the parabola from 𝑥 = −3 to 𝑥 = −2 plus the area of the smaller
rectangle bounded by 𝑦 = 4, the x-axis, and the lines 𝑥 = −2 and 𝑥 = 2, plus the area
under the right part of the parabola from 𝑥 = 2 to 𝑥 = 3. The area of the smaller rectangle
is (4 − 0) × (2 + 2) or 16. Thus,
−2 3
54 = 𝑅 + ∫ 𝑥 2 𝑑𝑥 + 16 + ∫ 𝑥 2 𝑑𝑥
−3 2
1 3 −2 1 33
54 = 𝑅 + 𝑥 | + 16 + 𝑥 |
3 −3 3 2
1 1
54 = 𝑅 + (−8 + 27) + 16 + (27 − 8)
3 3
19 19
54 = 𝑅 + + 16 +
3 3
76
𝑅= 𝑠𝑞. 𝑢𝑛𝑖𝑡𝑠 ANS.
3
A much simpler approach to the problem requires us to integrate with respect to
the variable 𝑦. Solving for 𝑥, 𝑥 = ±√𝑦. Hence, the “upper” curve is 𝑥 = +√𝑦 and the
“lower” curve is 𝑥 = −√𝑦. The area of the region bounded by these curves is
237
9
𝐴 = ∫ (√𝑦 − −√𝑦) 𝑑𝑦
4
9
2 3/2 9
= 2 ∫ √𝑦 𝑑𝑦 = 2 ∙ 𝑦 |
4 3 4
4 76
= (27 − 8) = 𝑠𝑞. 𝑢𝑛𝑖𝑡𝑠
3 3
_____________________________________________________________________________________________________
238
Try these.
1. Find the area of the region bounded by the lines 𝑦 = 𝑥 + 2, 𝑦 = 3𝑥 − 4, and the
𝑥-axis.
2. Find the area of the region bounded by 𝑦 = 𝑥 3 + 4𝑥 2 − 𝑥 − 4 and the 𝑥-axis.
3. Find the area of the region formed when a parabola 𝑦 = 𝑥 2 intersects with a
circle 𝑥 2 + 𝑦 2 = 4 in the first and second quadrant.
239
Applications of Definite Integrals:
13. Volume of Solids of Revolution
When a semicircle is revolved about an axis along its diameter, the result is a
sphere of the same diameter. When a rectangle whose dimensions are 𝑙 × 𝑤 is revolved
about an axis along its length, the result is a cylinder whose length is 𝑙 and its cross-
section is a circle whose diameter is 2𝑤. If the rectangle, however, is revolved about an
axis parallel to its length, the result is a hollow cylinder with inner and outer diameter
which depends on the distance of the axis of revolution from the length of the rectangle.
A cone is formed, on the other hand, if a right triangle is revolved about an axis along its
height or base. In all these examples, the shape, thus, formed is a solid with a circular
cross-section. Such solid in known as a solid of revolution and is formed by revolving a
plane figure about a chosen axis.
Consider a function defined by 𝑦 = 𝑓(𝑥). Let the plane figure be bounded by the
function, the x-axis, and the lines 𝑥 = 𝑎 and 𝑥 = 𝑏. If the plane figure is revolved about
the x-axis, a solid of revolution is formed. To compute the volume of the solid of
revolution, we use two methods: disk and cylinder. As the names suggest, the solid of
revolution is divided into an infinitesimal number of very thin concentric disks or rings
or an infinitesimal number of very thin hollow cylinders or shells.
Disk Method
The disk method involves dividing the solid into concentric disks whose thickness
is ∆𝑥. Note that the radius of the disk at any coordinate 𝑥𝑖 is the value of 𝑦 at 𝑥𝑖 or 𝑓(𝑥𝑖 ).
The volume of each component disk is equal to the area of its cross-section times its
thickness, or
∆𝑉 = 𝜋[𝑓(𝑥𝑖 )]2 ∆𝑥
240
where
𝑏−𝑎
∆𝑥 =
𝑛
f(x)
))
The total volume of the solid is approximated by the sum of the volumes of each disk or
𝑛
𝑉 ≈ ∑ 𝜋[𝑓(𝑥𝑖 )]2 ∆𝑥
𝑖=1
241
If the plane figure is bounded by 𝑦 = 𝑓(𝑥) and 𝑦 = 𝑔(𝑥) and 𝑓(𝑥) ≥ 𝑔(𝑥) and the
figure is revolved about the x-axis, then the solid of revolution can be partitioned into an
infinitesimal number of rings or washers that have an outer radius equal to 𝑓(𝑥) and an
inner radius equal to 𝑔(𝑥). Consequently, the volume of the solid of revolution, thus,
formed is
𝑏
𝑉 = 𝜋 ∫ {[𝑓(𝑥)]2 − [𝑔(𝑥)]2 } 𝑑𝑥 13.2
𝑎
A solid of revolution may be formed not only by revolving the plane figure about
the x-axis. It may also be revolved about any axis parallel to the x-axis. If that happens,
then corresponding adjustment must be made on the radius of the disk. Thus, if 𝑓(𝑥) >
0, the radius of any disk is equal to the value of the function 𝑓(𝑥) plus or minus a constant
which depends on the distance of the axis of revolution from the x-axis. If the axis of
revolution is a line 𝑦 = +𝑐 above the x-axis, the radius of the disk is 𝑓(𝑥) − 𝑐. If the axis
of revolution is a line 𝑦 = −𝑐 below the x-axis, the radius of the disk is 𝑓(𝑥) + 𝑐.
A solid of revolution may also be revolved about the y-axis and about any line
parallel to the y-axis. In that case, the integration is carried out with respect to the
variable 𝑦.
Cylinder Method
The cylinder method differs from the disk method because it uses hollow
cylinders or shells as approximation. The method involves dividing the solid of revolution
formed into an infinitesimal number of concentric hollow cylinders or shells whose
thickness is ∆𝑥.
Consider a function defined by 𝑦 = 𝑓(𝑥). Let the plane figure be bounded by the
function, the x-axis, and the lines 𝑥 = 𝑎 and 𝑥 = 𝑏. If the plane figure is revolved about
the y-axis, a solid of revolution is formed which we divide into hollow cylinders. Note
that each cylinder or shell has a radius 𝑥𝑖 and a length equal to the value of 𝑦 at 𝑥𝑖 or
𝑓(𝑥𝑖 ). The volume of each cylinder or shell is equal to the circumference of the cross-
section times its length times the thickness (if the cylinder is flattened into a thin
rectangular sheet). Thus,
∆𝑉 = 2𝜋𝑥𝑖 𝑓(𝑥𝑖 ) ∆𝑥
𝑏−𝑎
∆𝑥 =
𝑛
242
x
f(x)
The total volume of the solid is approximated by the sum of the volumes of each cylinder
or shell, or
𝑛
𝑉 ≈ 2𝜋 ∑ 𝑥𝑖 𝑓(𝑥𝑖 )∆𝑥
𝑖=1
If the plane figure is bounded by 𝑦 = 𝑓(𝑥) and 𝑦 = 𝑔(𝑥) and 𝑓(𝑥) ≥ 𝑔(𝑥) and the
figure is revolved about the x-axis, then the cylinder or shell has length equal to 𝑓(𝑥)
minus 𝑔(𝑥). Consequently, the volume of the solid of revolution, thus, formed is
243
𝑏
𝑉 = 2𝜋 ∫ 𝑥 [𝑓(𝑥) − 𝑔(𝑥)] 𝑑𝑥 13.4
𝑎
If the solid of revolution is revolved about an axis parallel to the y-axis, the radius
of the cross-section of the cylinder needs to be adjusted depending on how far apart is
the axis of revolution from the y-axis. Thus, if 𝑓(𝑥) > 0, the radius of the cylinder is 𝑥 ± 𝑐.
If the axis of revolution is a line 𝑥 = +𝑐 to the right of the y-axis, the radius is 𝑥 − 𝑐. If the
axis of revolution is a line 𝑥 = −𝑐 to the left of the y-axis, the radius is 𝑥 + 𝑐.
Using this method, a solid of revolution may also be revolved about the x-axis and
about any line parallel to the x-axis. In that case, the integration is carried out with
respect to the variable 𝑦.
_____________________________________________________________________________________________________
Example 13.1
Find the volume of the solid of revolution formed by revolving the region bounded by
𝑦 = 𝑥 2 , the x-axis, and the line 𝑥 = 3 about the x-axis, using both disk and cylinder
method.
Solution. The region is the plane figure bounded by the curve, the x-axis, and the line 𝑥 =
3. Hence, we integrate on the interval [0, 3]. The graph of the function and the solid of
revolution formed are shown below.
244
Using the disk method, the radius of the disk is 𝑦 = 𝑥 2 . The volume of the solid of
revolution, therefore, is
3 3
𝑉 = 𝜋 ∫ (𝑥 2 )2 𝑑𝑥 = 𝜋 ∫ 𝑥 4 𝑑𝑥
0 0
3
1 1
= 𝜋 ∙ 𝑥 5 | = (35 − 0)
5 0 5
243
= 𝜋 𝑐𝑢. 𝑢𝑛𝑖𝑡𝑠 ANS.
5
245
Using the cylinder method, we note that the radius of the cross-section of the
cylinder is 𝑦. The length of the cylinder is 3 minus the value of 𝑥 or 3 − √𝑦. The region
is bounded by the x-axis corresponding to 𝑦 = 0 up to 𝑦 = 𝑥 2 = 32 or 9. Hence, the
volume of the solid of revolution is
9 9
𝑉 = 2𝜋 ∫ 𝑦 (3 − √𝑦) 𝑑𝑦 = 2𝜋 ∫ (3𝑦 − 𝑦 3/2 ) 𝑑𝑦
0 0
3 2 2 5/2 9 3 2
= 2𝜋 ∙ ( 𝑦 − 𝑦 )| = 2𝜋 ∙ [( ∙ 92 − ∙ 95/2 ) − 0]
2 5 0 2 5
Choosing the method to use, whether the disk or ring method or the cylinder or
shell method, depends on circumstances, primarily, the axis of revolution. It is not a hard
and fast rule to use the disk or ring method if the axis of revolution is parallel to the x-
axis and the cylinder or shell method if the axis is parallel to the y-axis. In many instances,
both methods are convenient to use although in some, one is superior to the other.
Usually, it is easier to set up the integral using the disk or ring method because it is easier
to visualize. But the integral set up using the cylinder or shell method is easier to evaluate
because it avoids the complication of squaring the integrand.
_____________________________________________________________________________________________________
Example 13.2.
Find the volume of the solid of revolution in Example 12.1 by revolving the region about
the y-axis, using both disk and cylinder method.
Solution. The graph of the function and the solid of revolution are shown below.
246
Using the disk method, we note that the inner radius of the disk is equal to the value of
𝑥 which is equal to √𝑦 while the outer radius is 3. The region extends from 𝑥 = 0 to 𝑥 =
3 corresponding to 𝑦 = 0 to 𝑦 = 9. Hence, the volume of solid of revolution formed is
9 9
2
2
𝑉 = 𝜋 ∫ [3 − (√𝑦) ] 𝑑𝑦 = 𝜋 ∫ (9 − 𝑦) 𝑑𝑦
0 0
1 2 9 81
= 𝜋 (9𝑦 − 𝑦 )| = 𝜋 [(81 − ) − 0]
2 0 2
81𝜋
= 𝑐𝑢. 𝑢𝑛𝑖𝑡𝑠 ANS.
2
Using the cylinder method, we note that the radius of a cross-section of the
cylinder is 𝑥 while its length is the value of 𝑦 which is equal to 𝑥 2 . The region to be
revolved extends from 𝑥 = 0 to 𝑥 = 3. Hence, the volume of solid of revolution formed
is
3 3
𝑉 = 2𝜋 ∫ 𝑥 ∙ 𝑥 2 𝑑𝑥 = 2𝜋 ∫ 𝑥 3 𝑑𝑥
0 0
1 43 1
= 2𝜋 ∙ 𝑥 | = 2𝜋 ( ∙ 34 − 0)
4 0 4
247
81𝜋
= 𝑐𝑢. 𝑢𝑛𝑖𝑡𝑠 ANS.
2
_____________________________________________________________________________________________________
Example 13.3.
Find the volume of a sphere by revolving a semi-circle whose radius is 𝑟 about the x-axis.
Solution. Let us consider a circle whose center is (0, 0) and whose radius is 𝑟. The
equation of that circle is
𝑥2 + 𝑦2 = 𝑟2
-r r
248
Using the disk method more conveniently, the radius of the disk is the value of 𝑦 =
√𝑟 2 − 𝑥 2 . We take the positive root if we revolve the part of the circle in the upper
quadrants. The volume of solid of revolution is
𝑟 2
𝑉 = 𝜋 ∫ (√𝑟 2 − 𝑥 2 ) 𝑑𝑥
−𝑟
𝑟
= 𝜋 ∫ (𝑟 2 − 𝑥 2 ) 𝑑𝑥
−𝑟
𝑟
1
= 𝜋 (𝑟 2 𝑥 − 𝑥 3 )|
3 −𝑟
1 1
= 𝜋 [(𝑟 3 − 𝑟 3 ) − (−𝑟 3 + 𝑟 3 )]
3 3
4𝜋 3
= 𝑟 ANS.
3
We can approach the problem using the cylindrical method. We note that the
cylinder has radius equal to 𝑥 and a length equal to √𝑟 2 − 𝑥 2 at the top minus −√𝑟 2 − 𝑥 2
at the bottom or √𝑟 2 − 𝑥 2 − −√𝑟 2 − 𝑥 2 = 2√𝑟 2 − 𝑥 2 .
249
𝑟 0
2𝜋 ∫ 𝑥 ∙ 2√𝑟 2 − 𝑥 2 𝑑𝑥 = −2𝜋 ∫ √𝑢 𝑑𝑢
0 𝑟2
2 3/2 0
= −2𝜋 ∙ 𝑢 |
3 𝑟2
4
= − 𝜋(0 − 𝑟 3 )
3
4 3
𝑉= 𝜋𝑟 ANS.
3
By this time, it should be obvious that if you use the disk method with the disk
lying flat, we integrate with respect to 𝑦. But if the disk stands on its circular edge, we
integrate with respect to 𝑥. On the other hand, if you use the cylinder method, we
integrate with respect to 𝑥 if the cylinder stands erect and with respect to 𝑦 if it lies on
its edge.
___________________________________________________________________________________________________
Example 13.4.
Find the volume of the solid of revolution formed by revolving the region bounded by
𝑦 = 𝑥 3 , the line 𝑥 = 2, and the x-axis about an axis (a) through the line 𝑥 = 3 and (b)
through the line 𝑦 = 9, using both disk and cylinder method.
Solution. The graph of the function and its boundaries are shown below. Note that the
curve intersects the line 𝑥 = 2 at (2, 8).
250
(a) About an axis through the line 𝑥 = 3:
Using the disk method, we note that the inner radius of the disk is the distance
between the line 𝑥 = 2 and 𝑥 = 3, or 1 while the outer radius is 3 − 3√𝑦. The region spans
from 𝑦 = 0 to 𝑦 = 8. The volume of solid of revolution is
8
2
𝑉 = 𝜋 ∫ [(3 − 3√𝑦) − (3 − 2)2 ] 𝑑𝑦
0
8
= 𝜋 ∫ (9 − 6 3√𝑦 + 𝑦 2/3 − 1) 𝑑𝑦
0
9 4/3 3 5/3 8
= 𝜋 (8𝑦 − 𝑦 + 𝑦 )|
2 5 0
9 3
= 𝜋 [8(8 − 0) − (16 − 0) + (32 − 0)]
2 5
56𝜋
= 𝑐𝑢. 𝑢𝑛𝑖𝑡𝑠 ANS.
5
Using the cylinder method, we note that the length of the cylinder is 𝑦 = 𝑥 3 and
the radius of its cross-section is 3 − 𝑥 since the distance is measured from the line 𝑥 = 3.
The region to be revolved spans from 𝑥 = 0 to 𝑥 = 2. The volume of solid of revolution is
2
𝑉 = 2𝜋 ∫ (3 − 𝑥) (𝑥 3 ) 𝑑𝑥
0
2
= 2𝜋 ∫ (3𝑥 3 − 𝑥 4 ) 𝑑𝑥
0
3 4 1 5 2
= 2𝜋 ( 𝑥 − 𝑥 )|
4 5 0
3 1
= 2𝜋 [ (24 − 0) − (25 − 0)]
4 5
56𝜋
= 𝑐𝑢. 𝑢𝑛𝑖𝑡𝑠 ANS.
5
251
(b) About an axis through the line 𝑦 = 9:
Using the disk method, we note that the outer radius of the disk is 9 and its inner
radius is 9 minus the value of 𝑦 or 9 − 𝑥 3 since the distance is measured from the line
𝑦 = 9. The volume of solid of revolution is
2
𝑉 = 𝜋 ∫ [92 − (9 − 𝑥 3 )2 ] 𝑑𝑥
0
2
= 𝜋 ∫ [81 − (𝑥 6 − 18𝑥 3 + 81)] 𝑑𝑥
0
1 7 9 4 2
= 𝜋 (− 𝑥 + 𝑥 )|
7 2 0
1 9
= 𝜋 [− (27 − 0) + (24 − 0)]
7 2
376𝜋
= 𝑐𝑢. 𝑢𝑛𝑖𝑡𝑠 ANS.
7
252
Using the cylinder method, we note that the length of the cylinder is 2 − 3√𝑦
measured from the line 𝑥 = 2, and the radius of its cross-section is 9 − 𝑦 measured from
the line 𝑦 = 9. The volume of solid of revolution is
8
𝑉 = 2𝜋 ∫ (9 − 𝑦)(2 − 3√𝑦) 𝑑𝑦
0
8
= 2𝜋 ∫ (18 − 2𝑦 − 9 3√𝑦 + 𝑦 4/3 ) 𝑑𝑦
0
27 4/3 3 7/3 8
2
= 2𝜋 (18𝑦 − 𝑦 − 𝑦 + 𝑦 )|
4 7 0
27 3
= 2𝜋[18(8 − 0] − (64 − 0) − (16 − 0) + (128 − 0)]
4 7
188 376𝜋
= 2𝜋 ∙ = 𝑐𝑢. 𝑢𝑛𝑖𝑡𝑠 ANS.
7 7
As a hint, we integrate with respect to 𝑥 when using the disk method about an axis
parallel to the x-axis and the cylinder method about an axis parallel to the y-axis. On the
other hand, we integrate with respect to 𝑦 when using the disk method about an axis
parallel to the y-axis and the cylinder method about an axis parallel to the x-axis.
_____________________________________________________________________________________________________
253
Example 13.5.
Find the volume of the solid of revolution formed by revolving the region bounded by
𝑦 = 4 − 𝑥 2 , the line 𝑦 = 3𝑥, and the x-axis in the first quadrant about an axis (a) through
the line 𝑥 = −1 and (b) through the line 𝑦 = −1, using both disk and cylinder method.
Solution. The region referred to in the example is shown below. The line intersects the
parabola at (1, 3) in the first quadrant.
𝑦 = 3𝑥
𝑥 = −1
𝑦 = 4 − 𝑥2
𝑦 = −1
Using the disk or ring method, we note that the inner radius of the disk equals the
value of 𝑥 plus 1 for the line 𝑦 = 3𝑥 or 𝑥 = 𝑦/3 + 1, while its outer radius equals the value
of 𝑥 plus 1 for the parabola 𝑦 = 4 − 𝑥 2 or 𝑥 = √4 − 𝑦 + 1. We added 1 because the
radius is measured from the line 𝑥 = −1. We also note that the region extends from 𝑦 =
0 to 𝑦 = 3. Thus, the volume of solid of revolution is
3 2
2 1
𝑉 = 𝜋 ∫ [(√4 − 𝑦 + 1) − ( 𝑦 + 1) ] 𝑑𝑦
0 3
254
3
5 1
= 𝜋 ∫ (4 − 𝑦 + 2√4 − 𝑦 − 𝑦 2 ) 𝑑𝑦
0 3 9
5 2 4 3/2
1 3 3
= 𝜋 (4𝑦 − 𝑦 − (4 − 𝑦) − 𝑦 )|
6 3 27 0
5 4 1
= 𝜋 [4(3 − 0) − (9 − 0) − (1 − 8) − (27 − 0)]
6 3 27
77𝜋
= 𝑐𝑢. 𝑢𝑛𝑖𝑡𝑠 ANS.
6
Using the cylinder or shell method, we divide the region into two sub-regions: first,
from 𝑥 = 0 to 𝑥 = 1 under the straight line 𝑦 = 3𝑥, and second, from 𝑥 = 1 to 𝑥 = 2 under
the parabola 𝑦 = 4 − 𝑥 2 . The two sub-regions are distinct from one another because the
length of the cylinder for the first sub-region is 𝑦 = 3𝑥 while that for the second sub-
region is 𝑦 = 4 − 𝑥 2 . Note that the radius of the cylinder in either sub-region is 𝑥 + 1,
the distance being reckoned from the line 𝑥 = −1. The volume of solid of revolution is
1 2
𝑉 = 2𝜋 ∫ (𝑥 + 1) ∙ 3𝑥 𝑑𝑥 + 2𝜋 ∫ (𝑥 + 1) (4 − 𝑥 2 ) 𝑑𝑥
0 1
1 2
= 2𝜋 ∫ (3𝑥 2 + 3𝑥) 𝑑𝑥 + 2𝜋 ∫ (−𝑥 3 − 𝑥 2 + 4𝑥 + 4) 𝑑𝑥
0 1
3 2 1
3
1 4 1 3 2
2
= 2𝜋 (𝑥 + 𝑥 )| + 2𝜋 (− 𝑥 − 𝑥 + 2𝑥 + 4𝑥)|
2 0 4 3 1
3 1 1
= 2𝜋 [(1 + ) − 0] + 2𝜋 [− (16 − 1) − (8 − 1) + 2(4 − 1) + 4(2 − 1)]
2 4 3
47𝜋 77𝜋
= 5𝜋 + = 𝑐𝑢. 𝑢𝑛𝑖𝑡𝑠 ANS.
6 6
(b) About an axis through the line 𝑦 = −1:
Using the disk or ring method, we also divide the region into two sub-regions: first,
from 𝑥 = 0 to 𝑥 = 1 under the straight line 𝑦 = 3𝑥, and second, from 𝑥 = 1 to 𝑥 = 2 under
the parabola 𝑦 = 4 − 𝑥 2 . The two sub-regions are distinct from one another because the
outer radius of the disk for the first sub-region is the value of 𝑦 plus 1 or 3𝑥 + 1 while
that for the second sub-region is 𝑦 plus 1 or 4 − 𝑥 2 + 1 = 5 − 𝑥 2 . We added 1 because
the radii are measured from the line 𝑦 = −1. The inner radius of the disks for both sub-
regions is, of course, equal to 1 because that is how far the edge of the region, i.e., the x-
axis, is from the axis of revolution.
255
The volume of solid of revolution is
1 2
𝑉 = 𝜋 ∫ [(3𝑥 + 1)2 − 12 ] 𝑑𝑥 + 𝜋 ∫ [(5 − 𝑥 2 )2 − 12 ] 𝑑𝑥
0 1
1 2
= 𝜋 ∫ (9𝑥 2 + 6𝑥) 𝑑𝑥 + 𝜋 ∫ (𝑥 4 − 10𝑥 2 + 24) 𝑑𝑥
0 1
9 3 6 2 1 1 5 10 3 1
= 𝜋 ( 𝑥 + 𝑥 )| + 𝜋 ( 𝑥 − 𝑥 + 24𝑥)|
3 2 0 5 3 0
1 10
= 𝜋[(3 + 3) − 0] + 𝜋 [ (32 − 1) − (8 − 1) + 24(2 − 1)]
5 3
103𝜋 193𝜋
= 6𝜋 + = 𝑐𝑢. 𝑢𝑛𝑖𝑡𝑠 ANS.
15 15
Using the cylinder method, we note that the length of the cylinder is the value of
𝑥 for the parabola or √4 − 𝑦 minus the value of 𝑥 for the straight line or 𝑦/3. We also
note that the radius of the cross-section of the cylinder is 𝑦 + 1 because we reckon the
distance from the axis of revolution, i.e., the line 𝑦 = −1. The region that is revolved about
this axis spans from 𝑦 = 0 to 𝑦 = 3. The volume of solid of revolution is
3
1
𝑉 = 2𝜋 ∫ (𝑦 + 1) (√4 − 𝑦 − 𝑦) 𝑑𝑦
0 3
3 3
1
= 2𝜋 ∫ (𝑦 + 1)(√4 − 𝑦)𝑑𝑦 − 2𝜋 ∫ (𝑦 + 1) ( 𝑦) 𝑑𝑦
0 0 3
10 3/2 2 5/2 1 1 3 1 2 3
= 2𝜋 (− 𝑢 + 𝑢 )| − 2𝜋 ( 𝑦 + 𝑦 )|
3 5 4 9 6 0
10 2 1 1
= 2𝜋 ∙ [− (1 − 8) + (1 − 32)] − 2𝜋 [ (27 − 0) + (9 − 0)]
3 5 9 6
164 9 193𝜋
= 2𝜋 ∙ − 2𝜋 ∙ = 𝑐𝑢. 𝑢𝑛𝑖𝑡𝑠 ANS.
15 2 15
_____________________________________________________________________________________________________
256
Example 13.6.
Find the volume of the solid of revolution formed by revolving the region bounded by
𝑦 = 4𝑥 − 𝑥 2 and the x-axis about the y-axis using both methods.
Solution. The function is a parabola opening downward whose vertex is (2, 4). It
intersects the x-axis at 𝑥 = 0 and 𝑥 = 4. It may also be written as 𝑦 = −(𝑥 − 2)2 + 4 so
that 𝑥 = 2 ± √4 − 𝑦.
Using the disk method, we divide the solid into concentric disks of thickness 𝑑𝑦.
Each disk has an inner radius equal to 𝑥 = 2 − √4 − 𝑦 and outer radius equal to 𝑥 = 2 +
√4 − 𝑦. We integrate from the bottom of the region along the x-axis or 𝑦 = 0 to the top
of the region at its vertex or 𝑦 = 4. Thus,
4
2 2
𝑉 = 𝜋 ∫ [(2 + √4 − 𝑦) − (2 − √4 − 𝑦) ] 𝑑𝑦
0
4
= 𝜋 ∫ 8√4 − 𝑦 𝑑𝑦
0
4
2 3/2
= 8𝜋 ∙ [− (4 − 𝑦) ]
3 0
257
16
=− 𝜋(0 − 8)
3
128
= 𝜋 𝑐𝑢. 𝑢𝑛𝑖𝑡𝑠 ANS.
3
Using the cylinder method, we divide the solid into concentric hollow cylinders of
thickness 𝑑𝑥 whose radius is 𝑥 and whose length is 𝑦 = 4𝑥 − 𝑥 2 . We integrate from 𝑥 =
0 to 𝑥 = 4. Thus,
4
𝑉 = 2𝜋 ∫ 𝑥 (4𝑥 − 𝑥 2 ) 𝑑𝑥
0
4 3 1 4 4
= 2𝜋 ( 𝑥 − 𝑥 )|
3 4 0
256 256
= 2𝜋 ( − − 0)
3 4
128
= 𝜋 𝑐𝑢. 𝑢𝑛𝑖𝑡𝑠 ANS.
3
_____________________________________________________________________________________________________
Example 13.7.
Find the volume of a torus whose radius is 𝑎 and whose cross section has a radius of 𝑏.
Solution. A torus looks very much like a donut with a hole in the center. The region to
revolved is a circle of radius 𝑏 about the y-axis. The distance of the center of the cross-
section from the axis of revolution is 𝑎 as shown.
𝑎 − √𝑏 2 − 𝑦 2
𝑎 + √𝑏 2 − 𝑦 2
258
(𝑥 − 𝑎)2 + 𝑦 2 = 𝑏 2
so that
𝑥 = 𝑎 ± √𝑏 2 − 𝑦 2
The inner radius of the disk is 𝑎 − √𝑏 2 − 𝑦 2 while its outer radius is 𝑎 + √𝑏 2 − 𝑦 2 . The
volume of solid of revolution is
𝑏 2 2
𝑉 = 𝜋 ∫ [(𝑎 + √𝑏 2 − 𝑦 2 ) − (𝑎 − √𝑏 2 − 𝑦 2 ) ] 𝑑𝑦
−𝑏
𝑏
= 𝜋 ∫ [(𝑎2 + 2𝑎√𝑏 2 − 𝑦 2 + 𝑏 2 − 𝑦 2 ) − (𝑎2 − 2𝑎√𝑏 2 − 𝑦 2 + 𝑏 2 − 𝑦 2 )] 𝑑𝑦
−𝑏
𝑏 𝑏
= 4𝜋𝑎 ∫ √𝑏 2 − 𝑦 2 𝑑𝑦 = 8𝜋𝑎 ∫ √𝑏 2 − 𝑦 2 𝑑𝑦
−𝑏 0
Hence,
1
𝑉 = 8𝜋𝑎 ∙ 𝜋𝑏 2 = 2𝜋 2 𝑎𝑏 2 ANS.
4
_____________________________________________________________________________________________________
259
Try these.
260
Other Applications of Definite
14. Integrals
𝑓(𝑏) − 𝑓(𝑎)
𝑓 ′ (𝑐) =
𝑏−𝑎
𝑓(𝑏) − 𝑓(𝑎) = 𝑓′(𝑐)(𝑏 − 𝑎)
This is known as the mean value theorem, which you learned in differential calculus.
261
If we get the anti-derivative of 𝑓 ′ (𝑐), then we get 𝑓(𝑐) instead and the left side of
the equation can mean the anti-derivative of the function 𝑓(𝑥) evaluated on an interval
[𝑎, 𝑏]. Hence,
𝑏
∫ 𝑓(𝑥) 𝑑𝑥 = 𝑓(𝑐)(𝑏 − 𝑎)
𝑎
𝑏
1
𝑓(𝑐) = ∫ 𝑓(𝑥) 𝑑𝑥 14.1
𝑏−𝑎 𝑎
Since 𝑓(𝑐) lies between 𝑓(𝑎) and 𝑓(𝑏), 𝑓(𝑐) is known as the mean or average value of the
function 𝑓(𝑥) on an interval [𝑎, 𝑏].
_____________________________________________________________________________________________________
Example 14.1.
Find the mean or average value of 𝑦 = 𝐴 𝑠𝑖𝑛 𝑘𝑥 where 𝐴 and 𝑘 are constants, from
0 to 𝜋 and from 0 to 2𝜋.
262
1
=− 𝐴 (cos 𝑘𝜋 − cos 0)
𝑘𝜋
2𝐴
= ANS.
𝑘𝜋
From 0 to 2𝜋, the mean value of the function is
2𝜋
1
𝑓(𝑐) = ∫ 𝐴 𝑠𝑖𝑛 𝑘𝑥 𝑑𝑥
2𝜋 − 0 0
2𝜋
1
=− 𝐴 𝑐𝑜𝑠 𝑘𝑥|
2𝑘𝜋 0
1
=− 𝐴 (cos 2𝑘𝜋 − cos 0)
𝑘𝜋
=0 ANS.
That the mean value of the function is zero is quite obvious from the graph.
_____________________________________________________________________________________________________
Example 14.2.
Find the value of 𝑐 for the mean value of the function 𝑦 = 𝑥 2 + 3𝑥 − 4 from 1 to 5.
1 3
= (125 − 1) + (25 − 1) − (5 − 1)
12 8
46
=
3
If 𝑓(𝑐) =46/3, then
46
𝑥 2 + 3𝑥 − 4 =
3
3𝑥 2 + 9𝑥 − 58 = 0
263
Using the quadratic equation,
−𝑏 ± √𝑏 2 − 4𝑎𝑐
𝑥=
2𝑎
−9 ± √92 − 4(3)(−58)
=
2(3)
= 3.14, −6.14
Length of an Arc
The length of a line between two points can be easily measured. But measuring
the length of a curve between two points is more complicated. Nevertheless, we can
approximate the length of the curve between two points by measuring the length of the
line that joins them. In this case, we partition the entire curve into a number of segments
connecting successive points by straight lines.
Consider the curve below. Connecting P and Q with a straight line, the distance
between P and Q using the distance formula is
∆𝑠 = √(∆𝑥)2 + (∆𝑦)2
60
50
Q
40 ∆s
∆y
P
30 ∆x
20
10
0
0 2 4 6 8 10 12
264
Dividing the equation by ∆𝑥/∆𝑥,
∆𝑠 ∆𝑦 2
= √1 + ( )
∆𝑥 ∆𝑥
Using this approximation, the length of the entire curve on an interval [𝑎, 𝑏] is
𝑛 𝑛
∆𝑦 2
𝑠 ≈ ∑ ∆𝑠𝑖 ≈ ∑ √1 + ( ) ∆𝑥
∆𝑥
𝑖=1 𝑖=1
The length of the entire curve is better approximated if we use more but smaller
segments. This means that the length of the curve is equal to the limiting value of the sum
of the lengths of each segment, or
𝑛
∆𝑦 2
√
𝑠 = lim ∑ 1 + ( ) ∆𝑥
𝑛→∞ ∆𝑥
𝑖=1
In some instances, it would be easier to solve for the length of a curve by expressing 𝑥 as
a function of 𝑦. In this case, the length is
𝑑
𝑑𝑥 2
𝑠 = ∫ √1 + ( ) 𝑑𝑦 14.2(b)
𝑐 𝑑𝑦
_____________________________________________________________________________________________________
Example 14.3.
Find the length of a circle whose radius is 𝑟.
𝑥2 + 𝑦2 = 𝑟2
To find the length of an entire circle, we need to find the length of half circle and then
multiply the result by 2. The derivative of the equation is
2𝑥 𝑑𝑥 + 2𝑦 𝑑𝑦 = 0
265
𝑑𝑦 𝑥 𝑥
=− =−
𝑑𝑥 𝑦 √𝑟 2 − 𝑥 2
𝑟 2
𝑥
𝑠 = ∫ √1 + (− ) 𝑑𝑥
−𝑟 √𝑟 2 − 𝑥 2
𝑟
𝑥2
= ∫ √1 + 𝑑𝑥
−𝑟 𝑟2 − 𝑥2
𝑟
𝑟
=∫ 𝑑𝑥
−𝑟 √𝑟 2 − 𝑥2
𝑥 𝑟
= 𝑟 𝑠𝑖𝑛−1 |
𝑟 −𝑟
= 𝑟 (𝑠𝑖𝑛−1 1 − 𝑠𝑖𝑛−1 − 1)
𝜋 𝜋
= 𝑟 ( + ) = 𝜋𝑟
2 2
Hence, for the complete circle,
𝑠 = 2𝜋𝑟 ANS.
_____________________________________________________________________________________________________
Example 14.4.
Find the length of a parabola 𝑦 = 𝑥 2 from 𝑥 = −1 to 𝑥 = 1
1 1 1 1
= ( 𝑙𝑛 |2 + √5| + √5) − ( 𝑙𝑛 |−2 + √5| − √5)
4 2 4 2
266
1 √5 + 2
= 𝑙𝑛 | | + √5 ANS.
4 √5 − 2
_____________________________________________________________________________________________________
Example 14.5.
Find the length of the curve 𝑦 = 𝑥 3/2 from 𝑥 = 0 to 𝑥 = 5.
Solution. Getting the derivative of 𝑦 gives 𝑦 ′ = 32√𝑥. Thus, the length of the curve
from 𝑥 = 0 to 𝑥 = 5 is
5 2
3
𝑠 = ∫ √1 + ( √𝑥) 𝑑𝑥
0 2
5
9𝑥
= ∫ √1 + 𝑑𝑥
0 4
1 5
= ∫ √9𝑥 + 4 𝑑𝑥
2 0
5
1 2 1 3/2
= ∙ ∙ (9𝑥 + 4) |
2 3 9 0
1
= (493/2 − 43/2 )
27
335
= ANS.
27
_____________________________________________________________________________________________________
Consider a plane region bounded by the function 𝑦 = 𝑓(𝑥), the lines 𝑥 = 𝑎 and
𝑥 = 𝑏 and the x-axis. Let’s say that we revolve the plane region about the x-axis. To get
the area of the lateral surface of the solid, we need to cut the solid into many thin slices
which look like flattened, truncated cones, like this:
267
Now, the area of the lateral surface of such a cone is
𝑟1 + 𝑟2
𝐴 = 2𝜋 ( )𝑠
2
where 𝑟1 and 𝑟2 are the smaller and bigger radii and 𝑠 is the lateral length. It should come
as obvious by now that if the slice is thin enough, then the average of both radii is the
value of the function 𝑦 = 𝑓(𝑥) at their midpoint while 𝑑𝑠 is actually the length of a small
arc of the function 𝑦 = 𝑓(𝑥) which bounds the plane region revolved about the x-axis.
Hence, the area of a thin slice is
𝑑𝐴 = 2𝜋 𝑓(𝑥) 𝑑𝑠
𝑑𝑠 = √1 + (𝑑𝑦/𝑑𝑥)2 𝑑𝑥
_____________________________________________________________________________________________________
Example 14.6.
Find the area of the lateral surface of a plane region bounded by 𝑦 = 𝑥, the x-axis, and the
line 𝑥 = 1 revolved about the x-axis.
Solution. The plane region described is a right isosceles triangle whose altitude and base
are both equal to 1. It actually generates a cone whose lateral surface is 𝜋 × 𝑟 × 𝑠 where
𝑟 = 1 (the altitude of the triangle, or 𝑦 when 𝑥 = 1) and 𝑠 = √12 + 12 = √2 (its
hypotenuse). Hence, 𝐴 = √2 𝜋.
268
Using calculus, that cone has a lateral surface area equal to
𝑏
𝐴 = 2𝜋 ∫ 𝑓(𝑥) √1 + (𝑑𝑦/𝑑𝑥)2 𝑑𝑥
𝑎
1
= 2𝜋 ∫ 𝑥 √1 + 1 𝑑𝑥
0
1 21
= 2𝜋√2 ∙ 𝑥 |
2 0
Example 14.7.
Find the area of the lateral surface of a plane region bounded by 𝑦 = 𝑥 2 , the y-axis, and
the line 𝑦 = 4 revolved about the y-axis.
Solution. The plane region described above generates a paraboloid. If the paraboloid is
cut into thin slices, we have truncated cones lying flat whose radius is 𝑥 instead of 𝑓(𝑥).
Now, the thin slices start from 𝑥 = √𝑦 = √0 to 𝑥 = √4 = 2. Consequently, if that region
is revolved about the y-axis, we have a paraboloid whose lateral surface has an area equal
to
𝑏
𝐴 = 2𝜋 ∫ 𝑥 √1 + (𝑑𝑦/𝑑𝑥)2 𝑑𝑥
𝑎
269
2
= 2𝜋 ∫ 𝑥 √1 + (2𝑥)2 𝑑𝑥
0
2
= 2𝜋 ∫ 𝑥 √1 + 4𝑥 2 𝑑𝑥
0
1 2 3/2 17
= 𝜋∙ 𝑢 |
4 3 1
1
= (17√17 − 1) 𝜋 𝑠𝑞. 𝑢𝑛𝑖𝑡𝑠 ANS.
6
_____________________________________________________________________________________________________
Line Integrals
In Chapter 12, we obtained the area of the region under the curve 𝑦 = 𝑓(𝑥)
bounded by the x-axis and the lines 𝑥 = 𝑎 and 𝑥 = 𝑏. Visually, we can consider this area
as the region under the curve 𝑦 = 𝑓(𝑥) and above the line segment along the x-axis from
𝑥 = 𝑎 to 𝑥 = 𝑏. Now suppose, instead of a curve, we have a surface 𝑧 = 𝑓(𝑥, 𝑦). We can
obtain the area of the region under this surface and over a line segment on the 𝑥𝑦 plane
from, say, (𝑥1 , 𝑦1 ) to (𝑥2 , 𝑦2 ). To do this, we can also use rectangular strips whose height
is the value of 𝑓(𝑥, 𝑦) and whose width is 𝑑𝑠, so the area of each strip is 𝑑𝐴 = 𝑓(𝑥, 𝑦) 𝑑𝑠.
270
Hence, the area of the region under the surface 𝑧 = 𝑓(𝑥, 𝑦) and over a line segment on
the 𝑥𝑦 plane is
𝐴 = ∫𝑓(𝑥, 𝑦) 𝑑𝑠
𝑐
The notation 𝐶 in the integral sign simply means that the integration is carried not over
an interval [𝑎, 𝑏] but over a continuous line or even a curve C (e.g., parabola, hyperbola,
etc.). This is called the line integral because the integration is carried over a line or curve.
In order to evaluate a line integral over a curve C, it behooves that the curve on
the 𝑥𝑦 plane be described as parametric equations of 𝑥 and 𝑦 with respect to a third
variable, say, 𝑡 or 𝑥 = 𝑥(𝑡) and 𝑦 = 𝑦(𝑡). After this, following Eq. 13.2, we can express 𝑑𝑠
as
𝑑𝑥 2 𝑑𝑦 2
√
𝑑𝑠 = ( ) + ( ) 𝑑𝑡
𝑑𝑡 𝑑𝑡
𝑑𝑥 2 𝑑𝑦 2 14.4
𝐴 = ∫𝑓(𝑥(𝑡), 𝑦(𝑡)) √( ) + ( ) 𝑑𝑡
𝑐 𝑑𝑡 𝑑𝑡
_____________________________________________________________________________________________________
Example 14.8.
Find the area under the surface 𝑧 = 2𝑥 2 + 𝑦 2 and over a line segment from (1, 1) to
(2, 3).
Solution. The equation of the line joining (1, 1) and (2, 3) is 𝑦 = 2𝑥 − 1. In terms of the
variable 𝑡, the line can be described as
𝑥 = 𝑡, 𝑦 = 2𝑡 − 1, 1≤𝑡≤2
Hence,
𝑑𝑥 2 𝑑𝑦 2
𝐴 = ∫𝑓(𝑥(𝑡), 𝑦(𝑡)) √( ) + ( ) 𝑑𝑡
𝑐 𝑑𝑡 𝑑𝑡
271
2
= ∫ (6𝑡 2 − 4𝑡 + 1) √12 + 22 𝑑𝑡
1
2
= (2𝑡 3 − 2𝑡 2 + 𝑡)√5|1
_____________________________________________________________________________________________________
Example 14.9.
𝑥𝑦 2
Find the area under the surface 𝑧 = and over a parabola 𝑦 = 𝑥 2 from (0, 0) to
3
(1, 1).
𝑥 = 𝑡, 𝑦 = 𝑡2 , 0≤𝑡≤1
1 4
𝑧= (𝑡)(2𝑡)2 = 𝑡 3
3 3
The area of the region under the surface and over C is
𝑑𝑥 2 𝑑𝑦 2
√
𝐴 = ∫𝑓(𝑥(𝑡), 𝑦(𝑡)) ( ) + ( ) 𝑑𝑡
𝑐 𝑑𝑡 𝑑𝑡
1
4 3
=∫ 𝑡 √1 + 4𝑡 2 𝑑𝑡
0 3
1 5 3/2
= ∫ (𝑢 − √𝑢) 𝑑𝑢
24 1
1 2 5/2 2 3/2 5
= ( 𝑢 − 𝑢 )|
24 5 3 1
272
1 10 2 2
= [(10√5 − √5) − ( − )]
24 3 5 3
1 20 4
= ( √5 + )
24 3 15
25√5 + 1
= 𝑠𝑞. 𝑢𝑛𝑖𝑡𝑠 ANS.
90
_____________________________________________________________________________________________________
Centroid
The centroid or center of mass (traditionally known as center of gravity) is the
point on the body where its mass is said to be concentrated. For a perfectly geometrical
body of uniform density, the center of mass is also its geometric center. For example, the
center of mass of a sphere of uniform density lies at its geometric center. If the body
assumes an irregular shape, finding the center of mass is not as easy. But we can use the
method of calculus to find it. In this section, we will find the center of mass of a 2-
dimensional plane region.
If there are N particles, each of mass 𝑚𝑖 and located at a point (𝑥𝑖 , 𝑦𝑖 ), the center
of mass of these particles (𝑥̅ , 𝑦̅) is obtained by getting the “weighted’ average of their
distances from the origin, thus,
𝑚1 𝑥1 + 𝑚2 𝑥2 + 𝑚3 𝑥3 +. . . ∑𝑁
𝑖=1 𝑚𝑖 𝑥𝑖
𝑥̅ = = 𝑁
𝑚1 + 𝑚2 + 𝑚3 + ⋯ ∑𝑖=1 𝑚𝑖
𝑚1 𝑦1 + 𝑚2 𝑦2 + 𝑚3 𝑦3 +. . . ∑𝑁
𝑖=1 𝑚𝑖 𝑦𝑖
𝑦̅ = = 𝑁
𝑚1 + 𝑚2 + 𝑚3 + ⋯ ∑𝑖=1 𝑚𝑖
Now, imagine a plane region under the curve 𝑓(𝑥) between 𝑥 = 𝑎 and 𝑥 = 𝑏 whose mass
is evenly distributed throughout its area. If 𝜌 represents its mass density or mass per unit
area, the total mass 𝑚 of the plane region is
𝑏
𝑚 = 𝜌 ∫ 𝑓(𝑥) 𝑑𝑥
𝑎
If we divide the plane region into very small rectangles whose dimensions are 𝑓(𝑥) in
length and 𝑑𝑥 in width and whose mass is 𝑑𝑚 = 𝜌 𝑓(𝑥) 𝑑𝑥, then the center of mass of
each rectangle lies at its center at (𝑥, 12𝑦). Thus, the center of mass of the region along the
x-axis is the weighted average of all the x’s or
273
𝑏 𝑏
∫𝑎 𝜌𝑥𝑓(𝑥) 𝑑𝑥 ∫𝑎 𝑥𝑓(𝑥) 𝑑𝑥
𝑥̅ = 𝑏 = 𝑏 14.5
∫𝑎 𝜌𝑓(𝑥) 𝑑𝑥 ∫𝑎 𝑓(𝑥) 𝑑𝑥
On the other hand, the center of mass of the region along the y-axis is the weighted
average of all the ½y’s or
𝑏1 𝑏1
∫𝑎 2 𝜌𝑦𝑓(𝑥) 𝑑𝑥 ∫𝑎 2 [𝑓(𝑥)]2 𝑑𝑥
𝑦̅ = 𝑏 = 𝑏
14.6
∫𝑎 𝜌𝑓(𝑥) 𝑑𝑥 ∫𝑎 𝑓(𝑥) 𝑑𝑥
_____________________________________________________________________________________________________
Example 14.10.
Locate the center of mass of the plane region in Example 12.6.
Solution. The plane region lies between the curves 4𝑦1 = 𝑥 2 and 𝑦2 2 = 4𝑥. We have
already shown previously that its area is 16/3 square units.
𝑏 4 1
∫𝑎 𝑥[𝑓(𝑥) − 𝑔(𝑥)] 𝑑𝑥 ∫0 𝑥 (2√𝑥 − 4 𝑥 2 ) 𝑑𝑥
𝑥̅ = 𝑏 =
∫𝑎 [𝑓(𝑥) − 𝑔(𝑥)] 𝑑𝑥 4 1
∫0 (2√𝑥 − 4 𝑥 2 ) 𝑑𝑥
274
𝑏1
4 2 1 2 2
∫𝑎 2 ([𝑓(𝑥)]2 − [𝑔(𝑥)]2 ) 𝑑𝑥 ∫0 ([2√𝑥] − [4 𝑥 ] ) 𝑑𝑥
1
𝑦̅ = =
𝑏
∫𝑎 [𝑓(𝑥) − 𝑔(𝑥)] 𝑑𝑥 2 4 1
∫0 (2√𝑥 − 4 𝑥 2 ) 𝑑𝑥
Note the difference between these and Eqs. 11.3 and 11.4. Integrating reveals that
4 1
∫0 (2𝑥 3/2 − 4 𝑥 3 ) 𝑑𝑥
𝑥̅ =
4 1
∫0 (2√𝑥 − 4 𝑥 2 ) 𝑑𝑥
4 5/2 1 44
𝑥 − 𝑥 |
5 16 0
=
16
3
48
9
= 5 =
16 5
3
4 1 4
1 ∫0 (4𝑥 − 16 𝑥 ) 𝑑𝑥
𝑦̅ =
2 4 (2 𝑥 − 1 𝑥 2 ) 𝑑𝑥
∫0 √ 4
2 1 5 4
(2𝑥 −
1 80 𝑥 )| 0
=
2 16
3
96
1 5 9
= ∙ =
2 16 5
3
Applications in Physics
The application of definite integral in physical situations is very extensive. In most
instances, it allows us to compute the value of a certain parameter which changes with
respect to defined variable. For example, if the speed of a body depends on the time and
the distance that a body traverses depends on the speed, definite integral allows us to
compute the distance travelled after the lapse of a certain time.
275
There are also other applications of definite integral in physical situations. If a
particle has a certain property which is dependent on a particular variable, then it is
possible to measure the value of that property of an object or system given the
distribution of the particle in that object or system.
_____________________________________________________________________________________________________
Example 14.11.
A stone is thrown upward near the edge of a building 100 m high with an initial speed of
19.6 m/s. The speed of the stone at any time 𝑡 is given by
𝑣 = 𝑣0 − 𝑔𝑡
where 𝑣0 is the initial speed and 𝑔 = 9.8 m/s2 which is the acceleration due to gravity.
Where is the stone 6 seconds after it was thrown?
𝑑𝑦
𝑣=
𝑑𝑡
where 𝑦 is the distance travelled by the stone measured from the top of the building.
Hence,
𝑑𝑦
= 𝑣0 − 𝑔𝑡
𝑑𝑡
Integrating yields
1
𝑦 = 𝑣0 𝑡 − 2𝑔𝑡 2
So, 6 s after the stone was thrown, the distance it has travelled is
1
𝑦 = (19.6 𝑚/𝑠)(6 𝑠) − 2(9.8 𝑚/𝑠 2 )(6 𝑠)2
𝑦 = −58.8 𝑚 ANS.
This means that 6 s after it was thrown upward, the stone is at a point 58.8 m below the
top of the building or 41.2 m above the ground.
_____________________________________________________________________________________________________
Example 14.12.
The work done by a force 𝐹 in moving an object through a displacement 𝑥 is 𝑊 = 𝐹𝑥,
expressed in Joules. The force applied in stretching a spring of force constant 𝑘 is
276
proportional to its elongation 𝑥, or 𝐹 = −𝑘𝑥, where the negative sign shows that the
direction of the force is opposite its elongation. How much work is done in stretching a
spring of force constant 𝑘 = 10 𝑁/𝑚 from an original length of 0.2 𝑚 to a length of 0.5 𝑚.
𝑥2
= ∫ −𝑘𝑥 𝑑𝑥
𝑥1
𝑥2
1
= − 𝑘𝑥 2 |
2 𝑥1
1
= − 𝑘( 𝑥2 2 − 𝑥1 2 )
2
Originally, 𝑥1 = 0 until the spring was stretched to 𝑥2 = 0.5 − 0.2 or 0.3 𝑚. Thus, the
work done is
1
𝑊 = − (10 𝑁/𝑚)((0.3 𝑚)2 − 02 )
2
= −0.45 𝐽 ANS.
_____________________________________________________________________________________________________
Example 14.13.
When torque is applied to a body that is free to rotate about any axis, the body acquires
an angular acceleration that is inversely proportional to its moment of inertia 𝐼. The
moment of inertia of a body that is free to rotate about any axis is given by
𝐼 = ∫ 𝑟 2 𝑑𝑚
where 𝑟 is the distance of a particle of mass 𝑑𝑚 from the chosen axis of revolution. A
hollow disk of mass 𝑚, thickness 𝑙, and inner radius 𝑟1 and outer radius 𝑟2 is free to rotate
about an axis through its geometric center. Obtain an expression for the moment of
inertia of the hollow disk, a thin-walled disk, and a solid disk.
277
r
𝑑𝑚 = 2𝜋𝑟𝜌𝑙 𝑑𝑟
𝐼 = ∫ 𝑟 2 (2𝜋𝑟𝜌𝑙 𝑑𝑟)
𝑟2
= 2𝜋𝜌𝑙 ∫ 𝑟 3 𝑑𝑟
𝑟1
1 𝑟2
= 2𝜋𝜌𝑙 ∙ 𝑟 4 |
4 𝑟1
1
= 𝜋𝜌𝑙(𝑟2 4 − 𝑟1 4 )
2
1
= 𝜋𝜌𝑙(𝑟2 2 − 𝑟1 2 )(𝑟2 2 + 𝑟1 2 )
2
But the volume of the hollow cylinder is 𝜋𝑙(𝑟2 2 − 𝑟1 2 ). Hence, multiplying it by the
density 𝜌 gives the mass or
1
𝐼= 𝑚(𝑟2 2 + 𝑟1 2 ) ANS.
2
For thin-walled disks, we just allow 𝑟1 to equal 𝑟2 or just 𝑟. Hence,
1
𝐼 = 𝑚(𝑟 2 + 𝑟 2 ) = 𝑚𝑟 2 ANS.
2
For solid disks, we just make 𝑟1 = 0. Hence,
278
1 1
𝐼= 𝑚(𝑟2 2 + 0) = 𝑚𝑟 2 ANS.
2 2
_____________________________________________________________________________________________________
Example 14.14.
The electric field due to a point charge 𝑞 at a point a distance 𝑟 from the charge is given
by
𝑘𝑞
𝐸=
𝑟2
where 𝑘 = 9 × 109 𝑁𝑚2 /𝐶 2 . Obtain an expression for the electric field due to a charged
conducting ring of radius 𝑅 and total charge 𝑄 at a point along the line through its center.
Solution. If we partition the charged ring into very small segments, then the electric field
due to a segment is
𝑘 𝑑𝑞
𝑑𝐸 =
𝑟2
dq
r
R
θ dEx
x
dE
The y-component of 𝑑𝐸 will cancel out due to symmetry. The x-component of 𝑑𝐸,
however, will combine. If we let 𝑥 be the distance of the point from the center of the ring,
the x-component of 𝑑𝐸 is
𝑘 𝑑𝑞
𝑑𝐸𝑥 = 𝑐𝑜𝑠 𝜃
𝑟2
𝑘 𝑑𝑞 𝑥
=
𝑥 2 + 𝑅 2 √𝑥 2 + 𝑅 2
𝑘𝑥 𝑑𝑞
=
(𝑥 2 + 𝑅 2 )3/2
𝑘𝑄𝑥
𝐸= ANS.
(𝑥 2 + 𝑅 2 )3/2
_____________________________________________________________________________________________________
280
Try these.
3. Find the length of the portion of the parabola 𝑦 2 = 2𝑥 transected by the line
𝑦 = 𝑥 − 4.
4. Obtain an expression for the length of the portion of a circle of radius 𝑟 cut off
from it by a secant line whose length is 𝑙.
5. Find the area of the surface of revolution formed by revolving the plane region
bounded by 𝑦 = √𝑥, the x-axis, and the line 𝑥 = 4 about the x-axis. (Hint:
Integrate with respect to y.)
6. Find the area of the surface of revolution formed by revolving the plane region
bounded by 𝑦 = 𝑥 3 , the x-axis, and the line 𝑥 = 2 about the x-axis.
9. A 20 m heavy chain 100 kg in mass (weight is 980 N) is suspended from the top
of a building. How much work is done in lifting the entire chain?
10. Gauss’ Law states that the total electric flux through any closed surface is
proportional to the net electric charge inside the surface, or
𝑞
∮𝑬 ⃗⃗ ∙ 𝑑𝑨
⃗⃗ =
𝜖0
Consider an infinitely long charged wire whose uniform linear charge density
is 𝜆. Show that the electric field at a point a distance 𝑟 perpendicular to the wire
is
1 𝜆
𝐸=
2𝜋𝜖0 𝑟
281
15. Approximate Integration
In the previous chapters, we were able to evaluate integrals because the anti-
derivative of the function exists. In many instances, however, the anti-derivative of a
function does not exist or it cannot be expressed as an elementary function. An example
of such function is
2
∫ 𝑒 −𝑥 𝑑𝑥
In that case, it would be difficult to evaluate the definite integral. Fortunately, we can
estimate the value of the definite integral using approximation methods to a surprising
degree of accuracy.
Midpoint Rule
Geometrically, the definite integral of a function on an interval [𝑎, 𝑏] is equal to
the area of the region bounded by the function, the line 𝑥 = 𝑎 and 𝑥 = 𝑏, and the x-axis.
We can approximate the region by partitioning it into a number of rectangular strips of
uniform width ∆𝑥 such that
𝑏−𝑎
∆𝑥 =
𝑛
In Chapter 10, we did such approximation using strips whose top right edge touches the
function. We realize, however, that the strips excessively cover the region so the total
area of the strips is an overestimate of the area of the region. If we use strips whose top
left edge touches the function, we have this time an underestimate of the area of the
282
region because the strips deficiently cover the region. So, a better approximation is to use
strips which touch the function at the top middle or the midpoint of the strip.
f(x)
𝑥̅31
𝑥̅1 𝑥̅2
x=a x=b
If ∆𝑥 is the width of a strip, then the coordinate of the midpoint of the first strip is
𝑥̅1 = 𝑎 + 0.5 ∆𝑥. The length of the first strip is the value of the function at the midpoint
of the strip 𝑥̅1 or 𝑓(𝑥̅1 ) = 𝑓(𝑎 + 0.5 ∆𝑥). The coordinate of the midpoint of the second
strip would then be 𝑥̅2 = 𝑎 + 0.5 ∆𝑥 + ∆𝑥 = 𝑎 + 1.5 ∆𝑥 and its length is 𝑓(𝑥̅2 ) = 𝑓(𝑎 +
1.5 ∆𝑥). In general, the coordinate of the 𝑟𝑡ℎ strip is 𝑥̅𝑟 = 𝑎 + (𝑟 − 12)∆𝑥 and its length is
𝑓(𝑥̅𝑟 ) = 𝑓(𝑎 + (𝑟 − 12)∆𝑥). The area of the 𝑟𝑡ℎ strip is 𝑓(𝑥̅𝑟 ) ∆𝑥 = 𝑓(𝑎 + (𝑟 − 12)∆𝑥) ∙ ∆𝑥
so the area of the bounded region is
𝑛
𝐴 ≈ ∑ 𝑓(𝑥̅𝑟 ) ∆𝑥
𝑟=1
𝑛
1
≈ ∑ 𝑓(𝑎 + (𝑟 − 2) ∆𝑥) ∙ ∆𝑥
𝑟=1
Hence,
𝑏 𝑛
1
∫ 𝑓(𝑥) 𝑑𝑥 ≈ ∑ 𝑓(𝑎 + (𝑟 − 2) ∆𝑥) ∙ ∆𝑥 15.1
𝑎 𝑟=1
_____________________________________________________________________________________________________
283
Example 15.1.
Use the midpoint rule to approximate, accurate to three decimal places, the integral
3
∫ 𝑥 2 𝑑𝑥
1
Solution. If 𝑛 = 5,
3−1
∆𝑥 = = 0.4
5
The table below shows the required computation.
1
𝑟 𝑥̅𝑟 = 𝑎 + (𝑟 − 2) ∆𝑥 𝑓(𝑥̅𝑟 ) 𝑓(𝑥̅𝑟 ) ∆𝑥
1 1.2 1.44 0.576
2 1.6 2.56 1.024
3 2.0 4.00 1.600
4 2.4 5.76 2.304
5 2.8 7.84 3.136
Total 8.640
Hence,
3
∫ 𝑥 2 𝑑𝑥 ≈ 8.640 ANS.
1
3−1
∆𝑥 = = 0.2
10
The table below shows the required computation.
284
1
𝑟 𝑥̅𝑟 = 𝑎 + (𝑟 − 2) ∆𝑥 𝑓(𝑥̅𝑟 ) 𝑓(𝑥̅𝑟 ) ∆𝑥
1 1.1 1.21 0.242
2 1.3 1.69 0.338
3 1.5 2.25 0.450
4 1.7 2.89 0.578
5 1.9 3.61 0.722
6 2.1 4.41 0.882
7 2.3 5.29 1.058
8 2.5 6.25 1.250
9 2.7 7.29 1.458
10 2.9 8.41 1.682
Total 8.660
Hence,
3
∫ 𝑥 2 𝑑𝑥 ≈ 8.660 ANS.
1
The error is +0.0067. This shows that the more strips, the better the approximation.
_____________________________________________________________________________________________________
Example 15.2.
Use the midpoint rule to approximate the integral
6
1
∫ 𝑑𝑥
2 𝑥+1
accurate to four decimal places. Use 𝑛 = 4 and 𝑛 = 8. Compute also the error in both
instances.
Solution. If 𝑛 = 4,
6−2
∆𝑥 = = 1.0
4
The table below shows the required computation in four decimal places.
285
𝑟 𝑥̅𝑟 = 𝑎 + (𝑟 − 12)∆𝑥 𝑓(𝑥̅𝑟 ) 𝑓(𝑥̅𝑟 ) ∆𝑥
1 2.5 0.2857 0.2857
2 3.5 0.2222 0.2222
3 4.5 0.1818 0.1818
4 5.5 0.1538 0.1538
Total 0.8436
Hence,
6
1
∫ 𝑑𝑥 ≈ 0.8436 ANS.
2 𝑥+1
6−2
∆𝑥 = = 0.5
8
The table below shows the required computation in four decimal places.
286
Hence,
6
1
∫ 𝑑𝑥 ≈ 0.8464 ANS.
2 𝑥+1
Example 15.3.
The sine integral function is defined as
𝑥
𝑠𝑖𝑛 𝑡
𝑆𝑖(𝑥) = ∫ 𝑑𝑡
0 𝑡
Use the midpoint rule to estimate 𝑆𝑖(3) accurate to three decimal places. Use 𝑛 = 9.
1
Solution. Since 𝑛 = 9, ∆𝑥 = 3. The table below shows the required computation in four
decimal places.
Thus,
3
𝑠𝑖𝑛 𝑡
𝑆𝑖(3) = ∫ 𝑑𝑡 ≈ 1.8503
0 𝑡
287
Online sine integral calculator returns a value of 1.8486 in four decimal places so our
value is a little overestimated.
_____________________________________________________________________________________________________
Trapezoidal Rule
A different method of approximating definite integrals uses trapezoids instead of
rectangular strips. Trapezoids are better approximate of curves because the curve is
approximated by an oblique line segment instead of a horizontal line segment. But like
the midpoint rule, we also partition the region under consideration into 𝑛 trapezoids of
uniform width ∆𝑥 such that
𝑏−𝑎
∆𝑥 =
𝑛
The area of a trapezoidal strip is equal to 1/2(ℓ1 + ℓ2 ) × 𝑤. For the first strip, the
length of the left edge ℓ1 = 𝑓(𝑥0 ) = 𝑓(𝑎) while the length of the right edge ℓ2 = 𝑓(𝑥1 ) =
𝑓(𝑎 + ∆𝑥). For the second strip, the length of the left edge ℓ1 = 𝑓(𝑥1 ) = 𝑓(𝑎 + ∆𝑥) while
the length of the right edge ℓ2 = 𝑓(𝑥2 ) = 𝑓(𝑎 + 2 ∆𝑥). For the 𝑟𝑡ℎ strip, the length of the
left edge ℓ1 = 𝑓(𝑥𝑟−1 ) = 𝑓(𝑎 + (𝑟 − 1) ∆𝑥) while the length of the right edge ℓ2 =
𝑓(𝑥𝑟 ) = 𝑓(𝑎 + 𝑟 ∆𝑥). For the last strip, the length of the left edge ℓ1 = 𝑓(𝑎 + (𝑛 − 1) ∆𝑥)
while the length of the right edge ℓ2 = 𝑓(𝑥𝑛 ) = 𝑓(𝑏). The combined area of the
trapezoidal strips is
f(x)
x3
x2
x0 x1 2
x=a x=b
288
𝑓(𝑥0 ) + 𝑓(𝑥1 ) 𝑓(𝑥1 ) + 𝑓(𝑥2 ) 𝑓(𝑥𝑛−1 ) + 𝑓(𝑥𝑛 )
𝐴= ∙ ∆𝑥 + ∙ ∆𝑥 + ⋯ + ∙ ∆𝑥
2 2 2
1 1
= [ 𝑓(𝑎) + 𝑓(𝑥1 ) + 𝑓(𝑥2 ) + ⋯ + 𝑓(𝑥𝑛−1 ) + 𝑓(𝑏)] ∆𝑥
2 2
𝑛−1
1 1
= 𝑓(𝑎) ∙ ∆𝑥 + ∑ 𝑓(𝑥𝑟 ) ∆𝑥 + 𝑓(𝑏) ∙ ∆𝑥
2 2
𝑟=1
𝑛−1
1 1
= 𝑓(𝑎) ∙ ∆𝑥 + ∑ 𝑓(𝑎 + 𝑟 ∆𝑥) ∆𝑥 + 𝑓(𝑏) ∙ ∆𝑥
2 2
𝑟=1
Hence,
𝑏 𝑛−1
1 1
∫ 𝑓(𝑥) 𝑑𝑥 ≈ 𝑓(𝑎) ∙ ∆𝑥 + ∑ 𝑓(𝑎 + 𝑟 ∆𝑥) ∙ ∆𝑥 + 𝑓(𝑏) ∙ ∆𝑥 15.2
𝑎 2 2
𝑟=1
____________________________________________________________________________________________________
Example 15.4.
Use the trapezoidal rule to approximate the integral
3
∫ 𝑥 2 𝑑𝑥
1
accurate to three decimal places. Use 𝑛 = 5 and 𝑛 = 10. Compute also the error in both
instances.
Solution. If 𝑛 = 5, ∆𝑥 = 0.4. The table below shows the required computation. Note that
we computed up to the (𝑛 − 1)𝑡ℎ strip only.
𝑟 𝑥𝑟 = 𝑎 + 𝑟 ∆𝑥 𝑓(𝑥𝑟 ) 𝑓(𝑥𝑟 ) ∆𝑥
1 1.40 1.96 0.784
2 1.80 3.24 1.296
3 2.20 4.84 1.936
4 2.60 6.76 2.704
Total 6.720
289
3
1 1
∫ 𝑥 2 𝑑𝑥 ≈ (1)(0.4) + 6.720 + (9)(0.4)
1 2 2
≈ 8.72 ANS.
On the other hand, if 𝑛 = 10, ∆𝑥 = 0.2. The table below shows the required
computation.
𝑟 𝑥𝑟 = 𝑎 + 𝑟 ∆𝑥 𝑓(𝑥𝑟 ) 𝑓(𝑥𝑟 ) ∆𝑥
1 1.2 1.44 0.288
2 1.4 1.96 0.392
3 1.6 2.56 0.512
4 1.8 3.24 0.648
5 2.0 4.00 0.800
6 2.2 4.84 0.968
7 2.4 5.76 1.152
8 2.6 6.76 1.352
9 2.8 7.84 1.568
Total 7.680
Hence,
3
1 1
∫ 𝑥 2 𝑑𝑥 ≈ (1)(0.2) + 7.680 + (9)(0.2)
1 2 2
≈ 8.68 ANS.
The error is −0.013. Notice that the trapezoidal method appears to overestimate the
value of the definite integral while the midpoint method seems to underestimate it.
_____________________________________________________________________________________________________
Example 15.5.
Use the trapezoidal rule to approximate the integral
6
1
∫ 𝑑𝑥
2 𝑥+1
290
accurate to four decimal places. Use 𝑛 = 4 and 𝑛 = 8. Compute also the error in both
instances.
Solution. If 𝑛 = 4, ∆𝑥 = 1.0. The table below shows the required computation in four
decimal places.
𝑟 𝑥𝑟 = 𝑎 + 𝑟 ∆𝑥 𝑓(𝑥𝑟 ) 𝑓(𝑥𝑟 ) ∆𝑥
1 3 0.2500 0.2500
2 4 0.2000 0.2000
3 5 0.1667 0.1667
Total 0.6167
≈ 0.8548 ANS.
On the other hand, if 𝑛 = 8, ∆𝑥 = 0.5. The table below shows the required
computation in four decimal places.
𝑟 𝑥𝑟 = 𝑎 + 𝑟 ∆𝑥 𝑓(𝑥𝑟 ) 𝑓(𝑥𝑟 ) ∆𝑥
1 2.5 0.2857 0.1429
2 3.0 0.2500 0.1250
3 3.5 0.2222 0.1111
4 4.0 0.2000 0.1000
5 4.5 0.1818 0.0909
6 5.0 0.1667 0.0833
7 5.5 0.1538 0.0769
Total 0.7301
Hence,
291
6
1 1 1
∫ 𝑑𝑥 ≈ (0.3333)(0.5) + 0.7301 + (0.1429)(0.5)
2 𝑥+1 2 2
≈ 0.8491 ANS.
The error is −0.0018. Comparing with the error using the midpoint rule, it appears that
the trapezoidal rule is not better than the midpoint rule in approximating definite
integrals. The absolute error in using the trapezoidal rule is about twice that in using the
midpoint rule.
_____________________________________________________________________________________________________
Example 15.6.
Use the trapezoidal rule to estimate the integral
5
∫ (3𝑥 − 7) 𝑑𝑥
2
𝑟 𝑥𝑟 = 𝑎 + 𝑟 ∆𝑥 𝑓(𝑥𝑟 ) 𝑓(𝑥𝑟 ) ∆𝑥
1 2.5 0.5 0.25
2 3.0 2.0 1.00
3 3.5 3.5 1.75
4 4.0 5.0 2.50
5 4.5 6.5 3.25
Total 8.75
≈ 10.5 ANS.
292
3
= (25 − 4) − 7(5 − 2)
2
= 10.5
Thus, the error is zero. This should not come as a surprise because the integrand is a
sloping straight line. A trapezoid is a perfect fit for such straight line.
___________________________________________________________________________
Simpson’s Rule
Another approximation method uses parabolas to approximate curves instead of
line segments. This method is known as Simpson’s rule after Thomas Simpson, an English
mathematician in the 1700s. Similar to the two methods previously discussed, Simpson’s
rule allows the partitioning of the region into an even number of strips of uniform width
∆𝑥, whose top edge is a portion of a parabola of the form 𝑎𝑥 2 + 𝑏𝑥 + 𝑐. Because we need
at least three points to determine the values of the coefficients of the parabola 𝑎, 𝑏, and 𝑐,
it is not enough to use a single strip but two consecutive strips. This is the reason why
the rule applies only to even number of strips. The width ∆𝑥 is also given by
𝑏−𝑎
∆𝑥 = =ℎ
𝑛
with the condition that 𝑛 is even.
P1 (0, y1)
P0 (-h, y0)
P1 (h, y2)
∆x ∆x
0 x
To determine the area of two consecutive strips, we evaluate the integral
ℎ
𝐴 = ∫ (𝑎𝑥 2 + 𝑏𝑥 + 𝑐) 𝑑𝑥
−ℎ
293
It is left as an exercise to show that
ℎ
2
∫ (𝑎𝑥 2 + 𝑏𝑥 + 𝑐) 𝑑𝑥 = 𝑎ℎ3 + 2𝑐ℎ
−ℎ 3
If the parabola passes through at least three points in two consecutive strips, say, the first
and second, at 𝑥 = −ℎ, 𝑥 = 0, and 𝑥 = ℎ, then
𝑓(𝑥0 ) = 𝑎ℎ2 − 𝑏ℎ + 𝑐
𝑓(𝑥1 ) = 𝑐
𝑓(𝑥2 ) = 𝑎ℎ2 − 𝑏ℎ + 𝑐
These equations allow us to solve for the values of the coefficients 𝑎, 𝑏, and 𝑐. When
substituted to the equation for area, the result is
ℎ
𝐴= [𝑓(𝑥0 ) + 4 𝑓(𝑥1 ) + 𝑓(𝑥2 )]
3
If we consider the next pair, i.e., the third and fourth strip, the result is
ℎ
𝐴= [𝑓(𝑥2 ) + 4 𝑓(𝑥3 ) + 𝑓(𝑥4 )]
3
Therefore, the total area up to the last two strips is
ℎ
𝐴= [𝑓(𝑥0 ) + 4 𝑓(𝑥1 ) + 2 𝑓(𝑥2 ) + 4 𝑓(𝑥3 ) + 2 𝑓(𝑥4 ) + ⋯
3
+ 2 𝑓(𝑥𝑛−2 ) + 4 𝑓(𝑥𝑛−1 ) + 𝑓(𝑥𝑛 )]
Example 15.7.
Use Simpson’s rule to approximate the integral
294
3
∫ 𝑥 2 𝑑𝑥
1
accurate to three decimal places. Use 𝑛 = 10. Compute also the error in approximating.
3
0.2
∫ 𝑥 2 𝑑𝑥 ≈ ∙ 130 ≈ 8.667 ANS.
1 3
Note that there are 11 terms because we start with 𝑓(𝑥0 ). The error is zero.
It should not come as a surprise that this is the exact value of the definite integral
and not a mere approximation. The reason is that Simpson’s rule approximates the curve
with parabolas. But the function itself is a parabola so that there was really no
approximation used.
_____________________________________________________________________________________________________
295
Example 15.8.
Use Simpson’s rule to approximate the integral
6
1
∫ 𝑑𝑥
2 𝑥+1
accurate to four decimal places. Use 𝑛 = 4 and 𝑛 = 8. Compute also the error in both
instances.
≈ 0.8476 ANS.
0 2 0.3333 1 0.3333
1 2.5 0.2857 4 1.1429
2 3 0.2500 2 0.5000
3 3.5 0.2222 4 0.8889
4 4 0.2000 2 0.4000
5 4.5 0.1818 4 0.7273
6 5 0.1667 2 0.3333
7 5.5 0.1538 4 0.6154
8 6 0.1429 1 0.1429
Total 5.0839
6
1 0.5
∫ 𝑑𝑥 ≈ ∙ 5.0839
2 𝑥+1 3
≈ 0.8473 ANS.
296
The error is ~0. Compared with the errors in using the midpoint and trapezoidal rules,
the best approximation is through the use of Simpson’s rule.
_____________________________________________________________________________________________________
Example 15.9.
Use Simpson’s rule to approximate the integral
2
2
∫ 𝑒 𝑥 𝑑𝑥
1
≈ 15.0745 ANS.
Example 15.10.
Use Simpson’s rule to approximate the integral
𝜋/2
∫ √𝑐𝑜𝑠 𝑥 𝑑𝑥
0
Use 𝑛 = 6.
≈ 1.1873 ANS.
_____________________________________________________________________________________________________
297
Approximate Integration Using Infinite Power Series
In addition to the preceding numerical methods for estimating the value of a
definite integral, we can also use the method of infinite power series. This method
involves converting the integrand into a power series, integrating the terms in the series,
and evaluating the terms using the upper and lower limits of the definite integral.
Suppose that a function 𝑓(𝑥) can be expressed as a sum of the power series
If 𝑥 = 𝑎, then
𝑓(𝑎) = 𝑐0
If 𝑥 = 𝑎, then
𝑓′(𝑎) = 𝑐1
If 𝑥 = 𝑎, then
𝑓′′(𝑎) = 2𝑐2
If 𝑥 = 𝑎, then
𝑓 ′′′(𝑎) = 2 ∙ 3𝑐3 = 3! 𝑐3
298
𝑓 (𝑛) (𝑎) = 𝑛! 𝑐𝑛
𝑓 (𝑛) (𝑎)
𝑐𝑛 =
𝑛!
Finally, for a function 𝑓(𝑥) that can be expressed as a power series,
The Maclaurin series for common functions are summarized below for reference:
∞
1
= 1 + 𝑥 + 𝑥2 + 𝑥3+ . . . = ∑ 𝑥𝑛 , 𝑥 < 1
1−𝑥
𝑛=0
∞
𝑥3 𝑥5 𝑥7 𝑛
𝑥 2𝑛+1
𝑠𝑖𝑛 𝑥 = 𝑥 − + − + . . . = ∑(−1)
3! 5! 7! (2𝑛 + 1)!
𝑛=0
∞
𝑥2 𝑥4 𝑥6 𝑛
𝑥 2𝑛
𝑐𝑜𝑠 𝑥 = 1 − + − + . . . = ∑(−1)
2! 4! 6! (2𝑛)!
𝑛=0
∞
𝑥3 𝑥5 𝑥7 𝑥 2𝑛+1
𝑠𝑖𝑛ℎ 𝑥 = 𝑥 + + + + . . . = ∑
3! 5! 7! (2𝑛 + 1)!
𝑛=0
∞
𝑥2 𝑥4 𝑥6 𝑥 2𝑛
𝑐𝑜𝑠ℎ 𝑥 = 1 + + + + . . . = ∑
2! 4! 6! (2𝑛)!
𝑛=0
299
∞
−1
𝑥3 𝑥5 𝑥7 𝑥 2𝑛+1
𝑡𝑎𝑛 𝑥 = 𝑥 − + − + . . . = ∑(−1)𝑛
3 5 7 2𝑛 + 1
𝑛=0
∞
−1
𝑥3 𝑥5 𝑥7 𝑥 2𝑛+1
𝑡𝑎𝑛ℎ 𝑥 = 𝑥 + + + + . . .= ∑
3 5 7 2𝑛 + 1
𝑛=0
∞
1 1 1 𝑥𝑛
𝑒 = 1 + 𝑥 + 𝑥2 + 𝑥3+ . . . = ∑
𝑥
1! 2! 3! 𝑛!
𝑛=0
1 1 1
𝑙𝑛 |𝑥| = (𝑥 − 1) − (𝑥 − 1)2 + (𝑥 − 1)3 − (𝑥 − 1)4 + . . .
2 3 4
∞ 𝑛
(𝑥 − 1)
= ∑(−1)𝑛−1
𝑛
𝑛=0
∞
𝑘
𝑘(𝑘 − 1) 2 𝑘(𝑘 − 1)(𝑘 − 2) 3 𝑘
(1 + 𝑥) = 1 + 𝑘𝑥 + 𝑥 + 𝑥 + . . . = ∑ ( ) 𝑥𝑛
2! 3!
𝑛=0 𝑛
Although using infinite power series may be tempting to use in approximating the
value of a definite integral, care must be observed in using it. It behooves that the infinite
series is convergent, i.e., the sum of the terms is not infinite. One must follow the allowed
values of 𝑥 in using examples of Maclaurin series appearing above to arrive at a definite
value. But, of course, we don’t need to get there because we are only approximating. We
can stop after a certain number of terms depending on the amount of error of
approximation.
_____________________________________________________________________________________________________
Example 15.11.
Approximate
1
2
∫ 𝑒 −𝑥 𝑑𝑥
0
300
∞
(−1)𝑛 𝑥 2𝑛
=∑
𝑛!
𝑛=0
The definite integral becomes
1 1 ∞
(−1)𝑛 𝑥 2𝑛
−𝑥 2
∫ 𝑒 𝑑𝑥 = ∫ ∑ 𝑑𝑥
0 0 𝑛!
𝑛=0
∞ 1
(−1)𝑛 𝑥 2𝑛+1
=∑ |
(2𝑛 + 1)𝑛!
𝑛=0 0
1 1 1 1 1
≈ (1 − + − + − +⋯)−0
3 10 42 216 1322
≈ 0.7467 ANS.
Note that we carried out our calculation up to 𝑛 = 5 only since 1/1322 < 0.001.
_____________________________________________________________________________________________________
Example 15.12.
∫ 𝑥𝑒 𝑥 𝑑𝑥 = (𝑥 − 1)𝑒 𝑥 + 𝐶
Proof. Using power series, the integral becomes
∞
𝑥
𝑥𝑛
∫ 𝑥𝑒 𝑑𝑥 = ∫ 𝑥 ∑ 𝑑𝑥
𝑛!
𝑛=0
∞
𝑥 𝑛+1
= ∫∑ 𝑑𝑥
𝑛!
𝑛=0
∞
𝑥 𝑛+2
=∑ +𝐶
𝑛! (𝑛 + 2)
𝑛=0
∞
𝑥 𝑛+2
=∑ (𝑛 + 1) + 𝐶
(𝑛 + 2)!
𝑛=0
301
∞
𝑥
𝑥𝑛
(𝑥 − 1)𝑒 = (𝑥 − 1) ∑
𝑛!
𝑛=0
𝑥 𝑥2 𝑥3 𝑥4
= (𝑥 − 1) (1 + + + + +. . . )
1! 2! 3! 4!
𝑥2 𝑥3 𝑥3 𝑥4 𝑥 𝑥2 𝑥3 𝑥4
= (𝑥 + + + + +. . . ) − (1 + + + + +. . . )
1! 2! 3! 4! 1! 2! 3! 4!
1 1 1 1 1 1
= −1 + 𝑥 2 ( − ) + 𝑥3 ( − ) + 𝑥4 ( − ) + . . .
1! 2! 2! 3! 3! 4!
𝑥 2 2𝑥 3 3𝑥 4
= −1 + + + +. . .
2! 3! 4!
∞
𝑥 𝑛+2
= −1 + ∑ (𝑛 + 1)
(𝑛 + 2)!
𝑛=0
Hence,
∞
𝑥
𝑥 𝑛+2
∫ 𝑥𝑒 𝑑𝑥 = −1 + ∑ (𝑛 + 1) + 𝐶
(𝑛 + 2)!
𝑛=0
= (𝑥 − 1)𝑒 𝑥 + 𝐶 𝑞. 𝑒. 𝑑.
Example 15.13.
Approximate
𝜋/2
∫ 𝑥 𝑠𝑖𝑛 𝑥 𝑑𝑥
0
302
𝜋/2
𝜋/2
∫ 𝑥 𝑠𝑖𝑛 𝑥 𝑑𝑥 = 𝑠𝑖𝑛 𝑥 − 𝑥 𝑐𝑜𝑠 𝑥|0 =1
0
So, our answer should be approximately equal to 1. Using the power series for sine of a
function, we have
∞
𝑥 2𝑛+1
𝑥 𝑠𝑖𝑛 𝑥 = 𝑥 ∑(−1)𝑛
(2𝑛 + 1)!
𝑛=0
∞
𝑥 2𝑛+1 ∙ 𝑥
= ∑(−1)𝑛
(2𝑛 + 1)!
𝑛=0
∞
𝑥 2𝑛+2
= ∑(−1)𝑛
(2𝑛 + 1)!
𝑛=0
𝜋/2 3 5 7 9
(𝜋) (𝜋) (𝜋 ) (𝜋)
∫ 𝑥 𝑠𝑖𝑛 𝑥 𝑑𝑥 ≈ [ 2 − 2 + 2 − 2 + . . . ] − [0]
0 1! 3 3! 5 5! 7 7! 9
≈ 1.000001
303
Try these.
4. The error function is a special function with a sigmoid shape that is useful in
probability and statistics. It is defined as
2 𝑥 −𝑡 2
𝑒𝑟𝑓(𝑥) = ∫ 𝑒 𝑑𝑡
√𝜋 0
Use the midpoint rule to evaluate up to 4 decimal places 𝑒𝑟𝑓(1) Use 𝑛 = 6.
5. The Fresnel integrals find application in diffraction of light. They are defined as
follows:
𝑥 𝑥
𝜋𝑡 2 𝜋𝑡 2
𝐶(𝑥) = ∫ 𝑐𝑜𝑠 𝑑𝑡 𝑆(𝑥) = ∫ 𝑠𝑖𝑛 𝑑𝑡
0 2 0 2
Use the trapezoidal rule to evaluate 𝐶(12) and 𝑆(12) up to 5 decimal places. Use
𝑛 = 7.
304
16. Multiple Integrals
In some instances, a function is not only a function of one variable but by two or
three or even more variables. In the previous chapters, we dealt with the integration of a
function of a single variable. We also obtained the area of a plane region and volume of a
solid by evaluating the definite integral of a function of single variable. If we can evaluate
the definite integral of a single variable function, we can also evaluate the definite integral
of a function of several variables. The process is known as multiple integration and we
can also use it to find the volume of a solid. Integrals of a function of two variables over a
region R2 are called double integrals. Integrals of a function of three variables over a
region R3 are called triple integrals.
Double Integrals
Consider a function 𝑧 = 𝑓(𝑥, 𝑦). Such a function defines a surface in three
dimensional space. Just as we defined a function 𝑦 = 𝑓(𝑥) on an interval [𝑎, 𝑏](which is
a one-dimensional space), we can also define a function 𝑓(𝑥, 𝑦) on a rectangle (which is
a two-dimensional space)
So, if a function 𝑦 = 𝑓(𝑥) can define a plane region under the curve and over the interval,
a function 𝑧 = 𝑓(𝑥, 𝑦) can, on the other hand, define a volume under the surface and over
the rectangle, by analogy. If the area of the region under the curve 𝑦 = 𝑓(𝑥) and over the
interval [𝑎, 𝑏] is
𝑏
𝐴 = ∫ 𝑓(𝑥) 𝑑𝑥
𝑎
305
Then the volume of the region under the surface 𝑧 = 𝑓(𝑥, 𝑦) and over the rectangle
[𝑎, 𝑏] × [𝑐, 𝑑] is
𝑑 𝑏
𝑉 = ∫ ∫ 𝑓(𝑥, 𝑦) 𝑑𝑥 𝑑𝑦 16.1
𝑐 𝑎
_____________________________________________________________________________________________________
Example 16.1.
Find the volume of the region under the surface
𝑥2 𝑦2
𝑓(𝑥, 𝑦) = + +1
9 9
over a rectangular region 𝑅 = [1, 2] × [0, 3].
306
In evaluating the double integral, we can evaluate first with respect to the variable 𝑥 and
subsequently with respect to 𝑦. Integration yields
3 2 3 2
𝑥2 𝑦2 𝑥 3 𝑥𝑦 2
∫ ∫ ( + + 1) 𝑑𝑥 𝑑𝑦 = ∫ ( + + 𝑥)| 𝑑𝑦
0 1 9 9 0 27 9 1
3
34 𝑦 2
= ∫ ( + ) 𝑑𝑦
0 27 9
3
34𝑦 𝑦 3
= + |
27 27 0
34 43
= +1= 𝑐𝑢. 𝑢𝑛𝑖𝑡𝑠 ANS.
9 9
_____________________________________________________________________________________________________
In evaluating double integrals, it does not matter whether you will evaluate with
respect to 𝑥 first and 𝑦 next or with respect to 𝑦 first and 𝑥 next. In all cases,
𝑑 𝑏 𝑏 𝑑
∫ ∫ 𝑓(𝑥, 𝑦) 𝑑𝑥 𝑑𝑦 = ∫ ∫ 𝑓(𝑥, 𝑦) 𝑑𝑦 𝑑𝑥 16.2
𝑐 𝑎 𝑎 𝑐
This is known as Fubini’s theorem. In our example, we can evaluate first with respect to 𝑦
and 𝑥 next. The result is
2 3 2 3
𝑥2 𝑦2 𝑦𝑥 2 𝑦 3
∫ ∫ ( + + 1) 𝑑𝑦 𝑑𝑥 = ∫ ( + + 𝑦)| 𝑑𝑥
1 0 9 9 1 9 27 0
2
𝑥2
= ∫ ( + 4) 𝑑𝑥
1 3
2
𝑥3
= ( + 4𝑥)|
9 1
7 43
= +4= 𝑐𝑢. 𝑢𝑛𝑖𝑡𝑠
9 9
If the function 𝑓(𝑥, 𝑦) = 𝑔(𝑥) ∙ ℎ(𝑦), then
𝑑 𝑏 𝑏 𝑑
∫ ∫ 𝑓(𝑥, 𝑦) 𝑑𝑥 𝑑𝑦 = (∫ 𝑔(𝑥) 𝑑𝑥) (∫ ℎ(𝑦) 𝑑𝑦) 16.3
𝑐 𝑎 𝑎 𝑐
307
Although we evaluated the double integral of a function over a rectangular region,
we can also evaluate the double integral over any other closed region which can be of a
different shape. In both instances, the double integral is also called surface integral
because 𝑓(𝑥, 𝑦) describes a surface. Ordinarily, when the double integral is over a
rectangular region R, we write the integral as
∬ 𝑓(𝑥, 𝑦) 𝑑𝐴
𝑅
But when the double integral is over a general region D, we write the integral as
∬ 𝑓(𝑥, 𝑦) 𝑑𝐴
𝐷
The variable 𝑑𝐴 has been generalized for all coordinate systems (e.g., Cartesian, polar,
etc.) For Cartesian coordinate system, 𝑑𝐴 = 𝑑𝑥 𝑑𝑦.
There are two types of general region D. In type I, the region lies between two
continuous functions of 𝑥. Hence,
On the other hand, in type II, the region lies between two continuous functions of 𝑦.
Hence,
But if a function 𝑓(𝑥, 𝑦) is continuous on a type II region D, the double integral is written
as
𝑑 ℎ2 (𝑦)
∬ 𝑓(𝑥, 𝑦) 𝑑𝐴 = ∫ ∫ 𝑓(𝑥, 𝑦) 𝑑𝑥 𝑑𝑦 16.5
𝑐 ℎ1 (𝑦)
𝐷
_____________________________________________________________________________________________________
308
Example 16.2.
Evaluate
∬(𝑥 3 + 2𝑦 2 ) 𝑑𝐴
𝐷
1
Where 𝐷 is the region in the first quadrant bounded by 𝑦 = 4 𝑥 2 and 𝑦 = 2√𝑥.
Solution. The region 𝐷 is shown below. The curves which define the region intersect at
1
(4, 4) so that 𝑎 = 0 and 𝑏 = 4 while 𝑔1 (𝑥) = 4 𝑥 2 and 𝑔2 (𝑥) = 2√𝑥. Thus,
(4, 4)
4 2√𝑥
3 2) (𝑥 3 + 2𝑦 2 ) 𝑑𝑦 𝑑𝑥
∬(𝑥 + 2𝑦 𝑑𝐴 = ∫ ∫
1 2
0 𝑥
𝐷 4
4
3
2 3 2√𝑥
= ∫ (𝑥 𝑦 + 𝑦 )|1 𝑑𝑥
0 3 𝑥2
4
4
16 3/2 1 5 1 6
= ∫ (2𝑥 7/2 + 𝑥 − 𝑥 − 𝑥 ) 𝑑𝑥
0 3 4 96
4 9/2 32 5/2 1 6 1 7 4
=( 𝑥 + 𝑥 − 𝑥 − 𝑥 )|
9 15 24 672 0
309
31744
= ANS.
315
_____________________________________________________________________________________________________
Example 16.3.
Find the volume of the solid under the surface
𝑓(𝑥, 𝑦) = 𝑥 + 𝑦 + 6
1
over a region on the 𝑥𝑦 plane bounded by 𝑦 = −𝑥 2 + 4 and 𝑦 = 2 𝑥 2 − 2.
Solution. The surface and the region D are shown in the graph below. Note that the region
D which is a type I region is bounded on the left and right at
1 2
−𝑥 2 + 4 = 𝑥 −2
2
𝑥 = ±2
The volume of the solid under the surface and the region D is
2 −𝑥 2 +4
𝑉=∫ ∫ (𝑥 + 𝑦 + 6) 𝑑𝑦 𝑑𝑥
1 2
−2 𝑥 −2
2
310
−𝑥 +4 2
2
1
= ∫ (𝑥𝑦 + 𝑦 2 + 6𝑦)|1 𝑑𝑥
−2 2 𝑥 2 −2
2
2 2
1 1 1
= ∫ {𝑥 [(−𝑥 2 + 4) − ( 𝑥 2 − 2)] + [(−𝑥 2 + 4)2 − ( 𝑥 2 − 2) ]
−2 2 2 2
1
+ 6 [(−𝑥 2 + 4) − ( 𝑥 2 − 2)]} 𝑑𝑥
2
2
3 3
= ∫ ( 𝑥 4 − 𝑥 3 − 12𝑥 2 + 6𝑥 + 42) 𝑑𝑥
−2 8 2
2
3 5 3 4
= 𝑥 − 𝑥 − 4𝑥 3 + 3𝑥 2 + 42𝑥|
40 8 −2
544
= 𝑐𝑢. 𝑢𝑛𝑖𝑡𝑠 ANS.
5
_____________________________________________________________________________________________________
Example 16.4.
Using double integrals, calculate the volume of a cylinder whose height is ℎ and whose
radius is 𝑟.
Solution. To calculate the volume of a cylinder, we will consider the volume of solid under
a plane lying on 𝑧 = ℎ parallel to the 𝑥𝑦 plane and over a circle 𝑥 2 + 𝑦 2 = 𝑟 2 . Hence,
𝑟 √𝑟 2 −𝑥 2
𝑉=∫ ∫ ℎ 𝑑𝑦 𝑑𝑥
−𝑟 −√𝑟 2 −𝑥2
𝑟
2 2
= ∫ ℎ𝑦|√𝑟 −𝑥
−√𝑟 2 −𝑥 2
−𝑟
𝑟
= ∫ 2ℎ √𝑟 2 − 𝑥 2 𝑑𝑥
−𝑟
𝑟
𝑟2 𝑥 1
= 2ℎ ( 𝑠𝑖𝑛−1 + 𝑥√𝑟 2 − 𝑥 2 )|
2 𝑟 2 −𝑟
𝑟2 𝜋 𝜋
= 2ℎ [ ( + )] = 𝜋𝑟 2 ℎ ANS.
2 2 2
_____________________________________________________________________________________________________
311
Example 16.5.
1
Find the volume of the solid under the surface 𝑓(𝑥, 𝑦) = 3 𝑥𝑦 and over an ellipse
𝑥2 𝑦2
+ = 1.
4 9
The solid consists of two regions, one in the first octant and another in the third octant.
We carry out the integration with respect to 𝑦 from 0 to the positive root of 𝑦 =
3√1 − 𝑥 2 /4. We also carry out the integration with respect to 𝑥 from 𝑥 = 0 to 𝑥 = 2. The
volume of the solid is twice the volume of one region or
2 3√1−𝑥2 /4
1
𝑉 = 2∫ ∫ 𝑥𝑦 𝑑𝑦 𝑑𝑥
0 0 3
2 3√1−𝑥 2 /4
1
= 2 ∫ 𝑥𝑦 2 |
0 6 0
2
1 𝑥2
= 2∫ 𝑥 [9 (1 − ) − 0] 𝑑𝑥
0 6 4
2
3𝑥 2 3𝑥 4
= − |
2 16 0
312
3 3
= (4 − 0) − (16 − 0)
2 16
= 3 𝑐𝑢. 𝑢𝑛𝑖𝑡𝑠 ANS.
_____________________________________________________________________________________________________
Example 16.6.
Find the volume of the solid under the paraboloid 𝑧 = 𝑥 2 + 𝑦 2 and above a circle
𝑥 2 + 𝑦 2 = 1.
Solution. The solid under consideration is shown below. Since the radius of the circle is
1, we carry out the integration with respect to x from 𝑥 = −1 to 𝑥 = 1. If 𝑥 2 + 𝑦 2 = 1,
𝑦 = ±√1 − 𝑥 2 . Hence, we carry out the integration with respect to y from 𝑦 = −√1 − 𝑥 2
to 𝑦 = +√1 − 𝑥 2 . The volume of the solid under consideration is
1 √1−𝑥2
𝑉=∫ ∫ (𝑥 2 + 𝑦 2 ) 𝑑𝑦 𝑑𝑥
−1 −√1−𝑥 2
2
1
2
1 3 √1−𝑥
=∫ 𝑥 𝑦+ 𝑦 | 𝑑𝑥
−1 3 −√1−𝑥2
1
2
= ∫ [2𝑥 2 √1 − 𝑥 2 + (1 − 𝑥 2 )3/2 ] 𝑑𝑥
−1 3
This integral can be solved using trigonometric substitution with 𝑥 = 𝑠𝑖𝑛 𝜃. It will be left
as an exercise to show that
313
1 1
2 ∫ 𝑥 2 √1 − 𝑥 2 𝑑𝑥 = 2 [ 𝑠𝑖𝑛−1 𝑥 − 𝑥 √1 − 𝑥 2 (1 − 2𝑥 2 )]
8 8
2 2 3 1 1
∫(1 − 𝑥 2 )3/2 𝑑𝑥 = [ 𝑠𝑖𝑛−1 𝑥 + 𝑥 √1 − 𝑥 2 + 𝑥√1 − 𝑥 2 (1 − 2𝑥 2 )]
3 3 8 2 8
1 1 𝜋
𝑉= 𝜋 + 𝜋 = 𝑐𝑢. 𝑢𝑛𝑖𝑡𝑠 ANS.
4 4 2
_____________________________________________________________________________________________________
∫ 𝑓(𝑥) ∙ 𝑓 ′ (𝑥) 𝑑𝑥 = ∫ 𝑢 𝑑𝑢
𝑑𝑥
= ∫ 𝑓(𝑢) ∙ 𝑑𝑢
𝑑𝑢
𝜕(𝑥, 𝑦)
∬ 𝑓(𝑥, 𝑦) 𝑑𝑥 𝑑𝑦 = ∬ 𝑓(𝑢, 𝑣) | | 𝑑𝑢 𝑑𝑣
𝜕(𝑢, 𝑣)
𝑅 𝑆
The term between the absolute value signs is known as the Jacobian which is equal to
𝜕𝑥 𝜕𝑥
𝜕(𝑥, 𝑦)
| | = |𝜕𝑢 𝜕𝑣 |
𝜕(𝑢, 𝑣) 𝜕𝑦 𝜕𝑦
𝜕𝑢 𝜕𝑣
314
Example 16.7.
1
Find the volume of the solid under the surface 𝑓(𝑥, 𝑦) = 3 𝑥𝑦 and over an ellipse
𝑥2 𝑦2
+ = 1, using a change of variables.
4 9
Solution. In Example 15.5., we can evaluate the double integral using a change of
variables. If the surface R in the 𝑥𝑦 plane is the ellipse given by
𝑥2 𝑦2
+ =1
4 9
9𝑥 2 + 4𝑦 2 = 36
a suitable substitution is 𝑥 = 2𝑢 and 𝑦 = 3𝑣. The reason is simplicity. The surface S in the
𝑢𝑣 plane is the ellipse given by
9(2𝑢)2 + 4(3𝑣)2 = 36
𝑢2 + 𝑣 2 = 1
𝜕𝑥 𝜕𝑥
|𝜕𝑢 𝜕𝑣 | = |2 0| = 6
𝜕𝑦 𝜕𝑦 0 3
𝜕𝑢 𝜕𝑣
Hence, the volume of the solid under consideration is
2 3√1−𝑥2 /4 1 √1−𝑢 2
1 1
2∫ ∫ 𝑥𝑦 𝑑𝑦 𝑑𝑥 = 2 ∫ ∫ (2𝑢)(3𝑣)(6) 𝑑𝑣 𝑑𝑢
0 0 3 0 0 3
1 √1−𝑢2
= 24 ∫ ∫ 𝑢𝑣 𝑑𝑣 𝑑𝑢
0 0
2
1
1 2 √1−𝑢
= 24 ∫ 𝑢𝑣 | 𝑑𝑢
0 2 0
1
= 12 ∫ 𝑢(1 − 𝑢2 ) 𝑑𝑢
0
315
1 2 1 4 1
= 12 ( 𝑢 − 𝑢 )|
2 4 0
1 1
= 12 ( − − 0)
2 4
_____________________________________________________________________________________________________
𝑥 = 𝑟 𝑐𝑜𝑠 𝜃
𝑦 = 𝑟 𝑠𝑖𝑛 𝜃
𝜕𝑥 𝜕𝑥
| 𝜕𝑟 𝜕𝜃| = |𝑐𝑜𝑠 𝜃 −𝑟 𝑠𝑖𝑛 𝜃
|
𝜕𝑦 𝜕𝑦 𝑠𝑖𝑛 𝜃 𝑟 𝑐𝑜𝑠 𝜃
𝜕𝑟 𝜕𝜃
= 𝑟 𝑐𝑜𝑠 2 𝜃 − (−𝑟 𝑠𝑖𝑛2 𝜃) = 𝑟
where 𝑎 ≤ 𝑟 ≤ 𝑏 and 𝛼 ≤ 𝜃 ≤ 𝛽.
_____________________________________________________________________________________________________
Example 16.8.
Find the volume of the solid under the paraboloid 𝑧 = 𝑥 2 + 𝑦 2 and above a circle
𝑥 2 + 𝑦 2 = 1, using polar coordinates.
= 𝑟2
316
We integrate with respect to 𝑟 from 𝑟 = 0 to 𝑟 = 1 since the region has a maximum radius
of 1. We also integrate with respect to 𝜃 from 𝜃 = 0 to 𝜃 = 2𝜋 since we map the entire
circle from beginning to end. Hence, the volume of the solid under consideration is
2𝜋 1
𝑉=∫ ∫ 𝑟 2 ∙ 𝑟 𝑑𝑟 𝑑𝜃
0 0
2𝜋
1 41
=∫ 𝑟 | 𝑑𝜃
0 4 0
2𝜋
1
=∫ 𝑑𝜃
0 4
1 2𝜋
= 𝜃|
4 0
𝜋
𝑉= 𝑐𝑢. 𝑢𝑛𝑖𝑡𝑠 ANS.
2
Notice that we were able to arrive at the same answer minus the complications we
encountered in Example 15.6.
_____________________________________________________________________________________________________
Example 16.9.
Use double integrals to calculate the volume of a sphere of radius R.
Solution. The equation of a spherical surface with radius R whose center is at the origin
is
𝑥 2 + 𝑦 2 + 𝑧 2 = 𝑅2
Therefore, the volume of the region below the hemispherical surface 𝑧(𝑥, 𝑦) over a circle
on the 𝑥𝑦 plane with radius 𝑅 is
𝑅 𝑅
𝑉 = ∫ ∫ √𝑅 2 − 𝑥 2 − 𝑦 2 𝑑𝑥 𝑑𝑦
0 0
317
2𝜋 𝑅
1 2 2 2 3/2
=∫ − ∙ (𝑅 − 𝑟 ) | 𝑑𝜃
0 2 3 0
1 2𝜋
= − ∫ (0 − 𝑅 3 ) 𝑑𝜃
3 0
1 3 2𝜋
= 𝑅 𝜃|
3 0
2 3
= 𝜋𝑅
3
Hence, the volume of the entire sphere is twice that or
2 4
𝑉 = 2 ( 𝜋𝑅 3 ) = 𝜋𝑅 3 ANS.
3 3
Change of variables can be useful in evaluating the definite integral of functions
whose antiderivatives cannot be expressed in terms of elementary functions as this next
example will show.
_____________________________________________________________________________________________________
Example 16.10.
Evaluate
∞
2
∫ 𝑒 −𝑥 𝑑𝑥
0
so that
∞ ∞
2 +𝑦 2 )
𝑡 = ∫ ∫ 𝑒 −(𝑥
2
𝑑𝑥 𝑑𝑦
0 0
318
Converting to polar coordinates, 𝑥 2 + 𝑦 2 = 𝑟 2 . We carry out the integration with respect
to 𝑟 from 𝑟 = 0 to 𝑟 = ∞, consistent with the limits of 𝑥. Since 𝑥 = 𝑟 𝑐𝑜𝑠 𝜃, we carry out
the integration with respect to 𝜃 from 𝜃 = 0 corresponding to 𝑒 0 = 1 = 𝑐𝑜𝑠 0, to 𝜃 = 𝜋/2
corresponding to 𝑒 −∞ = 0 = 𝑐𝑜𝑠 𝜋/2. Thus,
𝜋/2 ∞
2 2
𝑡 =∫ ∫ 𝑒 −𝑟 𝑟 𝑑𝑟 𝑑𝜃
0 0
𝜋/2
1 −𝑟 2 ∞
=∫ (− 𝑒 )| 𝑑𝜃
0 2 0
𝜋/2
1
=∫ (0 + ) 𝑑𝜃
0 2
1 𝜋/2
= 𝜃|
2 0
𝜋
=
4
This means that
∞
2 √𝜋
𝑡 = ∫ 𝑒 −𝑥 𝑑𝑥 = ANS.
0 2
_____________________________________________________________________________________________________
Triple Integrals
In the previous section, we evaluated the double integral of a function of two
variables over a rectangular region and subsequently over a general region in a plane. If
we have a function of three variables, then we can evaluate the triple integral of the
function over a 3-D region, say, a box B, and eventually, over a general solid region E.
319
If, on the other hand, 𝑓(𝑥, 𝑦, 𝑧) is continuous on a 3-D region E, we can also define the
triple integral of the function over E. There are also two types of such 3-D region. In type
I, the solid region has a projection D which is also a type I 2-D region so that
In type II, the solid region has a projection D which is also a type II 2-D region so that
In the previous section, we evaluated the double integral as the volume of the solid
under the function and over the region D. For triple integrals, the interpretation is rather
more abstract and less useful because the triple integral may be considered as the
“volume” of a 4-dimensional “solid,” which is difficult to visualize.
_____________________________________________________________________________________________________
Example 16.11.
Evaluate the integral
∭(3𝑥𝑦 − 𝑧 2 ) 𝑑𝑉
320
1 3 1
9 2
∫ ( 𝑥𝑦 − 3𝑦)| 𝑑𝑥 = ∫ (36𝑥 − 6) 𝑑𝑥
0 2 1 0
Example 16.12.
Evaluate the iterated integral
2 𝑥2 𝑥/𝑦
∫ ∫ ∫ 3𝑥𝑦 3 𝑧 𝑑𝑧 𝑑𝑦 𝑑𝑥
1 0 0
2 𝑥2 𝑥/𝑦 2 𝑥2 𝑥/𝑦
3
3𝑥𝑦 3 𝑧 2
∫ ∫ ∫ 3𝑥𝑦 𝑧 𝑑𝑧 𝑑𝑦 𝑑𝑥 = ∫ ∫ | 𝑑𝑦 𝑑𝑥
1 0 0 1 0 2 0
3 𝑥2
3𝑥 3 𝑦
=∫ ∫ 𝑑𝑦 𝑑𝑥
1 0 2
2 𝑥2 2 𝑥2
3𝑥 3 𝑦 3𝑥 3 𝑦 2
∫ ∫ 𝑑𝑦 𝑑𝑥 = ∫ | 𝑑𝑥
1 0 2 1 4 0
2
3𝑥 3 𝑥 4
=∫ 𝑑𝑥
1 4
2
3𝑥 7
=∫ 𝑑𝑥
1 4
3
= (256 − 1)
32
321
765
= ANS.
32
_____________________________________________________________________________________________________
Example 16.13.
Evaluate the iterated integral
4 𝑥 √𝑦 2 −1
∫ ∫ ∫ (𝑥𝑦 + 𝑦𝑧 + 𝑥𝑧) 𝑑𝑧 𝑑𝑦 𝑑𝑥
1 1 −√𝑦 2 −1
2 5
= (√15) = 30√15 ANS.
15
_____________________________________________________________________________________________________
322
Try these.
3. Find the volume of the solid under 𝑓(𝑥, 𝑦) = 𝑥 2 + 𝑦 2 and over the plane region
bounded by 𝑦 = 𝑥 2 and 𝑦 = √𝑥.
323
4. Find the volume of solid under 𝑓(𝑥, 𝑦) = 𝑥 + 2𝑦 and over a plane region
bounded by the lines 𝑦 = 𝑥 − 1, 𝑦 = 7 − 𝑥, and the x-axis.
324
References
Rohde, Jain, Poddar, and Ghosh (2012). Introduction to Integral Calculus: Systematic
Studies with Engineering Applications for Beginners. New Jersey: John Wiley & Sons.
Stewart, James (2018). Calculus : International Metric Version, 8th ed. Singapore: Cengage
Learning Asia.
www.desmos.com/calculator
www.georgebra.org
www.integral-calculator.com
www.symbolab.com
www.wolframalpha.com
325
Standard Derivatives
Power, Exponential, and Logarithmic Functions
𝑑 𝑛
𝑥 = 𝑛𝑥 𝑛−1
𝑑𝑥
𝑑 1
√𝑥 =
𝑑𝑥 2√𝑥
𝑑 𝑎𝑥
𝑒 = 𝑎𝑒 𝑎𝑥
𝑑𝑥
𝑑 𝑎𝑥
𝑏 = 𝑎𝑏 𝑎𝑥 𝑙𝑛 𝑏
𝑑𝑥
𝑑 1
𝑙𝑛 𝑥 =
𝑑𝑥 𝑥
𝑑 1
𝑙𝑜𝑔𝑏 𝑥 =
𝑑𝑥 𝑥 𝑙𝑛 𝑏
Trigonometric Functions
𝑑
𝑠𝑖𝑛 𝑎𝑥 = 𝑎 𝑐𝑜𝑠 𝑎𝑥
𝑑𝑥
𝑑
𝑐𝑜𝑠 𝑎𝑥 = −𝑎 𝑠𝑖𝑛 𝑎𝑥
𝑑𝑥
𝑑
𝑡𝑎𝑛 𝑎𝑥 = 𝑎 𝑠𝑒𝑐 2 𝑎𝑥
𝑑𝑥
𝑑
𝑐𝑜𝑡 𝑎𝑥 = −𝑎 𝑐𝑠𝑐 2 𝑎𝑥
𝑑𝑥
𝑑
𝑠𝑒𝑐 𝑎𝑥 = 𝑎 𝑠𝑒𝑐 𝑎𝑥 𝑡𝑎𝑛 𝑎𝑥
𝑑𝑥
𝑑
𝑐𝑠𝑐 𝑎𝑥 = −𝑎 𝑐𝑠𝑐 𝑎𝑥 𝑐𝑜𝑡 𝑎𝑥
𝑑𝑥
326
Inverse Trigonometric Functions
𝑑 𝑥 1
𝑠𝑖𝑛−1 =
𝑑𝑥 𝑎 √𝑎2 − 𝑥 2
𝑑 𝑥 1
𝑐𝑜𝑠 −1 = −
𝑑𝑥 𝑎 √𝑎2 − 𝑥 2
𝑑 𝑥 𝑎
𝑡𝑎𝑛−1 = 2
𝑑𝑥 𝑎 𝑎 + 𝑥2
𝑑 𝑥 𝑎
𝑐𝑜𝑡 −1 = − 2
𝑑𝑥 𝑎 𝑎 + 𝑥2
𝑑 𝑥 𝑎
𝑠𝑒𝑐 −1 =
𝑑𝑥 𝑎 𝑥 √𝑥 2 − 𝑎2
𝑑 𝑥 𝑎
𝑐𝑠𝑐 −1 = −
𝑑𝑥 𝑎 𝑥 √𝑥 − 𝑎2
2
Hyperbolic Functions
𝑑
𝑠𝑖𝑛ℎ 𝑎𝑥 = 𝑎 𝑐𝑜𝑠ℎ 𝑎𝑥
𝑑𝑥
𝑑
𝑐𝑜𝑠ℎ 𝑎𝑥 = 𝑎 𝑠𝑖𝑛ℎ 𝑎𝑥
𝑑𝑥
𝑑
𝑡𝑎𝑛ℎ 𝑎𝑥 = 𝑎 𝑠𝑒𝑐ℎ2 𝑎𝑥
𝑑𝑥
𝑑
𝑐𝑜𝑡ℎ 𝑎𝑥 = −𝑎 𝑐𝑠𝑐ℎ2 𝑎𝑥
𝑑𝑥
𝑑
𝑠𝑒𝑐ℎ 𝑎𝑥 = −𝑎 𝑠𝑒𝑐ℎ 𝑥 𝑡𝑎𝑛ℎ 𝑥
𝑑𝑥
𝑑
𝑐𝑠𝑐ℎ 𝑎𝑥 = −𝑎 𝑐𝑠𝑐ℎ 𝑎𝑥 𝑐𝑜𝑡ℎ 𝑎𝑥
𝑑𝑥
327
𝑑 𝑥 1
𝑠𝑖𝑛ℎ−1 =
𝑑𝑥 𝑎 √𝑎2 + 𝑥 2
𝑑 𝑥 1
𝑐𝑜𝑠ℎ−1 =
𝑑𝑥 𝑎 √𝑥 − 𝑎2
2
𝑑 𝑥 𝑎
𝑡𝑎𝑛ℎ−1 = 2
𝑑𝑥 𝑎 𝑎 − 𝑥2
𝑑 𝑥 𝑎
𝑐𝑜𝑡ℎ−1 = 2
𝑑𝑥 𝑎 𝑎 − 𝑥2
𝑑 𝑥 𝑎
𝑠𝑒𝑐ℎ−1 = −
𝑑𝑥 𝑎 𝑥 √𝑎2 − 𝑥 2
𝑑 𝑥 𝑎
𝑐𝑠𝑐ℎ−1 = −
𝑑𝑥 𝑎 |𝑥| √𝑥 2 + 𝑎2
Standard Integrals
328
Power, Exponential, and Logarithmic Functions
𝑛
𝑥 𝑛+1
∫ 𝑥 𝑑𝑥 = + 𝐶, 𝑛 ≠ −1
𝑛+1
1
∫ 𝑑𝑥 = 𝑙𝑛 |𝑥| + 𝐶
𝑥
𝑒 𝑎𝑥
∫ 𝑒 𝑎𝑥 𝑑𝑥 = +𝐶
𝑎
𝑏 𝑎𝑥
∫ 𝑏 𝑎𝑥 𝑑𝑥 = +𝐶
𝑎 𝑙𝑛 𝑏
∫ 𝑙𝑛 𝑥 𝑑𝑥 = 𝑥 𝑙𝑛 𝑥 − 𝑥 + 𝐶
Trigonometric Functions
1
∫ 𝑠𝑖𝑛 𝑎𝑥 𝑑𝑥 = − 𝑐𝑜𝑠 𝑎𝑥 + 𝐶
𝑎
1
∫ 𝑐𝑜𝑠 𝑎𝑥 𝑑𝑥 = 𝑠𝑖𝑛 𝑎𝑥 + 𝐶
𝑎
1
∫ 𝑡𝑎𝑛 𝑎𝑥 𝑑𝑥 = − 𝑙𝑛 |𝑐𝑜𝑠 𝑎𝑥| + 𝐶
𝑎
1
∫ 𝑐𝑜𝑡 𝑎𝑥 𝑑𝑥 = 𝑙𝑛 |𝑠𝑖𝑛 𝑎𝑥| + 𝐶
𝑎
1
∫ 𝑠𝑒𝑐 𝑎𝑥 𝑑𝑥 = 𝑙𝑛 |𝑠𝑒𝑐 𝑎𝑥 + 𝑡𝑎𝑛 𝑎𝑥| + 𝐶
𝑎
1
∫ 𝑐𝑠𝑐 𝑎𝑥 𝑑𝑥 = 𝑙𝑛 |𝑐𝑠𝑐 𝑎𝑥 − 𝑐𝑜𝑡 𝑎𝑥| + 𝐶
𝑎
1
∫ 𝑠𝑒𝑐 2 𝑎𝑥 𝑑𝑥 = 𝑡𝑎𝑛 𝑎𝑥 + 𝐶
𝑎
1
∫ 𝑐𝑠𝑐 2 𝑎𝑥 𝑑𝑥 = − 𝑐𝑜𝑡 𝑎𝑥 + 𝐶
𝑎
329
1
∫ 𝑠𝑒𝑐 𝑎𝑥 ∙ 𝑡𝑎𝑛 𝑎𝑥 𝑑𝑥 = 𝑠𝑒𝑐 𝑎𝑥 + 𝐶
𝑎
1
∫ 𝑐𝑠𝑐 𝑎𝑥 ∙ 𝑐𝑜𝑡 𝑎𝑥 𝑑𝑥 = − 𝑐𝑠𝑐 𝑎𝑥 + 𝐶
𝑎
1
∫ 𝑠𝑖𝑛−1 𝑎𝑥 𝑑𝑥 = 𝑥 𝑠𝑖𝑛−1 𝑎𝑥 + √1 − 𝑎2 𝑥 2 + 𝐶
𝑎
1
∫ 𝑐𝑜𝑠 −1 𝑎𝑥 𝑑𝑥 = 𝑥 𝑐𝑜𝑠 −1 𝑎𝑥 − √1 − 𝑎2 𝑥 2 + 𝐶
𝑎
1
∫ 𝑡𝑎𝑛−1 𝑎𝑥 𝑑𝑥 = 𝑥 𝑡𝑎𝑛−1 𝑎𝑥 − 𝑙𝑛 (1 + 𝑎2 𝑥 2 ) + 𝐶
2𝑎
1
∫ 𝑐𝑜𝑡 −1 𝑎𝑥 𝑑𝑥 = 𝑥 𝑐𝑜𝑡 −1 𝑎𝑥 + 𝑙𝑛 (1 + 𝑎2 𝑥 2 ) + 𝐶
2𝑎
1
∫ 𝑠𝑒𝑐 −1 𝑎𝑥 𝑑𝑥 = 𝑥 𝑠𝑒𝑐 −1 𝑎𝑥 − 𝑐𝑜𝑠ℎ−1 𝑎𝑥 + 𝐶
𝑎
1
∫ 𝑐𝑠𝑐 −1 𝑎𝑥 𝑑𝑥 = 𝑥 𝑐𝑠𝑐 −1 𝑎𝑥 + 𝑐𝑜𝑠ℎ−1 𝑎𝑥 + 𝐶
𝑎
Hyperbolic Functions
1
∫ 𝑠𝑖𝑛ℎ 𝑎𝑥 𝑑𝑥 = 𝑐𝑜𝑠ℎ 𝑎𝑥 + 𝐶
𝑎
1
∫ 𝑐𝑜𝑠ℎ 𝑎𝑥 𝑑𝑥 = 𝑠𝑖𝑛ℎ 𝑎𝑥 + 𝐶
𝑎
1
∫ 𝑡𝑎𝑛ℎ 𝑎𝑥 𝑑𝑥 = 𝑙𝑛 |𝑐𝑜𝑠ℎ 𝑎𝑥| + 𝐶
𝑎
1
∫ 𝑐𝑜𝑡ℎ 𝑎𝑥 𝑑𝑥 = 𝑙𝑛 |𝑠𝑖𝑛ℎ 𝑎𝑥| + 𝐶
𝑎
1
∫ 𝑠𝑒𝑐ℎ 𝑎𝑥 𝑑𝑥 = 𝑡𝑎𝑛−1 (𝑠𝑖𝑛ℎ 𝑎𝑥) + 𝐶
𝑎
330
1
∫ 𝑐𝑠𝑐ℎ 𝑎𝑥 𝑑𝑥 = 𝑙𝑛 |𝑐𝑠𝑐ℎ 𝑎𝑥 − 𝑐𝑜𝑡ℎ 𝑎𝑥| + 𝐶
𝑎
1
∫ 𝑠𝑒𝑐ℎ2 𝑎𝑥 𝑑𝑥 = 𝑡𝑎𝑛ℎ 𝑎𝑥 + 𝐶
𝑎
1
∫ 𝑐𝑠𝑐ℎ2 𝑎𝑥 𝑑𝑥 = − 𝑐𝑜𝑡ℎ 𝑎𝑥 + 𝐶
𝑎
1
∫ 𝑠𝑒𝑐ℎ 𝑎𝑥 ∙ 𝑡𝑎𝑛ℎ 𝑎𝑥 𝑑𝑥 = − 𝑠𝑒𝑐ℎ 𝑎𝑥 + 𝐶
𝑎
1
∫ 𝑐𝑠𝑐ℎ 𝑎𝑥 ∙ 𝑐𝑜𝑡ℎ 𝑎𝑥 𝑑𝑥 = − 𝑐𝑠𝑐ℎ 𝑎𝑥 + 𝐶
𝑎
1
∫ 𝑠𝑖𝑛ℎ−1 𝑎𝑥 𝑑𝑥 = 𝑥 𝑠𝑖𝑛ℎ−1 𝑎𝑥 − √𝑎2 𝑥 2 + 1 + 𝐶
𝑎
1
∫ 𝑐𝑜𝑠ℎ−1 𝑎𝑥 𝑑𝑥 = 𝑥 𝑐𝑜𝑠ℎ−1 𝑎𝑥 − √𝑎2 𝑥 2 − 1 + 𝐶
𝑎
1
∫ 𝑡𝑎𝑛ℎ−1 𝑎𝑥 𝑑𝑥 = 𝑥 𝑡𝑎𝑛ℎ−1 𝑎𝑥 + 𝑙𝑛 (1 − 𝑎2 𝑥 2 ) + 𝐶
2𝑎
1
∫ 𝑐𝑜𝑡ℎ−1 𝑎𝑥 𝑑𝑥 = 𝑥 𝑐𝑜𝑡ℎ−1 𝑎𝑥 + 𝑙𝑛 (𝑎2 𝑥 2 − 1) + 𝐶
2𝑎
1
∫ 𝑠𝑒𝑐ℎ−1 𝑎𝑥 𝑑𝑥 = 𝑥 𝑠𝑒𝑐ℎ−1 𝑎𝑥 + 𝑠𝑖𝑛−1 𝑎𝑥 + 𝐶
𝑎
1
∫ 𝑐𝑠𝑐ℎ−1 𝑎𝑥 𝑑𝑥 = 𝑥 𝑐𝑠𝑐ℎ−1 𝑎𝑥 + 𝑠𝑖𝑛ℎ−1 𝑎𝑥 + 𝐶
𝑎
𝑑𝑥 𝑥
∫ = 𝑠𝑖𝑛−1 +𝐶
√𝑎2 − 𝑥 2 𝑎
331
𝑑𝑥 𝑥
∫ = −𝑐𝑜𝑠 −1 +𝐶
√𝑎2 − 𝑥 2 𝑎
𝑑𝑥 1 𝑥
∫ = 𝑡𝑎𝑛−1 + 𝐶
𝑎2 +𝑥 2 𝑎 𝑎
𝑑𝑥 1 𝑥
∫ = − 𝑐𝑜𝑡 −1 + 𝐶
𝑎2 +𝑥 2 𝑎 𝑎
𝑑𝑥 1 𝑥
∫ = 𝑠𝑒𝑐 −1 + 𝐶
𝑥√𝑥 2 − 𝑎2 𝑎 𝑎
𝑑𝑥 1 𝑥
∫ =− 𝑐𝑠𝑐 −1 + 𝐶
𝑥√𝑥 2 − 𝑎2 𝑎 𝑎
𝑑𝑥 𝑥
∫ = 𝑠𝑖𝑛ℎ−1 +𝐶
√𝑥 2 + 𝑎2 𝑎
𝑑𝑥 𝑥
∫ = 𝑐𝑜𝑠ℎ−1 +𝐶
√𝑥 2 − 𝑎2 𝑎
𝑑𝑥 1 −1
𝑥
∫ = 𝑡𝑎𝑛ℎ + 𝐶, 𝑥 2 < 𝑎2
𝑎2 − 𝑥 2 𝑎 𝑎
𝑑𝑥 1 −1
𝑥
∫ = 𝑐𝑜𝑡ℎ + 𝐶, 𝑥 2 > 𝑎2
𝑎2 − 𝑥 2 𝑎 𝑎
𝑑𝑥 1 𝑥
∫ =− 𝑠𝑒𝑐ℎ−1 + 𝐶
𝑥√𝑎2 − 𝑥 2 𝑎 𝑎
𝑑𝑥 1 𝑥
∫ =− 𝑐𝑠𝑐ℎ−1 + 𝐶
𝑥√𝑎2 + 𝑥 2 𝑎 𝑎
1 𝑎2 𝑥
∫ √𝑥 2 + 𝑎2 𝑑𝑥 = 𝑥√𝑥 + 𝑎 + 𝑠𝑖𝑛ℎ−1 + 𝐶
2 2
2 2 𝑎
1 𝑎2 𝑥
∫ √𝑥 2 − 𝑎2 𝑑𝑥 = 𝑥√𝑥 2 − 𝑎2 − 𝑐𝑜𝑠ℎ−1 + 𝐶
2 2 𝑎
332
1 𝑎2 𝑥
√ 2 2 √ 2 2
∫ 𝑎 − 𝑥 𝑑𝑥 = 𝑥 𝑎 − 𝑥 + 𝑠𝑖𝑛−1 + +𝐶
2 2 𝑎
1 𝑛−1
∫ 𝑠𝑖𝑛𝑛 𝑥 𝑑𝑥 = − 𝑐𝑜𝑠 𝑥 𝑠𝑖𝑛𝑛−1 𝑥 + ∫ 𝑠𝑖𝑛𝑛−2 𝑥 𝑑𝑥
𝑛 𝑛
1 𝑛−1
∫ 𝑐𝑜𝑠 𝑛 𝑥 𝑑𝑥 = 𝑠𝑖𝑛 𝑥 𝑐𝑜𝑠 𝑛−1 𝑥 + ∫ 𝑐𝑜𝑠 𝑛−2 𝑥 𝑑𝑥
𝑛 𝑛
1
∫ 𝑡𝑎𝑛𝑛 𝑥 𝑑𝑥 = 𝑡𝑎𝑛𝑛−1 𝑥 − ∫ 𝑡𝑎𝑛𝑛−2 𝑥 𝑑𝑥
𝑛−1
1
∫ 𝑐𝑜𝑡 𝑛 𝑥 𝑑𝑥 = − 𝑐𝑜𝑡 𝑛−1 𝑥 − ∫ 𝑐𝑜𝑡 𝑛−2 𝑥 𝑑𝑥
𝑛−1
1 𝑛−2
∫ 𝑠𝑒𝑐 𝑛 𝑥 𝑑𝑥 = 𝑠𝑒𝑐 𝑛−2 𝑥 𝑡𝑎𝑛 𝑥 + ∫ 𝑠𝑒𝑐 𝑛−2 𝑥 𝑑𝑥
𝑛−1 𝑛−1
1 𝑛−2
∫ 𝑐𝑠𝑐 𝑛 𝑥 𝑑𝑥 = − 𝑐𝑠𝑐 𝑛−2 𝑥 𝑐𝑜𝑡 𝑥 + ∫ 𝑐𝑠𝑐 𝑛−2 𝑥 𝑑𝑥
𝑛−1 𝑛−1
∫ 𝑥 𝑛 𝑒 𝑥 𝑑𝑥 = 𝑥 𝑛 𝑒 𝑥 − 𝑛 ∫ 𝑥 𝑛−1 𝑒 𝑥 𝑑𝑥
333
Basic Identities
Trigonometric Hyperbolic
1 1 1 1 1 1
𝑐𝑜𝑠 2 𝑥 = √2 + 2 𝑐𝑜𝑠 𝑥 𝑐𝑜𝑠ℎ 2 𝑥 = √2 𝑐𝑜𝑠ℎ 𝑥 + 2
1 1 1 1 1 1
𝑠𝑖𝑛 2 𝑥 = √2 − 2 𝑐𝑜𝑠 𝑥 𝑠𝑖𝑛ℎ 2 𝑥 = √2 𝑐𝑜𝑠ℎ 𝑥 − 2
1 𝑠𝑖𝑛 𝑥 1 𝑠𝑖𝑛ℎ 𝑥
𝑡𝑎𝑛 2 𝑥 = 𝑡𝑎𝑛ℎ 2 𝑥 =
1 + 𝑐𝑜𝑠 𝑥 1 + 𝑐𝑜𝑠ℎ 𝑥
𝑐𝑜𝑠 (𝑥 ± 𝑦) = 𝑐𝑜𝑠 𝑥 𝑐𝑜𝑠 𝑦 ∓ 𝑠𝑖𝑛 𝑥 𝑠𝑖𝑛 𝑦 𝑐𝑜𝑠ℎ (𝑥 ± 𝑦) = 𝑐𝑜𝑠ℎ 𝑥 𝑐𝑜𝑠ℎ 𝑦 ± 𝑠𝑖𝑛ℎ 𝑥 𝑠𝑖𝑛ℎ 𝑦
𝑠𝑖𝑛 (𝑥 ± 𝑦) = 𝑠𝑖𝑛 𝑥 𝑐𝑜𝑠 𝑦 ± 𝑐𝑜𝑠 𝑥 𝑠𝑖𝑛 𝑦 𝑠𝑖𝑛ℎ (𝑥 ± 𝑦) = 𝑠𝑖𝑛ℎ 𝑥 𝑐𝑜𝑠ℎ 𝑦 ± 𝑐𝑜𝑠ℎ 𝑥 𝑠𝑖𝑛ℎ 𝑦
334